You are on page 1of 147

AKASH SINGH

A
contact me @ITAKASHSINGH

अस्वीकरण
इस नोट्स में प्रदान की गई सामग्री, ले खकों द्वारा बनाई गई तथा उनके स्वामित्व में है, तथा जिसका लाइसेंस सॉर्टिंग हैट
टे क्नोलॉजीज प्राइवेट लिमिटे ड (कंपनी) को कंपनी के प्लेटफॉर्म से जुड़े शिक्षार्थि यों तक पहुंच प्रदान करने के एकमात्र उद्देश्य
से दिया गया है। कंपनी, नोट्स की सामग्री, छवियों या किसी भी अन्य सामग्री के संबध
ं में सभी अधिकारों और देनदारियों
को अस्वीकृति करती है। ले खकों का सामग्री पर एकमात्र स्वामित्व हैं, और अंतिम रुप, बिना किसी सीमा के इस नोट्स की
सामग्री के संबध
ं में उत्पन्न होने वाले किसी भी दावे, देनदारियों, क्षति, नुकसान या वाद के लिए पूरी तरह से जिम्द
मे ार होंगे।

NOT FOR SALE

B
विषय-सूची

03 14 28 39
अध्याय - 1 अध्याय - 2 अध्याय - 3 अध्याय - 4
विज्ञान और प्रोद्योगिकी का सूचना प्रौद्योगिकी जैव प्रौद्योगिकी स्वास्थ्य
विकास

50 53 71 89
अध्याय - 5 अध्याय - 6 अध्याय - 7 अध्याय - 8
नैनो तकनीक नाभिकीय ऊर्जा अंतरिक्ष प्रोद्योगिकी रक्षा

AKASH SINGH
97 108 118
अध्याय - 9 अध्याय - 10 विगत वर्षों के प्रश्न
कुछ उन्नत प्रौद्योगिकियां बौद्धिक संपदा अधिकार

2
AKASH SINGH
अध्याय - 1

विज्ञान और प्रोद्योगिकी का विकास


(EVOLUTION OF SCIENCE AND TECHNOLOGY)

• भारत के पास एक समृद्ध वैज्ञानिक और तकनीकी • शहरों में नवीन सिं चाई प्रणाली थी, और इसने शहरी केंद्रों
विरासत है। यह अध्याय प्राचीन सिं धु घाटी सभ्यता से के सतत विकास को सुनिश्चित किया।
संबंधित वैज्ञानिक और तकनीकी प्रगति के इतिहास की
एक त्वरित झलक प्रदान करता है।
3. वजन, माप और धातु विज्ञान (Weights,
Measurement and Metallurgy)

सिंधु घाटी सभ्यता (Indus • सिं धु घाटी सभ्यता के लोगों द्वारा वजन और उप-वजन
की एक अत्यंत प्रभावी प्रणाली को अपनाया गया था।
Valley Civilization, IVC) चूंकि व्यापार अर्थव्यवस्था का एक महत्वपूर्ण हिस्सा
था, इसलिए बाट और माप की आधुनिक प्रणाली का
• भारतीय पुरातत्व सर्वेक्षण (ASI) और अन्य एजेंसियों द्वारा
होना महत्वपूर्ण था। वर्तमान मानकीकरण और गुणवत्ता
की गई खुदाई सिं धु घाटी सभ्यता के क्षेत्रों में रहने वाले
नियंत्रण की उत्पत्ति प्राचीन काल से हुई है।
नागरिकों की विभिन्न वैज्ञानिक प्रगति को इं गित करती
है। • साक्ष्य बताते हैं कि लोगों द्वारा तेज गर्मी से बचाने के लिए
कपास का व्यापक रूप से उपयोग किया जाता था। इस
प्रकार, गर्मि यों में सूती कपड़े पहनने की हमारी वर्तमान
प्रवृत्ति प्राचीन काल से शुरू हुई।
1. वास्तुकला और सिविल इं जीनियरिं ग
(Architecture and Civil Engineering)

• शहरी केंद्र, वैज्ञानिक सड़कों; अपवाह प्रणाली (कॉर्बेल 4. मिट्टी के बर्तन और मनके बनाना (Pottery
तकनीक के साथ); और नियोजित संरचनाओ ं (उदाहरणों and Bead Making)
में विशाल स्नानागार और अन्न भंडार शामिल हैं) के
साथ एक निश्चित खाका स्वरूप पर आधारित थे। • लोगों द्वारा किया जाने वाला एक महत्वपूर्ण कार्य मिट्टी
के बर्तन बनाना था। विभिन्न एजेंसियों को कुम्हार का
• उन्हें लोथल में दुनिया का पहला ज्वारीय बंदरगाह बनाने
पहिया म‍ िला है।
का श्रेय दिया जाता है। इतिहासकार बताते हैं कि इससे
संकेत मिलता है कि वे ज्वार की भौगोलिक घटना • ताप-प्रौद्योगिकी जिसका उपयोग साधारण वस्तुओ ं
को समझते हैं (केंद्रीय बजट 2020 में भारत में समुद्री के उत्पादन के लिए ऊष्माक्षेपी (exothermic) और
संग्रहालय स्थापित करने की घोषणा की गई है और ऊष्माशोषी (Endothermic) प्रतिक्रियाओ ं को प्रेरित
लोथल गंतव्यों में से एक है)। करने के लिए किया जाता था, का व्यापक रूप से उपयोग
किया जाता था।
• जहां तक आवास का संबंध है, अलग-अलग घरों में निजी
स्नानघरों की सुविधा थी और सभी में जल आपूर्ति तंत्र • लोग भट्टियों से परिचित थे और मनका बनाना एक
भी था। अन्य महत्वपूर्ण आर्थि क गतिविधि थी जिसने रत्न और
आभूषण उद्योग के विकास में मदद की। इसका बड़ा भाग
• सरकार की कुछ हालिया योजनाओ ं जैसे स्मार्ट सिटी
अन्य सभ्यताओ ं को भी निर्यात किया जाता था।
परियोजना के पीछे की प्रेरणा का पता इस काल से
लगाया जा सकता है।

5. चिकित्सा विज्ञान (Medical Science)


2. सिं चाई प्रणाली (Irrigation System) • विभिन्न रिपोर्टों के अनुसार, इस सभ्यता के नागरिकों ने
• सिं धु घाटी सभ्यता में बड़े पैमाने पर शुष्क और अर्ध- दंत विज्ञान में भी महारत हासिल की थी, जिसमें छिद्रों का
शुष्क क्षेत्रों थे और इसके लिए कृत्रिम जलाशयों और जल इलाज करना और दंत समस्याओ ं का उपचार शामिल है,
संरचनाओ ं के निर्माण की आवश्यकता थी। कई नदियों जिसका रोगी को सामना करना पड़ता था। मेहरगढ़ के
की मौजूदगी के बावजूद, जल एक दुर्ल भ वस्तु थी। साक्ष्य इस बात का खुलासा करते हैं।

AKASH SINGH
• IVC के लोग ट्रेफिनेशन का भी अभ्यास करते थे, एक ऐसी • चरक ने चरक संहिता की रचना की। उन्होंने विभिन्न
तकनीक जिसमें माइग्रेन जैसी छोटी-मोटी समस्याओ ं के रोगों के वर्गीकरण पर भी चर्चा की जिससे आधुनिक
इलाज के लिए खोपड़ी के पास छे द करना शामिल था। समय के वर्गीकरण का मार्ग प्रशस्त हुआ।

• उन्होंने आयुर्वेद चिकित्सा पद्धति में महत्वपूर्ण योगदान


दिया। उन्होंने आयुर्वेद को स्वास्थ्य देखभाल की एक
6. गणित (Mathematics) व्यापक प्रणाली के रूप में माना जो निवारक और
उपचारात्मक दोनों पहलु ओ ं से उपचार करती है।
• हड़प्पावासी शायद सरल गणितीय समीकरणों और
अंकों के बारे में भी जानते थे। व्यापक व्यापार और बढ़ती
अर्थव्यवस्था ने इसे आवश्यक बना दिया।
पिं गला (Pingala)
• हड़प्पा की संख्यात्मक प्रणाली दशमलव प्रणाली का
अनुसरण करती हुई दिखाई दी। • आचार्य पिं गला एक प्राचीन गणितज्ञ थे, जिन्होंने छोटी
और लं बी प्रणालियों में द्विआधारी संख्याओ ं का उपयोग
किया था जो कि विकसित किए गए मोर्स कोड के समान
था।
वैदिक काल (Vedic Period) • आधुनिक समय में, विभिन्न आईटी कार्यक्रमों जैसे
• इतिहास का प्रत्येक काल पूर्व काल की उपलब्धियों पर कोडिं ग, कंप्यूटर भाषा प्रणाली आदि का विकास इस
निर्मि त होता है। वैदिक काल अलग नहीं था। चूंकि वे उन द्विआधारी प्रणाली के अध्ययन के कारण हो सका है।
क्षेत्रों में रहते थे जो भारत-गंगा के मैदानों का निर्माण
करते थे, कृषि, धातु विज्ञान, भूगोल में प्रगति अन्य लोगों
के बीच वैज्ञानिक संस्कृति का एक महत्वपूर्ण हिस्सा
बन गई। महत्वपूर्ण भौगोलिक घटनाएं जैसे मानसून के
उत्तर-वैदिक काल (Post-
दौरान पवनों का उलट जाना, बुवाई का मौसम ‍आदि से Vedic Period):
लोग परिचित थे और उनका अध्ययन किया जाता था।
आर्यभट्ट-I (Aryabhatta-I)

• गुप्त काल में वे एक महत्वपूर्ण गणितज्ञ के रूप में उभरे।


सुश्रुत (Sushruta): गणित में आर्यभट्ट के मुख्य योगदान में पाई (Pi) के मूल्य
• ऋषि सुश्रुत छठी शताब्दी ईसा पूर्व में रहते थे। सुश्रुत को का अनुमान, बीजगणित और त्रिकोणमिति से संबंधित
अक्सर प्लास्टिक सर्जरी और सामान्य सर्जरी का जनक अन्य समीकरण शामिल हैं।
कहा जाता है। • उनकी पाई की गणना यूनानियों की तुलना में कहीं
• सर्जरी के अध्ययन में उनके योगदान की दुनिया भर के अधिक सटीक है और गणितज्ञों ने इसे स्वीकार किया है।
चिकित्सा पेशेवरों ने सराहना की है। • पृथ्वी की अपनी धुरी पर घूमने की अवधारणा और इसके
• मानव शरीर रचना विज्ञान एक महत्वपूर्ण विषय था घूर्णन के कारण मौसमी परिवर्तन उनका एक और
जिसमें उन्होंने व्यापक शोध किया। योगदान था और यह गैलीलियो द्वारा की गई खोज से भी
पहले का है।
• कुष्ठ रोग, मोतियाबिं द सर्जरी और ऐसी कई अन्य
प्रक्रियाओ ं का वर्णन किया गया है। कई बीमारियों का • सरकार ने अपने पहले उपग्रह का नाम उन्हीं के नाम पर
इलाज अच्छी तरह से प्रले खित किया गया है। 760 से रखकर उन्हें सम्मानित किया।
अधिक पौधों का उनकी उपयोगिता के साथ वर्णन किया
गया है।

• सुश्रुत संहिता में 1100 से अधिक बीमारियों का उल्ले ख है,


जिनमें कई प्रकार के बुखार, शरीर की बीमारी और 21 वीं
सदी में होने वाले अन्य रोग शामिल हैं।

चरक (Charaka)

AKASH SINGH
'आर्यभटियम' पुस्तक को चार अध्यायों में विभाजित किया पुस्तक में साइन (Sine) के मान का भी अनुमान लगाया।
गया है और निम्नलिखित ब्रह्मांडीय सिद्धांत शामिल हैं:

• खगोलीय स्थिरांक और साइन सूची, भास्कर द्वितीय (1114 -1185 ई.) (Bhaskara-II)
• गणना के लिए आवश्यक गणित,
• भास्कर-द्वितीय मध्यकालीन समय के एक और
• सनकी और महाकाव्यों का उपयोग करके ग्रहों के महत्वपूर्ण गणितज्ञ थे। सिद्धांत सिरोमणि उनके महत्वपूर्ण
देशांतर की गणना के लिए समय और नियमों का कार्यों में से एक था, जो चार भागों से मिलकर बनता है:
विभाजन,
• लीलावती- अंकगणित और ज्यामिति से संबंधित।
• शस्त्रागार क्षेत्र, त्रिकोणमिति की समस्याओ ं से संबंधित
• बीजगणित- बीजगणित पर एक ग्रंथ।
नियम और ग्रहणों का अध्ययन।
• ग्रहगणिता-खगोल विज्ञान से संबंधित।

• गोलाध्याय-खगोल विज्ञान से संबंधित।


वराहमिहिर (500 ई.) (Varahamihira)

• वराहमिहिर की सबसे प्रसिद्ध कृति पंचसिद्धांतिका है,


जिसे खगोल विज्ञान पर एक पूर्ण ग्रंथ माना जाता है और उल्लेखनीय योगदान
यह पहले के पांच खगोलीय ग्रंथों, अर्थात् सूर्य, रोमक,
पौलिसा, वशिष्ठ और पैतामह सिद्धांत का सारांश प्रस्तुत
(Noteworthy
करता है। Contributions):
• उन्होंने रोमक सिद्धांत का परिचय दिया जो चंद्र कैलें डर
का आधार है। • निम्नलिखित को विज्ञान और प्रौद्योगिकी के क्षेत्र में
प्राचीन भारत का महत्वपूर्ण योगदान माना जा सकता है:

1. दशमलव प्रणाली (The Decimal System)


ब्रह्मगुप्त (598-668 ई.) (Brahmagupta)
• दशमलव प्रणाली में, प्रत्येक प्रतीक को स्थिति के मूल्य
• ब्रह्मगुप्त एक खगोलशास्त्री और गणितज्ञ थे। उन्होंने 628 के साथ-साथ एक निरपेक्ष मान भी प्राप्त होता है।
ई. में ब्रह्मस्फुटसिद्धांत में गणित और खगोल विज्ञान पर
• प्रणाली की सरलता के कारण, इसने अपने अनुप्रयोगों
एक महत्वपूर्ण ग्रंथ लिखा। उन्होंने खगोलीय समस्याओ ं
को व्यापक और अपनाने में आसान बना दिया।
के लिए बीजगणित को लागू करना शुरू किया।

• उन्होंने सामान्य रैखिक समीकरण के समाधान दिए,


सामान्य द्विघात समीकरण के दो समकक्ष समाधान,
उन्होंने समझाया कि एक पूर्णांक के घन और घनमूल
को कैसे खोजना है, वर्गों और वर्गमूलों की गणना को
सुविधाजनक बनाने के नियम, और पांच प्रकार के अंशों
के संयोजन नियमों से निपटने के नियम दिए।

• उन्होंने इस विचार का खंडन किया कि चंद्रमा सूर्य


चित्र 1.1 : फाइबोनैचि संख्या
की तुलना में पृथ्वी से अधिक दूर है, जिसे उन्होंने सूर्य
द्वारा चंद्रमा को आलोकित करने के बारे में बताते हुए
समझाया।
2. फाइबोनैचि संख्या (Fibonacci Numbers)

• फाइबोनैचि संख्याएं और उनका क्रम सबसे पहले


भास्कर प्रथम (Bhaskara-I) भारतीय गणित में मातृमेरु के रूप में प्रकट होता है।
पिं गला ने इसका उल्ले ख किया।
• उनकी टिप्पणी, आर्यभटीयभाष्य (629 सीई में लिखी
• आज आधुनिक कंप्यूटर विज्ञान में विभिन्न आईटी
गई), गणित और खगोल विज्ञान पर संस्कृत में सबसे
भाषाओ ं, कोडिं ग आदि में उनका महत्व है।
पुराना ज्ञात गद्य कार्य है।

• वह संभवत: शून्य वृत्त के साथ हिं दू दशमलव प्रणाली में


संख्याएं लिखने वाले पहले व्यक्ति थे। उन्होंने अपनी एक

AKASH SINGH
3. बाइनरी नंबर (Binary Numbers)
मध्यकाल (Medieval
• बाइनरी नंबर सिस्टम का वर्णन सबसे पहले पिं गला ने
अपनी पुस्तक में किया था।
Period)
• बाइनरी नंबर (0 और 1) प्राथमिक भाषा है जिसमें कंप्यूटर • इस अवधि में भारत, तुर्की, अरब दुनिया और मुगलों (मध्य
प्रोग्राम बनते हैं। संयोजन 0 और 1 को बिट्स और बाइट्स एशिया) के बीच बातचीत देखी गई। उनके विकास ने
कहा जाता है। इसने फिबोनाची संख्याओ ं के साथ आईटी विभिन्न क्षेत्रों में प्रगति की।
क्षेत्र के विकास में मदद की है।

मध्यकालीन युग में विज्ञान (Sciences in


4. सूर्य केन्द्रित सिद्धांत (The Heliocentric Medieval Age)
Theory)
• लगातार युद्धों और संघर्षों के कारण, विज्ञान और
• आर्यभट्ट और उनका काम आर्यभटीय उस समय प्रौद्योगिकी के विकास को बहुत नुकसान हुआ। फिर भी,
खगोलीय ज्ञान के शिखर का प्रतिनिधित्व करते थे। इस अवधि के दौरान विभिन्न क्षेत्रों में कई विकास हुए।
उन्होंने प्रतिपादित किया कि पृथ्वी गोल है, अपनी धुरी
पर घूमती है, और सूर्य के चारों ओर घूमती है, अर्थात
सूर्यकेन्द्रित सिद्धांत। इस घटना के माध्यम से ऋतुओ ं में गणित (Mathematics):
परिवर्तन को समझाया गया था।
• नारायण पंडित, गणित में अपने कार्यों के लिए प्रसिद्ध
थे। उनकी दो महत्वपूर्ण रचनाएं गणिताकुमुदी और
बीजगणितवत्सा हैं।
5. वूट्ज़ स्टील (Wootz Steel)
• नीलकंठ सोमसुतवन ने तंत्रसंग्रह का निर्माण किया,
• चेर वंश के तमिलों ने इसका निर्माण किया। चेर राजवंश जिसमें त्रिकोणमिति से जुड़ी विभिन्न समस्याओ ं को हल
के तमिलों के पास दुनिया का सबसे बेहतरीन स्टील करने के लिए नियम शामिल थे।
था। इसे चारकोल भट्टी के अंदर रखे सीलबंद मिट्टी के
• अकबर ने शिक्षा प्रणाली में अन्य विषयों के साथ-साथ
क्रूसिबल में कार्बन में ब्लै क मैग्नेटाइट अयस्क को
गणित को अध्ययन का विषय बनाने का भी आदेश दिया।
पिघलाकर बनाया जाता था।

जीव विज्ञान (Biology):


6. प्लास्टिक सर्जरी और मोतियाबिं द सर्जरी
(Plastic Surgery and Cataract • हाथी और घोड़ों जैसे घरेलू पशुओ ं की अच्छी नस्लों के
Surgery) उत्पादन में अकबर की विशेष रुचि थी। जहाँगीर ने अपनी
कृति तुजुक-ए-जहाँगीरी में प्रजनन और संकरण पर
• सुश्रुत संहिता में विभिन्न बीमारियों, पौधों, तैयारी, इलाज
अपनी टिप्पणियों और प्रयोगों को दर्ज किया।
और प्लास्टिक सर्जरी तकनीकों का उल्ले ख है। सुश्रुत
संहिता नाक के पुनर्निर्माण के तरीकों का वर्णन करती
है, जिसे राइनोप्लास्टी भी कहा जाता है। मोतियाबिं द के
रसायन शास्त्र (Chemistry):
पीछे के सटीक कारणों पर चर्चा की गई और सर्जरी के
माध्यम से इसका इलाज खोजा गया। • कश्मीर, सियालकोट, मुजफ्फराबाद, पटना, मुर्शि दाबाद,
औरंगाबाद, अहमदाबाद और मैसूर मध्यकालीन समय में
कागज उत्पादन के प्रसिद्ध केंद्र थे। यह तकनीक चीन से
7. आयुर्वेद (Ayurveda) अपनाई गई थी।

• शुक्राचार्य द्वारा रचित सुक्रान्ति में वर्णन है कि विभिन्न


• इसे इं डियन स्कूल ऑफ मेडिसिन माना जाता है। भारतीय
प्रकार की तोपों में उपयोग के लिए विभिन्न अनुपातों में
चिकित्सा के जनक के रूप में, चरक पहले चिकित्सक
साल्टपीटर, सल्फर और चारकोल का उपयोग करके
थे जिन्होंने पाचन, चयापचय और प्रतिरक्षा की अवधारणा
बारूद कैसे तैयार किया जा सकता है। आतिशबाजी के
प्रस्तुत की थी।
मुख्य प्रकार, जो हवा में चलते हैं, चिं गारी पैदा करते हैं,
विभिन्न रंगों से धधकते हैं, और विस्फोट के साथ समाप्त
होते हैं, शामिल हैं।

AKASH SINGH
• अकबर के काम आइन-ए-अकबरी में इत्र के नियमन स्थापित बॉम्बे एस्ट्रोफिजिकल ऑब्जर्वेटरी, विज्ञान के
की बात की गई है। गुलाब का अत्तर (इत्र) एक बहुत साथ भारतीय परिचितता को बढ़ाने का एक शानदार
लोकप्रिय इत्र था, जिसके बारे में माना जाता है कि इसकी उदाहरण था।
खोज नूरजहाँ ने की थी।
• भारतीय विज्ञान संस्थान, बंगलौर: जमशेदजी
नसरवानजी टाटा (1839-1904) ने तकनीकी शिक्षा के
लिए भारतीय पहल की। आज तक, यह एक महत्वपूर्ण
खगोल विज्ञान (Astronomy):
संस्थान है जो कार्यरत है।
• जयपुर के महाराजा सवाई जय सिं ह-द्वितीय (एक राजपूत • जमशेदपुर में टाटा स्टील मिल: यह टाटा समूह द्वारा
शासक) खगोल विज्ञान के संरक्षक थे। उन्होंने दिल्ली, स्थापित किया गया था और विज्ञान और प्रौद्योगिकी के
वाराणसी, उज्जैन, मथुरा और जयपुर में पांच खगोलीय विभिन्न अनुप्रयोगों में अनुसंधान और विकास यहां किये
वेधशालाओ ं की स्थापना की। उन्होंने विभिन्न नगरों की गये।
योजना भी बनाई और उनका निर्माण किया।

दवा (Medicine): विज्ञान और प्रौद्योगिकी में


• तेरहवीं शताब्दी में रचित सारंगधारा संहिता में नैदानिक भारत की प्रसिद्ध हस्तियां
प्रयोजनों के लिए इसकी भौतिक औषधि और मूत्र परीक्षण
में अफीम के उपयोग का वर्णन किया गया है। उल्लिखित
(Famous Personalities
दवाओ ं में रस चिकित्सा प्रणाली की धातु तैयारी और यहां of India In Science and
तक कि आयातित दवाएं भी शामिल हैं।

• रस चिकित्सा प्रणाली, मुख्य रूप से मर्क्यूरियल और गैर-


Technology)
मर्क्यूरियल दोनों तरह की खनिज दवाओ ं के साथ काम जगदीश चंद्र बोस (Jagadish Chandra Bose)
करती है। (1858-1937)

इस अवधि में आयुर्वेद पर कुछ महत्वपूर्ण ग्रंथ संकलित • वह एक जीवविज्ञानी, भौतिक विज्ञानी, वनस्पतिशास्त्री
किए गए थे जिनमें वनगसेन द्वारा सारंगधारा संहिता और और विज्ञान कथा के शुरुआती ले खक थे। उन्हें बंगाली
चिकित्सा संग्रह, भवमिस्र के यागरतबज्र और भवप्रकाश विज्ञान कथा का जनक माना जाता है।
शामिल थे।

सिद्ध:

सिद्धों ने अपने आध्यात्मिक और योगिक अभ्यासों द्वारा


वैथियाम (दवा), जोथिदम (ज्योतिष), मनिथिरिगम (तांत्रिक
अभ्यास), वाथम (कीमिया), योगम (ध्यान और योग
अभ्यास) और ज्ञानम (सर्वशक्तिमान के बारे में ज्ञान) में बड़े
पैमाने पर ज्ञान और अनुभव प्राप्त किया। तमिल परंपरा के
अनुसार 18 सिद्धों को सिद्ध चिकित्सा का स्तंभ माना जाता
है।

आधुनिक भारत में विज्ञान और प्रौद्योगिकी


(Science and Technology in Modern
India)

• इस खंड में, हम कुछ महत्वपूर्ण वैज्ञानिकों द्वारा निभाई गई


भूमिका का पता लगाएं गे जिनके योगदान केवल भारत
तक ही सीमित नहीं थे बल्कि पूरे विश्व में पहचाने गए थे।

संस्थान (Institutions):

• कावसजी दादाभाई नेगमवाला (1857 -1938) द्वारा चित्र 1.2 : जगदीश चंद्र बोस

AKASH SINGH
बोस की उल्ले खनीय उपलब्धियां (Notable • रे दूरदर्शी थे और उनका मानना था कि औद्योगीकरण
Achievements of Bose): भारत की प्रगति की दिशा में एक कदम है।

• हम सभी ने सुना होगा कि हमारी तरह पौधों में भी भावनाएँ


होती हैं। बोस ने ही इसकी खोज की थी। उन्हें क्रे स्कोग्राफ
पी.सी. रे की उल्ले खनीय उपलब्धियां (Notable
के आविष्कार का श्रेय दिया जाता है, जो पौधों के विकास
Achievements of PC Ray)
को मापने के लिए एक उपकरण है।
• उन्होंने अपने जीवनकाल में लगभग 150 शोध पत्र
• वायरले स टे लीग्राफी जिसने आधुनिक संचार प्रणालियों
प्रकाशित किए। विज्ञान पर उनके कई ले ख अपने समय
के विकास का रास्ता दिखाया, का श्रेय अक्सर बोस को
की प्रसिद्ध पत्रिकाओ ं में प्रकाशित हुए थे। उनके प्रकाशनों
दिया जाता है, जिन्होंने पेटेंट दाखिल करने से इनकार
में मर्क्यूरस नाइट्राइट पर शोध शामिल था, और इसके
कर दिया था। वर्षों बाद, इसे एक इतालवी भौतिक विज्ञानी
यौगिक ने उन्हें दुनिया भर से पहचान दिलाई।
ने हड़प लिया। उन्होंने रेडियो तरंगों पर भी शोध किया।
उन्होंने कोहेरर नामक एक उपकरण में भी सुधार किया। • एक शिक्षक के रूप में - उन्होंने भारत में युवा रसायनज्ञों
इसका उपयोग रेडियो तरंगों का पता लगाने के लिए की एक पीढ़ी को प्रेरित किया और इस तरह एक भारतीय
किया जाता था। रसायन शास्त्र स्कूल बनाने में मदद की। मेघनाद साहा
(भारतीय खगोल वैज्ञानिक) और शांति स्वरूप भटनागर
• बोस ने सूर्य से विद्युत चुम्बकीय विकिरण के अस्तित्व के
जैसे प्रसिद्ध भारतीय वैज्ञानिक उनके छात्रों में से थे।
बारे में बताया, जिसकी पुष्टि बाद में 1944 में हुई।
• उन्होंने बहुत कम संसाधनों के साथ भारत में पहली
• उन्होंने कलकत्ता में बोस संस्थान की स्थापना की जो
रासायनिक फैक्ट्री की स्थापना की। बंगाल केमिकल्स
मुख्य रूप से पौधों के अध्ययन के लिए समर्पि त था। आज,
की स्थापना स्वदेशी आंदोलन के दौरान हुई थी।
संस्थान अन्य क्षेत्रों में भी अनुसंधान करता है।
• 1901 में, उनके अग्रणी प्रयासों के परिणामस्वरूप बंगाल
• उनके सम्मान में चंद्रमा पर एक क्रेटर का नाम उनके
केमिकल एं ड फार्मास्युटिकल वर्क्स लिमिटे ड का गठन
नाम पर रखा गया है।
हुआ।

• रे में वैज्ञानिक और औद्योगिक उद्यमी दोनों प्रकार के गुण


प्रफुल्ल चंद्र रे (Prafulla Chandra Ray) (1861- थे। इसलिए उन्हें भारतीय फार्मा उद्योग का जनक कहा
1944) जा सकता है।

श्रीनिवास रामानुजन (Srinivasa Ramanujan) (1887-


1920)

• उन्होंने अपने आप से गणित पर काम करना शुरू किया,


ज्यामितीय और अंकगणितीय श्रृंखलाओ ं का योग किया
और जी.एस. कैर द्वारा लिखित पुस्तक सिनोप्सिस ऑफ
एलीमेंट्री रिजल्ट्स इन प्योर मैथमेटिक्स से अत्यधिक
प्रभावित हुए।

• एक वर्ष के लिए उन्हें इं डियन मैथमैटिकल सोसाइटी के


संस्थापक सदस्यों में से एक रामचंद्र राव का समर्थन
मिला, जिन्होंने उन्हें प्रति माह 25 रुपये दिए।

चित्र 1.3 : प्रफुल्ल चंद्र रे

8
चित्र 1.4 : श्रीनिवास रामानुजन चित्र 1.5 : सी.वी. रमन

रामानुजन की उल्ले खनीय उपलब्धियां (Notable सर सी.वी. रमन की उल्ले खनीय उपलब्धियां (Notable
Achievements of Ramanujan) Achievements of Sir CV Raman)

• उन्होंने गणितीय विश्ले षण, अनंत श्रृंखला, निरंतर भिन्न, • उन्होंने कंपन, ध्वनि, संगीत वाद्ययंत्र, अल्ट्रासोनिक्स,
संख्या सिद्धांत और खेल सिद्धांत सहित कई गणितीय विवर्तन, फोटोइले क्ट्रिसिटी, एक्स-रे विवर्तन, आदि में
क्षेत्रों में बहुत से योगदान दिए। अनुसंधान के लिए बहुत कार्य किया।
• 1911 में बर्नौली संख्याओ ं पर उनके कार्यों ने उन्हें पहचान • प्रकाश के प्रकीर्णन पर किए गये उनके कार्य ने उन्हें
दिलाई और उन्हें एक गणितीय प्रतिभा के रूप में जाना दुनिया भर में पहचान दिलाई।
गया।
• फरवरी 1928 में, एक विज्ञान सम्मेलन में, उन्होंने अपनी
• रामानुजन ने कई ऐसी गणितीय सवालों के समाधान खोज की घोषणा की जिसे वर्तमान में रमन प्रभाव के रूप
प्रदान किये जिन्हें तब तक अनसुलझा माना जाता था। में जाना जाता है।
• 1914 में, रामानुजन ने (पाई) के मान की गणना के लिए
एक सूत्र खोजा। यह सूत्र वर्तमान में पाई की गणना के रमन प्रभाव एक माध्यम के अणुओ ं द्वारा प्रकाश के प्रकीर्णन
लिए उपयोग किए जाने वाले सबसे तेज़ एल्गोरिदम का की प्रक्रिया है। माध्यम में प्रवेश करने पर प्रकाश की तरंग
आधार है। दैर्ध्य में होने वाले परिवर्तन के कारण प्रकाश का प्रकीर्णन
होता है।

• 1930 में, वे भौतिकी के नोबेल पुरस्कार से सम्मानित


सर चंद्रशेखर वेंकट रमन (Sir Chandrasekhara
होने वाले पहले एशियाई वैज्ञानिक बने। यह पुरस्कार
Venkata Raman) (1888-1970)
प्रकाश के प्रकीर्णन या रमन प्रभाव से संबंधित उनकी
• सी.वी. रमन ने नोबेल पुरस्कार जीता और वे यह प्रतिष्ठित खोज के लिए दिया गया था।
पुरस्कार जीतने वाले कुछ भारतीयों में से एक हैं।
• भारत सरकार ने इस वैज्ञानिक को भारत रत्न से भी
सम्मानित किया है।

9
सत्येंद्र नाथ बोस (Satyendra Nath Bose)
(1894-1974)

• 1958 में उन्हें रॉयल सोसाइटी का फेलो चुना गया। भारत


सरकार ने उन्हें राष्ट्रीय प्रोफेसर नामित किया और उन्हें
पद्म विभूषण के सम्मान से सम्मानित किया।

चित्र 1.7 : एस. एस. भटनागर

एस. एस. भटनागर की उल्ले खनीय उपलब्धियां (Notable


Achievements of S S Bhatnagar)

• उन्हें भारत में 'अनुसंधान प्रयोगशालाओ ं के जनक' के


चित्र 1.6 : एस. एन. बोस रूप में जाना जाता है।

• वे 19 वर्षों से अधिक समय तक रसायन शास्त्र के प्रोफेसर


रहे एवं वैज्ञानिक और औद्योगिक अनुसंधान परिषद
एस. एन. बोस की उल्ले खनीय उपलब्धियां (Notable
Achievements of S N Bose) (CSIR) के पहले महानिदेशक थे। उनके कार्यकाल
में 12 प्रयोगशालाओ ं की स्थापना हुई, और आज CSIR
• वह अपने 1924 के पेपर के लिए सबसे ज्यादा जाने जाते प्रयोगशालाओ ं की कुल संख्या लगभग 40 है।
हैं। उन्होंने क्लासिक इले क्ट्रोडायनामिक्स के संदर्भ
• उन्होंने स्वतंत्रता प्राप्ति के बाद विज्ञान और प्रौद्योगिकी
के बिना ब्लै क बॉडी विकिरण के लिए मैक्स प्लैं क
के बुनियादी ढांचे और नीतियों के निर्माण और होमी
(सैद्धांतिक भौतिक विज्ञानी) नियम का प्रतिपादन किया,
भाभा, पी.सी. महालनोबिस, विक्रम साराभाई, और अन्य
जिसके परिणामस्वरूप क्वांटम यांत्रिकी की बोस-
वैज्ञानिकों को तैयार करने में महत्वपूर्ण भूमिका निभाई।
आइं स्टीन प्रणाली की खोज हुई। इसे पदार्थ की पांचवीं
अवस्था के रूप में भी जाना जाता है। • नव स्वतंत्र भारत में, उन्हें सरकार के शिक्षा सचिव और
शैक्षिक सलाहकार के रूप में नियुक्त किया गया था।
• प्रो. बोस का शोध आइं स्टीन के सापेक्षता के सिद्धांत और
प्लैं क के क्वांटम विकिरण नियम के बारे में था।

• वे एक्स-रे क्रिस्टलोग्राफी पर अपने काम के लिए भी


होमी जहांगीर भाभा (Homi Jehangir Bhabha)
प्रसिद्ध हैं और उन्होंने अपनी एक्स-रे क्रिस्टलोग्राफी (1909-1966)
प्रयोगशाला की स्थापना की थी।
• 1927 में, वे इं जिनियरिं ग की पढ़ाई के लिए कैम्ब्रिज
विश्वविद्यालय, इं ग्लैं ड गए। उन्होंने अपने परिवार की इच्छा
के अनुसार इं जीनियरिं ग की पढ़ाई शुरू की, ले किन वे
शांति स्वरूप भटनागर (Shanti Swarup
Bhatnagar) (1894-1955) जल्दी ही भौतिकी की ओर आकर्षि त हो गए।

• वे भारतीय विज्ञान संस्थान में शामिल होने के लिए


• 1911 में, उन्होंने दबाव और तापमान में शीरा और
द्वितीय विश्व युद्ध से पहले भारत लौट आए। यहां उन्होंने
कार्बनयुक्त पदार्थ का उपयोग करके 'बैटरी में कार्बन
कॉस्मिक रे रिसर्च इं स्टीट्यूट की स्थापना की। 1945 में,
इले क्ट्रोड का विकल्प कैसे बनाया जाए' पर एक पत्र
उन्होंने टाटा इं स्टीट्यूट ऑफ फंडामेंटल रिसर्च (TIFR) की
प्रकाशित किया। 1954 में भारत सरकार ने उन्हें पद्म
स्थापना की, जहां भारत के परमाणु कार्यक्रम के लिए
विभूषण से सम्मानित किया।

10
प्रारंभिक शोध शुरू हुआ। • 1948 में, भाभा को अंतर्राष्ट्रीय नाभिकीय ऊर्जा आयोग
का अध्यक्ष नियुक्त किया गया।

• उन्हें भारत के परमाणु कार्यक्रम के मुख्य वास्तुकार के


रूप में जाना जाता है। 1954 में उन्हें पद्म विभूषण पुरस्कार
से सम्मानित किया गया।

• उन्हें परमाणु विज्ञान में अपने उत्कृष्ट कार्य के लिए


अंतरराष्ट्रीय स्तर पर पहचान मिली। उन्होंने नाभिकीय
ऊर्जा के शांतिपूर्ण उपयोग (1955 में जिनेवा में आयोजित)
पर संयुक्त राष्ट्र के पहले सम्मेलन के अध्यक्ष के रूप में
कार्य किया।

सुब्रमण्यम चंद्रशेखर (Subramaniam


Chandrasekhar) (1910-1995)

चित्र 1.8 : होमी जहांगीर भाभा

• वह एक बहुआयामी व्यक्तित्व थे। उन्हें पद्म भूषण, एडम


अवार्ड, अमेरिकन एकेडमी ऑफ आर्ट्स एं ड साइं सेज
के मानद फेलो और संयुक्त राज्य अमेरिका में नेशनल
एकेडमी ऑफ साइं सेज के विदेशी सहयोगी सहित कई
पुरस्कारों से सम्मानित किया गया।

कॉस्मिक किरणें तेज गति से चलने वाले , बाहरी अंतरिक्ष से


आने वाले अत्यंत छोटे कण हैं। जब ये कण पृथ्वी के वायुमं-
डल में प्रवेश करते हैं, तो वे हवा के परमाणुओ ं से
टकराते हैं और इले क्ट्रॉनों की बौछार पैदा करते हैं।
चित्र 1.9 : सुब्रमण्यम चंद्रशेखर

होमी जहांगीर भाभा की उल्ले खनीय उपलब्धियां (Notable


Achievements of Homi Jehangir Bhabha): एस. चंद्रशेखर की उल्ले खनीय उपलब्धियां (Notable
Achievements of S Chandrasekhar):
• 1937: जर्मन भौतिक विज्ञानी डब्ल्यू. हेटलर के साथ भाभा
ने कॉस्मिक किरणों के बारे में पहेली को सुलझाया। इन • उनका पहला वैज्ञानिक पत्र, 'कॉम्पटन स्कैटरिं ग एं ड द
इले क्ट्रानों की बौछारों में परमाणु कणों की उपस्थिति न्यू स्टैटिस्टिक्स' 1928 में रॉयल सोसाइटी प्रोसीडिं ग्स में
की उनकी खोज, जिसे उन्होंने मेसन कहा, का उपयोग प्रकाशित हुआ था। इस पत्र के आधार पर उन्हें कैम्ब्रिज
आइं स्टीन के सापेक्षता के सिद्धांत को जांचने के लिए विश्वविद्यालय में आर.एच. फाउलर द्वारा एक शोध छात्र के
किया गया था। रूप में स्वीकार किया गया।

• भाभा ने जेआरडी टाटा से धन की मांग की और मौलिक • उन्होंने सफेद बौने तारों का सिद्धांत प्रतिपादित किया।
अनुसंधान के लिए पूरी तरह समर्पि त संस्थान की इस सिद्धांत के अनुसार, एक तारा जिसका द्रव्यमान सूर्य
आवश्यकता को महसूस किया। इससे 1945 में मुंबई के द्रव्यमान के 1.45 गुना से अधिक है, वह सफेद बौना
में टाटा इं स्टीट्यूट ऑफ फंडामेंटल रिसर्च (TIFR) की तारा नहीं बन सकता है। इस सीमा को अब 'चंद्रशेखर
स्थापना हुई और भाभा इसके निदेशक बने। लिमिट' के नाम से जाना जाता है।

11
• तारों की संरचना और उनके विकास के लिए महत्वपूर्ण • 1975-76 में शुरू किए गए सैटेलाइट इं स्ट्रक्शनल
भौतिक प्रक्रियाओ ं पर उनके काम के लिए उन्हें 1983 में टे लीविज़न एक्सपेरिमेंट (SITE) ने भारत के 2400
भौतिकी के नोबेल पुरस्कार से सम्मानित किया गया। गांवों में 50 लाख लोगों को शिक्षा दी। उन्होंने बच्चों के
बीच विज्ञान को लोकप्रिय बनाने के उद्देश्य से 1965 में
अहमदाबाद में सामुदायिक विज्ञान केंद्र की स्थापना की।
विक्रम साराभाई (Vikram Sarabhai) (1919- • अंतर्राष्ट्रीय खगोलीय संघ ने विज्ञान में उनके योगदान के
1971) सम्मान में चंद्रमा पर (शांति के सागर में) एक क्रेटर का
नाम उनके नाम पर रखा।

सी. आर. राव (C. R. Rao)

• 1941 में वे कलकत्ता विश्वविद्यालय के एम.ए. सांख्यिकी


पाठ्यक्रम में प्रवेश लिया। 1943 में एम.ए. की परीक्षा
उत्तीर्ण करने पर, उन्होंने विश्वविद्यालय का स्वर्ण पदक
जीता और पहले ही उनके कुछ शोध पत्र प्रकाशित हो
चुके थे। 1943 में उन्होंने एक तकनीकी प्रशिक्षु के रूप
में आईएसआई (भारतीय सांख्यिकी संस्थान) में प्रवेश
लिया, संस्थान के प्रशिक्षण अनुभाग में शोध, शिक्षण और
कलकत्ता विश्वविद्यालय में, सांख्य, इं डियन जर्नल ऑफ
स्टैटिस्टिक्स के संपादन में प्रोफेसर महला नोबिस की
सहायता की।

चित्र 1.10 : विक्रम साराभाई

विक्रम साराभाई के उल्ले खनीय योगदान (Notable


Contributions of Vikram Sarabhai):

• उन्होंने 1948 में एम.जी. विज्ञान संस्थान, अहमदाबाद


के कुछ कमरों में भौतिक अनुसंधान प्रयोगशाला की
स्थापना की। प्रोफेसर के.के. रामनाथन इसके निदेशक
बने।

• अप्रैल 1954 में, पीआरएल एक नए भवन में चला गया


और डॉ. साराभाई ने इसे भारतीय अंतरिक्ष कार्यक्रम का
उद्गम स्थल बनाया।

• 28 वर्ष की छोटी उम्र में, उन्होंने अहमदाबाद टे क्सटाइल


इं डस्ट्रीज़ रिसर्च एसोसिएशन का गठन किया, और 1949- चित्र 1.11 : सी. आर. राव
1956 तक इसके मानद निदेशक रहे।

• उन्होंने 1962-1965 तक भारतीय प्रबंधन संस्थान,


सी. आर. राव के उल्ले खनीय योगदान (Notable
अहमदाबाद के निर्माण और निर्देशन में भी मदद की।
Contributions by CR Rao)
• साराभाई ने भारतीय अंतरिक्ष अनुसंधान संगठन का
• राव ने कुछ काम पहले ही पूरा कर लिया था जिसमें
विस्तार करके भारत के अंतरिक्ष युग की शुरुआत की।
उनका नाम है: क्रै मर-राव असमानता, राव-ब्लै कवेल
इसकी कई परियोजनाओ ं में से एक, भारत का पहला
प्रमेय, राव का स्कोर परीक्षण, और राव के ऑर्थोगोनल
उपग्रह- आर्यभट्ट 1975 में लॉन्च किया गया, भाभा की
सरणी।
तरह, साराभाई चाहते थे कि विज्ञान का व्यावहारिक
अनुप्रयोग आम आदमी तक पहुंचे। • वे 1948 में आईएसआई में लौट आए और 1949 में उन्हें 29

12
वर्ष की छोटी उम्र में ही प्रोफेसर बना दिया गया। उन्होंने डॉ. वेंकटरमन रामकृष्णन (Dr Venkataraman
आईएसआई के अनुसंधान और प्रशिक्षण अनुभाग का Ramakrishnan)
नेतृत्व और विकास किया और आईएसआई के निदेशक
बने।

हरगोबिं द खुराना (Hargobind Khorana)

चित्र 1.13 : डॉ. वेंकटरमन रामकृष्णन

• राइबोसोम की संरचना और कार्यों का सफलतापूर्वक


पता लगाने के लिए उन्हें 2009 में रसायन विज्ञान का
चित्र 1.12 : हरगोबिं द खुराना नोबेल पुरस्कार मिला।

एक जीव के महत्वपूर्ण कार्यों को कोशिकाओ ं राइबोसोम


एच. जी. खुराना के उल्ले खनीय योगदान (Notable में उत्पादित बड़े जटिल प्रोटीन अणुओ ं द्वारा प्रबंधित किया
Contributions by HG Khorana):
जाता है।
• डॉ. हर गोबिं द खुराना ने जेनेटिक कोड को क्रै क करने
• अन्य अनुप्रयोगों में, यह एं टीबायोटिक दवाओ ं के उत्पादन
के लिए मार्शल निरेनबर्ग और रॉबर्ट होली के साथ 1968
में महत्वपूर्ण रहा है।
में मेडिसिन और फिजियोलॉजी के लिए नोबेल पुरस्कार
साझा किया। उन्होंने इसका कोड स्थापित किया, सभी
जीवित जीवों के लिए सामान्य प्राकृतिक भाषा, तीन
अक्षरों वाले शब्दों में लिखी गई है: तीन न्यूक्लियोटाइड
कोड का प्रत्येक सेट एक विशिष्ट अमीनो एसिड के लिए
है।

13
अध्याय - 2

सूचना प्रौद्योगिकी
(INFORMATION TECHNOLOGY)

• सूचना प्रौद्योगिकी (IT), सूचनाओ ं को संग्रहीत करने, • डिजिटल हस्ताक्षर के लिए कानूनी मान्यता।
पुनः प्राप्त करने और साझा करने की प्रणालियों का
• इले क्ट्रॉनिक रिकॉर्ड और डिजिटल हस्ताक्षर के लिए
अध्ययन और उपयोग है। इसमें सॉफ्टवेयर, हार्डवेयर
सुरक्षा उपाय मौजूद हैं।
और सॉफ्टवेयर और हार्डवेयर का उपयोग करने वाले
विभिन्न एप्लिकेशन शामिल हैं। सूचना प्रौद्योगिकी की • अधिनियम के तहत जांच करने के लिए न्यायनिर्णायक
व्यवसाय, शिक्षा, शासन और यहां तक कि स्वास्थ्य सेवा अधिकारियों की नियुक्ति के लिए प्रक्रिया निर्धारित
में व्यापक उपयोगिता है। करता है।

• यह क्षेत्र भारत की अर्थव्यवस्था और रोजगार के लिए • अधिनियम के तहत साइबर अपीलीय न्यायाधिकरण
अविश्वसनीय रूप से फायदेमंद रहा है। नेसकॉम के स्थापित करने का प्रावधान। यह न्यायाधिकरण, नियंत्रक
अनुसार, आईटी क्षेत्र ने 2019 में 180 बिलियन अमेरिकी या न्यायनिर्णायक अधिकारी के आदेश के खिलाफ की
डॉलर का कुल राजस्व पैदा किया, जिसमें निर्यात राजस्व गई सभी अपीलों की सुनवाई करेगा।
99 बिलियन अमेरिकी डॉलर और घरेलू राजस्व 48 • साइबर अपीलीय न्यायाधिकरण के आदेश के खिलाफ
बिलियन अमेरिकी डॉलर था, जो 13% से अधिक बढ़ गया। अपील केवल उच्च न्यायालय में ही दर्ज की जा सकती है।
2020 तक, भारत के आईटी कार्यबल में 4.36 मिलियन
• प्रमाणन प्राधिकारियों के कार्य को नियंत्रित करने और
कर्मचारी हैं।
लाइसेंस देने के लिए नियंत्रक प्राधिकारी (CCA) की
नियुक्ति का प्रावधान। नियंत्रक सभी डिजिटल हस्ताक्षरों
के भंडार के रूप में कार्य करता है।

आईटी अधिनियम, 2000 (IT • अधिनियम भारत के बाहर किए गए अपराधों या


उल्लं घनों पर भी लागू होता है।
Act, 2000):
• इस अधिनियम के तहत उल्लिखित अपराधों पर वरिष्ठ
• 1996 में, अंतरराष्ट्रीय व्यापार कानून पर संयुक्त राष्ट्र पुलिस अधिकारी और अन्य अधिकारी किसी भी
आयोग (UNCITRAL) ने विभिन्न देशों में कानून सार्वजनिक स्थान में प्रवेश कर सकते हैं और वारंट के
में एकरूपता लाने के लिए इले क्ट्रॉनिक कामर्स बिना तलाशी और गिरफ्तारी कर सकते हैं।
(ई-कॉमर्स) पर मॉडल कानून अपनाया। सूचना • केंद्र सरकार और नियंत्रक को सलाह देने के लिए
प्रौद्योगिकी अधिनियम, 2000 (आईटीए-2000, या साइबर विनियम सलाहकार समिति की स्थापना के लिए
आईटी अधिनियम के रूप में भी जाना जाता है) भारत प्रावधान।
में साइबर अपराध और इले क्ट्रॉनिक कॉमर्स से निपटने
वाला प्राथमिक कानून है।

• सूचना प्रौद्योगिकी अधिनियम, 2000 डेटा के इले क्ट्रॉनिक यह कानून निम्नलिखित मामलों में लागू होता है (The Law
is Applicable in the Following Matters):
आदान-प्रदान और संचार के अन्य इले क्ट्रॉनिक माध्यमों
या इले क्ट्रॉनिक कॉमर्स ले नदेन के माध्यम से किए गए • यह अधिनियम पूरे देश में प्रभावी है, जिसमें जम्मू और
ले नदेन को कानूनी मान्यता प्रदान करता है। कश्मीर भी शामिल है। जम्मू और कश्मीर को शामिल
• उद्देश्य: इले क्ट्रॉनिक डेटा इं टरचेंज के माध्यम से किए गए करने के लिए, अधिनियम संविधान के अनुच्छेद 253 का
ले नदेन के लिए कानूनी मान्यता प्रदान करना। उपयोग करता है।

• धारा 1 (2), धारा 75 के साथ, यह उल्ले ख करती है कि


अधिनियम के प्रावधान भारत के बाहर किए गए किसी
सूचना प्रौद्योगिकी अधिनियम, 2000 की विशेषताएं भी अपराध पर लागू होते हैं।
(Features of the Information Technology • मान लीजिए कि अपराध करने वाले व्यक्ति ने भारत
Act, 2000):
में स्थित किसी कंप्यूटर या कम्प्यूटरीकृत प्रणाली या
• सुरक्षित इले क्ट्रॉनिक चैनलों के माध्यम से किए गए नेटवर्क का प्रयोग किया है, फिर चाहे उसकी राष्ट्रीयता
सभी इले क्ट्रॉनिक अनुबंध कानूनी रूप से मान्य हैं। कुछ भी हो। इस मामले में, व्यक्ति अधिनियम के तहत

14

AKASH SINGH
दंडनीय है।
राष्ट्रीय ज्ञान नेटवर्क
(National Knowledge
धारा 66 ए और अभिव्यक्ति की स्वतंत्रता पर प्रतिबंध
(Section 66A and Restrictions on Freedom Network)
of Speech):
• राष्ट्रीय ज्ञान नेटवर्क (NKN) का उद्देश्य ज्ञान साझा करने
• 2008 में मूल अधिनियम में संशोधन के रूप में इसको और सहयोगी अनुसंधान की सुविधा के लिए उच्च गति
शामिल किए जाने के बाद, धारा 66ए अपनी असंवैधानिक डेटा संचार नेटवर्क के साथ उच्च शिक्षा और अनुसंधान के
प्रकृति के कारण विवादों में रही है। सभी संस्थानों को आपस में जोड़ना है।

• वैश्विक स्तर पर, अग्रणी अनुसंधान और नवाचार


बहु-विषयक और सहयोगात्मक दृष्टिकोणों की ओर
धारा 66ए (Section 66A):
स्थानांतरित होते हैं जिनके लिए कुशल संचार और
• कोई भी व्यक्ति जो कंप्यूटर संसाधन के किसी भी माध्यम कम्प्यूटे शनल शक्ति की आवश्यकता होती है।
से ऐसी कोई भी जानकारी भेजता है जो घोर आपत्तिजनक • भारत में, NKN का लक्ष्य, अपनी बहु-गीगाबिट क्षमता के
हो या खतरनाक चरित्र वाली हो; या कोई भी जानकारी साथ, ऐसे प्रतिमान बदलाव को संबोधित करने के लिए
जिसे वह झूठा जानता है, ले किन झुंझलाहट, असुविधा, सभी विश्वविद्यालयों, अनुसंधान संस्थानों, पुस्तकालयों,
खतरे, बाधा, अपमान का कारण बनता है; प्रयोगशालाओ ं, स्वास्थ्य देखभाल और कृषि संस्थानों को
जोड़ना है।
» कारावास से, जिसकी अवधि तीन वर्ष तक की हो सकती
है और जुर्माने से दंडनीय होगा। • परमाणु, अंतरिक्ष और रक्षा अनुसंधान में प्रमुख मिशन-
उन्मुख एजेंसियां भी NKN का हिस्सा हैं।

• सूचना और ज्ञान के प्रवाह को सुगम बनाकर, नेटवर्क


श्रेया सिं घल केस (Shreya Singhal Case): पहुंच के महत्वपूर्ण मुद्दे को हल करता है। यह देश में
अनुसंधान के प्रयासों को समृद्ध करने के लिए सहयोग
• दिल्ली निवासी कानून की छात्रा श्रेया सिं घल ने भारत के
का एक नया प्रतिमान बनाता है।
सर्वोच्च न्यायालय में एक जनहित याचिका (PIL) दायर
की और तर्क दिया कि धारा 66ए, संविधान के अनुच्छेद • नेटवर्क डिजाइन एक सक्रिय दृष्टिकोण पर आधारित है।
14, 19 (1) (ए), और अनुच्छेद 21 का उल्लं घन करती है। यह उपयोग और संभावित लाभों दोनों के संदर्भ में भविष्य
की आवश्यकताओ ं और नई संभावनाओ ं पर विचार
• 2015 में, सुप्रीम कोर्ट ने कहा कि धारा 66ए असंवैधानिक
करता है।
है। अदालत ने कहा कि आईटी अधिनियम 2000 की
धारा 66ए भारत के संविधान के अनुच्छेद 19 (1) के तहत • यह ‍एक ज्ञान क्रांति लाएगा और समाज को बदलने और
प्रदान किए गए अभिव्यक्ति की आजादी के अधिकार समावेशी विकास को बढ़ावा देने में सहायक होगा।
का "मनमाने ढं ग से, अत्यधिक और असमान रूप से
अतिक्रमण करती है"। हालाँकि, न्यायालय ने अधिनियम
की धारा 69ए और 79 को रद्द करने की याचिका को एनकेएन की स्थापना निम्नलिखित विशेषताओ ं को ध्यान
ठु करा दिया, जो इस तरह की वेबसाइटों को अवरुद्ध में रखते हुए की गई है (NKN has been Established
Keeping the Following Features in Mind):
करने की प्रक्रिया और सुरक्षा उपायों से संबंधित है।

• आईटी अधिनियम, 2000 की धारा 69ए केंद्र को एक • एक उच्च गति नेटवर्क कनेक्टिविटी स्थापित करना जो
मध्यस्थ के लिए सार्वजनिक पहुंच को अवरुद्ध करने की ज्ञान और सूचना साझा करने में सक्षम होगा।
अनुमति देती है। एक मध्यस्थ में "दूरसंचार सेवा प्रदाता, • सहयोगी अनुसंधान, विकास और नवाचार को सक्षम
नेटवर्क सेवा प्रदाता, इं टरनेट सेवा प्रदाता, वेब-होस्टिं ग करना।
सेवा प्रदाता, खोज इं जन, ऑनलाइन भुगतान साइट
• इं जीनियरिं ग, विज्ञान, चिकित्सा, अंतरिक्ष, कानून आदि
आदि" शामिल हैं।
जैसे विशिष्ट क्षेत्रों में उन्नत दूरस्थ शिक्षा की सुविधा
प्रदान करना।

• ई-गवर्नेंस के लिए अल्ट्रा-हाई-स्पीड बैकबोन की सुविधा।

• अनुसंधान, शिक्षा, स्वास्थ्य, वाणिज्य और शासन में


विभिन्न क्षेत्रीय नेटवर्क के एकीकरण को सुगम बनाना।

15
• विश्व भर में अनुसंधान समुदायों के साथ सहयोग करने उपाय।
के लिए वैश्विक नेटवर्क से जोड़ना।
• साइबर घटना पर प्रतिक्रियाओ ं का समन्वय।

• सूचना सुरक्षा प्रथाओ ं, प्रक्रियाओ ं, रोकथाम, प्रतिक्रिया


इं टरनेट क्षेत्र में विभिन्न हितधारक (Different और साइबर घटनाओ ं की रिपोर्टिंग से संबंधित दिशानिर्देश,
Stakeholders in the Internet Space) सलाह, भेद्यता नोट और श्वेत पत्र जारी करना।

• ये जानना महत्वपूर्ण है कि इं टरनेट क्षेत्र में चार अलग- • साइबर सुरक्षा से संबंधित ऐसे अन्य कार्य जो निर्धारित
अलग प्रकार के हितधारक हैं: किए जा सकते हैं।

i. किसी भी इं टरनेट सेवा के उपभोक्ता,

ii. दूरसंचार सेवा प्रदाता (TSP) या इं टरनेट सेवा प्रदाता


(ISP), ऑप्टिकल संचार (Optical
iii. ओवर-द-टॉप (OTT) सेवा प्रदाता (वे जो वेबसाइट Communication)
और एप्लिकेशन जैसी इं टरनेट एक्सेस सेवाएं प्रदान
करते हैं), और • ऑप्टिकल संचार वह संचार है जिसमें सिग्नल को सुदर

छोर तक ले जाने के लिए विद्युत धारा के स्थान पर प्रकाश
iv. सरकार, जो इन हितधारियों के बीच संबंधों को
का उपयोग किया जाता है। ऑप्टिकल संचार अपने गंतव्य
विनियमित और परिभाषित करती है। ट्राई दूरसंचार
तक सिग्नल ले जाने के लिए ऑप्टिकल फाइबर पर
क्षेत्र में एक स्वतंत्र नियामक है, जो मुख्य रूप से TSP
निर्भर करता है। विद्युत संचरण पर इसके कई लाभों के
और उनकी लाइसेंसिंग शर्तों को नियंत्रित करता है।
कारण, विकसित दुनिया के मुख्य नेटवर्क में बड़े पैमाने
पर कॉपर संचार की जगह ऑप्टिकल फाइबर का प्रयोग
हो रहा है।
भारतीय कंप्यूटर • 1970 के दशक में कम-नुकसान वाले ऑप्टिकल

आपातकालीन प्रतिक्रिया टीम फाइबर केबल के विकास के बाद से, ऑप्टिकल संचार
सबसे लोकप्रिय संचार विधियों में से एक बन गया है।
(The Indian Computer इसके मुख्य लाभों में उच्च बैंडविड्थ, असाधारण रूप से
डेटा या सिग्नल की कम हानि, अधिक ट्रांसमिशन रेंज,
Emergency Response और कोई विद्युत चुम्बकीय हस्तक्षेप न होना, शामिल है।

Team, CERT-In) हालांकि, इसके नुकसान केबल, ट्रांसमीटर/रिसीवर


और अन्य सहायक उपकरण की उच्च लागत और केबल
• भारतीय कंप्यूटर आपातकालीन प्रतिक्रिया टीम (सर्ट- स्थापना और इं टरकनेक्शन के दौरान आवश्यक कौशल
इन), भारत सरकार के इले क्ट्रॉनिक्स और सूचना और विशेषज्ञता हैं।
प्रौद्योगिकी मंत्रालय के अंतर्गत एक कार्यालय है। यह
हैकिंग और फ़िशिं ग जैसे साइबर सुरक्षा खतरों से निपटने
के लिए नोडल एजेंसी है। यह भारतीय इं टरनेट डोमेन की ऑप्टिकल फाइबर केबल के लाभ (Advantages
सुरक्षा से संबंधित तंत्र को मजबूत करता है। मार्च 2014 में, of Optical Fiber Cable)
सर्ट-इन ने एं ड्रॉइड जेली बीन के वीपीएन कार्यान्वयन में
• बैंडविड्थ (Bandwidth): फाइबर ऑप्टिक केबल में
एक बड़ी कमी की सूचना दी।
धातु केबल्स की तुलना में बहुत अधिक बैंडविड्थ होती है।
• सर्ट-इन जनवरी 2004 से चालू है। सर्ट-इन का अधिकार फाइबर के प्रति यूनिट समय में प्रसारित किए जा सकने
क्षेत्र भारतीय साइबर समुदाय है। वाली सूचनाओ ं की मात्रा अन्य ट्रांसमिशन मीडिया पर
इसका सबसे महत्वपूर्ण लाभ है।
सर्ट-इन को साइबर सुरक्षा में निम्नलिखित कार्यों को करने
• बिजली की कम हानि (Low Power Loss): ऑप्टिकल
के लिए राष्ट्रीय एजेंसी के रूप में नामित किया गया है:
फाइबर में बिजली की हानि कम होती है, जो लं बी दूरी तक
• साइबर घटनाओ ं पर सूचना का संग्रह, विश्ले षण और ट्रांसमिशन की अनुमति देता है, कॉपर तार के माध्यम से
प्रसार। ट्रांसमिशन की सबसे लं बी अनुशंसित दूरी 100 मीटर है
जबकि फाइबर के साथ यह 2 किमी है।
• साइबर सुरक्षा घटनाओ ं का पूर्वानुमान और चेतावनी।
• इं टरफरेन्स (Interference): फाइबर ऑप्टिक केबल
• साइबर सुरक्षा घटनाओ ंसे निपटने के लिए आपातकालीन
विद्युत चुम्बकीय इं टरफरेन्स से प्रतिरक्षित हैं। इसे विद्युत

16
शोर वाले वातावरण में भी बिना किसी चिं ता के चलाया बैकअप और उत्तरजीविता को ध्यान में रखना आवश्यक
जा सकता है क्योंकि बिजली का शोर फाइबर को है।
प्रभावित नहीं करेगा।
• मोड़ा नहीं जा सकता (Can't Be Curved): ऑप्टिकल
• आकार (Size): एक फाइबर ऑप्टिक केबल की क्षमता फाइबर पर ट्रांसमिशन के लिए थोड़ी-थोड़ी दूरी के
तांबे के केबल की क्षमता से लगभग 4.5 गुना अधिक अंतराल पर दोहराने की आवश्यकता होती है। केवल कुछ
होती है जबकि क्रॉस सेक्शनल क्षेत्र 30 गुना कम होता है। सेंटीमीटर त्रिज्या का मोड़ होने पर फाइबर टू ट सकते हैं
या ट्रांसमिशन हानि हो सकता है।
• वजन (Weight): फाइबर ऑप्टिक केबल धातु के तारों
की तुलना में बहुत पतले और हल्के होते हैं। वे समान
क्षमता के केबलों की तुलना में कम जगह घेरते हैं। हल्का नेशनल ऑप्टिकल फाइबर नेटवर्क (NOFN):
वजन फाइबर को स्थापित करना आसान बनाता है।
• वर्तमान में सभी राज्यों की राजधानियों, जिला
• सुरक्षा (Security): ऑप्टिकल फाइबर को टै प करना मुख्यालयों और ब्लॉक स्तर तक ऑप्टिकल
मुश्किल होता है। चूंकि वे विद्युत चुम्बकीय ऊर्जा का कनेक्टिविटी उपलब्ध है।
विकिरण नहीं करते हैं, इसलिए उत्सर्जन को बाधित नहीं
• सरकार देश की सभी 2,50,000 ग्राम पंचायतों को
किया जा सकता है। चूंकि फाइबर को भौतिक रूप से टै प
NOFN से जोड़ने की योजना बना रही है। यह सार्वजनिक
करना मुश्किल है, इसलिए संवेदनशील डेटा ले जाने के
क्षेत्र के उपक्रमों (बीएसएनएल, रायटे ल और पावर
लिए फाइबर सबसे सुरक्षित उपलब्ध माध्यम है।
ग्रिड) के मौजूदा फाइबर्स का उपयोग करके और जहां
• लचीलापन (Flexibility): एक ऑप्टिकल फाइबर भी ग्राम पंचायतों को जोड़ने के लिए आवश्यक हो, और
केबल में तांबे या उसी व्यास के स्टील के केबल की अधिक फाइबर बिछाकर किया जाएगा।
तुलना में अधिक तन्यता होती है। यह अधिक लचीला है,
• इस प्रकार सृजित डार्क फाइबर नेटवर्क को उपयुक्त
आसानी से झुकता है और ज्यादातर जंग उत्पन्न करने
तकनीक से रोशन किया जाएगा जिससे ग्राम
वाले तत्वों का प्रतिरोध करता है। जबकि यह कमी कॉपर
पंचायतों में पर्याप्त बैंडविथ उपलब्ध होगी। इसे नेशनल
केबल को कमजोर बनाती है।
ऑप्टिकल फाइबर नेटवर्क (NOFN) कहा जाएगा। इस
• लागत (Cost): तांबे की तुलना में कांच के लिए कच्चा प्रकार, ग्राम पंचायतों और ब्लॉकों के बीच संपर्क की
माल भरपूर मात्रा में उपलब्ध होता है। इसका मतलब खाई को भर दिया जाएगा।
है कि कांच को तांबे की तुलना में सस्ते में बनाया जा
• सभी सेवा प्रदाताओ ं को NOFN तक गैर-भेदभावपूर्ण
सकता है।
पहुंच प्रदान की जाएगी। ये सेवा प्रदाता जैसे दूरसंचार
सेवा प्रदाता (TSP), इं टरनेट सेवा प्रदाता, केबल टीवी
ऑपरेटर और सामग्री प्रदाता ग्रामीण क्षेत्रों में विभिन्न
ऑप्टिकल फाइबर केबल के नुकसान सेवाएं शुरू कर सकते हैं।
(Disadvantages of Optical Fiber Cable)
• इन ऑपरेटरों द्वारा विभिन्न श्रेणियों के आवेदन जैसे
• जोड़ना मुश्किल (Difficult to Splice): ऑप्टिकल ई-स्वास्थ्य, ई-शिक्षा और ई-गवर्नेंस आदि उपलब्ध
फाइबर को जोड़ना मुश्किल होता है, और फाइबर में कराए जा सकते हैं।
प्रकीर्णन के कारण प्रकाश की हानि होती है। फाइबर
• NOFN परियोजना की अनुमानित लागत लगभग
केबलों को सीमित कोण तक ही मोड़ा जा सकता है। अगर
20,000 करोड़ रूपये है। इसे 2 वर्ष में पूरा करने का
हम उन्हें ज्यादा मोड़ेंगे तो वे टू ट जाएं गे।
प्रस्ताव है। इस परियोजना को यूनिवर्सल सर्वि स द्वारा
• स्थापित करने में महंगा (Expensive to Install): वित्त पोषित किया जाएगा।
ऑप्टिकल फाइबर स्थापित करने में अधिक महंगे हैं, और
उन्हें विशेषज्ञों द्वारा स्थापित किया जाना है। वे तारों की
तरह मजबूत नहीं हैं। ऑप्टिकल फाइबर के लिए अक्सर
विशेष परीक्षण उपकरण की आवश्यकता होती है। क्लाउड कंप्यूटिंग (Cloud
• अत्यधिक संवेदनशील (Highly Susceptible): Computing):
फाइबर ऑप्टिक केबल छोटा और कॉम्पैक्ट होता है, और
इसके स्थापना या निर्माण गतिविधियों के दौरान कटने • क्लाउड कंप्यूटिंग तेजी से नवाचार, लचीले संसाधनों
या क्षतिग्रस्त होने की अत्यधिक आशंका होती है। फाइबर और पैमाने की अर्थव्यवस्थाओ ं की पेशकश करने के
ऑप्टिक केबल जबरदस्त डेटा ट्रांसमिशन क्षमता प्रदान लिए इं टरनेट पर सर्वर, स्टोरेज, डेटाबेस, नेटवर्किंग,
करते हैं। इसलिए, जब फाइबर ऑप्टिक केबलिंग को सॉफ्टवेयर, एनालिटिक्स और इं टे लिजेंस सहित
ट्रांसमिशन माध्यम के रूप में चुना जाता है, तो बहाली, कंप्यूटिंग सेवाओ ं ("क्लाउड") की डिलीवरी है। क्लाउड

17
कंप्यूटिंग बिना प्रत्यक्ष उपयोगकर्ता या प्रत्यक्ष सक्रिय को बचाया जा सकता है।
प्रबंधन के कंप्यूटर प्रणाली संसाधनों विशेष रूप से डेटा
• सुरक्षा (Security): कई क्लाउड प्रदाता नीतियों,
स्टोरेज (क्लाउड स्टोरेज) और कंप्यूटिंग पावर की ऑन-
प्रौद्योगिकियों और नियंत्रणों के व्यापक सेट उपलब्ध
डिमांड डिलीवरी है।
कराते हैं। यह आपके डेटा, ऐप्स और बुनियादी ढांचे को
• क्लाउड एक संगठन (एं टरप्राइज क्लाउड) तक सीमित संभावित खतरों से बचाने में मदद करते हुए, आपकी
हो सकते हैं, या कई संगठनों (सार्वजनिक क्लाउड) के समग्र सुरक्षा को मजबूत करता है।
लिए उपलब्ध हो सकते हैं। क्लाउड कंप्यूटिंग सुसंगतता
और किफायत के मामले में संसाधनों के बंटवारे पर
निर्भर करता है। क्लाउड कंप्यूटिंग के उपयोग (Uses of cloud
computing):

• क्लाउड-नेटिव एप्लिकेशन बनाना (Create Cloud-


क्लाउड कंप्यूटिंग के लाभ (Benefits of Cloud
Native Applications): क्लाउड कंप्यूटिंग वेब
Computing):
आधारित या मोबाइल आधारित एप्लिकेशन के निर्माण,
• लागत (Cost): यह हार्डवेयर और सॉफ्टवेयर खरीदने परिनियोजन और स्केल में मदद करता है।
और साइट पर डेटा केंद्र स्थापित करने और चलाने, • एप्लीकेशन का परीक्षण और निर्माण (Test and Build
बिजली और शीतलन के लिए चौबीसों घंटे बिजली Applications): क्लाउड इन्फ्रास्ट्रक्चर का उपयोग
आपूर्ति तथा बुनियादी ढांचे के प्रबंधन के लिए आईटी करके एप्लिकेशन विकास लागत और समय को कम
विशेषज्ञ के पूंजीगत व्यय को बचाता है। करके इसे आसानी से ऊपर या नीचे बढ़ाया जा सकता है।
• गति (Speed): अधिकांश क्लाउड कंप्यूटिंग सेवाएं स्वयं • डेटा स्टोरेज, बैकअप और रिकवरी (Store, Back Up
सेवा और मांग पर प्रदान की जाती हैं, इसलिए बड़ी मात्रा and Recover Data): इं टरनेट पर डेटा को ऑफ़साइट
में कंप्यूटिंग संसाधनों को भी, आम तौर पर केवल कुछ क्लाउड स्टोरेज सिस्टम में स्थानांतरित करके डेटा को
माउस क्लिक के साथ, मिनटों में उपलब्ध कराया जा अधिक किफायती और बड़े पैमाने पर सुरक्षित रखे, यह
सकता है यह व्यवसायों को बहुत अधिक लचीलापन देता स्टोरेज सिस्टम किसी भी स्थान और किसी भी डिवाइस
है और क्षमता नियोजन के दबाव को दूर करता है। से एक्सेस किया जा सकता है।
• वैश्विक स्तर (Global Scale): क्लाउड कंप्यूटिंग सेवाओ ं • डेटा का विश्ले षण (Analyze Data): क्लाउड कई
के लाभों में व्यापक पैमाने पर विस्तार करने की क्षमता टीमों, डिवीजनों, विभागों और भौगोलिक स्थानों पर
शामिल है। क्लाउड में, यह सही मात्रा में आईटी संसाधनों डेटा को एकीकृत करता है। अधिक जानकार निर्णयों के
- उदाहरण के लिए, कंप्यूटिंग शक्ति, भंडारण, बैंडविड्थ लिए, मशीन लर्निं ग और आर्टिफिशियल इं टे लिजेंस जैसी
को आवश्यकतानुसार और सही उचित जगह से उपलब्ध क्लाउड सेवाओ ं का उपयोग करें।
कराने में मदद करता है।
• ऑडियो और वीडियो स्ट्रीम करना (Stream Audio
• उत्पादकता (Productivity): ऑन-साइट डेटा केंद्रों and Video): ग्लोबल डिस्ट्रीब्यूशन की सहायता से
को आमतौर पर बहुत सारे हार्डवेयर सेटअप, सॉफ़्टवेयर हाई-डेफिनिशन वीडियो और ऑडियो वाले किसी भी
पैचिंग और अन्य समय ले ने वाले आईटी प्रबंधन कार्यों डिवाइस पर कहीं भी, कभी भी दर्शकों से जुड़ें। उदाहरण:
की आवश्यकता होती है। क्लाउड कंप्यूटिंग सेवाओ ं में नेटफ्लिक्स, अमेज़न प्राइम, हॉटस्टार आदि।
आईटी टीमें यह समय अधिक महत्वपूर्ण व्यावसायिक
• सॉफ्टवेयर की ऑन डिमांड ‍डिलीवरी (Deliver
लक्ष्यों को प्राप्त करने में लगा सकती हैं।
Software on Demand): सॉफ्टवेयर एज ए ‍सर्वि स
• कार्य (Performance): निष्पादन: सबसे बड़ी क्लाउड (SaaS) के रूप में जाना जाता है और यह ऑन-डिमांड
कंप्यूटिंग सेवाएं सुरक्षित डेटा केंद्रों के विश्वव्यापी नेटवर्क सॉफ्टवेयर ग्राहकों को नवीनतम सॉफ्टवेयर संस्करण
पर चलती हैं, जिन्हें नियमित रूप से नवीनतम पीढ़ी के और अपडेट प्रदान करने में सक्षम है। SaaS एक
तेज और कुशल कंप्यूटिंग हार्डवेयर में अपग्रेड किया सॉफ्टवेयर लाइसेंसिंग और डिलीवरी मॉडल है जिसमें
जाता है। यह एकल कॉर्पोरेट डेटासेंटर पर कई लाभ सॉफ्टवेयर को सब्सक्रिप्शन के आधार पर लाइसेंस दिया
प्रदान करता है, जिसमें अनुप्रयोगों के लिए कम नेटवर्क जाता है और इसे केंद्रीय रूप से होस्ट किया जाता है।
विलं बता और अधिक ‍किफायत शामिल हैं।

• विश्वसनीयता (Reliability): क्लाउड कंप्यूटिंग डेटा


बैकअप, डिजास्टर रिकवरी और व्यापार निरंतरता को सरकार की पहल (Government Initiatives):
आसान और कम खर्चीला बनाती है। क्योंकि क्लाउड
ई-ग्राम पंचायत (E-Gram Panchayat)
प्रदाता के नेटवर्क पर कई अनावश्यक साइटों पर डेटा

18
• शासन की गुणवत्ता में सुधार के लिए, भारत सरकार ने
आंतरिक सरकारी कार्यों को सरल और मजबूत बनाने
ग्रिड कंप्यूटिंग (Grid
के लिए ई-पंचायत के रूप में जानी जाने वाली एक Computing)
ई-गवर्नेंस योजना शुरू की।
• संयुक्त कार्यों को पूरा करने के लिए एक साथ काम
करने के लिए ग्रिड कंप्यूटिंग कई कंप्यूटरों का उपयोग
किसान सुविधा (Kisan Suvidha) करता है, जो अक्सर भौगोलिक रूप से दूर होते हैं ले किन
नेटवर्क से जुड़े होते हैं। यह आम तौर पर कंप्यूटर के एक
• भारत सरकार ने किसानों को प्रासंगिक जानकारी के
सेट "डेटा ग्रिड" पर चलाया जाता है, यह सीधे बातचीत
साथ तुरत
ं मदद करने के लिए किसान सुविधा पोर्टल
करके विभिन्न कार्यों और कार्यों का समन्वय करता है।
शुरू किया है। यह किसानों को मौसम, बाजार मूल्य, बीज,
उर्वरक, कीटनाशकों, कृषि मशीनरी, डीलरों, कृषि सलाह,
पौधों की सुरक्षा और एकीकृत कीट प्रबंधन (IPM) आदि
पर विस्तृत सूचना प्रदान करता है। यह उन्हें चरम मौसम ग्रिड कम्प्यूटिंग कैसे काम करता है? (How Grid
की स्थिति और बदलते बाजार मूल्य के बारे में सूचित Computing Works?)
करता है। • ग्रिड कंप्यूटिंग प्रत्येक कंप्यूटर पर विशेष सॉफ्टवेयर
चलाकर काम करता है। यह डेटा ग्रिड में भाग ले ता है।

• सॉफ्टवेयर पूरी प्रणाली के प्रबंधक के रूप में कार्य करता


डिजिलॉकर (DigiLocker)
है और पूरे ग्रिड में विभिन्न कार्यों का समन्वय करता है।
• डिजिलॉकर सार्वजनिक क्लाउड-आधारित भंडार है।
• सॉफ़्टवेयर विशेष रूप से, प्रत्येक कंप्यूटर को उप-कार्य
यह आपकी सुविधा के आधार पर आपके दस्तावेज़ों को
प्रदान करता है ताकि वे अपने संबंधित उप-कार्यों पर एक
अपलोड करने के लिए एक ऑनलाइन ड्राइव से कहीं
साथ कार्य कर सकें।
अधिक है। दस्तावेजों को डिजिटल रूप से सत्यापित किया
जाता है और डिजिलॉकर सत्यापन की एक प्रामाणिक • उप-कार्यों के पूरा होने के बाद, आउटपुट एकत्रित किए
मुहर के साथ हस्ताक्षरित किया जाता है। इसका उद्देश्य जाते हैं फिर बड़े पैमाने के कार्य को पूरा किया जाता है।
नागरिकों के डिजिटल दस्तावेज़ वॉले ट को प्रामाणिक • सॉफ्टवेयर प्रत्येक कंप्यूटर को अन्य कंप्यूटरों के साथ
डिजिटल दस्तावेज़ों तक पहुँच प्रदान करके नागरिकों का नेटवर्क पर संचार करने की सुविधा देता है ताकि प्रत्येक
'डिजिटल सशक्तिकरण' करना है। कंप्यूटर के उप-कार्यों के हिस्से पर जानकारी साझा की
जा सके, ताकि आउटपुट को कैसे समेकित और वितरित
किया जाए।
मेघराज (MeghRaj)

• इसका उद्देश्य सरकार के आईसीटी खर्च को इष्टतम


करते हुए देश में ई-सेवाओ ं के वितरण में तेजी लाना ग्रिड कंप्यूटिंग के अनुप्रयोग (Applications of
है। यह बुनियादी ढांचे के इष्टतम उपयोग को सुनिश्चित Grid Computing):
करेगा और ई-गवर्नेंस अनुप्रयोगों के विकास और तैनाती • आदर्श ग्रिड कंप्यूटिंग परिदृश्य में, प्रत्येक मौजूद संसाधन
में तेजी लाएगा। को अधिकृत कंप्यूटरों के बीच साझा किया जाता है,
जिससे उन्हें सुपर कंप्यूटर में बदल दिया जाता है।

• अनुप्रयोग विभाजन: इसमें समस्या को अलग-अलग


पीएम वाणी (वाई-फाई एक्सेस नेटवर्क इं टरफेस) (PM
WANI) टु कड़ों में तोड़ना शामिल है।

• ग्रिड कंप्यूटिंग की मदद से विभिन्न भौगोलिक क्षेत्रों और


• यह पूरे देश में वाई-फाई की पहुंच बढ़ाने के लिए बनाई
डेटा स्रोतों जैसे उपग्रह, सेंसर, पिछले डेटा आदि से ढे र सारे
गई एक योजना है। इसके तहत कनेक्टिविटी विकल्पों
इकट्ठे कर आपदाओ ं का बेहतर प्रबंधन किया जा सकता
को बढ़ाने और डिजिटल पहुंच में सुधार के लिए देश भर में
है।
बड़े पैमाने पर वाई-फाई हॉटस्पॉट बनाए जाएं गे।
• ग्रिड कंप्यूटिंग पूरे डेटा के बिट्स को अलग-अलग ग्रिड
नोड्स में वितरित करके एक साथ डेटा को संसाधित
करने में मुट्ठी भर है।

• फिल्म उद्योग में इसका उपयोग कार्य को अधिक कुशलता

19
से करने और समय बचाने की क्षमता के कारण किया
जाता है। कंप्यूटर न केवल प्रभाव डालने में मदद करते
क्वांटम कम्प्यूटिंग
हैं, बल्कि वे ग्रिड कंप्यूटिंग का उपयोग करके फिल्म को (Quantum Computing):
सामान्य से तेज गति से बनाने में मदद भी करते हैं।

• ऑनलाइन गेमिं ग में अधिक रुचि रखने वाले • क्वांटम कंप्यूटिंग एक प्रकार की कंप्यूटिंग है जो क्वांटम
उपयोगकर्ताओ ं की वृद्धि के साथ, ट्रैफ़िक में वृद्धि हुई है, यांत्रिकी के सिद्धांतों के आधार पर कंप्यूटर प्रौद्योगिकी
और यहीं पर ग्रिड कंप्यूटिंग तेजी से और बेहतर प्रासेसिंग विकसित करने पर केंद्रित है, जो परमाणु और उप-
में मदद करती है। परमाणु स्तरों पर ऊर्जा और पदार्थों के व्यवहार की
व्याख्या करता है।

• आज हम जिन क्लासिकल कंप्यूटरों का उपयोग करते


भारत में ग्रिड कंप्यूटिंग (Grid Computing in हैं, वे केवल बिट्स में जानकारी को एन्कोड कर सकते
India): हैं। इसमें 1 या 0 लगता है। यह उनकी क्षमता को सीमित
करता है।
गरुड़ (Garuda):
• दूसरी ओर, क्वांटम कंप्यूटिंग, क्वांटम बिट्स या
• सी-डैक ने वैज्ञानिक, इं जीनियरिं ग और सामाजिक-
क्वबिट्स का उपयोग करती है। यह उप-परमाणु कणों
आर्थि क विकास के लिए भारतीय राष्ट्रीय ग्रिड कंप्यूटिंग
की अद्वितीय क्षमता का उपयोग करता है। यह उन्हें
पहल - गरुड़ की शुरुआत की।
क्लासिकल कंप्यूटरों की तुलना में एक ही समय में एक
• गरुड़ एक राष्ट्रव्यापी ग्रिड जिसमें कम्प्यूटे शनल नोड्स, से अधिक रूपों में, यानी 1 और 0 में रहने की अनुमति
मास स्टोरेज और वैज्ञानिक उपकरण शामिल हैं, पर देता है।
वैज्ञानिक और तकनीकी शोधकर्ताओ ं के साथ सहयोग
• सुपरइम्पोजिशन और एं टें गलमेंट क्वांटम भौतिकी की
करता है।
दो विशेषताएं हैं जिस पर क्वांटम कंप्यूटर आधारित हैं।
• इसका उद्देश्य 21वीं सदी के लिए डेटा उपलब्ध कराने और यह क्वांटम कंप्यूटरों को पारंपरिक कंप्यूटरों की तुलना
गहन विज्ञान और इं जीनियरिं ग अनुप्रयोगों की गणना में अधिक गति से और बहुत कम ऊर्जा खपत पर संचालन
करने के लिए आवश्यक तकनीकी प्रगति प्रदान करना को तेजी से संभालने के योग्य बनाता है।
है।
• क्वांटम कंप्यूटिंग अंतरिक्ष विज्ञान, आपदा प्रबंधन, वित्त,
• गरुड़ की सबसे महत्वपूर्ण चुनौतियों में से एक है साइबर सुरक्षा, सैन्य मामलों, खुफिया, दवा डिजाइन और
शोधकर्ताओ ं और जटिल वैज्ञानिक और इं जीनियरिं ग खोज, एयरोस्पेस डिजाइनिं ग, उपयोगिताओ ं (परमाणु
प्रयासों में नवाचार के बीच सही संतुलन बनाना। संलयन), पॉलिमर डिजाइन, आर्टिफिशियल इं टे लिजेंस
(AI) और भारी डेटा विश्ले षण, और डिजिटल निर्माण के
• सूचना प्रौद्योगिकी विभाग (DIT) ने प्रगत संगणन विकास
क्षेत्रों में बहुत योगदान दे सकती है।
केंद्र (सी-डैक) देश भर में कम्प्यूटे शनल ग्रिड 'गरुड़' को
तैनात करने के लिए, जो देश भर के 17 शहरों को जोड़ेगी, • क्वांटम सर्वोच्चता: 2019 में Google ने घोषणा की कि
को वित्त पोषित किया है ताकि "ग्रिड" नेटवर्क कंप्यूटिंग उसने "क्वांटम सर्वोच्चता" हासिल कर ली है, जिसका अर्थ
को अनुसंधान प्रयोगशालाओ ं और उद्योग में लाया जा है कि उन्होंने एक समस्या को शीघ्र हल करने के लिए
सके। क्वांटम कंप्यूटर का उपयोग किया था। एक पारंपरिक
कंप्यूटर में इसे हल करने में अव्यावहारिक रूप से लं बा
• ग्रिड कंप्यूटिंग और आपदा प्रबंधन: परियोजना की
समय (हजारों वर्ष) लगता। आईबीएम ने तुरत
ं यह कहते
संकल्पना गरुड़ ग्रिड के तहत प्रगत संगणन विकास
हुए दावे को खारिज कर दिया कि उनके पारंपरिक सुपर
केंद्र (सी-डैक) और स्पेस एप्लीकेशन सेंटर (SAC)
कंप्यूटर कुछ ही दिनों में समस्या का समाधान कर सकते
के बीच 2006 में की गई थी। यह परियोजना सिं थेटिक
हैं और इसके लिए Google द्वारा दावा किए गए हजारों
एपर्चर रडार (SAR), ग्रिड कंप्यूटिंग और GEO सिं क्रोनस
वर्षों की आवश्यकता नहीं होगी।
सैटेलाइट (जीसैट) तकनीक का उपयोग करके नदी की
बाढ़ का आकलन करने के बारे में है।

• जब बाढ़ आती है, तो सिं थेटिक अपर्चर रडार (SAR) से


क्वांटम कंप्यूटिंग के लाभ और अनुप्रयोग
युक्त वायुयान को बाढ़ प्रभावित क्षेत्र के ऊपर से उड़ाया
(Benefits & Applications of Quantum
जाता है। SAR खराब मौसम की स्थिति (बादल, कोहरा,
Computing):
मंद प्रकाश) में भी बाढ़ स्थल के बारे में डेटा कैप्चर कर
सकता है। 1. आपदा प्रबंधन:

• जलवायु डेटा का आसान और शीघ्र संग्रहण प्रारंभिक

20

AKASH SINGH
चरणों में आपदाओ ं को कम कर सकता है। कृषि भूमि को 2. नैतिक सरोकार: उन्नत मशीनों द्वारा मनुष्यों का
सीमित करने वाली कृषि खाद्य प्रौद्योगिकी श्रृंखलाओ ं पर प्रतिस्थापन।
गहरा प्रभाव।
3. भारी बेरोजगारी।
2. सुरक्षित संचार:
4. डेटा सुरक्षा चिं ता।
• उपग्रह,सैन्य उद्देश्य साइबर सुरक्षा
5. R&D और इं फ्रा में भारी निवेश।
3. इं टरनेट ऑफ थिं ग्स, बिग डेटा, आर्टिफिशियल इं टे लिजेंस

4. अनुसंधान:
भारत में उठाए गए कदम (Steps Taken in
(क) जीनोम सीक्वेंसिं ग जैसे जैविक अध्ययन, लक्षित
India):
दवा वितरण आदि।

(ख) ब्रह्मांड की खोज, गुरुत्वाकर्षण तरंगें, ब्लै क होल 1. राष्ट्रीय क्वांटम प्रौद्योगिकी मिशन।
आदि। 2. QUEST -क्वांटम सक्षम विज्ञान और प्रौद्योगिकी।

3. बजट 2020 में, अगले 5 वर्षों के लिए 8000 करोड़


परिव्यय का प्रस्ताव - क्वांटम प्रौद्योगिकी और अनुप्रयोगों
क्वांटम कंप्यूटिंग की चुनौतियाँ (Challenges of पर राष्ट्रीय मिशन।
Quantum Computing):
4. इसरो+रमन वैज्ञानिक संस्थान द्वारा अंतरिक्ष में सुरक्षित
1. नई तकनीक से संबंधित कौशल विकास। क्वांटम संचार विकसित करना।

नेट न्यूट्रैलिटी (Net Neutrality):

• नेट न्यूट्रैलिटी का सिद्धांत कहता है कि इं टरनेट उपयोगकर्ता दूरसंचार सेवा प्रदाताओ ं (TSP) या इं टरनेट सेवा प्रदाताओ ं
द्वारा बिना किसी भेदभाव के इं टरनेट पर सभी सामग्री तक पहुंचने में सक्षम होना चाहिए। संक्षेप में इसका अर्थ है (i) सभी
वेबसाइटों या एप्लिकेशन को TSPs द्वारा समान माना जाना चाहिए, (ii) सभी एप्लिकेशन को एक ही इं टरनेट स्पीड पर
एक्सेस करने की अनुमति दी जानी चाहिए, और (iii) सभी एप्लिकेशन समान लागत पर सुलभ होने चाहिए।

• दूसरे शब्दों में, इन कंपनियों को ग्राहकों को अपने केबल पैकेज रखने या एक अलग वीडियो-स्ट्रीमिं ग सेवा खरीदने के
लिए प्रोत्साहित करने के लिए स्काइप जैसी किसी सेवा तक पहुंचने से रोकने, या नेटफ्लिक्स को धीमा नहीं करने में
सक्षम नहीं होना चाहिए।

• अभिव्यक्ति की स्वतंत्रता के लिए नेट न्यूट्रैलिटी के महत्व पर प्रकाश डालते हैं: मुट्ठी भर बड़ी दूरसंचार कंपनियां ब्रॉडबैंड
बाजार पर हावी हो सकती हैं, जो विशेष विचारों को दबाने या सबसे अधिक भुगतान करने वालों को ऑनलाइन अपने
विचार रखने की अनुमति देने संबंधी बड़ी शक्ति उन्हें दे सकती है।

भारत में नेट न्यूट्रैलिटी को कैसे विनियमित किया जाता है? (How is Net Neutrality Regulated in India?)

• अब तक, भारत में नेट न्यूट्रैलिटी को सीधे तौर पर किसी भी कानून या नीतिगत ढांचे द्वारा नियंत्रित नहीं किया गया
है। पिछले वर्ष के दौरान, नेट न्यूट्रैलिटी नीति के निर्माण के संबंध में कुछ कार्य हुए हैं। TRAI ने डेटा सर्वि सेज के लिए
डिफरेंशियल प्राइसिं ग के साथ-साथ ओवर-द-टॉप सर्वि सेज (OTT) के लिए रेगुलेटरी फ्रेमवर्क पर परामर्श पत्रों पर
सुझाव आमंत्रित किए थे।

• दूरसंचार विभाग (DoT) द्वारा गठित एक समिति ने भी नेट न्यूट्रैलिटी के मुद्दे की जांच की थी।

• अंतरराष्ट्रीय स्तर पर, संयुक्त राज्य अमेरिका, जापान, ब्राजील, चिली, नॉर्वे जैसे देशों में कुछ के कानून, व्यवस्था या
नियामक ढांचे हैं जो नेट न्यूट्रैलिटी को प्रभावित करते हैं।

नेट न्यूट्रैलिटी पर TRAI के 2016 के विनियम (TRAI’s 2016 Regulations on Net Neutrality):

TRAI के नवीनतम नियमों में कहा गया है कि (The Latest TRAI Regulations State that):

• किसी भी सेवा प्रदाता को कोई ऐसा समझौता या अनुबंध करने की अनुमति नहीं है, जिसके परिणामस्वरूप सामग्री (डेटा
सेवाओ ं) के आधार पर उपभोक्ता से भेदभावपूर्ण शुल्क लगाया जाएगा।

21
• इस तरह के टै रिफ की अनुमति केवल क्लोज्ड इले क्ट्रॉनिक कम्युनिकेशन नेटवर्क में दी जाएगी, जो ऐसे नेटवर्क हैं जहां
डेटा न तो प्राप्त किया जाता है और न ही इं टरनेट पर डाला जाता है,

• एक सेवा प्रदाता आपातकालीन सेवाओ ं तक पहुँचने या सेवा प्रदान करने के लिए टै रिफ कम कर सकता है,

• इन नियमों के उल्लं घन के मामले में, सेवा प्रदाता को उल्लं घन के लिए प्रति दिन 50,000 रुपये का भुगतान करना पड़
सकता है, जो अधिकतम 50 लाख रुपये तक है।

• यहां ध्यान दिया जा सकता है कि, 2006 और 2008 में, TRAI ने सुझाव दिया था कि इं टरनेट क्षेत्र अनरेगुलेटे ड और भेदभाव
रहित (net neutral) रहता है।

साइबर अपराध साइबर अपराध के प्रकार (Types of


Cybercrime):
(Cybercrime) सेवाओ ं की अनुपलब्धता से संबंधित (DoS) हमले (Denial
of Services (DoS) Attacks)
• साइबर अपराध को ऐसे अपराध के रूप में परिभाषित
• इनका उपयोग ऑनलाइन सेवा को अनुपलब्ध बनाने
किया जाता है जहां अपराध का उद्देश्य या अपराध करने
और विभिन्न स्रोतों से ट्रैफ़िक के माध्यम से साइट को
के लिए उपकरण के रूप में कंप्यूटर का उपयोग किया
हैवी करके नेटवर्क को डाउन करने के लिए किया जाता
जाता है।
है। बॉटनेट के नाम से जाने जाने वाले संक्रमित उपकरणों
• साइबर अपराधी उपयोगकर्ता की व्यक्तिगत जानकारी, के बड़े नेटवर्क उपयोगकर्ताओ ं के कंप्यूटर पर मॉलवेयर
गोपनीय व्यावसायिक जानकारी सरकारी जानकारी जमा करके बनाए जाते हैं। नेटवर्क डाउन होने के बाद
तक पहुंचने या किसी डिवाइस को अक्षम के लिए किसी हैकर सिस्टम को हैक कर ले ता है।
डिवाइस का उपयोग कर सकते हैं। उपरोक्त जानकारी
को ऑनलाइन बेचना या प्राप्त करना भी एक साइबर
अपराध है। बॉटनेट (Botnets):

• बॉटनेट छे ड़छाड़ किए गए कंप्यूटरों के नेटवर्क होते हैं


जिन्हें रिमोट हैकर्स द्वारा बाहरी रूप से नियंत्रित किया
विभिन्न प्रकार के साइबर खतरे और साइबर
अपराध (Various Types of Cyber Threats जाता है। रिमोट हैकर्स इन बॉटनेट के जरिए स्पैम भेजते हैं
and Cybercrimes): या दूसरे कंप्यूटरों पर हमला करते हैं। बॉटनेट का उपयोग
मॉलवेयर के रूप में कार्य करने और दुर्भावनापूर्ण कार्य
• डिजिटल दुनिया में साइबर अपराध बड़े पैमाने पर बढ़ करने के लिए भी किया जा सकता है।
रहे हैं। वर्ल्ड वाइड वेब के साइबर अपराधी अपने लाभ के
लिए इं टरनेट उपयोगकर्ताओ ं की व्यक्तिगत जानकारी
का दुरूपयोग करते हैं। वे अवैध उत्पादों और सेवाओ ं को पहचान की चोरी (Identity Theft):
खरीदने और बेचने के लिए डार्क वेब में गहराई तक जाते
हैं। वे वर्गीकृत सरकारी जानकारी तक भी पहुँच जाते हैं। • यह तब होता है जब कोई अपराधी धन चोरी करने, गोपनीय
जानकारी तक पहुंचने, कर या स्वास्थ्य बीमा धोखाधड़ी
• साइबर अपराध अब तक के सबसे ऊंचे स्तर पर हैं,
करने के लिए उपयोगकर्ता की व्यक्तिगत जानकारी तक
जिससे कंपनियों और लोगों को प्रतिवर्ष अरबों डॉलर का
पहुंचता है। वे आपके नाम से एक फोन/इं टरनेट खाता भी
नुकसान होता है।
खोल सकते हैं, आपराधिक गतिविधि की योजना बनाने
• प्रौद्योगिकी के विकास और स्मार्ट तकनीक की बढ़ती के लिए आपके नाम का उपयोग कर सकते हैं और आपके
पहुंच का अर्थ है कि हैकर्स के दुरूपयोग के लिए नाम पर सरकारी लाभों का दावा कर सकते हैं। वे हैकिंग,
उपयोगकर्ताओ ं के घरों के भीतर कई रास्ते हैं। सोशल मीडिया से व्यक्तिगत जानकारी प्राप्त करने या
फ़िशिं ग ईमेल भेजकर उपयोगकर्ताओ ं के पासवर्ड का
जबकि कानून प्रवर्तन एजेंसियां बढ़ते मुद्दों से निपटने का पता लगाकर ऐसा कर सकते हैं।
प्रयास कर रहीं हैं, इं टरनेट पर गोपनीयता का फायदा उठाते
हुए आपराधियों की संख्या बढ़ती जा रही है।
साइबरस्टॉकिंग (Cyberstalking):

• इसमें ऑनलाइन उत्पीड़न शामिल है जहां उपयोगकर्ता

22
के पास ढे रों ऑनलाइन संदेश और ईमेल आते हैं। बीच यौन गतिविधि, तीव्र हिं सक वीडियो और आपराधिक
साइबरस्टॉकर उपयोगकर्ता को डराने और उसके अंदर गतिविधि के वीडियो शामिल हो सकते हैं, ले किन यह
डर पैदा करने के लिए सोशल मीडिया, वेबसाइटों और इन्हीं तक सीमित नहीं है। अवैध सामग्री में आतंकवादी
सर्च इं जनों का उपयोग करते हैं। आमतौर पर, साइबर गतिविधियों की वकालत करने वाली तथा बाल शोषण
स्टॉकर अपने शिकार को जानता है और उसको अपनी से संबंधित सामग्री शामिल है।
सुरक्षा के लिए डर या चिं तित महसूस कराता है।

ऑनलाइन घोटाले (Online Scams):


सोशल इं जीनियरिं ग (Social Engineering):
• ये आमतौर पर विज्ञापनों या स्पैम ईमेल के रूप में होते हैं।
• सोशल इं जीनियरिं ग में ऐसे अपराधी शामिल होते हैं जो इसमें पुरस्कार के वादे या अवास्तविक राशि के प्रस्ताव
आमतौर पर फोन या ईमेल द्वारा आपसे सीधे संपर्क शामिल हैं। ऑनलाइन घोटालों में "टू गुड टू बी ट्रू" तरह
करते हैं। वे आपका विश्वास हासिल करना चाहते हैं और के आकर्षक ऑफ़र शामिल हैं और जब उन पर क्लिक
आमतौर पर ग्राहक सेवा एजेंट के रूप में पेश आते हैं ताकि किया जाता है तो मॉलवेयर हस्तक्षेप और जानकारी लीक
आप उन्हें आवश्यक जानकारी दे सकें। इसमें आमतौर पर हो सकता है।
एक पासवर्ड, जिस कंपनी के लिए आप काम करते हैं,
या बैंक की जानकारी शामिल होती है। साइबर क्रिमिनल
इं टरनेट से आपके बारे में सभी संभंव जानकारी जुटाकर भारत में साइबर अपराध – आंकड़े और तथ्य (Cybercrime
फिर आपको सोशल अकाउं ट पर एक दोस्त के बनाने in India - Statistics & Facts):
का प्रयास करते हैं। एक बार जब वे किसी खाते तक पहुंच
• 2017 की उद्योग रिपोर्ट के अनुसार, साइबर अपराधों के
प्राप्त कर ले ते हैं, तो वे आपकी जानकारी या सुरक्षित
कारण भारतीय उपभोक्ताओ ं को 18 बिलियन अमेरिकी
खाते आपके नाम पर बेच सकते हैं।
डॉलर से अधिक का नुकसान हुआ था।

• 2018 में, देश में साइबर अपराधों के 27 हजार से अधिक


पीयूपीएस (PUPs): मामले दर्ज किए गए, जो कि दो वर्ष पहले के मामलों की
संख्या की तुलना में 121 प्रतिशत से अधिक की वृद्धि को
• पीयूपीएस या पोटे न्शियली अनवांटेड प्रोग्राम अन्य दर्शाता है।
साइबर अपराधों की तुलना में कम खतरनाक, ले किन
• जबकि अपराध छोटे ऑनलाइन धोखाधड़ी से ले कर
एक प्रकार का मॉलवेयर होते हैं। वे आपके सिस्टम
लॉटरी घोटाले और यौन उत्पीड़न तक होते हैं, ले किन
में सर्च इं जन और पहले से डाउनलोड किए गए ऐप्स
सबसे ज्यादा अपराध बैंकिंग और वित्त क्षेत्र में होते हैं।
सहित आवश्यक सॉफ़्टवेयर को अनइं स्टॉल कर देते
हैं। उनमें स्पाइवेयर या एडवेयर शामिल हो सकते हैं, • आर्थि क नुकसान के साथ, वे सार्वजनिक सुरक्षा खासकर
और एं टीविषाणु सॉफ़्टवेयर स्थापित करने से ऐसे फर्जी नाबालिगों और समाज के कमजोर वर्गों को भी साइबर
डाउनलोड को रोका जा सकता है। दुरूपयोग और शोषण के माध्यम से प्रभावित करते हैं।

• अकेले 2018 में, भारत में यौन उत्पीड़न से संबंधित दो


हजार से अधिक साइबर अपराध और महिलाओ ं और
फ़िशिं ग (Phishing): नाबालिगों के खिलाफ साइबर बुलिंग के 700 से अधिक
मामले दर्ज किए गए।
• यह एक प्रकार का हमला है जिसमें हैकर्स उपयोगकर्ताओ ं
को उनके खातों या कंप्यूटर तक पहुंचने के लिए फर्जी
ईमेल अटै चमेंट या यूआरएल भेजते हैं। उपयोगकर्ताओ ं को
साइबर सुरक्षा के लिए भारत सरकार द्वारा की
ईमेल में यह कहा जाता है कि उन्हें अपना पासवर्ड बदलने
गई पहल (Initiatives Taken by Indian
या अपनी बिलिंग जानकारी अपडेट करने की आवश्यकता Government for Cyber Security):
है, इससे अपराधियों को एक्सेस मिल जाता है।
सर्ट-इन (CERT-In):

• देश की साइबर सुरक्षा से निपटने के लिए राष्ट्रीय


निषिद्ध/अवैध सामग्री (Prohibited/Illegal Content): एजेंसी के रूप में काम करने वाली भारतीय कंप्यूटर
• इसमें ऐसे अपराधी शामिल होते हैं जो अत्यधिक कष्टकारी आपातकालीन प्रतिक्रिया टीम (CERT-In) उन्नतीकरण
और आपत्तिजनक अनुपयुक्त सामग्री को साझा और ने सरकारी नेटवर्क पर साइबर हमलों की दर को कम
वितरित करते हैं। आपत्तिजनक सामग्री में वयस्कों के करने में मदद की है।

23
साइबर सुरक्षित भारत (Cyber Surakshit Bharat) है। विधेयक का तात्पर्य केवल भारत में व्यक्तियों से
संबंधित किसी भी महत्वपूर्ण जानकारी के भंडारण और
• भारत में साइबर सुरक्षा पारिस्थितिकी तंत्र को मजबूत
प्रसंस्करण से है। यह सख्ती से कहता है कि व्यक्ति के
करने के उद्देश्य से - 'डिजिटल इं डिया' के लिए सरकार
संवेदनशील वैयक्तिक डाटा को स्थानीय रूप से संग्रहीत
के दृष्टिकोण के अनुरूप, इले क्ट्रॉनिक्स और सूचना
किया जाए; हालाँकि, इसे कुछ शर्तों के अधीन विदेशों में
प्रौद्योगिकी मंत्रालय (MeitY) ने साइबर सुरक्षित भारत
प्रासेस किया जा सकता है, विधेयक का उद्देश्य सोशल
पहल शुरू की है। यह कार्यक्रम राष्ट्रीय ई-गवर्नेंस
मीडिया कंपनियों को अधिक जवाबदेह बनाना और उन्हें
डिवीजन (NeGD) के सहयोग से आयोजित किया गया।
आपत्तिजनक सामग्री के प्रसार को रोकने के लिए बाध्य
करना है।

राष्ट्रीय महत्वपूर्ण सूचना मूल संरचना संरक्षण केंद्र


(National Critical Information Infrastructure
Protection Centre, NCIIPC): साइबर स्वच्छता केंद्र (Botnet Cleaning and Malware
Analysis Centre)
• NCIIPC केंद्र सरकार की एक स्थापना है, जो हमारे देश
• यह फर्जी प्रोगामों का पता लगाने और ऐसे प्रोगामों को
की महत्वपूर्ण जानकारी की सुरक्षा के लिए बनाई गई है,
हटाने के लिए फ्री टु ल प्रदान करने के लिए शुरू किया
जिसका राष्ट्रीय सुरक्षा, आर्थि क विकास या सार्वजनिक
गया है।
स्वास्थ्य सेवा पर व्यापक प्रभाव पड़ता है।

राष्ट्रीय साइबर सुरक्षा नीति, 2013 (National Cyber


NCIIPC ने मोटे तौर पर निम्नलिखित को 'महत्वपूर्ण क्षेत्रों'
Security Policy, 2013):
के रूप में पहचाना है (NCIIPC has Broadly Identified
the Following as ‘Critical Sectors):
• नीति का उद्देश्य नागरिकों, व्यवसायों और सरकार के
• विद्युत और ऊर्जा लिए एक सुरक्षित और लचीला साइबर स्पेस बनाना है।
इसका मिशन साइबरस्पेस सूचना और बुनियादी ढांचे
• बैंकिंग, वित्तीय सेवाएं और बीमा
की रक्षा करना, साइबर हमलों को रोकने और उनका
• दूरसंचार सामना करने के लिए क्षमताओ ं का निर्माण करना और
• परिवहन संस्थागत संरचनाओ ं, लोगों, प्रक्रियाओ ं और प्रौद्योगिकी
के समन्वित प्रयासों के माध्यम से नुकसान को कम
• सरकारी
करना है।
• सामरिक और सार्वजनिक उद्यम
• क्षमता निर्माण, कौशल विकास और प्रशिक्षण के माध्यम
से अगले 5 वर्षों में साइबर सुरक्षा में 500,000 कुशल
पेशेवरों का कार्यबल तैयार करना।
मुख्य सूचना सुरक्षा अधिकारियों की नियुक्ति
(Appointment of Chief Information Security • साइबर अपराध की प्रभावी रोकथाम, जांच और
Officers) अभियोजन को सक्षम बनाना और उचित विधायी हस्तक्षेप
के माध्यम से कानून प्रवर्तन क्षमताओ ं में वृद्धि करना।
• भारत सरकार ने हाल ही में सरकारी संगठनों के CISOs
के लिए एक लिखित दिशानिर्देश जारी किया है, जिसमें
एप्लीकेशनों, बुनियादी ढांचे की सुरक्षा और अनुपालन
हेतु सर्वोत्तम तरीकों पर प्रकाश डाला गया है। मुख्य सूचना सुपर कंप्यूटर और उसके अनुप्रयोग
सुरक्षा अधिकारी (CISOs) प्रत्येक तकनीकी नवाचार (Supercomputers and Its Applications):
के साथ उत्पन्न होने वाली सुरक्षा आवश्यकताओ ं की
• सुपरकंप्यूटर हाई-परफारमेंस कंप्यूटिंग (HPC) का
पहचान और दस्तावेजीकरण कर सकते हैं।
भौतिक अवतार हैं, जो संगठनों को उन समस्याओ ं को हल
करने की क्षमता प्रदान करते हैं जो नियमित कंप्यूटरों
के साथ असंभव होती हैं। यह समानांतर में काम कर रहे
वैयक्तिक डाटा संरक्षण विधेयक (Personal Data
कई कंप्यूटर प्रणाली (यानी, एक "सुपर कंप्यूटर") के
Protection Bill)
केंद्रित गणना संसाधनों का उपयोग करके व्यापक रूप
• भारतीय उपयोगकर्ताओ ं को वैश्विक उल्लं घनों से बचाने से जटिल या डेटा से भरी समस्याओ ं के प्रसंस्करण को
के लिए केंद्र सरकार द्वारा वैयक्तिक डाटा संरक्षण (PDP) संदर्भि त करता है। सुपरकंप्यूटिंग में किसी भी कंप्यूटर
विधेयक का अनुमोदन भारतीय नागरिकों के लिए के अधिकतम संभावित प्रदर्शन पर काम करने वाली
सबसे महत्वपूर्ण है, यह डाटा स्थानीयकरण पर केंद्रित प्रणाली शामिल होती है, जिसे आमतौर पर पेटाफ्लॉप्स

24
में मापा जाता है। मशीनों में मानव बुद्धि के अनुकरण को संदर्भि त करता
है। यह शब्द किसी भी मशीन के संदर्भ में प्रयोग किया जा
सकता है जो सीखने और समस्या-समाधान जैसे मानव
सुपरकंप्यूटिंग के अनुप्रयोग (Applications of दिमाग से जुड़े कार्य कर सकती है।
Supercomputing):
AI के संभावित अनुप्रयोग (Potential
(a) खगोल भौतिकी की तरह अकादमिक अनुसंधान Application of AI):
(b) ड्रग्स और दवा की खोज
1. खनन कार्य
(c) मौसम पूर्वानुमान/आपदा प्रबंधन
2. सेल्फ ड्राइविं ग कार
(d) वैकल्पिक ऊर्जा स्रोतों का विकास
3. अंतरिक्ष क्षेत्र
(e) आणविक गतिविज्ञान
4. डिजिटल सहायक और स्मार्ट स्पीकर
(f) भविष्य में AI का अनुप्रयोग सुपरकंप्यूटिंग पर
5. रक्षा क्षेत्र
निर्भर करता है
6. चेहरे की पहचान
(g) अंतरिक्ष यान वायुगतिकी सुरक्षा में सुधार हेतु
अनुसंधान एवं विकास 7. कृषि:

(h) बिग डेटा माइनिं ग और एनालिटिक्स (a) बुवाई, कीट नियंत्रण आदि के लिए परामर्शी

(b) इमेज वर्गीकरण उपकरण

(c) रोगों की प्रारंभिक अवस्था में पहचान।


राष्ट्रीय सुपरकंप्यूटिंग मिशन (National
Supercomputing Mission): 8. स्वास्थ्य और चिकित्सा:

उद्दे श्य (Mission): (a) रोगियों का आकलन

(b) कृत्रिम सर्जरी सिमुलेटर


1. विभिन्न शैक्षणिक और अनुसंधान संस्थानों को जोड़ने
वाले 70 सुपर कंप्यूटर का क्लस्टर बनाना। (c) मानसिक स्वास्थ्य रोगियों की काउं सलिंग

2. मिशन को विज्ञान और प्रौद्योगिकी विभाग (DST) और (d) रेडियो सर्जरी


इले क्ट्रॉनिक्स और सूचना प्रौद्योगिकी विभाग (DeitY)
द्वारा संयुक्त रूप से कार्यान्वित और संचालित किया
जाएगा। AI की सीमाएं और चिं ताएं (Limitations &
3. निम्नलिखित क्षेत्रों में सुपरकंप्यूटिंग इन्फ्रा की बढ़ती
Concerns of AI):
मांगों को पूरा करने के लिए 1. मानव श्रम की मांग में कमी और बेरोजगारी।
(a) अकादमिक 2. नैतिक सरोकार।
(b) शोधकर्ताओ ं 3. कंप्यूटिंग इं फ्रा की उच्च लागत और कम उपलब्धता।
(c) MSMEs 4. AI आतंकवादियों, नक्सलियों आदि के हाथों में पड़ सकता
(d) स्टार्टअप है।

5. डाटा से संबंधित गोपनीयता और सुरक्षा संबंधी चिं ताएं ।

6. मनोवैज्ञानिक संकट और समाज का नैतिक पतन।


आर्टिफिशियल इं टे लिजेंस
(अध्याय 9 में विस्तार से आर्टि फिशियल इं टे लिजेंस और भारत (Artificial
चर्चा की गई) (Artificial Intelligence & India):

Intelligence) 1. केंद्रीय बजट 2018-19 में, सरकार ने नई और उभरती


प्रौद्योगिकियों में अनुसंधान एवं विकास के मार्गदर्शन हेतु
• आर्टिफिशियल इं टे लिजेंस (AI) मानव की तरह सोचने AI पर राष्ट्रीय कार्यक्रम बनाने के लिए नीति आयोग को
और उनके समान कार्य करने के लिए प्रोग्राम की गई कहा।

25

AKASH SINGH
2. नीति आयोग द्वारा तीन आयामी दृष्टिकोण: 5. स्पेक्ट्रम का उपयोग: स्पेक्ट्रम की हर एक बिट का
उपयोग।
(a) विभिन्न क्षेत्रों में AI परियोजनाओ ं का अन्वेषणात्मक
प्रमाण देना। 6. क्षमता: तेज ब्रॉडबैंड, IoT।

(b) एक जीवंत AI पारिस्थितिकी तंत्र के निर्माण के लिए


एक राष्ट्रीय रणनीति बनाना।
अनुप्रयोग (Applications):
(c) विभिन्न विशेषज्ञों और हितधारकों के साथ सहयोग
करना। 1. स्मार्ट इं फ्रा और स्मार्ट सिटी
3. ऐरावत (AIRAWAT): AI रिसर्च, एनालिटिक्स और 2. महत्वपूर्ण बुनियादी ढांचे की सुरक्षा
नॉले ज एसिमिले शन प्लेटफॉर्म एक क्लाउड कंप्यूटिंग
3. आपदा प्रबंधन और मौसम पूर्वानुमान
प्लेटफॉर्म है।
4. टे ली-मेडिसिन और टे ली-एजुकेशन
4. नीति आयोग द्वारा AI के लिए राष्ट्रीय रणनीति (2018)
का मसौदा: अनुसंधान, अपनाने और व्यावसायीकरण 5. मनोरंजन और मल्टीमीडिया
के लिए इसके दायरे की योजना बनाना। स्वास्थ्य, कृषि, 6. दैनिक जीवन में AI को बढ़ावा देना
शिक्षा, स्मार्ट सिटी, बुनियादी ढांचा और आपदा प्रबंधन में
7. IoT की सुविधा
AI का इस्तेमाल।
8. डिजिटल आर्थि क नीति पर OECD नीति के अनुसार,
5जी तकनीक से जीडीपी बढ़ेगी, रोजगार पैदा होगा और
अर्थव्यवस्था का डिजिटलीकरण होगा।
5जी प्रौद्योगिकी (5G 9. कृषि: संपूर्ण मूल्य श्रृंखला में सुधार, सटीक खेती, स्मार्ट
Technology) सिं चाई, बेहतर मृदा और फसल निगरानी।

10. निर्माण: सटीक निर्माण के लिए रोबोट का उपयोग,


• 5G 5वीं पीढ़ी का मोबाइल नेटवर्क है। यह 1G, 2G, 3G विशेष रूप से जहां मनुष्य सुरक्षा या सटीकता के कारण
और 4G नेटवर्क के बाद एक नया वैश्विक वायरले स काम नहीं कर सकते।
मानक है। 5G एक नए तरह का नेटवर्क है जिसे मशीनों,
11. हेल्थकेयर: प्रभावी टे लीमेडिसिन डिलीवरी, सर्जि कल
वस्तुओ ं और उपकरणों सहित लगभग सभी और हर चीज
रोबोट का टे ली-कंट्रोल।
को आभासी रूप से जोड़ने के लिए डिज़ाइन किया गया है।
12. ऊर्जा क्षेत्र: स्मार्ट ग्रिड, स्मार्ट मीटरिं ग, बढ़ती नवीकरणीय
• 5G वायरले स तकनीक हाई मल्टी-जीबीपीएस पीक
और भंडारण तकनीक के बाद ग्रिड के प्रबंधन के लिए लो
डेटा स्पीड, अल्ट्रा-लो ले टें सी अधिक विश्वसनीयता,
ले टे न्सी संचार महत्वपूर्ण है।
विशाल नेटवर्क क्षमता, बढ़ी हुई उपलब्धता, और अधिक
उपयोगकर्ताओ ं के लिए एक समान अनुभव प्रदान
करने के लिए है, उच्च प्रदर्शन और बेहतर दक्षता, नए
उपयोगकर्ता अनुभवों को सशक्त बनाती है और नए भारत में चुनौतियां (Challenges in India):
उद्योगों को जोड़ती है।
• लगभग 80% सेल साइटों में अक्षम तकनीकी सहायता
के कारण 4G जैसे मौजूदा स्पेक्ट्रमों में हाई ले टे न्सी और
क्षमता का सीमित उपयोग हो पा रहा है।
विशेषताएं (Features):
• मौजूदा फोन को 5G फोन से बदलना महंगा है। 5G
1. लो ले टे न्सी: यह बहुत कम देरी के साथ वास्तविक समय स्पेक्ट्रम की कम क्षेत्र में होने के कारण 5G मॉडेम अभी
डेटा पहुंच को सक्षम बनाती है। नोट: ले टे न्सी देरी मापने भी व्यापक पैमाने पर उपलब्ध नहीं हैं। लागत का बोझ
का एक उपाय है। किसी नेटवर्क में, ले टे न्सी कुछ डेटा को मोबाइल निर्माताओ ं और ग्राहकों दोनों पर पड़ता है।
नेटवर्क के दूसरे गंतव्य तक पहुंचने में लगने वाले समय • भारत में 5G इं फ्रास्ट्रक्चर बनाना भी महंगा है। इसके लिए
को मापती है। मौजूदा टावरों के उन्नयन, कार्यबल के कौशल विकास
2. डेटा उपलब्ध कराने में बेहतर लचीलापन। की आवश्यकता होगी। कंपनियों को पुराने उपकरणों
को चलाने के लिए 4G टावरों को बनाए रखने की भी
3. क्षमता: यातायात क्षमता और नेटवर्क दक्षता में 100 गुना
आवश्यकता होगी। इससे कंपनियों को अतिरिक्त लागत
वृद्धि।
आएगी।
4. गति: 1 gbps से अधिक।
• भारत में स्पेक्ट्रम की कीमतें बाकी जगहों की तुलना में

26
सबसे अधिक कीमतों में से एक हैं। • मानव, पक्षियों और वन्यजीवों के स्वास्थ्य पर विद्युत
चुम्बकीय क्षेत्रों के संभावित प्रभाव के खिलाफ आवाज
• 5G की तैनाती से भारत की डिजिटल अर्थव्यवस्था को
उठाई जा रही है।
बल मिले गा, ले किन इससे साइबर सुरक्षा का खतरा भी
बढ़ेगा।

27
अध्याय - 3

जैव प्रौद्योगिकी
(BIOTECHNOLOGY)

परिचय (Introduction) • दही, ब्रैड या वाईन सभी को बनाने में भी सूक्ष्मजीव


संबंधित प्रक्रियाओ ं का उपयोग होता है, इसीलिए यह भी
जैव प्रौद्योगिकी का ही एक रूप है। उदाहरण: पात्रे निषेचन
• जैव प्रौद्योगिकी का तात्पर्य जीवित जीवों या एं जाइमों
या विट्रो फर्टि लाइज़ेशन (IVF) से टे स्टट्यूब बेबी की
का उपयोग करके मनुष्यों के लिए उपयोगी उत्पादों और
तकनीक, 'जीन' का संक्ष्ले षण कर उपयोग करना, डीएनए
प्रक्रियाओ ं का उत्पादन करना है।
पर आधारित वैक्सीन बनाना या किसी खराब जीन को
• यह जीवित प्राणियों या एं जाइमों और उनकी जैविक सही करना ये सभी जैव प्रौद्योगिकी के ही उदाहरण है।
प्रक्रियाओ ं जैसे: बायोकेमिस्ट्री, माइक्रोबायोलॉजी
• अधिक गंभीर विषयों को समझने से पहले , जैव प्रौद्योगिकी
और जेनेटिक-इं जीनियरिं ग और सूक्ष्मजीवों का उचित
से जुड़े कुछ महत्वपूर्ण शब्दों को समझना भी बहुत ही
औद्योगिकीय प्रयोग कर मनुष्य के लिए सहायक एवं
जरूरी है।
लाभदायक उत्पादनिर्मि त करना है।

जैव प्रौद्योगिकी से जुड़े कुछ महत्वपूर्ण शब्द (Terms Related to Bio-Technology)


परिभाषाएँ स्पष्टीकरण
कोशिकाएँ (Cell) • यह सभी जीवित प्राणियों की आधारभूत संरचनात्मक एवं क्रियात्मक इकाई है।

• एक संपूर्ण कोशिका के अलावा कुछ भी स्व-अस्तित्व सुनिश्चित नही कर सकता।

• सभी जीवो में कोशिकाएँ पाई जाती है, जिनसे वे बने होते है। कुछ जीव मात्र एक ही
कोशिका से बने होते है, जिस कारण उन्हे, 'एककोशिकीय' जीव कहा जाता है, जबकि
अन्य जीव जो अनेक कोशिकाओ ं से बने होते है उन्हें 'बहुकोशिकीय' जीव कहा जाता है।

• प्रत्येक कोशिका के अंदर एक घनी झिल्ली (मेम्ब्रेन) से बनी संरचना होती है जिसे
न्यूक्लिअस/केन्द्रक कहते है। जिसमे क्रोमोजोम्स होते है, जिनमे डीएनए के कई जीन
का समावेश होता है।

• सुकेंद्रकी (Eukaryotic) कोशिकाएँ : इनके केन्द्रक झिल्ली से घिरे होते है।

• अकेंद्रिक (Prokaryotic) कोशिकाएँ : इनके केंद्रक झिल्ली से घिरे नही होते है।

• साईटोप्लाज्म: एक आंशिक रूप से द्रव्य पदार्थ होता है जो कोशिकाओ ं में पाया जाता
है। और यह अकेंद्रिक और सुकेंद्रक दोनों ही कोशिकाओ ं में पाए जाते है। साईटोप्लाज्म
पौधों एवं जानवरों दोनों की ही कोशिकाओ ं में कोशिकीय गतिविधियों का मुख्य स्थान
होता है। कोशिकाओ ं को 'जीवित अवस्था' में बनाए रखने के लिए इसमें कई रसायनिक
प्रतिक्रियाएं होती रहती है।
डीएनए (DNA) • डिऑक्सीराइबोन्यूक्लिक अम्ल (DNA) एक ऐसा अणु है जिसमें प्रत्येक प्रजाति की
जीवविज्ञान संबंधी वह जानकारियाँ मौजूद होती है, जो उन्हें अन्य प्रजातियों से अलग
बनाती है।

• डीएनए से जुड़ी जानकारियाँ किसी व्यस्क जीव से प्रजनन के दौरान उसके बच्चों
(वंशजों) में जाती है।

• डीएनए में चार तरह के रासायनिक तत्व मौजूद होते है, एडिनिन, थाइमिन, साइटोसिन
और गुआनाइन जिन्हें संक्षिप्त रूप में A,T,O एवं G अक्षरों से भी दर्शाया जाता है।

28
जीन (Gene) • वंशानुक्रम की कार्यात्मक इकाई।

• जीन एक नाभिकीय अम्लका अंश होता है जिसमें एक क्रियाशील उत्पाद बनाने के लिए
जरूरी सूचनाएँ होती है, सामान्यतः यह प्रोटीन होता है।
टीकाकरण (Vaccination) • टीकाकरण लोगो को किसी भी बीमारी से संक्रमित होने से बचाने का सबसे अच्छा और
सफल तरीका है।

• इसके जरिए शरीर की प्रतिरक्षा शक्ति (इम्यून सिस्टम) को बढ़ाकर किसी विशेष
संक्रमण के प्रति प्रतिरक्षा सुनिश्चित की जाती है, जिसके लिए शरीर की प्राकृतिक
प्रतिरक्षा प्रणाली का ही उपयोग किया जाता है।
अ नु व ां श ि क - अ भि य ां त्रि क ी • किसी जीव की विशिष्टताओ ं में किसी प्रकार के परिवर्तन लाने के लिए डीएनए में
(Genetic-Engineering) प्रत्यक्ष तौर पर बदलाव करना।

• इसमें एक जीव के जीनोम से डीएनए निकालकर अन्य जीव के डीएनए के साथ जोड़ा
जाता है।
जी न - चिकि त् सा / उ प च ा र • इसका उपयोग किसी रोगी में अनुवांशिक बीमारी का उपचार करने के लिए किसी नए
(Gene Therapy) डीएनए को डालने के लिए किया जाता है।

• नए डीएनए में आमतौर पर ऐसे क्रियाशील जीन मौजूद होते है जो शरीर में बीमारी का
कारण बनाने वाले परिवर्तनों को रोकते है।
जैवनैतिकता (Bioethics) • जैवनैतिकता, नीतिशास्त्र (ऐथिक्स), दर्शनशास्त्र और सामाजिक टिप्पणियों का
सम्मिश्रण है। यह जैविक विज्ञान और समाज पर उसके प्रभावों का अध्ययन करता है।
जीनोम रिसोर्स बैंक • यह जैविक पदार्थों जैसे: भ्रूण, ऊतक, रक्त से संबंधित पदार्थ, डीएनए आदि का फ्रोज़न
(Genome Resource भण्डार कोष होता है।
Bank)
• यह जैव-विविधता को बनाए रखने के लिए संरक्षण एवं संचय में प्रयोग होता है।
मानव जीनोम परियोज- • यह हमारी प्रजाति, यानि होमोसेपियन्स के सभी जीन (जिन्हें एक साथ जीनोम कहा
ना (Human Genome जाता है) को क्रमबद्ध करने और क्रमवार रूप से समझने से संबंधित है, इसीलिए इतिहास
Project) के रहस्यों को समझने में सबसे बड़ा कदम रही थी।

• इससे संबंधित सूचनाओ ं का उचित विश्ले षणात्मक अध्ययन करने के लिए इसके
रिकोर्ड तैयार करने, सूचनाओ ं को एकत्रित करने और इसके अध्ययन हेतु संसाधनों का
निर्माण करने के प्रयास भी किए गए थे।
बायोपाइरेसी (Biopiracy) • बायोपाइरेसी से तात्पर्य है किसी संगठन या बहुराष्ट्रीय कंपनियो, संबंधित देशो एवं
वंचित वर्ग द्वारा बिना किसी आधिकारिक स्वीकृति के जैविक संसाधनो को अवैध रूप
से प्राप्त करना।
बायोसेंसर्स (Biosensors) • यह बायोलॉजी सेंसर्स का ही अन्य संक्षिप्त रूप है।

• बायोसेंसर्स ऐसे यंत्र होते है, जिनसे किसी वस्तु में जैविक पदार्थों की मौजूदगी और
उनकी मात्रा के बारे में जाना जा सकता है। जैसे- बायोमोले क्यूल, कोई जैविक संरचना
या कोई सूक्ष्मजीव।

• अलग-अगल उपयोगों के आधार पर बायोसेंसर्स को इम्यूनोसेंसर्स, ओपट्रोड्स, रेज़ोनेंट


मीरर, केमिकल केनरी, बायोचिप, ग्लू कोमीटर और बायोकम्प्यूटर्स आदि नामों से भी
जाना जाता है।
बायोरिएक्टर (Bioreactor) • बायोरिएक्टर एक ऐसा मानवकृत यंत्र होता है जिसमें जैविक प्रतिक्रिया या बदलाव
होते है।

• यह जैविक जैविक प्रतिक्रिया के लिए बनी एक ऐसी बंद प्रणाली होती है, जिसमें
कोशिकाओ ं और ऊतकों का विकास होता है। यह प्रक्रिया वायवीय या अवायवीय
(Aerobic or Anaerobic) दोनों में से कोई भी हो सकती है।

29

AKASH SINGH
बायोरेमेडिएशन • बायोरेमेडिएशन, जैव प्रौद्योगिकी एक ऐसी शाखा है, जिसमें जीवित प्राणियों जैसे-
(Bioremediation) माइक्रोब्स, बैक्टीरिया (जीवाणु) का उपयोग कर मृदा, जल और अन्य पर्यावरणीय
घटकों से प्रदूषक तत्वों को हटाने या साफ करने का कार्य किया जाता है।

• अन्य शब्दों में, बायोरेमेडिएशन में जीवित प्राणियों मुख्यतः सूक्ष्मजीवों का उपयोग कर
प्रदूषक तत्वों को या तो कम किया जाता है या उनकी हानिकारक क्षमता को कम कर
दिया जाता है, जिससे वे पर्यावरण को ज्यादा नुकसान ना पहुंचाए।

• इसके लिए सबसे उपयुक्त जीव होते है- बैक्टीरिया (जीवाणु), फंगाए (कवक) या ऐसे
पौधे जिनमें प्राकृतिक रूप से ही ऐसे गुण होते है जो हानिकारक प्रदूषक तत्वों का प्रभाव
कम कर सकते है।
जैव क्षेत्रीयवाद • यह एक राजनीतिक, सांस्कृतिक, और पारिस्थितिक तंत्र या प्राकृतिक रूप से परिभाषित
(Bioregionalism) क्षेत्रों पर आधारित विचारों का समूह है, जिसे पर्यावरण क्षेत्रों के समान जैव-क्षेत्र कहा
जाता है।

• जैविक क्षेत्र ऐसी पर्यावरणीय या भौतिक विशेषताएं होती है जिनमें जल -विभाजक


सीमाएं , मृदा या क्षेत्र से जुड़ी कुछ विशेषताएँ शामिल होती है।

• जैव क्षेत्रीयवाद का विचार है कि इस तरह के जैव-क्षेत्र का निर्धारण सांस्कृतिक तौर पर


भी होता है इसीलिए यह क्षेत्रीय जनसंख्या, ज्ञान और समाधानों का समर्थन करता है।
जैविक नकल • बायोमाईमेटिक्स मानव सृजित ऐसे उत्पाद, प्रक्रियाएँ , यंत्र या प्रणालियाँ होती है जो
(Biomimetics/ प्रकृति की नकल करती है।
Biomimicry)
• बायोमाईमेटिक उपकरणों को बनाने की कला और विज्ञान को बायोमीमीक्री भी कहते
है। चूँकि यह जैविक प्रणालियों की नकल ही होती है।

• नैनोटे क्नोलॉजी रोबोटिक्स, आर्टिफिशल इं टे लिजंस (AI) चिकित्सा और अन्य क्षेत्र के


शोधकर्ताओ ं की इस क्षेत्र में एक विशेष रूचि है।

जैव प्रौद्योगिकी के विभिन्न (स) पौधे के आणविक चिह्न (मोलीक्यूलर मार्क र)-
प्रजनन में सहायक
प्रकार (Different Types of • उदाहरण- बीटी कॉटन

Biotechnology)
हरी जैव-प्रौद्योगिकी (Green Biotechnology) नीली जैव-प्रौद्योगिकी (Blue Biotechnology)

• इनका उपयोग मुख्यतः कृषि के लिए किया जाता है। • समुंद्री जीवो या साफ जल में रहने वाले जीवो में
कोशिकीय जैविक पद्धतियों का उपयोग करने से
• इस तकनीक का उपयोग कर पौधो को सूखे और कीट
संबंधित।
आदि के प्रति सहनशील बनाया जाता है।
• इसमें इन जीवों का उपयोग किया जाता है और इनके
• लक्ष्यः पोषक तत्वों की गुणवत्ता बढ़ाना, मात्रा में भी वृद्धि
डेरिवेटिव (व्युत्पन्न प्रजाति) का भी कई कारणों में
करना और पौधों/फसलों के लिए उत्पाद की आर्थि क
उपयोग किया जाता है जिसमें औद्योगिक उपयोग भी एक
स्थिति बेहतर करना।
कारण है।
• इसमें आर्थि क रूप से महत्वपूर्ण पौधों या फसलों की
• इसका सबसे अच्छा घटक समुंद्री पर्यावास में मिलने
प्रजाति में विदेशी जीन डालकर उन्हें बेहतर बनाया जाता
वाले नए क्रियाशील तत्वों को पहचानने और विकास की
है।
प्रक्रिया है।
• इसमें मुख्यतः तीन क्षेत्रों पर ध्यान दिया जाता है:
• उदाहरण के लिए-घावों पर पट्टी करने के लिए काइटोसन
(अ) पौधों/फसलों का टिशू कल्चर/पादप ऊतक संवर्धन (काइटोसन मूल रूप से एक प्रकार का चीनी जैसा पदार्थ
(ब) पौधों की जेनेटिक-इं जीनियरिं ग होता है जो समुंद्री झींगे या कीड़ो के बाहरी सख्त कंकाल
(शंख) से बनता है।) का उपयोग।

30
लाल जैव-प्रौद्योगिकी (Red Biotechnology)
जैव प्रौद्योगिकी के अनुप्रयोग
• लाल जैव-प्रौद्योगिकी (बायोफार्मा), जैव-प्रौद्योगिकी की
ही एक अलग शाखा है जो जैव-प्रौद्योगिकी के चिकित्सीय
(Applications of
या दवाइयों के अनु प्रयोगात्मक पहलू से संबंधित है। Biotechnology)
• इसका उपयोग वैक्सीन, एं टीबायोटिक्स, रिजेनेरटि
े व
थेरप
े ी, जीन थेरप
े ी, स्टैम सैल थेरप
े ी आदि में किया जाता • जैव प्रौद्योगिकी मुख्यतः बायोफार्मा स्यूटिकल्स और
है। जैविक उत्पादों के औद्योगिक स्तरीय उत्पादन को देखती
है। जिसमें जेनेटिक रूप से परिवर्ति त जीवाणु, कवक,
पादप और जंतु र उपयोग में लाए जाते है। इसके अनुप्रयोग
से संबंधित प्रमुख क्षेत्र है; चिकित्साशास्त्र, डायग्नोसिस,
सफेद जैव-प्रौद्योगिकी (White Biotechnology)
कृषि में जीएम फसलों के लिए, संसाधित भोजन पदार्थ,
• यह ओद्यौगिकीय जैव-प्रौद्योगिकी से संबंधित होती है और बायोरेमेडिएशन, अवशेष निष्पादन और ऊर्जा उत्पादन के
कई ओद्यौगिक प्रक्रियाओ ं में प्रयोग की जाती है। क्षेत्र। जैव प्रौद्योगिकी के तीन प्रमुख शोध क्षेत्र है:

• इसमें कम - संसाधनों से पूर्ण होने वाली प्रक्रियाओ ं एवं


» एक संशोधित जीव के रूप में एक उत्प्रेरक की भूमिका
उत्पादों को तैयार करने पर विशेष ध्यान दिया जाता है
निभाना। जो सामान्यतः जीवाणु/सूक्ष्मजीव या शुद्ध
और इसीलिए यह ऊर्जा-कुशल भी होता है।
एं जाइम ही होता है।
• उदाहरण:
» इं जीनियरिं ग के जरिए, केटे लिस्ट की उत्प्रेरक के अनुरूप
परिस्थिति बनाना।
» रासायनिक उत्पाद
» अनुप्रवाह प्रसंस्करण प्रौद्योगिकी (downstream
» नए प्रकार के प्लास्टिक/कपड़े का उत्पादन और डिजाइन
processing technologies) का उपयोग कर प्रोटीन/
» बायो ईधन के जैसे अन्य पर्यावरण सापेक्ष ऊर्जा स्रोतों की जैविक घटकों को शुद्ध करना।
खोज एवं पहचान।
• चलिए अब हम समझते है कि मनुष्य ने जैव प्रौद्योगिकी
का उपयोग किस प्रकार किया है, विशेषतः भोज्यपदार्थ
भूरी जैव-प्रौद्योगिकी (Grey Biotechnology) के उत्पादन और स्वास्थ्य आदि क्षेत्र में और जीवन की
गुणवत्ता को सुधारा है।
• यह एक पर्यावरणीय अनुप्रयोग है जो जैव-विविधता को
बनाए रखने पर केंद्रित है। यह सूक्ष्मजीवों का प्रयोग कर
प्रदूषक तत्वों को हटाने या कम करने के साथ-साथ
पौधों का उपयोग करना है जिसमें विभिन्न पदार्थों जैसे- कृषि में जैव प्रौद्योगिकी का
हाइड्रोकार्बन, हेवीमैट्ल्स को भी हटाया या अलग किया
जा सकें।
उपयोग (Application
• उदाहरण: कारलो स्थित एसएमई माइक्रोजेन बायोटै क, of Biotechnology In
चाईना में औद्योगिकीकरण से प्रदूषित भूमि को साफ
करने में सहयोग दे रही है।
Agriculture)
जैव ईंधन (Biofuels)

पीली जैव-प्रौद्योगिकी (Yellow Biotechnology) • इन जैव ईंधन उत्पादों की गुणवत्ता को सुधारने के लिए
और इनका अधिक से अधिक उत्पादन करने के लिए
• इसमें ऐसे कीट-पतंगों का उपयोग किया जाता है जो हरी जेनेटिक-इं जीनियरिं ग और एनज़ाईम अनुकूलन
(पौधों से संबंधित) जैव प्रौद्योगिकी या लाल (जानवरों से तकनीकों का उपयोग किया जाता है।
संबंधित) जैव प्रौद्योगिकी के अनुरूप होते है। इन कीटों के
• अधिक पैदावार देने वाली और ऊर्जा से परिपूर्ण फसलों
जीन में मौजूद क्रियाशील तत्वों का प्रयोग शोधों में और
का अधिक उत्पादन कर फसल-कटाई और यातायात
चिकित्सा एवं कृषि क्षेत्र के लिए दवाईयों के निर्माण मे भी
(प्राप्त ऊर्जा की प्रति इकाई पर) से जुड़े खर्चों को कम
किया जाता है।
किया जा सकता है। जिससे ईंधन उत्पादों का मूल्य बढ़ेगा।
• उदाहरण: किण्वन द्वारा वाइन, पनीर और बियर बनाना।

31
पादप और जंतु प्रजनन (Plant and Animal जैविक तनाव का सामना (Biotic Strain
Reproduction) Confrontation)

• प्रजनन के तरीके जैसे; पार परागण, ग्राफ्टिं ग और पार- • धरती का 20% से भी कम हिस्सा ही ऐसा है जो कृषि योग्य
प्रजनन के माध्यम से पादप और जंतु में गुण वृद्धि करना उपयुक्त है। ले किन कुछ फसलों को अनुवांशिक रूप से
एक बहुत ही लं बी और ज्यादा समय ले ने वाली प्रक्रिया है। इस प्रकार परिवर्ति त किया गया है कि वे गंभीर प्राकृतिक
परिवेश जैसे: ठं ड, सूखे और मृदा में लवण की अधिक
• जीन अभिव्यक्ति नियंत्रण तंत्र का उपयोग कर इसे संभव
मात्रा को भी सहन कर सकते है।
बनाया जा सकता है, जैसे: विशेष जीन प्रवर्तक और
प्रतिरूपण पद्धतियों का उपयोग करना। • सोडियम की मात्रा के संदर्भ में पौधों के जीन की जांच
से अधिक लवणता क्षेत्र में उगने वाले पौधो की उपज में
• (GMOs) जेनेटिक रूप से परिवर्ति त जीवों के संबंध में
वृद्धि हुई है।
किसी भी विवाद को बढ़ाए बिना मार्क र सहायक पद्धति
का उपयोग कर पशुओ ं में निर्देशित प्रजनन की गुणवत्ता • सूचनाओ ं का अप- या डाउन-विनियमन आमतौर पर
को सुधारा जा सकता है। पौधों/फसलों में सूखे के प्रति सहनशीलता को बदलने के
लिए इस्तेमाल की जाने वाली विधि है।

• मक्का और सफेद-सरसों ऐसी ही फसलें है जो कम


कीटनाशकों के प्रति सहनशील फसलें (Pesticide सुविधा स्थितियों में में भी उगाई जा सकती है इसका
Resistant Crops) परीक्षण केलिफोर्नि या और कोलोराडो के खेतों में किया
गया है।
• ये पौधे कीटनाशकों के प्रति सहनशील होते है जिससे
किसान फ़सल खराब करने वाले कई कीट-पतंगों को
मारकर अपनी फ़सलों की उपज बेहतर कर सकते है।
मजबूत रेशम बनाना (Manufacturing Power
• उदाहरण: राउं डअप-रेडी तकनीक, मोनसेंटो द्वारा
Fibers)
शहरीकृत।
• मकड़ी से मिलने वाला रेशम का धागा अभी तक मानव
• पहली बार 1998 में जीएम सोयाबीन के रूप में पेश किया
की जानकारी में आया सबसे मजबूत धागा है। (इसे
गया, राउं डअप-रेडी पौधे शाकनाशी ग्लाइफोसेट से
बुलेटप्रूफ कपड़ा बनाने में उपयोग किया जाता है। यह
अप्रभावित रहते हैं, जिसे प्रचुर मात्रा में लगाया जा सकता
स्टील से भी ज्यादा मजबूत है।)
है।
• अगस्त 2000, में कनाडा की एक कंपनी नेक्सीया
• इसके फायदों में निम्न है; समय की बचत, जुताई के खर्च
(Nexia) ने ट्रांसजेनिक बकरियों के विषय में सूचना
में बचत, जिससे खरपतवार की मात्रा में भी कमी आएगी
साझा की इनके दूध में मकड़ी के रेशम के प्रोटीन मौजूद
साथ ही अलग-अगल कीटनाशकों आदि का भी प्रयोग
थे। इससे इस रेशम की अधिक उत्पाद की समस्या तो
किया जा सकता है।
हल हुई परंतु इस प्रयोग को जब स्थापित करना पड़ा तब
वैज्ञानिक यह नही समझ पाए कि इससे उस प्रकार का
कुछ महत्वपूर्ण तथ्य (Quick Facts):
धागा कैसे बुना जाए जैसे मकड़ियां बुनती है।
बी टी कपास, भारत की इकलौती जेनेटिक रूप से परिव-
र्ति त फसल है। जो व्यवसायिक रूप से मान्य है।

गोल्डन राइस में बीटा-कैरोटीन होता है, जो हमारे शरीर टीका (Vaccines)
में विटामिन ए के निर्माण का अग्रदूत होता है। यह नाम,
बीटा-कैरोटीन की अधिक अभिव्यक्ति के कारण ट्रांसजे- • वैक्सीन/टीका एक जैविक रूप से तैयार दवाई होती है
निक अनाज के रंग से आया है, जो गाजर को उनका नारंगी जो किसी विशेष संक्रामक रोग से हमारा प्रतिरक्षा तंत्र
रंग देता है। मजबूत कर हमे सुरक्षा प्रदान करती है। ओरल-वैक्सीन
(मुँह के जरिए दी जाने वाली) अविकसित देशों में लोगों
को रोगों के संक्रमण से बचाने का सबसे अच्छा तरीका है।

पोषण – प्रतिपूर्ति (Nutrient Supplementation) • जीएम फसलों में हस्तांतरणीय रोगजनकों के जरिए
ऐ ंटीजेनिक प्रोटीन डालने से रोग प्रतिरोधक क्षमता बढ़
• स्वास्थ्य में सुधार के लिए, वैज्ञानिक आनुवंशिक रूप जाती है जिससे ऐसे रोगों से निपटने में मदद मिलती है।
से विखंडित भोज्य पदार्थों को बना रहे हैं जिनमें रोग या
• उदाहरण: तंबाकू के पौधे का उपयोग कर एं टी लोम्फोमा
भुखमरी से लड़ने में मदद करने के लिए ज्ञात पोषक तत्व
वैक्सीन बनाई गई है। जिसमें नकली हानिकारक बी-
होते हैं। उदाहरण - गोल्डन राइस।
कोशिकाओ ं से लिए आरएनए का भी उपयोग होता है। यह

32
कैंसर के इलाज में कारगर है। इसमें किसी भी प्रकार के रसायन या कृत्रिम पदार्थ का
उपयोग नहीं किया जाता।

• ये प्रोटीन होते है (जिसमें एं टीबाॅडीज़ भी शामिल है),


एं टीबायोटिक दवाएँ (Antibiotics) न्यूक्लिक अम्ल (DNA), RNA आदि जिसका चिकित्सा
या फिर विवो नैदानिक में उपयोग होता है। इसका उत्पादन
• मानव और जंतु दोनों के लिए एं टीबायोटिक दवाएँ बनाने
प्रत्यक्ष तौर पर मूल जैविक स्त्रोत से किया जाता है। शरीर
में पौधों का उपयोग होता है।
में लक्षित होने पर, वे रोगों के छिपे हुए तंत्र पर हमला
• खाद्य भंडार में एं टीबायोटिक प्रोटीन डालना, जो सीधे करते हैं और बिना किसी दुष्प्रभाव के उन्हें नष्ट कर देते हैं।
जानवरों को खिलाया जाता है, पारंपरिक एं टीबायोटिक
उत्पादन की तुलना में कम खर्चीला है।

• हालांकि, यह अभ्यास कई बायोएथिक्स मुद्दों को उठाता जीन थैरप


े ी (Gene Therapy)
है और संभवतः एं टीबायोटिक दवाओ ं के अनावश्यक
• जीन थेरप
े ी जैव प्रौद्योगिकी की एक और तकनीक
उपयोग एं टीबायोटिक-प्रतिरोधी जीवाणु तनाव के
है जिसका उपयोग कैंसर और पार्किंसंस रोग जैसी
विस्तार को प्रोत्साहित कर सकते हैं।
बीमारियों के निदान के लिए किया जाता है।

• इसका तरीका ऐसा है कि अगर शरीर में मौजूद जीन पर


फसल-कैफेटे रिया (Crop Cafeteria) हमला होता है तो यह या तो खराब कोशिकाओ ं को हटा
देता है या फिर उन्हें बदल देता है।
• फसल-कैफेटे रिया में एक क्षेत्र-विशेष के लिए उपयुक्त
• कुछ स्थितियों में, स्वस्थ जीन, जेनेटिक सूचना को सही
सभी तरह की फसलों की विविधता एक ही खेत में लगाई
करता है, जिससे जीन पुनः शरीर को स्वस्थ करने में लग
जाती है। जिससे अपने क्षेत्र में उपजाई जा सकने वाली
जाते है।
फसलों के कई विकल्प किसानों को पता लग जाते है।
जिसमे से अपनी जरूरत के अनुसार फसल चुनकर
किसान उसे उगा सकते है।
जेनेटिक जाँच (Genetic Testing)
• कृषक समाज में तकनीकी के प्रसार करने वालों के लिए
यह काफी सुविधाजनक है। • जेनेटिक जाँच एक ऐसी चिकित्सा जाँच होती है जो
क्रोमोजोम्स में, जीन और प्रोटीन में परिवर्तनों की पहचान
करती है।
फूल (Flowers) • जेनेटिक जाँच में डीएनए की जाँच कर जीनोम में
परिवर्तन कर क्रम तय किया जाता है।
• जैव प्रौद्योगिकी मात्र पौधो के रोगों और कीट-पतंगों से
ही रक्षा या फसल की गुणवत्ता ही सुनिश्चित नही करती • इसके जरिए अपराधियों की भी पहचान की जाती है और
इसके और भी कई फायदे है। साथ ही बच्चों के माता-पिता का भी पता लगाया जा
सकता है।
• फूलों के रंग, गंध, आकार और अन्य विशेषताओ ं में सुधार
के लिए जीन पहचान और स्थानांतरण तकनीक जैसे फार्माकोजेनोमिक्स (Pharmacogenomics)
कुछ सरल सौंदर्य अनुप्रयोग हैं।
• इसमें यह समझा जाता है कि कैसे जीन किसी व्यक्ति के
शरीर में दवाओ ं के प्रति क्रिया करते है।

चिकित्सा में जैव प्रौद्योगिकी • यह फार्मास्यूटिकल और जीनोमिक्स का मिश्रण है।

का अनुप्रयोग (Application
of Biotechnology in
Medicine)
बायोफार्मास्युटिकल (Biopharmaceuticals)

• बायोफार्मास्युटिकल एक प्रकार की दवाएँ होती है जिन्हें


जैव प्रौद्योगिकी का उपयोग कर बनाया जाता है ले किन

33
खाद्य प्रसंस्करण में जैव भोज्य पदार्थों का फोर्टि फिकेशन (Food
Fortification)
प्रौद्योगिकी का अनुप्रयोग • फूड फोर्टिफिकेशन एक पद्धति है जो वैज्ञानिक रूप
(Application of से सिद्ध, लागत प्रभावी, मापनीय और स्थायी वैश्विक
हस्तक्षेप पर आधारित है जिससे भोज्य पदार्थों में सूक्ष्म
Biotechnology in Food पोषक तत्वों की कमी को पूरा किया जा सकता है।

Processing) • कभी-कभी यह पद्धति पूर्ण रुप से व्यवसायिक हो जाती


है, इसके अलावा इसे जन स्वास्थ्य नीतियों के अंतर्गत
• खाद्य प्रसंस्करण एक ऐसी प्रक्रिया है जिसके द्वारा गैर- कुपोषण से प्रभावित या कम पोषकतत्वों का भोजन
स्वादिष्ट और आसानी से खराब होने वाले कच्चे माल करने वाले लोगों में कृत्रिम माध्यमों से पोषण की कमी
को खाद्य और पीने योग्य खाद्य पदार्थों और पेय पदार्थों को पूरा करने की कोशिश की जाती है।
में परिवर्ति त किया जाता है, जिनकी शेल्फ लाइफ लं बी • किसी क्षेत्र विशेष की प्रमुख फसलों में भी वहाँ की मृदा या
होती है। वह विधि जिसके द्वारा सूक्ष्मजीव जीवों और उनके अन्य किसी कारण से पोषण-तत्वों की कमी हो सकती है।
व्युत्पन्नों का उपयोग खाद्य पदार्थों की खाद्य क्षमता और
• इस माध्यम को अपनाकर इन मुख्य फसलों और मसालों
शेल्फ जीवन को बढ़ाने के लिए किया जाता है, किण्वन
में पोषण-तत्वों की मात्रा बढ़ाकर इस समस्या का निदान
के रूप में जाना जाता है।
पाया जा सकता है।

प्रोटीन इं जीनियरिं ग (Protein Engineering)

• प्रोटीन इं जीनियरिं ग में जैव प्रौद्योगिकी का एक बहुत बड़ा


पर्यावरण में जैव प्रौद्योगिकी
एवं महत्वपूर्ण योगदान होता है। का अनुप्रयोग (Application
• बेहतर किण्वन के लिए जिम्मेदार सूक्ष्मजीवों के उपयोगी
एं जाइम, टैं कों आदि में जीवों की खेती करके बड़े पैमाने
of Biotechnology In
पर व्यावसायिक रूप से उत्पादित किए जाते हैं। Environment)
• पर्यावरणीय समस्याओ ं और पारिस्थितिक तंत्र को
किण्वन प्रक्रिया (Fermentation Process) हल करने के लिए जैव प्रौद्योगिकी के उपयोग को
पर्यावरण जैव प्रौद्योगिकी के रूप में जाना जाता है।
• यह विटामिन के जैवसंश्लेषण के माध्यम से खाद्य पदार्थों इसका उपयोग प्राकृतिक पर्यावरण को समझने और
के आहार मूल्य में सुधार करता है, आवश्यक अमीनो इसका अध्ययन करने में होता है। इं टरनेशनल सोसाइटी
एसिड और प्रोटीन को बढ़ाता है, प्रोटीन और फाइबर की फॉर एनवायरमेंटल बायोटे क्नोलॉजी के अनुसार,
पाचनशक्ति में सुधार करता है, सूक्ष्म पोषक तत्वों की जैव पर्यावरण जैव प्रौद्योगिकी को "एक ऐसे वातावरण के
उपलब्धता को बढ़ाता है और पोषण विरोधी कारकों को रूप में परिभाषित किया गया है जो जैविक प्रणालियों
कम करता है। को विकसित करने, कुशलतापूर्वक उपयोग करने और
विनियमित करने में मदद करता है और पर्यावरण को
FSSAI ने भारत में सूक्ष्म पोषक तत्वों के कुपोषण के उच्च प्रदूषण या भूमि, वायु और जल के प्रदूषण से रोकता है।"
बोझ को कम करने के लिए खाद्य सुरक्षा और मानक (खाद्य पर्यावणीय जैव प्रौद्योगिकी के कुछ मुख्य उपयोग एवं
पदार्थों का फोर्टिफिकेशन) विनियम, 2016 का संचालन प्रकार निम्न है-
किया।, उदाहरण:

• गेहूं का आटा और चावल (आयरन, विटामिन बी 12 और


जैवसूचक (Biomarker)
फोलिक एसिड के साथ)

• दूध एवं खाद्य तेल (विटामिन ए और डी) • पर्यावरणीय जैव प्रौद्योगिकी का उपयोग करके ही
रसायनों को प्रतिक्रिया मिलती है। इसके माध्यम से
• डबल फोर्टि फाईड नमक (आयोडीन और आयरन के
यह समझने में सहायता मिलती है कि किस हद तक
साथ) "+F" चिह्न से इसकी (नमक की) पहचान की
नुकसान हुआ है और यह जानने में भी कि प्रदूषण और
जाती है।
जहरीले रसायनों का कितना प्रभाव पड़ा है। इन पद्धतियों
के प्राथमिक प्रयोग से ही हम तेल एवं उनके उद्गम स्त्रोतों

34

AKASH SINGH
का संबंध समझ पाते है। • यह जीवों और इं जीनियरों को यह समझने में मद्द करता
है कि कैसे पर्यावरण स्वच्छ रखा जा सके और कैसे
पर्यावरण में हो रहे परिवर्तनों के अनुरूप स्वयं को ढ़ाला
जैव-ऊर्जा (Bioenergy) जा सके।

• पर्यावरणीय जैवप्रौद्योगिकी के लाभ यह है कि ये हमें


• बायोगैस, बायोमास, ईंधन और हाइड्रोजन का समेकित
हानिकारक प्रदूषक तत्वों से बचने और पर्यावरण एवं
रूप बायोएनर्जी या जैव-ऊर्जा कहलाता है। पर्यावरणीय
प्राकृतिक संसाधनों पर पड़ने वाले दुष्प्रभावों को समझने
जैव प्रौद्योगिकी के उपयोग से जुड़े प्रमुख क्षेत्र- औद्योगिक,
में मदद करता है।
गृह-संबंधी और अंतरिक्ष-क्षेत्र है। वर्तमान आवश्यकता
के अनुसार इन ईंधनों से स्वच्छ ऊर्जा की स्थिति का • समाज का विकास इस प्रकार किया जाना चाहिए कि यह
निष्कर्ष निकाला जाता है और स्वच्छ ऊर्जा प्राप्त करने पर्यावरण को बनाए रखने में मदद कर सकें।
के वैकल्पिक तरीके समय की मांग है। हरित ऊर्जा के • पर्यावरणीय जैवप्रौद्योगिकी प्रदूषक तत्वों को कम करने
अग्रणी उदाहरणों में से एक जैविक और बायोमास कचरे में योगदान देती है।
से एकत्रित अपशिष्ट है; और स्वच्छ ऊर्जा का उत्पादन
• वैज्ञानिकों और पर्यावरणविदों के लिए कचरे को पुन:
करने के लिए संसाधित किया गया। प्रत्येक देश में
उपयोग योग्य उत्पादों में बदलने के तरीके खोजना
बायोमास ऊर्जा की आपूर्ति एक गहन महत्व का विषय
लाभदायक बनता जा रहा है।
बन चुका है।

क्रिस्पर-कस 9 तकनीक (CRISPR-Cas9


बायोरेमेडिएशन/जैव-उपचार (Bioremediation)
Technology)
• बायोरेमेडिएशन, बायोटे क्नोलॉजी की एक ऐसी ब्राँच है जो
• यह एक ऐसी तकनीक है जो आनुवंशिकीविदों और
भूमि, जल एवं अन्य पर्यावरणीय तत्वों में जहरीले पदार्थों
चिकित्सा शोधकर्ताओ ं को डीएनए अनुक्रम अनुभागों को
एवं प्रदूषक तत्वों का अध्ययन करती है। यह सूक्ष्मजीवों
हटाने, जोड़ने या बदलने के द्वारा जीनोम के कुछ हिस्सों
जैसे माइक्रोब और बैक्टीरिया का भी अध्ययन करती है।
को संपादित करने में सक्षम बनाती है।
बायोरेमेडिएशन का उपयोग कर तले में मौजूद गंदे जल
एवं जल में फैले तेल को साफ करने में किया जाता है। • इस समय जेनेटिक मेनीप्यूलेशन (जीन में परिवर्तन
उदाहरण: ऑयलज़ेपर (जिसे टे री-टीइआरआई, ने तैयार करना) का सबसे सटीक, अच्छा और आसान तरीका है।
किया) हाईड्रोकार्बन घटकों पर जीवित रहता है, जो कच्चे
तेल में मौजूद होते हैं और एक प्रकार की तेलीय कीचड़
(एक ऐसा हानिकारक हाईड्रोकार्बन मलबा जो ऑयल यह कैसे कार्य करता है ? (How It Works ?)
रिफाइनरी से निकलता है) को नुक्सान न पहुंचाने वाले
• क्रिस्पर-कस 9 तकनीक एक प्रकार से 'कट एवं पेस्ट'
CO2 और जल के रूप में परिवर्ति त कर देते हैं।
पद्धति पर कार्य करती है, यह उन डीएनए स्ट्रैं ड पर कार्य
करती है जिनमें जेनेटिक सूचनाएँ होती है या जीन संबंधी
जानकारियाँ होती है।

• इस तकनीक के माध्यम से डीएनए स्ट्रें ड पर जीन के उन


बायोट्रांसफोर्मेशन (Biotransformation) हिस्सों का पता लगाया जाता है जहाँ बदलाव करने है और
फिर, Cas9 प्रोटीन का उपयोग कर, जो किसी कैंची की
• पर्यावरण के जीव-विज्ञान में परिवर्तन, जिसमें गंभीर
तरह कार्य करता है, इस हिस्से को डीएनए स्ट्रैं ड से काट
प्रकृति वाले घटकों को साधारण अहानिकारक घटकों में
दिया जाता है। टू टे हुए डीएनए स्ट्रैं ड में प्राकृतिक रूप से
परिवर्ति त कर दिया जाता है, और इसी को जैव-परिवर्तन
पुनः स्वस्थ होने की शक्ति होती है।
प्रक्रिया कहते है। इसका उपयोग मुख्यतः निर्माण क्षेत्र
में होता है जहाँ जहरीले पदार्थों को परिवर्ति त कर बाय- • इस प्रकार प्राकृतिक रूप से स्वयं स्वस्थ होने की प्रक्रिया
प्रोडक्ट में बदल दिया जाता है। (ऑटो-रिपेयर) में वैज्ञानिक हस्तक्षेप करते है और जरूरत
अनुसार जेनेटिक कोड डालकर इनमें सुधार करते है। इस
प्रकार पुनः स्वस्थ हुए डीएनए स्ट्रैं ड में जीन सूचनाएँ डाल
दी जाती है।
लाभ (Benefits)

• यह हमारे पर्यावरण को आने वाली पीढ़ी के लिए सुरक्षित


और साफ रखने में मद्द करता है।

35
• जैव सुरक्षा पर कार्टाजेना प्रोटोकॉल के अनुच्छेद 20 में
जीवित संशोधित जीवों (LMOs) पर वैज्ञानिक, तकनीकी,
पर्यावरण और कानूनी जानकारी के आदान-प्रदान की
सुविधा के लिए भारत जैव सुरक्षा समाशोधन गृह की
आवश्यकता है।

• BCH एक सूचना विनिमय तंत्र है जिसे पार्टि यों को इसके


प्रावधानों को लागू करने में सहायता करने और LMOs
के साथ जानकारी और अनुभव साझा करने की सुविधा
प्रदान करने के लिए स्थापित किया गया है।

जैव सुरक्षा पर कार्टाजेना प्रोटोकॉल (The


Cartagena Protocol On Biosafety)

• जैव सुरक्षा पर कार्टाजेना प्रोटोकॉल एक अंतर्राष्ट्रीय


समझौता है। इसका उद्देश्य आधुनिक जैव प्रौद्योगिकी के
परिणामस्वरूप जीवित संशोधित जीवों (LMO) के सुरक्षित
संचालन, परिवहन और उपयोग को सुनिश्चित करना है।
इसका जैविक विविधता और मानव स्वास्थ्य पर प्रतिकूल
प्रभाव पड़ सकता है।

• इसे 2000 में अपनाया गया और 2003 से यह प्रभाव में


आया।

चित्र 3.1 : क्रिस्पर-Cas 9


जीएम सरसों (GM Mustard, DMH-11)

• डीएमएच-ll पारंपरिक सरसों के मुकाबले 30% ज्यादा


जेनेटिक इं जीनियरिं ग मूल्यांकन समिति
पैदावार देती है।
(Genetic Engineering Appraisal
Committee, GEAC) • इससे खाद्य तेलों की पैदावार एवं उत्पाद में वृद्धि होगी
जिससे देश के खाद्य तेल आयात बिल में कमी आएगी।
• जेनेटिक इं जीनियरिं ग मूल्यांकन समिति पर्यावरण, वन
एवं जलवायु परिवर्तन मंत्रालय (MoEF&CC) के अंतर्गत • सर्वोच्च न्यायालय ने जीएम सरसों को तैयार करने की
कार्य करती है। अनुमति पर रोक लगा रखी है।

• नियम, 1989 के अनुसार, यह पर्यावरण के दृष्टिकोण • जीएम सरसों कीट प्रतिरोधी है।
से अनुसंधान और औद्योगिक उत्पादन में खतरनाक
सूक्ष्मजीवों और पुनः संयोजकों के बड़े पैमाने पर उपयोग
से संबंधित गतिविधियों के मूल्यांकन के लिए जिम्मेदार प्रमुख फसलों का बायोफोर्टि फिकेशन
है। (Biofortification of Staple Crops)
• प्रायोगिक क्षेत्र परीक्षणों सहित पर्यावरण में आनुवंशिक • फोर्टिफिकेशन खाद्य आपूर्ति की पोषण गुणवत्ता में सुधार
रूप से इं जीनियर (GE) जीवों और उत्पादों की रिहाई से करने और न्यूनतम स्वास्थ्य जोखिम के साथ सार्वजनिक
संबंधित प्रस्तावों के मूल्यांकन के लिए भी समिति लाभ प्रदान करने के लिए भोजन में आवश्यक सूक्ष्म
जिम्मेदार है। पोषक तत्व, यानी विटामिन और खनिज पदार्थों (ट्रे स
तत्वों सहित) को बढ़ाने का प्रयास करता है।

• बायोफोर्टिफिकेशन के माध्यम से कृषि-विज्ञान संबंधी


भारत जैव सुरक्षा समाशोधन गृह (India पद्धतियों, पारंपरिक फ़सल प्रजनन तरीकों एवं आधुनिक
Biosafety Clearing House, BCH) जैव-प्रौद्योगिक (बायोटे क्नोलॉजी) के माध्यम से खाद्य
• भारत जैव सुरक्षा पर कार्टाजेना प्रोटोकॉल का एक फसलों की पोषणात्मक गुणवत्ता बढ़ाई व सुधारी जा
हस्ताक्षरकर्ता है और 2003 में इसकी पुष्टि की थी। सकती है।

36
• हालाँकि बायोफोर्टिफिकेशन, पारंपरिक फोर्टिफिकेशन अर्थ बायोजेनोम परियोजना (The Earth Bio-
जैसा नही होता। बायोफोर्टिफिकेशन का उद्देश्य फसलों Genome Project)
के प्रसंस्करण के दौरान मानवीय साधनों के बजाय पौधों
• एक अंतर्राष्ट्रीय सहभागिता परियोजना है जो धरती
की वृद्धि के दौरान फसलों में पोषक तत्वों के स्तर को
पर मौजूद यूकेरियोटिक जैव-विविधता के जीनोम को
बढ़ाना है।
प्रत्येक 10 वर्ष में डिजिटल बनाने और इनका अनुक्रमण
• इसीलिए, इसका उपयोग उन स्थानों व स्थितियों में किया करने का काम करती है।
जा सकता है जहाँ पारंपरिक फोर्टिफिकेशन पद्धतियां या
• यह लगभग 1.5 मिलियन प्रजातियों के जीनोम की मैपिंग
तो उपलब्ध नहीं है या उनके लिए इनका कार्यान्वयन
(सूची बनाना) करेंगे: उदाहरण- मानव के ज्ञान में आने
अत्यंत कठिन है।
वाले सभी जटिल जीवन स्वरूप।
• बायोफोर्टिफिकेशन के कुछ उदाहरण:
• यह परियोजना, मानव जीनोम पहल से प्रेरित होकर शुरू
(a) आयरन- चावलों, दालों, शकरकंद, कसावा और की गई। यह एक ओपन-सोर्स डीएनए डेटाबेस था जो
लै ग्यूम में 2003 में बंद कर दिया गया।
(b) जींक- गेंहू, चावल, दालों, शकरकंद और मकई में • यह वैज्ञानिक शोधों के लिए एक मंच प्रदान करता है और
(c) प्रोविटामिन कैरोटीनोईड बायोफोर्टिफिकेशन- पर्यावरण एवं संरक्षण संबंधी लक्ष्यों की पूर्ति में मद्द करता
शकरकंद, मक्का और कसावा में है।

(d) कसावा और सॉरघुम (चारा) में- अमिनो एसिड और


प्रोटीन का फोर्टिफिकेशन करना।

DNA प्रोफाइलिंग (DNA


मानव- ह्यूमन एटलस पहल (Manav – Human Profiling)
Atlas Initiative)
परिभाषा (Definition)
• एक ऐसी योजना है जो नागरिक-विज्ञान आधारित
• DNA प्रोफाइलिंग या DNA फिंगरप्रिंटिंग एक ऐसी
मानव एटलस बनाने को लक्ष्य करती है, इसमें माइक्रो
अवधारणा है जिसमें बायोमेट्रिक डेटाबेस संकल्पना है
से ले कर मैक्रो सभी स्तरों से जीवन-विज्ञान-साहित्य
जिसमें व्यक्तियों की DNA सूचनाएँ शामिल होती है।
और सार्वजनिक जानकारियाँ इक्ट्ठा कर मानव एटलस
बनाने का प्रयास है।

• यह जैव प्रोद्योगिकी विभाग द्वारा बनाई गई योजना है।


डीएनए प्रोफाइल बनाना (Establishing DNA
• यह एक ऐसी परियोजना है जिसमें मानव शरीर के प्रत्येक Profiles)
ऊतक की मैपिंग (स्थान निर्धारित करना) शामिल हैं।
• किसी भी व्यक्ति के जेनेटिक मेकअप (संरचना) का
• इसमें मानव शरीर में मौजूद सभी ऊतकों के स्तर पर आधार डीएनए ही होता है। मानव जाति में लगभग 99.9%
प्राकृतिक या रोगग्रस्त स्तर पर मानव शरीर का विस्तृत डीएनए सभी व्यक्तियों में समरूप होता है।
खाका बनाने का प्रयास है।
• बाकि बचा डीएनए परिवर्तनशील क्षेत्र (variable
• इसके उद्देश्य: region) 0.1 फीसदी होता है जिसे अग्रानुक्रम दोहराव की
(a) मानव शरीर संरचना में बेहतर जीव-वैज्ञानिक परिवर्तनीय संख्या (VNTR) भी कहा जाता है। जो प्रत्येक
समझ व्यक्ति के लिए अद्वितीय होता है और अलग पहचान
दिलवाता है, और यहीं एक डीएनए में व्यक्ति की डीएनए
(b) विभिन्न बिमारियों के संबंध में ऊतकों और
प्रोफाइल बनाता है।
कोशिकाओ ं की भूमिका समझना
• किसी व्यक्ति का डीएनए प्रोफाइल न्यूक्लियोटाइड बेस
(c) प्रेडक्टिव कम्प्यूटरिं ग के जरिए रोगों से संबंधित
जोड़े के क्रम को दोहराए जाने की संख्या के संदर्भ में
मॉडल का विकास करना
भिन्न होता है।
(d) दवाईयों की खोज करना।
• इस प्रकार, इन चर क्षेत्रों में इन बेस-जोड़े के अनुक्रमों को
दोहराए जाने की संख्या की गणना करके, एक व्यक्ति का
डीएनए प्रोफाइल स्थापित किया जाता है।

37
डीएनए फिंगरप्रिंटिंग तकनीक (Technique For इसके प्रयोग (Application)
DNA Fingerprinting)
डीएनए फिंगरप्रिंट के विभिन्न प्रयोगों एवं उपयोगों को निम्न
I. डीएनए को नमूने या जैविक इकाई से निकाला जाता है- के माध्यम से समझा जा सकता है:
सफेद रक्त कोशिकाओ ं या शुक्राणुजोआ या फिर बालों
की फोलिकल कोशिकाओ ं में से। I. व्यैक्तिकता: डीएनए फिंगरप्रिंट के माध्यम से एक व्यक्ति
को दूसरे व्यक्ति से अलग कर पहचाना जा सकता है,
II. डीएनए अणु सबसे पहले एं जाइम, प्रतिबंध एं डोन्यूक्लीज
सिवाय एक-जैसे दिखने वाले जुड़वा लोगों या बच्चों के।
(जिसे एक प्रकार का 'रसायनिक चाकू' भी कहा जाता है)
की मद्द से टू टते है, जिससे यह विभिन्न टु कड़ो में विभाजित II. अनुवांशिक बीमारियाँ: इस तकनीक की मदद से
हो जाते है। डीएनए के टु कड़ो में भी VNTR भरा होता है। उन जीन्स की पहचान की जा सकती है, जिनमें कोई
अनुवांशिक बीमारी हो या उससे संबंधित हो।
III. जेल वैद्युतकणसंचलन द्वारा टु कड़ों को आकार के
अनुसार अलग किया जाता है| III. मानव वंशावली: विभिन्न प्रकार के डीएनए का अध्ययन
करके ही मानव वंशावली से जुड़ी जानकारियाँ प्राप्त होती
IV. पीसीआर तकनीक की सहायता से इन वीएनटीआर
है।
युक्त विशेष आकार वाले टु कड़ो को गुना किया जाता है
और यह बढ़ जाते है। फिर इन्हे एक स्ट्रैं ड वाले डीएनए में IV. पितृत्व/मातृत्व संबंधी संदेह: डीएनए फिंगरप्रिंटिंग की
परिवर्ति त करने के लिए क्षारीय रसायन की मद्द से तोड़ मद्द से ही असली पिता, माता या बच्चे की पहचान की जा
दिया जाता है। सकती है।

V. इसके पश्चात एक स्ट्रैं ड डीएनए में विभाजित टु कड़ों को V. समाजशास्त्र: इसकी मद्द से विभिन्न प्रजातीय समूहों
एक नायलोन मेम्ब्रेन में भेज दिया जाता है। की पहचान की जा सकती हो, उनके उद्भव, एतिहासिक
पलायन एवं आक्रमण आदि कई तथ्यों की जानकारियाँ
VI. एक X- रे फिल्म के माध्यम से नायलोन मेम्ब्रेन पर वह
प्राप्त की जा सकती है। जीनोग्राफी के माध्यम से ही
स्थान चिह्नित किए जाते है जहाँ रेडियोएक्टिव डीएनए
मानव प्रजाति के पलायन संबंधी इतिहास का अध्ययन
प्रोब्स ने डीएनए के विभाजित टु कड़ों को लगाया हो। इन
किया जाता है।
जगहों को डार्क बैंड के रूप में चिह्नित किया जाता है जब
इनकी X-रे फिल्म बनाई जाती है। VI. फोरेंसिक: किसी अपराधी का पता लगाने या कानूनी
मामलों को सुलझाने में भी डीएनए फिंगरप्रिंटिंग का
VII. X-रे फिल्म पर चिह्नित यहीं डार्क बैंड डीएनए फिंगरप्रिंट
उपयोग किया जाता है।
(डीएनए प्रोफाइल) कहलाते है।

38
अध्याय - 4

स्वास्थ्य
(HEALTH)

रोग और टीके (Diseases करते हैं।

2. फैलने वाले रोगः शरीर के अन्य भागों में फैलते हैं।


and Vaccines) 3. प्रणालीगत रोगः पूरे शरीर को प्रभावित करते हैं।

रोग (Diseases)
रोगों को वर्गीकृत किया जा सकता हैः
• रोग एक असामान्य स्थिति है जो एक जीवित प्राणी को
− संचारी रोग
प्रभावित करती है। रोगों को आमतौर पर चिकित्सीय
स्थितियों के रूप में समझा जाता है जिसमें लक्षणों के एक − गैर-संचारी रोग
विशिष्ट समूह से जुड़ी एक रोग प्रक्रिया शामिल होती है।
विभिन्न प्रकार के रोग शरीर के भिन्न अंगों को प्रभावित • आइए अब हम संचारी और गैर-संचारी रोगों को बेहतर
करते हैं। ढं ग से समझने के लिए इनके बीच के अंतर को जानते हैं
कि विभिन्न रोगों पर टीके और दवाइयाँ कैसे काम करते
1. स्थानीयकृत रोगः शरीर के विशिष्ट भागों को प्रभावित
हैं, जिनका उल्ले ख अध्याय के उत्तरार्ध में किया गया है।

संचारी और गैर-संचारी रोगों के बीच अंतर (Differences Between Communicable and Non-Communicable
Diseases)

मानदंड संचारी रोग गैर-संचारी रोग (NCD)

ये रोग संक्रामक एजेंटों के कारण होते हैं और एक ये रोग एक संक्रमित व्यक्ति से दूसरे व्यक्ति में नहीं
परिभाषा संक्रमित व्यक्ति से दूसरे व्यक्ति आदि में फैलते हैं। ये फैलते बल्कि अस्वस्थ जीवन-शैली जैसे कारकों के
जल, हवा आदि के माध्यम से फैल सकते हैं। कारण होते हैं।

अन्य नाम संक्रामक रोग। जीर्ण रोग।

कारण रोगजनक कारण प्राथमिक होते हैं। कुपोषण, हार्मोनल, असंतुलन आदि।

एजेंट/वेक्टर विषाणु, बैक्टीरिया और कवक आदि। वेक्टर-जनित नहीं होते।

कई संचारी रोगों के लिए यथार्थ नैदानिक जाँच उप- अधिकांश गैर-संचारी रोगों के लिए यथार्थ नैदानिक
निदान
लब्ध है। जाँच उपलब्ध नहीं है।

यह एक पीढ़ी से दूसरी पीढ़ी में वंशागत नहीं किया यह एक पीढ़ी से दूसरी पीढ़ी में वंशागत किया जा
वंशानुगति
सकता। सकता है।

सामान्य रूप से कम गंभीर है और रोगियों के लिए दुनियाभर में मौतों के लिए अधिक गंभीर और जिम्मे-
कठोरता
कम समय तक खतरा। दार। रोगियों के लिए लम्बे समय तक खतरा।

कुछ पारम्परिक तरीकों को रोका जा सकता है, जैसे जीवनशैली में परिवर्तन, व्यायम व और खान-पान की
निवारण व्यक्तिगत स्वच्छता बनाए रखना, खाने के बर्तन आदतें इसकी रोकथाम करने में सहायक होती हैं।
साझा करने से बचना आदि।

39
रोगजनक के प्रवेश के बाद एक या दो दिन पहले एक या अधिक वर्षों तक लक्षण दिखाई नहीं दे सकते
लक्षण
लक्षण देखे जा सकते हैं। जिससे रोग के घातक होने का खतरा बढ जाता है।

सबसे आम संचारी रोग श्वसन तंत्र से जुडे होते हैं जैसे गैर-संचारित रोग विविध होते हैं जैसे ह्रदय रोग, मधुमेह
प्रभावित अंग
सामान्य सर्दी, इन्फ्लू एं जा और तपोदिक। आदि।

टाइफाइड, हैजा, मले रिया, तपेदिक, कोढ जैसे रोग कैंसर, मधुमेह, अल्जाइमर रोग, डाऊन सिं ड्रोम,
उदाहरण संचारी रोगों के उदाहरण हैं। क्वाशियोरकोर जैसे रोग गैर-संचारी रोगों के उदाहरण
हैं।

रोगाणु (Microbes) किया जा सकता है। एं टीबायोटिक दवाइयाँ जीवाणुओ ं को


खत्म करती हैं या कम-से-कम उन्हें बढने से रोकती हैं।
• रोगाणु हमारे आस-पास पाए जाने वाले सूक्ष्म जीव होते
हैं जो इतने सूक्ष्म होते हैं कि इन्हें नग्न आँखों से नहीं देखा • प्रायः जीवाणु टोक्सिन नामक रसायन पैदा करते हैं
जा सकता। ये जल, मृदा और हवा में रहते हैं। मानव शरीर जो असल में प्रभावित करते हैं और बीमारी का कारण
एसे लाखों रोगाणुओ ं का घर है जिन्हें सूक्षम जीव भी कहा बनते हैं। आमतौर पर जीवाणुओ ं को पेनिसिलिन जैसी
जाता है। एं टीबायोटिक दवाओ ं से ख़त्म किया जा सकता है।

• सभी रोगाणु हमारे लिे ए हानिकारक नहीं होते। कुछ • साल्मोनेला व स्टेफिलोकोकस जीवाणुओ ं के नामी
रोगाणु हमें बीमार करते हैं तो कुछ हमारे स्वास्थय के उदाहरण हैं।
लिए जरूरी होते हैं। विशेष सामान्य प्रकार हैंः

1. जीवाणु

2. विषाणु

3. कवक

4. प्रोटोजोआ

जीवाणु (Bacteria)

• जीवाणु एकल-कोशिकीय जीव होते हैं।

• ये सभी, मनुष्यों के लिए खतरनाक नहीं होते। इनमें


से बहुत से हमारे शरीर पर व शरीर के अंदर रहते हैं जो
हमें स्वस्थ रखने में सहायक होते हैं। उदाहरण- आतों
में लै क्टिक अम्ल जीवाणु भोजन को पचाने में हमारी
सहायता करते हैं। अन्य जीवाणु रोगाणुओ ं से लड कर
प्रतिरक्षा तंत्र में सहायता करते हैं।

• जीवाणु संक्रमण का इलाज एं टीबायोटिक दवाओ ं से चित्र 4.1: प्रोकार्योटिक II

रोग बैक्टीरिया प्रभाव लक्षण

कब्ज, खाने में कठिनाई, थकान, लार गिरना,


मस्तिष्क नियंत्रण और माँसपेशियों की
बोटु लिज़्म क्लोस्ट्रीडियम बोटु लिज़्म तंत्रिका तंत्र
कमजोरी के कारण फ्लोपी गति में हानि,
पक्षाघात

40
AKASH SINGH
शरीर के विभिन्न भाग जैसे चर्म खुजली, गले में ख़राश, बुख़ार, माँसपेशि-
एं थ्रेक्स बेसिलस एं थ्रेसिस
त्वचा, फेफडे और आँत आदि यों में दर्द, सांस ले ने में कठिनाई, थकान

खूनी खाँसी या बलगम (कफ़), बिना काम


तपेदिक माइकोबैक्टीरियम तपेदिक श्वसन तंत्र (फेफडे) किये थकान होना, बुख़ार, रात को पसीना
आना आदि

टाइफ़ाइड साल्मोनेला टाइफ़ी आँत तेज़ बुखार, सिरदर्द

तेज़ बुखार, शरीर में अकडन, जाडी अकडना


टे टनस क्लोस्ट्रीडियम टे टनी तंत्रिका तंत्र
आदि

निमोनिया डिप्लोकोकस निमोनिया फ़ेफ़डे तेज़ बुखार, फ़ेफ़डों में सूजन

डिप्थिरिया रेस्पिरेटरी ट्यूब कोरीन-बैक्टीरियम डिप्थिरिया सांस ले ने में कठिनाई और दम घुटना

रोगाणुरोधी प्रतिरोध (Antimicrobial Resistance – जाती हैं।


AMR)
• रोगाणुरोधी प्रतिरोध अस्पतालों में लं बे समय तक रहने
• डब्लू एचओ के अनुसार, रोगाणु प्रतिरोध तब होता है जब और अधिक गहन देखभाल की आवश्यकता के साथ
सूक्ष्मजीव (जैसे बैक्टीरिया, कवक, विषाणु और परजीवी) स्वास्थ्य देखभाल की लागत को बढ़ाता है।
रोगाणुरोधी दवाओ ं (जैसे एं टीबायोटिक्स, एं टीफंगल, • रोगाणुरोधी प्रतिरोध सहस्राब्दी विकास लक्ष्यों के लाभ
एं टीवायरल, एं टीमले रिया और कृमिनाशक) के संपर्क में को व सतत विकास लक्ष्यों की उपलब्धि को खतरे में डाल
आने पर बदल जाते हैं। जो सूक्ष्मजीव रोगाणुरोधी प्रतिरोध रहा है। डब्ल्यूएचओ रोगाणुरोधी प्रतिरोध को संबोधित
विकसित करते हैं उन्हें कभी-कभी "सुपरबग" कहा करने के लिए कई पहलों का नेतृत्व कर रहा है, जैसा कि
जाता है। जैसे-जैसे दवाएं अप्रभावी होती जाती हैं, शरीर में नीचे वर्णन किया गया है:
संक्रमण बना रहता है, जिससे दूसरों में फैलने का खतरा
1. वैश्विक रोगाणुरोधी प्रतिरोध निगरानी प्रणाली (GLASS)
बढ़ जाता है।
2. वैश्विक एं टीबायोटिक अनुसंधान और विकास भागीदारी
• एएमआर (AMR) के कारण: चिकित्सक की सलाह के
(GARDP)
बिना अपने-आप दवा ले ना, नुस्खे वाली दवा का उपयोग,
बड़े पैमाने पर अनियंत्रित मुर्गीपालन, गंदे नालों से 3. एं टी माइक्रोबियल प्रतिरोध (IACG) पर इं टरएजेंसी
जल निकायों का संदष
ू ण और फार्मा उद्योग से निर्वहन, समन्वय समूह।
सांस्कृतिक गतिविधियाँ जैसे सामूहिक स्नान आदि।

एं टीबायोटिक प्रतिरोध (Antibiotic Resistance)


रोगाणुरोधी प्रतिरोध एक वैश्विक चिं ता क्यों है? (Why is
Antimicrobial Resistance a Global Concern?) • एं टीबायोटिक्स दवाओ ं का उपयोग जीवाणु संक्रमण को
रोकने और उनका इलाज करने के लिए किया जाता है।
• विश्व स्तर पर नए प्रतिरोध तंत्र उभर और फैल रहे हैं एं टीबायोटिक प्रतिरोध तब होता है जब इन दवाओ ं के
जिससे हमारी आम संक्रामक रोगों के इलाज की क्षमता उपयोग के जवाब में बैक्टीरिया बदल जाते हैं। बैक्टीरिया,
को खतरा है, इसके परिणामस्वरूप दीर्घकालीन बीमारी, इं सान या जानवर नहीं, एं टीबायोटिक प्रतिरोधी बन जाते
विकलांगता और मृत्यु हो सकती है। हैं। ये बैक्टीरिया मनुष्यों और जानवरों को संक्रमित कर
• संक्रमणों को रोकने और उनका इलाज करने के लिए सकते हैं, और उनके कारण होने वाले संक्रमणों का
प्रभावी रोगाणुरोधी दवाओ ं के बिना, अंग प्रत्यारोपण, इलाज गैर-प्रतिरोधी बैक्टीरिया की तुलना में कठिन
कैंसर केमोथेरप
े ी, मधुमेह प्रबंधन और प्रमुख सर्जरी होता है। एं टीबायोटिक प्रतिरोध स्वाभाविक रूप से होता
(उदाहरण के लिए, सिजेरियन सेक्शन या हिप रिप्ले समेंट) है, ले किन मनुष्यों और जानवरों में एं टीबायोटिक दवाओ ं
जैसी चिकित्सा प्रक्रियाएँ बहुत अधिक जोखिम वाली हो का दुरुपयोग प्रक्रिया को तेज कर रहा है।

41
एक कोर होता है जो प्रोटीन की एक आवरण परत से घिरा
होता है (कोई कोशिका द्रव्य, कोशिका झिल्ली, या कुछ
भी नहीं)।

• उनके मूल में, कोरोनावायरस (COVID-19) में RNA


(बेज) नामक आनुवंशिक ब्लूप्रिंट होता है। एकल-फंसे
आरएनए एक आणविक संदेश के रूप में कार्य करता है
जो विषाणु के अन्य तत्वों के लिए आवश्यक प्रोटीन के
उत्पादन को सक्षम बनाता है।

• कई विषाणु बीमारियों के लिए जिम्मेदार होते हैं। कुछ


हानिरहित हैं और केवल मामूली सर्दी को ट्रिगर करते हैं,
जबकि अन्य एड्स जैसी गंभीर बीमारियों का कारण बन
सकते हैं।

• विषाणु से होने वाली अन्य बीमारियों में कोविड-


19, इन्फ्लू एं जा, खसरा, जिगर की सूजन (वायरल
हेपेटाइटिस), आदि शामिल हैं।
चित्र 4.2: बैक्टीरिया को रोकती दवा
• विषाणु स्वस्थ कोशिकाओ ं पर आक्रमण करते हैं और इन
कोशिकाओ ं से गुणा करना शुरू करते हैं। एक विषाणु
इन मेजबान कोशिकाओ ं के बिना पुनरुत्पादन नहीं कर
एं टीबायोटिक और रोगाणुरोधी प्रतिरोध के बीच अंतर (The
Difference Between Antibiotic and Antimicrobial सकता है। यही कारण है कि वे जीवित कोशिका के बाहर
Resistance) निष्क्रिय हैं।

• एं टीबायोटिक प्रतिरोध तब होता है जब जीवाणु संक्रमण


(जैसे मूत्र पथ के संक्रमण, निमोनिया, रक्त प्रवाह
संक्रमण) का इलाज करने के लिए उपयोग किए जाने
वाले एं टीबायोटिक दवाओ ं के जवाब में बैक्टीरिया बदल
जाते हैं, जिससे वे अप्रभावी हो जाते हैं। रोगाणुरोधी प्रतिरोध
एक व्यापक शब्द है, जिसमें परजीवी (जैसे, मले रिया),
विषाणु (जैसे, एचआईवी), और कवक (जैसे, कैंडिडा) जैसे
अन्य रोगाणुओ ं के कारण होने वाले संक्रमण के इलाज
के लिए दवाओ ं के प्रतिरोध को शामिल किया गया है।

विषाणु (Viruses)

• विषाणु अर्थात वायरस की अपनी कोई कोशिका नहीं


होती, जैसे जीवाणु। इसका मतलब है, सटीक शब्दों
चित्र 4.3 विषाणु
में, उन्हें आमतौर पर जीवित जीव नहीं माना जाता है।
वे बस एक या एक से अधिक अणुओ ं से बने होते हैं जो
एक प्रोटीन खोल से घिरे होते हैं। विषाणु को पुन: उत्पन्न
करने के लिए खोल के अंदर पाई जाने वाली अनुवांशिक • दवा के साथ विषाणु से लड़ना अपेक्षाकृत कठिन है।
जानकारी की आवश्यकता होती है। एं टीबायोटिक्स विषाणु के खिलाफ अप्रभावी हैं। कुछ
विषाणु से बचाव के लिए, टीकाकरण द्वारा प्रतिरक्षा
• उनमें केवल न्यूक्लिक एसिड (डीएनए या आरएनए) का
प्रणाली को "प्रशिक्षित" किया जा सकता है ताकि शरीर
विषाणु से लड़ने के लिए बेहतर तरीके से तैयार हो सके।

42

AKASH SINGH
क्या टीका
रोग विषाणु प्रभाव लक्षण से संचारित
उपलब्ध है?

SARS श्वसन तंत्र और बुखार, सांस ले ने में


कोविड-19 व्यक्ति से व्यक्ति हाँ
Cov-2 विषाणु फेफड़े तकलीफ, खाँसी आदि।

अत्यधिक थकान, दस्त,


बुखार, सिरदर्द, मांसपेशि-
जंतुओ ं और मनुष्यों
इबोला इबोला यों में दर्द, पेट दर्द, अस्प- हाँ
द्वारा
ष्टीकृत रक्तस्राव या चोट
लगना

संक्रमित जानवर की
लार या मस्तिष्क/
तंत्रिका तंत्र के ऊतकों
तेज़ सिरदर्द और तेज़
के साथ प्रत्यक्ष संपर्क
रेबीज रेबीज विषाणु केंद्रीय तंत्रिका तंत्र बुखार से मरीज पागल हो हाँ
(जैसे कि फटी त्वचा
जाते हैं।
या आँखों, नाक या मुँह
में श्ले ष्मा झिल्ली के
माध्यम से)

बुखार, भ्रम की स्थिति,


आक्षेप, मांसपेशी में
वेस्ट नाइल
वेस्ट नाइल केंद्रीय तंत्रिका तंत्र कमज़ोरी, दृष्टि खोना, संक्रमित मच्छर नहीं
विषाणु
स्तब्ध हो जाना, पक्षाघात,
प्रगाढ़ बेहोशी

ढीले और फ्लॉपी अंग,


कभी-कभी शरीर के एक
तरफ अचानक पक्षाघात,
पोलियो पोलियो विषाणु रीढ़ की हड्डी अस्थायी या स्थायी विकृत व्यक्ति से व्यक्ति हाँ
अंग, विशेष रूप से कूल्हों,
टखनों और पैरों में, उल्टी
थकान, मेनिंगिटि

एक भरी हुई या बहती


नाक सिरदर्द, हल्का
त्वचा और लिम्फ बुखार लाल, सूजी हुई
रूबेला रूबेला विषाणु व्यक्ति से व्यक्ति हाँ
नोड्स आंखें, तंत्रिका सूजन, बढ़े
हुए और कोमल लिम्फ
नोड्स

श्वसन तंत्र को
संक्रमित करता है, शरीर पर लाल लाल
खसरा रूबेला विषाणु व्यक्ति से व्यक्ति हाँ
फिर पूरे शरीर में चकत्ते पड़ जाते हैं।
फैल जाता है

43
हेपेटाइटिस (Hepatitis) तालिका में चर्चा की गई है)।

• हेपेटाइटिस यकृत की सूजन है। यह कई स्वास्थ्य • विशेष रूप से, प्रकार B और C लाखों लोगों में पुरानी
समस्याओ ं का कारण बन सकता है और घातक हो बीमारी का कारण बनते हैं और एक साथ, यकृत
सकता है। हेपेटाइटिस विषाणु के पांच मुख्य उपभेद हैं: सिरोसिस, कैंसर, और वायरल हेपेटाइटिस से संबंधित
प्रकार A,B,C,D और E। जबकि ये सभी यकृत रोग का मौतों का सबसे आम कारण हैं। कुछ प्रकार के हेपेटाइटिस
कारण बनते हैं, वे भिन्न होते हैं (जैसा कि नीचे दी गई को टीकाकरण के माध्यम से रोका जा सकता है।

क्या टीका इसकी रोकथाम


प्रकारांतर (वेरिएं ट) कारण संचरण
कर सकता है?

हैपेटाइटस A विषाणु दूषित भोजन और जल के अंतर्ग्रहण या किसी


हेपेटाइटिस A हाँ
(HAV) संक्रामक व्यक्ति के सीधे संपर्क के माध्यम से

प्रसव के दौरान मां से बच्चे के जन्म के दौरान,


साथ ही रक्त या शरीर के अन्य तरल पदार्थों के
हैपेटाइटस B विषाणु संपर्क के माध्यम से, संक्रमित साथी के साथ
हेपेटाइटिस B हाँ
(HAV) यौन संबंध, इं जेक्शन-दवा के उपयोग में सुई,
सीरिं ज, या दवा-तैयारी उपकरण और सुई की
छड़ें या तेज उपकरणों के संपर्क में शामिल हैं।

यह इं जेक्शन दवा के उपयोग, असुरक्षित इं -


हैपेटाइटस C विषाणु जेक्शन प्रथाओ ं, असुरक्षित स्वास्थ्य देखभाल,
हेपेटाइटिस C नहीं
(HCV) बिना जांचे हुए रक्त और रक्त उत्पादों के आधान
और यौन प्रथाओ ं के माध्यम से हो सकता है।

हेपेटाइटिस D विषाणु
(HDV) एक विषाणु
है इसकी प्रतिकृति जन्म के समय प्रसव के दौरान, साथ ही रक्त
के लिए हेपेटाइटिस या शरीर के अन्य तरल पदार्थों के संपर्क के
B विषाणु (HBV) की माध्यम से, संक्रमित साथी के साथ यौन संबंध, हेपेटाइटिस B टीकाकरण से
हेपेटाइटिस D
आवश्यकता होती है। इं जेक्शन-दवा के उपयोग में सुई, सीरिं ज, या रोका जा सकता है
HDV संक्रमण केवल दवा-तैयारी उपकरण और सुई की छड़ें या तेज
एक साथ या HBV के उपकरणों के संपर्क में शामिल हैं।
साथ सुपर-संक्रमण
के रूप में होता है।

हेपेटाइटिस E विषाणु के सं-


क्रमण को रोकने के लिए
खराब जल की गुणव-
मुख्य रूप से दूषित जल के माध्यम से फेकल- एक टीका विकसित किया
हेपेटाइटिस E त्ता वाले क्षेत्रों में दूषित
ओरल मार्ग से फैलता है। गया है और चीन में लाइसेंस
जल, फेकल संदष
ू ण
प्राप्त है ले किन कहीं और
उपलब्ध नहीं है।

रूबेला को कभी-कभी "तीन-दिवसीय खसरा" के रूप में जाना जाता है क्योंकि लक्षण समान हो सकते हैं ले किन रूबेला के
लक्षण खसरे की तुलना में हल्के होते हैं।

AKASH SINGH
44
कवक (Fungi) प्रतिरक्षा प्रणाली के कुछ घटकों में श्वेत रक्त कोशिकाएं
(जिसे स्मृति कोशिकाएं भी कहा जाता है), प्लीहा, थाइमस,
• कवक कई अलग-अलग वातावरणों में रह सकता है।
अस्थि मज्जा और त्वचा आदि शामिल हैं।
सबसे प्रसिद्ध कवक में मशरूम जैसे खमीर, मोल्ड और
खाद्य कवक शामिल हैं।

• बैक्टीरिया की तरह ही, कुछ कवक स्वाभाविक रूप से जन्मजात और अनुकूली प्रतिरक्षा प्रणाली (Innate and
त्वचा पर या शरीर में होते हैं। ले किन कवक भी बीमारियों Adaptive Immune System)
का कारण बन सकता है।
• प्रतिरक्षा प्रणाली के भीतर दो उपतंत्र होते हैं: जन्मजात
• कवक के कारण होने वाले रोगों को मायकोसेस कहा (गैर-विशिष्ट) प्रतिरक्षा प्रणाली और अनुकूली (विशिष्ट)
जाता है। प्रतिरक्षा प्रणाली। जब भी कोई रोगाणु या हानिकारक
• सामान्य उदाहरणों में शामिल हैं एथलीट फुट या नाखूनों पदार्थ प्रतिरक्षा प्रतिक्रिया को ट्रिगर करता है तो ये उप-
के फंगल संक्रमण। प्रणाली निकटता से जुड़े होते हैं और एक साथ काम करते
हैं।
• फंगल संक्रमण कभी-कभी मुंह में या प्रजनन अंगों में
फेफड़ों या श्ले ष्मा झिल्ली की सूजन का कारण बन • जन्मजात प्रतिरक्षा प्रणाली: हानिकारक कीटाणुओ ं
सकता है और कमजोर प्रतिरक्षा प्रणाली वाले लोगों के और पदार्थों के खिलाफ एक सामान्य रक्षा प्रदान करती
लिए जीवन के लिए खतरा बन सकता है। है, इसलिए इसे गैर-विशिष्ट प्रतिरक्षा प्रणाली भी कहा
जाता है। यह ज्यादातर प्राकृतिक हत्यारे कोशिकाओ ं और
• ले किन कुछ कवकों के उपयोगी गुणों से मनुष्यों को भी
फागोसाइट्स ("खाने वाली कोशिकाओ ं") जैसी प्रतिरक्षा
लाभ हुआ है।
कोशिकाओ ं का उपयोग करके लड़ता है। जन्मजात
• पेनिसिलिन की खोज को एक प्रकार के साँचे के रूप में प्रतिरक्षा प्रणाली का मुख्य कार्य हानिकारक पदार्थों
माना जाता है जिसका उपयोग इसे एं टीबायोटिक बनाने और कीटाणुओ ं से लड़ना है जो शरीर में प्रवेश करते हैं,
के लिए किया जाता है। उदाहरण के लिए, त्वचा या पाचन तंत्र के माध्यम से।

• अनुकूली (विशिष्ट) प्रतिरक्षा प्रणाली: एं टीबॉडी बनाती


है और उनका उपयोग विशेष रूप से कुछ कीटाणुओ ं से
प्रोटोजोआ (Protozoa)
लड़ने के लिए करती है जिनसे शरीर पहले संपर्क कर
• ये आम तौर पर सच्चे नाभिक और एक कोशिका झिल्ली चुका है। इसे "अधिग्रहित" (सीखा) या विशिष्ट प्रतिरक्षा
वाले एकल-कोशिका वाले जीव होते हैं, इसलिए ये प्रतिक्रिया के रूप में भी जाना जाता है क्योंकि अनुकूली
काफी हद तक हमारे शरीर की कोशिकाओ ं की तरह होते प्रतिरक्षा प्रणाली लगातार सीख रही है और अनुकूलन
हैं। प्रोटोजोआ के कारण होने वाले रोगों के उदाहरण हैं - कर रही है, शरीर बैक्टीरिया या विषाणु से भी लड़ सकता
मले रिया, चागास रोग, पेचिश, आदि। है जो समय के साथ बदलते हैं।

प्रतिरक्षा प्रणाली (Immune System) झुंड प्रतिरक्षा (Herd Immunity)

• प्रतिरक्षा संक्रमण के खिलाफ प्रतिरोध है। प्रतिरक्षा प्रणाली • जब अधिकांश आबादी किसी संक्रामक रोग से प्रतिरक्षित
शरीर की रोग पैदा करने वाले रोगाणुओ ं जैसे बैक्टीरिया, होती है, तो यह उन लोगों को अप्रत्यक्ष सुरक्षा प्रदान करती
विषाणु, कवक और प्रोटोजोआ आदि से लड़ने की क्षमता है - या झुंड प्रतिरक्षा (जिसे झुंड सुरक्षा भी कहा जाता है) -
को संदर्भि त करती है। जब भी ये रोगाणु हमारे शरीर पर जो रोग से प्रतिरक्षित नहीं हैं।
आक्रमण करने की कोशिश करते हैं, तो प्रतिरक्षा प्रणाली • उदाहरण के लिए, यदि 80% आबादी विषाणु से प्रतिरक्षित
किसी भी विदेशी से शरीर की रक्षा करने में महत्वपूर्ण है, तो हर पांच में से चार व्यक्ति जो किसी न किसी व्यक्ति
भूमिका निभाती है। हानिकारक पदार्थ, रोगाणु और से इस बीमारी का सामना करते हैं, वे बीमार नहीं होंगे
कोशिका परिवर्तन। यह व्यक्ति को बीमार कर सकता है। (और आगे इस बीमारी को नहीं फैलाएं गे)। इस तरह
• प्रतिरक्षा प्रणाली प्रत्येक रोगाणु (सूक्ष्मजीव) का रिकॉर्ड संक्रामक रोगों के प्रसार को नियंत्रित रखा जाता है।
रखती है जिसे उसने कभी भी शरीर में दोबारा प्रवेश करने संक्रमण कितना संक्रामक है, इस पर निर्भर करते हुए,
पर सूक्ष्म जीव को पहचानने और नष्ट करने के लिए आमतौर पर 50% से 90% आबादी को झुंड प्रतिरक्षा प्राप्त
पराजित किया है। प्रतिरक्षा प्रणाली एक जटिल तंत्र है और करने के लिए प्रतिरक्षा की आवश्यकता होती है।
विभिन्न अंगों, कोशिकाओ ं और प्रोटीन आदि से बना है।

45
SARS-CoV-2 के साथ झुंड प्रतिरक्षा प्राप्त करने में
क्या लगेगा? (What will It Take to Achieve Herd एं टीबॉडी बनाम एं टीजन
Immunity With SARS-CoV-2 ?)
एं टीबॉडी रोग से लड़ने के लिए प्रतिरक्षा प्रणाली द्वारा स्वा-
• किसी भी अन्य संक्रमण के साथ, झुंड प्रतिरक्षा प्राप्त भाविक रूप से उत्पादित प्रोटीन होते हैं।
करने के दो तरीके हैं: एं टीजन: पदार्थ जो एक प्रतिरक्षा प्रतिक्रिया को उत्तेजित कर
सकता है, विशेष रूप से लिम्फोसाइट्स को सक्रिय करता है,
» आबादी का एक बड़ा हिस्सा संक्रमित हो जाता है या
जो शरीर के संक्रमण से लड़ने वाली सफेद रक्त कोशिकाएं हैं।
» आबादी के एक बड़े हिस्से को एक सुरक्षात्मक टीका
मिलता है।
• टीके किसी व्यक्ति के प्रतिरक्षी तंत्र को एं टीबॉडी बनाने
के लिए प्रशिक्षित करते हैं, ठीक वैसे ही जैसे वह किसी
• सबसे खराब स्थिति में (उदाहरण के लिए, यदि हम SARS-
बीमारी के संपर्क में आने पर करता है। हालाँकि, क्योंकि
CoV-2 के प्रसार को धीमा करने के लिए शारीरिक दूरी
टीकों में विषाणु या बैक्टीरिया जैसे कीटाणुओ ं के केवल
का पालन नहीं करते हैं या अन्य उपाय नहीं करते हैं),
मारे गए या कमजोर रूप होते हैं, वे बीमारी का कारण नहीं
तो विषाणु कुछ महीनों में कई लोगों को संक्रमित कर
बनते हैं या किसी को भी इसकी जटिलताओ ं के जोखिम
सकता है। यह अस्पतालों पर भारी पड़ेगा और उच्च मृत्यु
में नहीं डालते हैं। एक इं जेक्शन अधिकांश टीके देता है,
दर को जन्म देगा।
ले किन कुछ को मौखिक रूप से (मुंह से) दिया जाता है या
• सर्वोत्तम स्थिति में, जब तक कोई टीका उपलब्ध नहीं हो नाक में छिड़का जाता है।
जाता, तब तक हम संक्रमण के वर्तमान स्तर को बनाए
रखते हैं—या इन स्तरों को कम भी करते हैं। अत्यधिक
प्रभावी टीका विकसित करने, परीक्षण करने और बड़े टीका का कार्य (Working of Vaccine)
पैमाने पर उत्पादन करने से पहले , एक विस्तारित अवधि
के लिए, संभवतः एक वर्ष या उससे अधिक समय तक • जब कोई व्यक्ति टीका प्राप्त करता है, तो उसकी प्रतिरक्षा
निरंतर शारीरिक दूरी के साथ, पूरी आबादी पर ठोस प्रणाली निम्नलिखित तरीकों से प्रतिक्रिया करती है।
प्रयास करना होगा।
» आक्रमणकारी रोगाणु, जैसे कि विषाणु या बैक्टीरिया को
• सबसे अधिक संभावना मामला बीच में कहीं है, जहां पहचानता है।
संक्रमण दर बढ़ जाती है और गिरावट का समय हो जाता
» प्रतिरक्षी उत्पन्न करता है।
है; जब संक्रमणों की संख्या गिरती है तो हम सामाजिक
दूरी के उपायों में ढील दे सकते हैं, और फिर संख्या बढ़ने » बीमारी को याद रखें और उससे कैसे लड़ें। यदि आप
पर इन उपायों को फिर से लागू करने की आवश्यकता भविष्य में रोगाणु के संपर्क में आते हैं, तो आपके अस्वस्थ
हो सकती है। वैक्सीन विकसित होने तक बड़े प्रकोपों होने से पहले आपकी प्रतिरक्षा प्रणाली इसे जल्दी से नष्ट
को रोकने के लिए लं बे समय तक प्रयास करने की कर सकती है।
आवश्यकता होगी। फिर भी, SARS-CoV-2 टीकाकरण
से पहले या वयस्कों की प्रतिरक्षा कम होने के बाद भी • इसलिए, टीका बिना बीमारी के शरीर में प्रतिरक्षा
बच्चों को संक्रमित कर सकता है। प्रतिक्रिया उत्पन्न करने का एक सुरक्षित और चतुर
तरीका है।

टीकाकरण (Vaccination)

• विश्व स्वास्थ्य संगठन की परिभाषा के अनुसार, लोगों


को हानिकारक बीमारियों का सामना करने से पहले
उन्हें हानिकारक बीमारियों से बचाने का एक सरल,
सुरक्षित और प्रभावी तरीका टीकाकरण है। यह विशिष्ट
संक्रमणों के लिए प्रतिरोध बनाने के लिए किसी के शरीर
की प्राकृतिक सुरक्षा का उपयोग करता है और उनकी
प्रतिरक्षा प्रणाली को मजबूत बनाता है।

चित्र 4.4: टीका कैसे काम करता है

46
• हमारी प्रतिरक्षा प्रणाली को याद रखने के लिए डिज़ाइन उपस्थित नहीं होते हैं।
किया गया है। एक बार टीके की एक या अधिक खुराक
• विश्व स्वास्थ्य संगठन ने निम्नलिखित व्यापक विवरण
के संपर्क में आने के बाद, हम आम तौर पर वर्षों, दशकों
को अपनाया है: "रोग और चोटों के निदान, उपचार और
या यहां तक कि जीवन भर किसी बीमारी से सुरक्षित रहते
रोकथाम के लिए वैध सूचनाओ ं के आदान-प्रदान के लिए
हैं। यह वही है जो टीकों को इतना प्रभावी बनाता है। किसी
सूचना और संचार प्रौद्योगिकियों का उपयोग करने वाले
बीमारी के होने के बाद उसका इलाज करने के बजाय,
सभी स्वास्थ्य पेशेवरों द्वारा स्वास्थ्य सेवाओ ं का वितरण,
टीके हमें पहली बार में बीमार होने से रोकते हैं।
जहां दूरी एक महत्वपूर्ण कारक है और अनुसंधान और
मूल्यांकन, और स्वास्थ्य देखभाल प्रदाताओ ं की सतत
शिक्षा के लिए, सभी व्यक्तियों और उनके समुदायों के

मैसेंजर आरएनए स्वास्थ्य को आगे बढ़ाने के हित में।"

(Messenger RNA, mRNA):


टे लीमेडिसिन के प्रकार (Types of
• एमआरएनए जीन का एक आरएनए संस्करण है जो Telemedicine):
कोशिका के केंद्रक को छोड़ कर साइटोप्लाज्म में चला
जाता है जहां प्रोटीन बनते हैं। यह एक एकल-फंसे हुए • टे लीमेडिसिन के 3 सामान्य प्रकार हैं, जिनमें शामिल हैं,
आरएनए अणु है जो एक जीन के डीएनए स्ट्रैं ड में से एक ले किन इन्हीं तक सीमित नहीं हैं:
का पूरक है। कोशिका में mRNA पाया जाता है।
» इं टरएक्टिव मेडिसिन - जो रोगियों और चिकित्सकों को
• हाल ही में मैसेंजर आरएनए के टीके या mRNA के टीके वास्तविक समय में संवाद करने की अनुमति देता है।
संयुक्त राज्य में उपयोग के लिए अधिकृत हैं।
» स्टोर और फॉरवर्ड - जो प्रदाताओ ं को किसी अन्य स्थान
• ये टीके एक नए प्रकार के टीके हैं जिनका उद्देश्य पर एक व्यवसायी के साथ रोगी की जानकारी साझा
संक्रामक रोगों से बचाव करना है और कोविड-19 का करने की अनुमति देता है।
कारण बनने वाले जीवित विषाणु का उपयोग नहीं करना
» दूरस्थ रोगी निगरानी - जो दूरस्थ देखभाल करने वालों
है।
को रोगियों की निगरानी करने की अनुमति देता है। यह
• कोविड-19 के लिए mRNA के टीके कोशिकाओ ं को डेटा एकत्र करने के लिए मोबाइल चिकित्सा उपकरणों
"स्पाइक प्रोटीन" का एक हानिरहित टु कड़ा बनाने के का उपयोग करके घर पर रहता है (जैसे रक्त शर्क रा या
लिए निर्देश देते हैं। स्पाइक प्रोटीन विषाणु की सतह पर रक्तचाप)।
पाया जाता है जो कोविड-19 का कारण बनता है।

• एमआरएनए कभी भी कोशिका के केंद्रक (जहां हमारा • टे लीमेडिसिन के लिए ये बुनियादी दृष्टिकोण टे ली-
डीएनए/आनुवंशिक सामग्री रखी जाती है) में प्रवेश नहीं डर्मेटोलॉजी, टे लीपैथोलॉजी और टे लीरेडियोलॉजी सहित
करता है, इसलिए वे हमारे डीएनए को प्रभावित नहीं विविध सेटिंग्स में सेवाओ ं की एक विस्तृत श्रृंखला पर
करते हैं परस्पर प्रभाव नहीं डालते हैं। लागू होते हैं।

• निर्देशों का उपयोग करने के तुरत


ं बाद सेल टू ट जाता है
और एमआरएनए से छुटकारा पाता है।
टे ली-मेडिसिन के तत्व (Elements of Tele-
Medicine):


टे ली-मेडिसिन (Tele-
टे लीमेडिसिन के लिए चार तत्व प्रासंगिक हैं:

1. इसका उद्देश्य नैदानिक सहायता प्रदान करना है।


Medicine): 2. इसका उद्देश्य भौगोलिक बाधाओ ं को दूर करना है, उन
उपयोगकर्ताओ ं को जोड़ना जो एक ही भौतिक स्थान पर
परिभाषा (Definition)
नहीं हैं।
• इसे केवल "स्वास्थ्य सेवाओ ं की दूरस्थ डिलीवरी" के रूप
3. इसमें विभिन्न प्रकार के आईसीटी का उपयोग शामिल है।
में परिभाषित किया गया है। टे लीमेडिसिन दूर से रोगियों
की देखभाल करने के अभ्यास को संदर्भि त करता है 4. इसका लक्ष्य स्वास्थ्य परिणामों में सुधार करना है।
जब प्रदाता और रोगी एक दूसरे के साथ शारीरिक रूप से

47
टे ली-मेडिसिन के लाभ (Benefits of Tele- • स्वास्थ्य पेशेवर प्रमाणीकरण; और
Medicine): • टे लीमेडिसिन सेवाओ ं की पेशकश करने वाले स्वास्थ्य
• टे लीमेडिसिन में निदान की परिवर्तनशीलता को कम पेशेवरों के लिए चिकित्सा दायित्व का जोखिम।
करने के साथ-साथ नैदानिक प्रबंधन और दुनिया भर
में स्वास्थ्य देखभाल सेवाओ ं के वितरण में सुधार, पहुंच,
गुणवत्ता, दक्षता और लागत-प्रभावशीलता को बढ़ाकर तकनीकी चुनौतियां (Technological Challenges):
काफी संभावनाएं हैं। • उपयोग की जा रही प्रणालियां जटिल हैं, और इसमें खराबी
• विशेष रूप से, टे लीमेडिसिन समुदायों की सहायता कर की संभावना है, जो सॉफ्टवेयर या हार्डवेयर विफलता को
सकता है पारम्परिक रूप से कम सेवा प्राप्त - दूरदराज ट्रिगर कर सकती है।
या ग्रामीण क्षेत्रों में कुछ स्वास्थ्य सेवाओ ं और कर्मचारियों • यह रोगियों की रुग्णता या मृत्यु दर और स्वास्थ्य देखभाल
के साथ - क्योंकि यह स्वास्थ्य देखभाल प्रदाताओ ं और प्रदाताओ ं के दायित्व को भी बढ़ा सकता है।
रोगियों के बीच दूरी और समय की बाधाओ ं को खत्म कर
देता है।

• इसके अलावा, साक्ष्य रोगियों, परिवारों, स्वास्थ्य समय की मांग (Need of the Hour):
चिकित्सकों और स्वास्थ्य प्रणाली के लिए महत्वपूर्ण
• इन चुनौतियों से पार पाने के लिए टे लीमेडिसिन को
सामाजिक आर्थि क लाभों की ओर इशारा करते हैं, जिसमें
निश्चित और व्यापक दिशानिर्देशों द्वारा विनियमित किया
रोगी-प्रदाता संचार और शैक्षिक अवसरों में वृद्धि शामिल
जाना चाहिए, जो व्यापक रूप से, आदर्श रूप से दुनिया भर
है।
में लागू होते हैं।

• समवर्ती रूप से, गोपनीयता, पहुंच और दायित्व को


टे लीमेडिसिन के लिए संभावित बाधाए (Potential Barriers नियंत्रित करने वाले कानून को स्थापित करने की
To Telemedicine): आवश्यकता है।

• स्वास्थ्य सेवाओ ं और इष्टतम गुणवत्ता को अधिकतम


मानव और सांस्कृतिक कारक (Human and Cultural
Factors): करने के लिए टे लीमेडिसिन को बुद्धिमानी से तैनात
किया जाएगा। इसे लाभकारी प्रयासों की गारंटी देना और
• मरीज और स्वास्थ्य कार्यकर्ता सेवा मॉडल को अपनाने नागरिकों को मौलिक सार्वजनिक स्वास्थ्य सेवाओ ं तक
का विरोध करते हैं क्योंकि यह पारंपरिक दृष्टिकोण या पहुंच से वंचित नहीं करना।
स्वदेशी प्रथाओ ं से अलग है।
• सभी देशों में, गोपनीयता, गरिमा और गोपनीयता के मुद्दे
• टे लीमेडिसिन दृष्टिकोण को प्रभावी ढंग से उपयोग करने टे लीमेडिसिन में आईसीटी के उपयोग से संबंधित नैतिक
के लिए आईसीटी साक्षरता की कमी भी योगदान कारक है। चिं ता का विषय हैं।
• सभी में सबसे अधिक चुनौतीपूर्ण रोगियों (विशेषकर उन
सभी व्यक्तियों की गरिमा को बनाए रखने और शिक्षा, भाषा,
लोगों के बीच) और सेवा प्रदाताओ ं के बीच भाषाई और
भौगोलिक स्थिति, शारीरिक और मानसिक क्षमता, उम्र और
सांस्कृतिक अंतर हैं।
लिंग में अंतर सुनिश्चित करने के लिए टे लीमेडिसिन को
• टे लीमेडिसिन अनुप्रयोगों के आर्थि क लाभ और लागत- समान रूप से और उच्चतम नैतिक मानकों पर लागू किया
प्रभावशीलता का दस्तावेजीकरण करने वाले अध्ययनों जाना चाहिए।
की कमी भी एक चुनौती है।

कानूनी विचार (Legal Considerations) ई-सिगरेट (E-Cigarettes)


• इसमें स्वास्थ्य पेशेवरों को विभिन्न न्यायालयों और देशों ई-सिगरेट क्या हैं? (What are E-Cigarettes?)
में सेवाएं देने की अनुमति देने के लिए एक अंतरराष्ट्रीय
कानूनी ढांचे की अनुपस्थिति शामिल है; • ई-सिगरेट इले क्ट्रॉनिक उपकरण होते हैं जो एक द्रव्य को
गर्म कर और एक एयरोसोल या हवा में छोटे कणों का
• स्वास्थ्य पेशेवरों और क्षेत्राधिकारों के बीच डेटा हस्तांतरण,
मिश्रण उत्पन्न करते हैं।
भंडारण, और साझा करने की तुलना में रोगी की
गोपनीयता और गोपनीयता को नियंत्रित करने के लिए • ई-सिगरेट कई रूप और आकारों में आती हैं। अधिकतरों
नीतियों का अभाव, में एक बैटरी, एक हीटिंग तत्व और द्रव्य रखने की जगह
होती है।

48
चित्र 4.5 ई-सिगरेट

• कुछ ई-सिगरेट नियमित सिगरेट, सिगार या पाइप की निकोटीन बच्चों, किशोरों और युवा वयस्कों के लिए
तरह दिखती हैं। असुरक्षित क्यों है? (Why is Nicotine Unsafe for
Kids, Teens, and Young Adults?)
• ई-सिगरेट को कई अलग-अलग नामों से जाना जाता
है। उन्हें कभी-कभी “ई-सिग,” “ई-हुक्का,” “मोड,” “वेप • अधिकांश ई-सिगरेट में निकोटिन होता है - नियमित
पेन,” ‘वेप्स,” ‘टैं क सिस्टम,” और “इले क्ट्रॉनिक निकोटीन सिगरेट, सिगार और अन्य तंबाकू उत्पादों में नशे की लत
डिलीवरी सिस्टम (ENDS)" कहा जाता है। वाली दवा।
• ई-सिगरेट का उपयोग करने को कभी-कभी “वेपिंग” • निकोटीन, विकसित हो रहे युवा मस्तिष्क को खराब कर
कहा जाता है। सकता है। लगभग 25 वर्ष की आयु तक मस्तिष्क का
विकास होता रहता है।

• किशोरावस्था में निकोटिन का प्रयोग मस्तिष्क के


क्या ई-सिगरेट, सिगरेट से ज्यादा सुरक्षित होती हैं? अंगों को नुकसान पहुंचा सकता है। यह ध्यान, सीखने,
(Are E-Cigarettes Safer Than Cigarettes?)
मनोदशा और आवेग नियंत्रण को नियंत्रित करता है।
• ई-सिगरेट जली हुई सिगरेट की तुलना में उपयोगकर्ताओ ं • हर बार जब कोई नई स्मृति बनती है या कोई नया कौशल
को कम हानिकारक रसायनों के संपर्क में लाती है। सीखा जाता है, तो मस्तिष्क की कोशिकाओ ं के बीच
• ले किन जलने वाली सिगरेट बहुत खतरनाक होती है, जो मजबूत संबंध - या अन्तर्ग्रथन - बन जाते हैं। युवा लोगों का
लं बे समय तक धूम्रपान करने वालों में से आधे लोगों की दिमाग वयस्क दिमाग की तुलना में तेजी से अन्तर्ग्रथन
मृत्यु का कारण बनती है। का निर्माण करता है। निकोटीन इन अन्तर्ग्रथन के बनने
के तरीके को बदल देता है।
• ई-सिगरेट के वाष्प में सिगरेट के धुएं की तुलना में कम
मात्रा में जहरीले रसायन होते हैं। • किशोरावस्था में निकोटीन का उपयोग करने से भविष्य
में अन्य नशीले पदार्थों की लत का खतरा भी बढ़ सकता
• ई-सिगरेट सहित किसी भी तंबाकू उत्पाद का उपयोग
है।
युवा लोगों के लिए असुरक्षित है।

49
अध्याय - 5

नैनो तकनीक
(NANOTECHNOLOGY)

• नैनो तकनीक विज्ञान, इं जीनियरिं ग और प्रौद्योगिकी करते हैं जो केवल लगभग 5 से 100 एनएम व्यास का
है जो नैनो पैमाने पर संचालित होती है, जो लगभग 1 से होता है।व्यास में 5 से 100 एनएम। वे एम्फीफिलिक हैं,
100 नैनोमीटर है। यह अत्यंत छोटी चीजों का अध्ययन यानी, एक हाइड्रोफिलिक बाहरी आवरण और एक
और अनुप्रयोग है और इसका उपयोग अन्य सभी विज्ञान हाइड्रोफोबिक इं टीरियर है। यह दोहरी संपत्ति उन्हें दवा के
क्षेत्रों में किया जा सकता है, जैसे कि रसायन विज्ञान, जीव अणुओ ं को पहुंचाने के लिए एक आदर्श वाहक बनाती है।
विज्ञान, भौतिक विज्ञान, आदि। जैसा कि दवा वितरण को लक्षित किया जाता है, आवश्यक
दवा की खुराक और दुष्प्रभाव कम से कम होते हैं।
• नैनोस्केल में किसी भी सामग्री के रूपांतरण के
परिणामस्वरूप उसके भौतिक-रासायनिक, जैविक, • नैनोस्पंज: वे लाल रक्त कोशिका झिल्ली के साथ ले पित
यांत्रिक, ऑप्टिकल, इले क्ट्रॉनिक और अन्य गुणों में बहुलक नैनोकण हैं, जिनका उपयोग विषाक्त पदार्थों को
परिवर्तन होता है जिसका उपयोग विभिन्न उपयोगी अवशोषित करने और उन्हें रक्तप्रवाह से निकालने के
गतिविधियों के लिए किया जा सकता है। लिए किया जा सकता है।

• रक्त शोधन: इनका उपयोग रक्त के शुद्धिकरण में किया


जाता है। नैनोकणों से पदार्थों का विशिष्ट लक्ष्यीकरण

अनुप्रयोग (Applications): संभव है।

• जीवाणुरोधी उपचार: जीवाणु को मारने के लिए स्वर्ण


ऊर्जा (Energy): नैनोकणों और अवरक्त प्रकाश का उपयोग किया जा
सकता है।
• नैनोटे क का उपयोग करके निर्मि त फोटोवोल्टिक पेंट्स
में सौर पैनलों को बदलने की क्षमता है। किसी भी सतह • न्यूरो-इले क्ट्रॉनिक इं टरफेसिं ग नैनो उपकरणों के
पर सोलर पेंट लगाने से यह सतह को सूर्य से ऊर्जा ले ने निर्माण से संबंधित एक दूरदर्शी लक्ष्य है जो कंप्यूटर को
और उसे बिजली में बदलने में सक्षम बनाएगी। तंत्रिका तंत्र से जुड़ने और जोड़ने की अनुमति देगा।

• इनका उपयोग पवन ऊर्जा उत्पादन में किया जा सकता • इमेजिं ग: नैनोपार्टि कल कंट्रास्ट एजेंटों के उपयोग से,
है। नैनोजेनरेटर लचीली पतली चादरें होती हैं जो मुड़ने अल्ट्रासाउं ड और एमआरआई जैसी छवियों ने कंट्रास्ट में
पर पारंपरिक ब्ले ड की तुलना में अधिक शक्ति उत्पन्न सुधार किया है।
कर सकती हैं। • कैंसर का इलाज: लोहे के नैनोकणों या सोने के गोले
• ऐसी नैनोबैटरी जिनका उपयोग रिचार्जेबल लिथियम- कैंसर के इलाज में महत्वपूर्ण रूप से उपयोग कर रहे हैं।
आयन बैटरी को लं बे समय तक चलने में मदद के लिए • स्टेम सेल प्रौद्योगिकी: चुंबकीय नैनोकणों (MNP) का
किया जाता है। उपयोग मूल कोशिका को अलग करने और समूह बनाने
के लिए सफलतापूर्वक किया गया है।

निर्माण (Construction):

• जल के रिसने के लिए डामर और कंक्रीट को और अधिक


वातावरण (Environment):
मजबूत बनाने के लिए नैनो आणविक संरचनाएं । • जल शोधन, विलवणीकरण और विषहरण के लिए नैनो
• पराबैंगनी और अवरक्त विकिरण को अवरुद्ध करने के झिल्ली के माध्यम से जल उपचार और इलाज।
लिए गर्मी प्रतिरोधी नैनो सामग्री; आदि। • संदष
ू कों और रोगजनकों आदि का पता लगाने के लिए
नैनोसंवेदक।

स्वास्थ्य देखभाल और चिकित्सा (Healthcare कृषि: नैनो तकनीक कृषि उत्पादन को बढ़ा सकती है, और
and Medicine): इसके अनुप्रयोगों में शामिल हैं:

• नैनोमिकेल्स तब बनते हैं जब एम्फीफिलिक अणु • फसल सुधार के लिए कीटनाशकों और उर्वरकों को लागू
एक गोलाकार संरचना बनाने के लिए खुद को इकट्ठा करने के लिए कृषि रसायनों के नैनोसूत्रीकरण।

50
• कृषि रसायनों के रोगों और अवशेषों की पहचान के लिए भारत में नैनो तकनीक (Nanotechnology in
फसल सुरक्षा में नैनो संवेदकों का अनुप्रयोग। India):
• पौधों की आनुवंशिक इं जीनियरिं ग के लिए नैनो नैनो विज्ञान और प्रौद्योगिकी पर मिशन (Mission on
उपकरण। Nano Science and Technology, Nano Mission):

• पादप रोग निदान। यह क्षमता निर्माण के लिए एक व्यापक कार्यक्रम है जिसमें


• पशु स्वास्थ्य, पशु प्रजनन, कुक्कुट उत्पादन; तथा देश में नैनो प्रौद्योगिकी के समग्र विकास की परिकल्पना की
गई है और देश के विकास के लिए इसकी कुछ अनुप्रयुक्त
• कटाई उपरांत प्रबंधन।
क्षमता का दोहन किया गया है।

नैनो-मिशन के उद्देश्य हैं:


रक्षा (Defence):
• नैनो प्रौद्योगिकी का बुनियादी संवर्धन
• नैनो प्रौद्योगिकी अनुसंधान चिकित्सा सुविधाओ ं में सुधार
• बुनियादी ढांचे का विकास
और कवच के रूप में हल्के वजन, मजबूत और बहु-
कार्यात्मक सामग्री के उत्पादन पर केंद्रित है। • नैनोविज्ञान अनुप्रयोगों में अनुसंधान एवं विकास की
स्थापना
• सैन्य नैनो-ड्रोन में कुछ ग्राम विस्फोटक भी शामिल हो
सकते हैं जो खोपड़ी में घुसने और सामग्री को नष्ट करने • नैनो विज्ञान के लिए विकास केंद्र की स्थापना

के लिए पर्याप्त हैं। ये नैनो-ड्रोन सर्जि कल परिशुद्धता के • नैनो तकनीक में मानव विकास
हवाई हमलों की सुविधा प्रदान करेंगे।
• अंतर्राष्ट्रीय सहयोग

अन्तरिक्ष (Space):
अन्य पहलें (Other Initiatives):
• उल्ले खनीय छोटे प्रक्षेपण वाहनों के साथ-साथ रूप से
• हाल ही में नैनो मिशन, विज्ञान और प्रौद्योगिकी विभाग
छोटे उपग्रहों के लिए नैनो तकनीक का उपयोग। इस
के तत्वावधान में नैनोविज्ञान और नैनो तकनीक
प्रकार, इन उपग्रहों को लागत प्रभावी बनाना।
(ICONSAT) पर अंतर्राष्ट्रीय सम्मेलन आयोजित किया
• अंतरिक्ष लिफ्ट: कार्बन नैनोट्यूब का उपयोग करके, एक गया था। यह सम्मेलन भौतिकी, रसायन विज्ञान और
बेहद हल्की ले किन मजबूत सामग्री, पृथ्वी की सतह से पदार्थ के कार्यक्षेत्र के विकास के लिए अत्याधुनिक नैनो
लगभग 36,000 किमी की ऊंचाई तक एक वास्तविक तकनीक लाने और देश और विदेश के युवा शोधकर्ताओ ं
भौतिक संरचना का निर्माण किया जा सकता है। और छात्रों के लिए एक संभावित मंच प्रदान करने के

• जल या अन्य रसायनों के अंश के लिए मंगल जैसे ग्रहों के लिए आयोजित किया गया था।

बड़े क्षेत्रों की खोज करने के लिए नैनोसंवेदक के नेटवर्क • विज्ञान और प्रौद्योगिकी विभाग और विज्ञान और
को तैनात करना। इं जीनियरिं ग अनुसंधान बोर्ड ने नैनो कोटिंग कोविड-
19 के लिए एक अनुसंधान अनुदान वित्त पोषित किया
है। इसके तहत एं टी-कोविड-19 ट्रिपल ले यर मेडिकल
खाद्य प्रसंस्करण (Food Processing): मास्क बनाने के लिए उपयुक्त सामग्री पर एं टीवायरल
नैनो-कोटिंग की कोटिंग की जाएगी।
• खाद्य प्रसंस्करण उद्योग में रोगाणुओ ं को मारने के लिए
रोगाणुरोधी नैनो इमल्शन का उपयोग किया जाता है। • इले क्ट्रॉनिक्स और सूचना प्रौद्योगिकी मंत्रालय (MeitY)
द्वारा प्रदान की जाने वाली विश्वेश्वरैया पीएचडी अध्येतावृत्ति
• खाद्य उपकरण, पैकेजिं ग, या भोजन के परिशोधन में।
देश में विभिन्न नैनो प्रौद्योगिकी गतिविधियों का समर्थन
• रोगजनकों के संदष
ू ण की पहचान के लिए नैनो- करती है।
आधारित एं टीजन का पता लगाने वाले बायोसेंसर।
• इले क्ट्रॉनिक्स और सूचना प्रौद्योगिकी मंत्रालय (INSPIRE)
योजना अनुसंधान अध्येताओ ं को नैनो प्रौद्योगिकी, नैनो
विज्ञान और नैनो-जैव प्रौद्योगिकी क्षेत्रों में काम करने के
लिए सहायता करती है।

• विज्ञान और प्रौद्योगिकी विभाग द्वारा स्थापित नैनो विज्ञान


और नैनो प्रौद्योगिकी में उत्कृष्टता केंद्र नैनो मिशन
विभिन्न क्षेत्रों में अनुसंधान और पीजी छात्रों की मदद

51

AKASH SINGH
करता है जैसे: 2. सेंसर (स्वास्थ्य, कृषि, सुरक्षा आदि)

• नैनो मिशन विभिन्न क्षेत्रों में अनुसंधान और पीजी छात्रों 3. कार्बनिक इले क्ट्रॉनिक्स
की मदद करता है जैसे:
4. कम्प्यूटे शनल नैनोइले क्ट्रॉनिक्स
1. पोस्ट मूर इले क्ट्रॉनिक्स 10 nm से आगे
5. उपकरणों/उत्पादों की एक श्रृंखला के लिए मूलरूप और
ऊष्मायन या उद्भवन सुविधाएं स्थापित करना।

52
अध्याय - 6

नाभिकीय ऊर्जा
(NUCLEAR TECHNOLOGY)

परिचय (Introduction) "परमाणु" (Atom) शब्द का अर्थ अविभाज्य (Indivisible)


है, जो ग्रीक भाषा से लिया गया है, क्योंकि पूर्व में यह
विचार कि परमाणु ब्रह्मांड में सबसे छोटी इकाई होती हैं
• नाभिकीय ऊर्जा वह ऊर्जा होती है जो किसी परमाणु के
और इन्हें विभाजित नहीं किया जा सकता है। किन्तु अब
नाभिक में होने वाले परिवर्तन के कारण या तो नाभिकीय
हम जानते हैं कि परमाणु तीन कणों से बने होते हैंः प्रोटॉन,
विखंडन या संलयन के कारण निकलती है। परमाणु के
न्यूट्रॉन और इले क्ट्रॉन और यहां तक कि यह मूल कण
घने नाभिक में भारी मात्रा में ऊर्जा होती है। परमाणु छोटी
भी क्वार्क जैसे छोटे कणों से बने होते हैं।
इकाइयाँ होती हैं जो ब्रह्मांड में सभी प्रकार के पदार्थों को
बनाती हैं। • नाभिक (Nucleus): इसे नाभिक या केन्द्र कहते हैं
जिसकी खोज 1911 में न्यूजीलैं ड के एक भौतिक विज्ञानी
अर्नेस्ट रदरफोर्ड ने की थी। वस्तुतः परमाणु का सारा
द्रव्यमान (द्रव्यमान) उसके नाभिक या केन्द्र में ही होता
नाभिकीय ऊर्जा (Nuclear Power)
है। प्रोटॉन और न्यूट्रॉन से मिलकर नाभिक निर्मि त
• नाभिकीय ऊर्जा वह ऊर्जा होती है जो नाभिकीय ऊर्जा होता है, जिसमें इनका द्रव्यमान (प्रोटॉन का अपेक्षाकृत
संयंत्रों से उत्पन्न होती है जो विखंडन के माध्यम से थोड़ा कम) और इनका कोणीय संवेग (Angular
बिजली उत्पादन के लिए कम समृद्ध यूरनि
े यम ईंधन का Momentum) व चक्रण (Spin) भी लगभग समान होता
उपयोग करती है। है।
• नाभिकीय विखंडन वह विखंडन होता है जो एक परमाणु • प्रोटॉन (Proton): प्रोटॉन परमाणु नाभिक के भीतर
के नाभिक के दो या दो से अधिक छोटे भागों (हल्के पाए जाने वाले धनावेशित कण होते हैं। रदरफोर्ड ने उन्हें
नाभिक) में विभाजित होने की प्रक्रिया और गर्मी, गतिज कैथोड-रे ट्यूबों के साथ किये गये प्रयोगों में खोजा था।
ऊर्जा और विद्युत चुम्बकीय विकिरण के रूप में न्यूट्रॉन प्रोटॉन उतने ही वृहद् और शक्तिशाली (लगभग 99.86
और ऊर्जा को उत्सर्जि त करता है। प्रतिशत) होते हैं जितने कि न्यूट्रॉन होते हैं।
• यूरनि
े यम ईंधन छोटे , सख्त सिरेमिक छर्रे से बनता है जो • इले क्ट्रॉन (Electron): प्रोटॉन और न्यूट्रॉन की तुलना
लं बी, ऊर्ध्वाधर ट्यूबों में पैक होता है। रिएक्टर में ईंधन के में इले क्ट्रोन छोटे होते हैं, जो या तो प्रोटॉन या न्यूट्रॉन से
बंडल डाले जाते हैं। 1,800 गुना छोटे होते हैं।
• एक यूरनि
े यम पेलेट, पेंसिल इरेज़र से थोड़ा बड़ा होता • न्यूट्रॉन (Neutrons): न्यूट्रॉन के अस्तित्व के सिद्धांत
है, जिसमें एक टन कोयला, 3 बैरल तेल, या 17,000 को 1920 में रदरफोर्ड द्वारा प्रतिपादित किया गया था
क्यूबिक फीट प्राकृतिक गैस के समान ऊर्जा होती है। और अमेरिकन फिजिकल सोसाइटी के अनुसार, 1932
प्रत्येक यूरनि
े यम ईंधन पेलेट बिजली का उत्पादन करने में चैडविक द्वारा खोजा गया था। जब परमाणुओ ं को
के लिए पांच साल तक की गर्मी प्रदान करता है। बेरिलियम की एक पतली शीट पर लक्ष्यवेध किया गया
• नाभिकीय ऊर्जा पर्यावरण के लिए भी कई प्रकार से था उस दौरान किये जा रहे प्रयोगों में न्यूट्रॉन पाए गए थे।
फायदेमंद होती हैं। ये आग, गर्मी या अन्य सामग्री उत्पन्न उस समय बिना किसी आवेश वाले अवपरमाणुक कण
नहीं करती हैं, इसलिए इसका कोई दहन उप-उत्पाद नहीं विमुक्त हुए थे - जिन्हें न्यूट्रॉन कहा गया।
होता हैं। इसके अलावा ये ग्रीनहाउस गैसों का उत्पादन • समस्थानिक (Isotopes): समस्थानिक एक ही तत्व
नहीं करती हैं। इसलिए, परमाणु संयंत्र वायु गुणवत्ता की के परमाणु होते हैं? जिसमें समान संख्या में प्रोटॉन
रक्षा करने और जलवायु परिवर्तन को कम करने में मदद होते हैं, ले किन न्यूट्रॉन की एक अलग संख्या होती है।
करती हैं। न्यूट्रॉन की अलग संख्या होने के बावजूद, एक ही तत्व
के समस्थानिकों (आइसोटोप) में बहुत समान भौतिक
गुण होते हैं। कुछ समस्थानिक (आइसोटोप) अस्थिर होते
मूलभूत तत्व (Basics) हैं और किसी दूसरे तत्त्व में परिवर्ति त होने के लिए इन्हें
रेडियोधर्मी क्षय से गुजरना होता है। विभिन्न क्षयकारी
• अणु (Atom): अणु या परमाणु पदार्थ की बुनियादी समस्थानिकों का अनुमानित (पूर्वसूचनीय) आधा जीवन
इकाइयाँ और तत्वों की परिभाषित संरचना होती हैं। वैज्ञानिकों को इसकी समस्थानिक संरचना, जैसे कार्बन

53
-14 के आधार पर सामग्री को दिनांकित करने की ड्यूटे रियम और ट्रिटियम के संयोजन से पृथ्वी पर सबसे
अनुमति देता है। आसानी से प्राप्त होता है। हाइड्रोजन सभी तत्वों में सबसे
हल्का होता है, जो एक प्रोटॉन और एक इले क्ट्रॉन
• न्यूट्रिनो (Neutrinos): वे परिहारकारी उपपरमाण्विक
से बनता है। ड्यूटे रियम के नाभिक में एक अतिरिक्त
कण हैं जो विभिन्न प्रकार की परमाणु प्रक्रियाओ ं में
न्यूट्रॉन होता है; यह हाइड्रोजन परमाणुओ ं में से एक को
निर्मि त होते हैं। उनका नाम, जिसका अर्थ है “थोड़ा तटस्थ
एच 20 में बदल सकता है जिसे "भारी जल" कहते हैं।
होना“, जो इस तथ्य को संदर्भि त करता है कि उनके पास
ट्रिटियम में दो अतिरिक्त न्यूट्रॉन होते हैं और इसलिए यह
कोई विद्युत आवेश नहीं है। ब्रह्मांड में चार मूलभूत बलों में
हाइड्रोजन से तीन गुना भारी होता है। एक संलयन चक्र
से, न्यूट्रिनो केवल दो गुरुत्वाकर्षण और परमाणुओ ं के
में, ट्रिटियम और ड्यूटे रियम एक साथ मिलाया जाता है
रेडियोधर्मी क्षय के लिए जिम्मेदार कमजोर बल के साथ
और परिणामस्वरूप हीलियम का निर्माण होता है, जो
प्रतिक्रियाएं करते हैं। इनका कोई भी द्रव्यमान न होने के
आवधिक तालिका में अगला सबसे भारी तत्व, और एक
कारण ये ब्रह्माण्ड में लगभग प्रकाश की गति से यात्रा
मुफ्त न्यूट्रॉन रिलीज होता है।
करते हैं।
• नाभिकीय विखंडन (Nuclear Fission): परमाणु
• नाभिकीय संलयन (Nuclear Fusion): यह एक
विखंडन एक अभिक्रिया होती है जिसमें न्यूट्रॉन द्वारा एक
परमाणु प्रक्रिया होती है, जहां हल्के परमाणुओ ं को एक
भारी नाभिकीय बमबारी की जाती है और इस प्रकार
साथ मिलाकर ऊर्जा का उत्पादन किया जाता है। यह
यह अस्थिर जैसी स्थिति हो जाती है, जिसके कारण यह
विखंडन की विपरीत अभिक्रिया है, जहां भारी आइसोटोप
ऊर्जा के एक बड़े पृथक्करण और दो या तीन न्यूट्रॉन के
अलग हो जाते हैं। संलयन (फ्यूजन) वह प्रक्रिया है
उत्सर्जन के साथ समान आकार और परिमाण के साथ दो
जिसके द्वारा सूर्य और अन्य तारे प्रकाश और ऊष्मा या
नाभिकों में विघटित हो जाती है।
गर्मी उत्पन्न करते हैं। यह हाइड्रोजन के दो समस्थानिकोंः

परमाणु विखंडन और परमाणु संलयन के बीच अंतर (Differences Between Nuclear Fission and Nuclear
Fusion)

विशेषता नाभिकीय विखंडन नाभिकीय संलयन

विखंडन में एक बड़े परमाणु का दो या दो संलयन में दो या दो से अधिक हल्के परमा-


परिभाषा से अधिक छोटे परमाणुओ ं में विभाजन णुओ ं का एक बड़े परमाणु के रूप में संलयन
होता है। होता है।

विखंडन से अत्यधिक रेडियोधर्मी कण संलयन प्रतिक्रिया से कुछ रेडियोधर्मी कण


उत्पन्न होते हैं। उत्पन्न होते हैं, ले किन यदि एक विखंडन
प्रतिक्रिया के उप-उत्पाद
“ट्रिगर“ का प्रयोग किया जाता है तो इससे रे-
डियोधर्मी कण उत्पन्न होंगे।

विखंडन अभिक्रिया में दो परमाणुओ ं दो या दो से अधिक प्रोटॉनों को पर्याप्त रूप से


को विभाजित करने में बहुत कम ऊर्जा पास लाने के लिए अत्यधिक उच्च ऊर्जा की
ऊर्जा की आवश्यकता
लगती है। आवश्यकता होती है। यह परमाणु बल अपने
इले क्ट्रोस्टैटिक प्रतिकर्षण को दूर करते हैं।

विखंडन की ऊर्जा रासायनिक अभिक्रि- संलयन से निकलने वाली ऊर्जा, विखंडन


याओ ं से निकली ऊर्जा से दस लाख गुना से निकलने वाली ऊर्जा से तीन से चार गुना
ऊर्जा का उत्सर्जन
अधिक होती है, ले किन परमाणु संलयन अधिक होती है।
से निकली ऊर्जा से कम होती है।

परमाणु हथियार एक प्रकार का विखंडन परमाणु हथियार एक प्रकार का हाइड्रोजन


बम होता है, जिसे परमाणु बम के रूप में बम होता है, जो एक संलयन अभिक्रिया को
परमाणु हथियार
भी जाना जाता है। “ट्रिगर“ करने के लिए एक विखंडन अभिक्रि-
याका उपयोग करता है।

54
परमाणु विखंडन और परमाणु संलयन (Nuclear Fission चरण 1ः दबावयुक्त भारी जल रिएक्टर का उपयोग (Stage 1:
and Nuclear Fusion) Pressurized Heavy Water Reactor Using)

• परिभाषा (Definition): विखंडन एक बड़े परमाणु का दो • ईंधनः ईंधन मैट्रिक्स के रूप में प्राकृतिक यूओ2 (UO2)
या दो से अधिक छोटे परमाणुओ ं के रूप में विभाजन होता
• मंदक और शीतलकः भारी जल
है। संलयन दो या दो से अधिक हल्के परमाणुओ ं का एक
बड़े परमाणु के रूप में संलयन होता है। • 0.7 प्रतिशत U-235 ऊर्जा विमुक्त करने के लिए विखंडन
की प्रक्रियासे गुजरता है। चूंकि शेष 99 प्रतिशत U-238
• प्रक्रिया का प्राकृतिक रूप से घटना (Natural
विखंडनीय नहीं है, इसलिए यह परिवर्तन से गुजरता है
Occurrence of the Process): विखंडन अभिक्रिया
और प्लूटोनियम-239 (PU-239) बनाता है।
सामान्यतः स्वाभाविक रूप से नहीं होती और संलयन
अभिक्रिया मुख्यतः तारों में घटित होती है, जैसे सूर्य। दाबयुक्त भारी जल रिएक्टर का कार्य (Working of
• अभिक्रिया का उप-उत्पाद (Byproducts of the Pressurized Heavy Water Reactor)
Reaction): विखंडन से अत्यधिक रेडियोधर्मी कण
• दाबयुक्त भारी जल रिएक्टर (PHWR) एक नाभिकीय
उत्पन्न होते हैं। ले किन संलयन अभिक्रिया में भी विखंडन
ऊर्जा रिएक्टर होता है, जो आमतौर पर अपने ईंधन के रूप
"ट्रिगर" का प्रयोग करने के परिणामस्वरूप रेडि योधर्मी
में अपरिष्कृ त प्राकृतिक यूरनि
े यम का उपयोग करता है।
कण उत्पन्न होंगे।
इसमें भारी जल (ड्यूटे रियम ऑक्साइड D2O) शीतलक
और मंदक का उपयोग किया जाता है।

नाभिकीय ऊर्जा के लिए भारत की रणनीति • जबकि भारी जल सामान्य हल्के जल की तुलना में
(India’s Strategy for Nuclear Energy) काफी अधिक महंगा होता है, यह बहुत अधिक न्यूट्रॉन
अर्थव्यवस्था उत्पन्न करता है, जिससे रिएक्टर को ईंधन
• भारत ने सचेतन रूप से विद्युत उत्पादन के उद्देश्य से संवर्धन सुविधाओ ं के बिना संचालित करने की अनुमति
नाभिकीय ऊर्जा के दोहन की संभावना का पता लगाया मिलती है (भारी जल की अतिरिक्त मूल लागत को कम
है। नाभिकीय ऊर्जा अधिनियम को प्राकृतिक रूप से पाए करता है) और आम तौर पर रिएक्टर की क्षमता को
जाने वाले दो तत्वों यूरनि
े यम और थोरियम का उपयोग बढ़ाने के लिए वैकल्पिक ईंधन चक्रों का कुशलतापूर्वक
करके तैयार और कार्यान्वित किया गया था, जिनका उपयोग किया जाता है।
भारतीय नाभिकीय ऊर्जा रिएक्टरों में परमाणु ईंधन के
रूप में उपयोग करने की अच्छी क्षमता है। भारत में इन
तत्वों के अनुमानित प्राकृतिक भण्डार (निक्षेप) हैंः

» प्राकृतिक यूरनि
े यम निक्षेप - ्70,000 टन

» थोरियम निक्षेप - ् 3,60,000 टन

• भारतीय नाभिकीय ऊर्जा उत्पादनः एक तीन चरण के


कार्यक्रम की परिकल्पना की गई है:

चरण 1 >> दाबित भारी जल


रिएक्टर उपयोग
चित्र्र 6.1ः दाबयुक्त भारी जल रिएक्टर
चरण 2 >> फ़ास्ट ब्रीडर रिएक्टर
चरण 2 - फास्ट ब्रीडर रिएक्टर (Stage 2 - Fast Breeder
Reactor)
चरण 3 >> ब्रीडर रिएक्टर
• भारत के नाभिकीय ऊर्जा उत्पादन के दूसरे चरण में
फास्ट ब्रीडर रिएक्टरों (FBR) में मुख्य ईंधन के रूप में
पहले चरण के रिएक्टर संचालन से प्राप्त PU-239 के
उपयोग की परिकल्पना की गई है।

55

AKASH SINGH
• FBR की मुख्य विशेषताएं हैंः है और शीतलक के रूप में सोडियम का उपयोग किया
जाता है।
» PU-239 FBR में मुख्य विखंडनीय तत्व के रूप में कार्य
» समवर्ती रूप से, उन्नत भारी जल रिएक्टरों (AHWR)
करता है।
में थोरियम-आधारित ईंधन और प्लूटोनियम-आधारित
» फ्यूल कोर के आसपास यू-238 की एक परत ताजा ईंधन की एक छोटी फ़ीड का उपयोग होता है। AHWRs से
पीयू-239 का उत्पादन करने के लिए परमाणु रूपांतरण बड़े पैमाने पर थोरियम के उपयोग के चरण तक पहुंचने
से गुजरती है। की अवधि को कम करने की उम्मीद रहती है।
» एफबीआर कोर के चारों ओर टीएच-232 की एक परत
के अलावा न्यूट्रॉन कैप्चर प्रतिक्रियाओ ं से भी गुजरता
है जिससे यू-233 का निर्माण होता है। यू-233 भारत चरण 3ः ब्रीडर रिएक्टर (Stage 3: Breeder Reactor)
के नाभिकीय ऊर्जा कार्यक्रम के तीसरे चरण के लिए
• भारत के नाभिकीय ऊर्जा उत्पादन कार्यक्रम का तीसरा
परमाणु रिएक्टर ईंधन होता है।
चरण U-233 ईंधन का उपयोग करने वाले ब्रीडर रिएक्टर
» दूसरे चरण के लिए यूरनि
े यम-238 और प्लूटोनियम-239 हैं। भारत का विशाल थोरियम भण्डार U-233 ईंधन वाले
ईंधन के रूप में होता है। ब्रीडर रिएक्टरों के डिजाइन और संचालन के अनुरूप है।
» इस अवस्था में किसी मंदक की आवश्यकता नहीं होती

चित्र्र 6.2: फास्ट ब्रीडर रिऐक्टर

• U-233 को TH-232 के परमाणु रूपांतरण से प्राप्त किया आवश्यकता को बनाए रखने में मदद करता है।
जाता है जिसे दूसरे चरण में एक परत के रूप में उपयोग
• यू-233/टीएच-232 आधारित ब्रीडर रिएक्टर विकास के
किया जाता है।
अधीन होते हैं और भारतीय परमाणु कार्यक्रम के अंतिम
• इसके अलावा, यू-233 ईंधन वाले ब्रीडर रिएक्टरों में थोरियम उपयोग चरण के मुख्य आधार के रूप में काम
U-233 रिएक्टर कोर के आसपास एक टीएच-232 की करेंगे। वर्तमान में ज्ञात भारतीय थोरियम भंडार विद्युत
एक परत होगी जो अधिक यू-233 का उत्पादन करेगा ऊर्जा का 358,000 ळॅ म-लत भण्डार होता है और आसानी
क्योंकि रिएक्टर चालू हो जाता है जिसके परिणामस्वरूप से अगली सदी के दौरान और उसके बाद भी ऊर्जा की
टीएच-232 से अधिक से अधिक यू-233 ईंधन का आवश्यकताओ ं को पूरा कर सकते है
उत्पादन होता है जो दीर्घकालिक विद्युत उत्पादन ईंधन की

56
भारत के नाभिकीय ऊर्जा संयंत्र (India’s Nuclear Power Plants)

क्र. क्षमता वाणिज्यिक


संयंत्र इकाई प्रकार
सं. (मेगावाट) संचालन की तिथि

01 तारापुर नाभिकीय ऊर्जा स्टेशन (TAPS), महाराष्ट्र 1 BWR 160 अक्टू बर 28, 1969

02 तारापुर नाभिकीय ऊर्जा स्टेशन (TAPS), महाराष्ट 2 BWR 160 अक्टू बर 28, 1969

03 तारापुर नाभिकीय ऊर्जा स्टेशन (TAPS), महाराष्ट्र 3 PHWR 540 अगस्त 18, 2006

04 तारापुर नाभिकीय ऊर्जा स्टेशन (TAPS), महाराष्ट्र 4 PHWR 540 सितंबर 12, 2005

05 राजस्थान नाभिकीय ऊर्जा स्टेशन (RAPS), राजस्थान 1 PHWR 100 दिसंबर 16, 1973

06 राजस्थान नाभिकीय ऊर्जा स्टेशन (RAPS), राजस्थान 2 PHWR 200 अप्रैल 01, 1981

07 राजस्थान नाभिकीय ऊर्जा स्टेशन (RAPS), राजस्थान 3 PHWR 220 जून 01, 2000

08 राजस्थान नाभिकीय ऊर्जा स्टेशन (RAPS), राजस्थान 4 PHWR 200 दिसंबर 23, 2000

09 राजस्थान नाभिकीय ऊर्जा स्टेशन (RAPS), राजस्थान 5 PHWR 220 फरवरी 04, 2010

10 राजस्थान नाभिकीय ऊर्जा स्टेशन (RAPS), राजस्थान 6 PHWR 220 मार्च 31, 2010

11 मद्रास नाभिकीय ऊर्जा स्टेशन (MAPS), तमिलनाडु 1 PHWR 220 जनवरी 27,1984

12 मद्रास नाभिकीय ऊर्जा स्टेशन (MAPS), तमिलनाडु 2 PHWR 220 मार्च 21,1986

13 कैगा जनरेटिंग स्टेशन (KGS), कर्नाटक 1 PHWR 220 नवंबर 16, 2000

14 कैगा जनरेटिंग स्टेशन (KGS), कर्नाटक 1 PHWR 220 मार्च 16, 2000

15 कैगा जनरेटिंग स्टेशन (KGS), कर्नाटक 3 PHWR 220 जनवरी 20, 2011

16 कैगा जनरेटिंग स्टेशन (KGS), कर्नाटक 4 PHWR 220 जनवरी 20, 2011

कुडनकुलम नाभिकीय ऊर्जा स्टेशन (KKNPS), VVER- 1000


17 1 1000 दिसंबर 31, 2014
तमिलनाडु (PWD)

कुडनकुलम नाभिकीय ऊर्जा स्टेशन (KKNPS) VVER-1000


18 2 1000 मार्च 31, 2017
तमिलनाड (PWD)

19 नरोरा परमाणु बिजलीघर (NAPS), उत्तरप्रदेश 1 PHWR 220 जनवरी 1,1991

20 नरोरा परमाणु बिजलीघर (NAPS), उत्तरप्रदेश 2 PHWR 220 जुलाई 1,1992

21 काकरापार नाभिकीय ऊर्जा स्टेशन (KAPS), गुजरात 1 PHWR 220 मई, 6, 1993

22 काकरापार नाभिकीय ऊर्जा स्टेशन (KAPS), गुजरात 2 PHWR 220 सितंबर 1,1995

57
कुल नाभिकीय ऊर्जा संयंत्र क्षमताः 6780 मेगावाट (Total • रिएक्टर का उपयोग अनुसंधान और रेडियोआइसोटोप के
Nuclear Power Plant Capacity: 6780 Mwe) उत्पादन के लिए भी किया जाता है।

ज़र्लि ना (ZERLINA)

• यह एक शून्य ऊर्जा टैं क प्रकार का अनुसंधान रिएक्टर


है जिसे 1961 में अपने ही देश में स्वदेशी रूप से बनाया
गया था।

• साइरस (CIRUS), 40 मेगावाट का एक टैं क प्रकार का


रिएक्टर है, जिसे सामग्री परीक्षण और रेडियोआइसोटोप
उत्पादन की सुविधाओ ं के साथ इं जीनियरिं ग प्रायोगिक
कार्य के लिए कनाडा की सहायता से 1960 में तारापुर में
अधिकृत और प्रमाणित किया गया था।

पूर्णि मा- 1 और पूर्णि मा- 2 (PURNIMA-I & PURNIMA-II)

• इन्हें क्रमशः 1972 और 1984 में अधिकृत और प्रमाणित


किया गया था।

• इसके साथ भारत ने अपने ’फास्ट रिएक्टर’ कार्यक्रम में


एक महत्वपूर्ण उपलब्धि हासिल की।

चित्र्र 6.3ः भारत में परिचालित नाभिकीय ऊर्जा संयंत्र


ध्रुव (DHRUVA)

• यह देश में बना स्वदेशी टैं क के प्रकार का 100 मेगावाट


• अप्सरा भारत का सबसे पुराना अनुसंधान रिएक्टर है।
रिएक्टर है जिसे 1985 में उन्नत परमाणु भौतिकी और
• इस रिएक्टर को भाभा परमाणु अनुसंधान केंद्र (BARC) आइसोटोप उत्पादन में अनुसंधान के लिए इसका
द्वारा तैयार और डिजाइन किया गया था और यूनाइटे ड इस्तेमाल किया गया था।
किंगडम की सहायता से बनाया गया था (जिसने 80
प्रतिशत सवंर्धि त यूरनि
े यम से मिलकर (युक्त) प्रारंभिक
ईंधन आपूर्ति भी प्रदान की थी)।े पूर्णि मा 3 (PURNIMA III)
• अगस्त 1956 में अप्सरा पहली बार महत्त्वपूर्ण और • यह भी 1 मेगावाट का एक टैं क के प्रकार का रिएक्टर है
आवश्यक हो गया था। जिसे नवंबर 1990 में क्रांतिकता और महत्वपूर्णता मिली
• अप्सरा एक हल्के जल का स्विमिं ग पूल-प्रकार का थी।
रिएक्टर है जिसमें एक मेगावाट थर्मल अधिकतम • इस रिएक्टर का एकमात्र उद्देश्य कामिनी रिएक्टर के
बिजली का उत्पादन होता है। लिए मॉकअप अध्ययन करना है।
• रिएक्टर समृद्ध यूरनि
े यम को एल्युमिनियम
मिश्रधातुयुक्त (एलॉयड) घुमावदार प्लेटों के रूप में
जलाता है। कामिनी (KAMINI)

• रिएक्टर के लिए ईंधन की आपूर्ति यूनाइटे ड किंगडम से • 1996 में यूरनि


े यम-233 आधारित परमाणु ईंधन में
एक अनुबंध के तहत की जाती है, बशर्ते ईंधन सुरक्षित महारत हासिल करने के भारत के प्रयास में कामिनी
हो। रिएक्टर स्वयं अंतर्राष्ट्रीय नाभिकीय ऊर्जा एजेंसी के (कलपक्कम मिनी रिएक्टर) का निर्माण एक महत्वपूर्ण
सुरक्षा उपायों के अधीन नहीं है। मील का पत्थर साबित हुआ है।
• अप्सरा रिएक्टर का उपयोग न्यूट्रॉन सक्रियण विश्ले षण, • फ्रांस के रैप्सोडी रिएक्टर के आधार पर डिजाइन किया
विकिरण क्षति अध्ययन, फोरेंसिक अनुसंधान, न्यूट्रॉन, गया, यह विश्व स्तर पर एकमात्र रिएक्टर है जो ईंधन के
रेडियोग्राफी, और परिरक्षण प्रयोग सहित विभिन्न प्रयोगों रूप में U-233 का उपयोग करता है।
के लिए किया जाता है।

58
• इसका उपयोग मुख्य रूप से कलपक्कम में FBTR से भारत के असैन्य परमाणु सौदे (India’s Civil
निकलने वाले अत्यधिक रेडियोधर्मी ईंधन तत्वों का Nuclear Deals):
अध्ययन करने के लिए किया जाएगा।
वर्तमान में, भारत के 14 देशों के साथ असैन्य परमाणु समझौते
• यह अगली सदी में बनाए जाने वाले प्रोटो-टाइप FBR हुए हैं जो निहितार्थ और भावना में भिन्न हैं। भारत में अपनी
के लिए उच्च प्रदर्शन वाले प्लूटोनियम ईंधन तत्वों को मौजूदा साझेदारियों को मजबूत करके और नई साझेदारी
विकसित करने में मदद करेगा। बनाकर अपने क्षेत्र का विस्तार करने की क्षमता है। बदलती
• इसे ’जीरो-पावर’ रिएक्टर भी कहा जाता है क्योंकि वैश्विक व्यवस्था में अपने नेतृत्व की स्थिति पर जोर देते हुए
उत्पादित बिजली की मात्रा (40 मेगावाट) की खपत इसकी बढ़ती ऊर्जा मांगों को पूरा करने की आवश्यकता है।
रिएक्टर द्वारा ही अनुसंधान उद्देश्यों के लिए की जाती है।
• भू-राजनीतिक क्षेत्र में भारत के उदय को आंशिक रूप
से नाभिकीय ऊर्जा क्षेत्र में इसकी गहरी भागीदारी से
नाभिकीय ऊर्जा आयोग (Atomic Energy प्रोत्साहित किया गया है और इसने वैश्विक असैन्य
Commission) परमाणु ढांचे में अपनी उपस्थिति को सुगम बनाया है।

• भारतीय नाभिकीय ऊर्जा आयोग की स्थापना पहली बार • इन समझौतों का एक केंद्रीय सिद्धांत नाभिकीय ऊर्जा
अगस्त 1948 में वैज्ञानिक अनुसंधान विभाग में की गई के “शांतिपूर्ण उद्देश्यों“ पर केंद्रित है; इसमें “अनुसंधान,
थी, जिसे कुछ महीने पहले जून 1948 में बनाया गया था। बिजली उत्पादन, चिकित्सा, कृषि और उद्योग जैसे क्षेत्रों में
सूचना, परमाणु सामग्री, उपकरण या घटकों का उपयोग“
को शामिल किया गया है।
नाभिकीय ऊर्जा नियामक बोर्ड (The Atomic Energy • 14 देशों के साथ भारत के समझौतों में अर्जेंटीना,
Regulatory Board) ऑस्ट्रेलिया, कनाडा, चेक गणराज्य, फ्रांस, जापान,
• यह अधिनियम के तहत कुछ नियामक और सुरक्षा कार्यों कजाकिस्तान, मंगोलिया, नामीबिया, रूस, दक्षिण
को करने के लिए नाभिकीय ऊर्जा अधिनियम, 1962 द्वारा कोरिया, यूनाइटे ड किंगडम, अमेरिका और वियतनाम
प्रदत्त शक्तियों का प्रयोग करके भारत के राष्ट्रपति द्वारा जैसे देश शामिल हैं। परमाणु अप्रसार में भारत का
15 नवंबर, 1983 को गठित किया गया था। असाधारण स्वच्छ ट्रै क रिकॉर्ड सबसे महत्वपूर्ण कारक है
जिसने इन समझौतों को सुगम बनाया है।
• AERB का नियामक प्राधिकरण नाभिकीय ऊर्जा
अधिनियम और पर्यावरण (संरक्षण) अधिनियम, 1986 • भारत ने 1998 में पोखरण के दूसरे दौर के बाद परमाणु
के तहत प्रख्यापित नियमों और अधिसूचनाओ ं से लिया परीक्षण करने पर स्व-स्थगन का पालन किया है। इसने
गया है। NPT के सिद्धांतों का पालन अपने कुछ हस्ताक्षरकर्ताओ ं
की तुलना में कहीं बेहतर किया है।
• मिशनः AERB का मिशन यह सुनिश्चित करना है कि
भारत में आयनकारी विकिरण और नाभिकीय ऊर्जा का
उपयोग लोगों के स्वास्थ्य और पर्यावरण के लिए अनुचित
जोखिम का कारण न बने। डार्क एनर्जी (Dark Energy)
• जनादेशः नाभिकीय ऊर्जा नियामक बोर्ड (AERB) के पास • सामान्यतः ब्रह्मांड का 68 प्रतिशत हिस्सा डार्क एनर्जी
नाभिकीय ऊर्जा अधिनियम, 1962 के तहत प्रख्यापित है। यह अंतरिक्ष का एक ऐसा गुण है जो अंतरिक्ष के
नियमों और विनियमों को लागू करने का अधिदेश है। विस्तार के रूप में मिश्रित नहीं होता है। जैसे-जैसे अधिक
नाभिकीय ऊर्जा नियामक बोर्ड (AERB) के पास देश स्थान अस्तित्व में आता है, अंतरिक्ष की यह ऊर्जा उतनी
में रेडियोलॉजिकल सुरक्षा सुनिश्चित करने के लिए ही अधिक प्रकट होती रहती है। नतीजतन, डार्क एनर्जी
नाभिकीय ऊर्जा अधिनियम, 1962 के तहत प्रख्यापित ब्रह्मांड को तेजी से और शीघ्रता रूप से विस्तारित करने
नियमों और विनियमों को लागू करने का अधिदेश है। का कारण बनती है।
नाभिकीय ऊर्जा नियामक बोर्ड (AERB) उन इकाइयों
• यद्यपि डार्क मैटर ब्रह्मांड पर एक “दबाव“ डालता है,
में औद्योगिक सुरक्षा सुनिश्चित करने के लिए कारखाना
डार्क एनर्जी का एक विपरीत विस्तारवादी प्रभाव होता
अधिनियम, 1948 के प्रावधानों का भी संचालन करता है
है। जैसा कि स्पष्ट है, हमारे ब्रह्मांड का विस्तार हो रहा
जो बोर्ड के दायरे में हैं।
है, जो यह दर्शाता है कि डार्क एनर्जी में डार्क मैटर की
तुलना में अधिक प्रचुरता रहती है।

59

AKASH SINGH
(IAEA) और NSG सहित वैश्विक असैन्य नाभिकीय ऊर्जा
• ब्रह्मांड विज्ञान के नियमों के अनुसार, ब्रह्मांड में द्रव्य- ढांचे में बदलाव का भी आह्वान किया गया।
मान की कुल मात्रा में वृद्धि नहीं हो सकती है। इसलिए
• इसके अलावा, भारत ने परमाणु परीक्षण पर अपनी
जब डार्क मैटर की मात्रा स्थिर रहती है, डार्क एनर्जी जो
एकतरफा रोक को जारी रखने और जिन राज्यों और
कि अंतरिक्ष का एक गुण है, उसका तेजी से बढ़ना तय
अभिकर्ताओ ं के पास यह तकनीक उपलब्ध नहीं है, उन
होता है। आखिरकार, डार्क एनर्जी डार्क मैटर के प्रभाव
तक परमाणु प्रौद्योगिकियों के हस्तांतरण को प्रतिबंधित
को दूर कर देगी और ब्रह्मांड के विस्तार की ओर ले
करने के उपायों को सशक्त करने के लिए खुद को
जाएगी।
प्रतिबद्ध किया।

• समझौते के कारण भारत ने अपने घरेलू निर्यात नियंत्रण


नियमों को और अधिक अद्यतन और कड़ा किया।
भारत-अमेरिका (India-US)
• भारत परमाणु हथियारों वाला एकमात्र देश बन गया जो
• 18 मई 1974 को, भारत ने ’ऑपरेशन 6 स्माइलिंग बुद्धा’ - NPT का पक्षकार नहीं है। यह शेष विश्व के साथ परमाणु
या पोखरण-1 द्वारा देश का पहला सफल परमाणु परीक्षण व्यापार में संलग्न हो सकता है। इसलिए, एनएसजी छूट
किया। ने भारत के लिए असैन्य परमाणु व्यापार ढांचे में एक
• इसके बाद, 11 और 13 मई 1998 को भारत के सफल मूल्यवान भागीदार के रूप में अपनी जगह बनाने का मार्ग
परमाणु परीक्षणों ने दुनिया को शक्ति संपन्न राष्ट्रों के प्रशस्त किया। 2 फरवरी 2009 को, भारत ने अंतरराष्ट्रीय
मध्य भारत के प्रवेश को स्वीकार करने के लिए मजबूर नाभिकीय ऊर्जा एजेंसी (जिसे भारत सेफगार्ड एग्रीमेंट
किया। 1998 के परमाणु परीक्षणों ने उस संदर्भ को पूरी कहा जाता है) के साथ एक समझौते पर हस्ताक्षर किए,
तरह से बदल दिया जिसके तहत अमेरिका ने भारत के जिसने भारत की असैनिक परमाणु सुविधाओ ं को (IAEA)
साथ बातचीत की और जो परिणामी वार्ता के सन्दर्भ में भी की सुरक्षा उपायों के अधीन किया।
समझौते पर हस्ताक्षर करने में महत्वपूर्ण थी।
भारत-अमेरिका असैन्य परमाणु समझौते की प्रमुख विशेष-
• एक जिम्मेदार परमाणु शक्ति के रूप में भारत की मान्यता ताएं और लक्षण भारत की भावी चर्चाओ ं और अन्य असैन्य
की दावेदारी और वैश्विक अप्रसार व्यवस्था के साथ परमाणु परमाणु भागीदारों के साथ समझौतों के संदर्भ के कुछ मह-
हथियार संपन्न देश के रूप में अपनी स्थिति के बारे में त्त्वपूर्ण बिं दु बन गए।
अंतरराष्ट्रीय धारणा और परमाणु हथियारों के अप्रसार
पर संधि (NPT) पर हस्ताक्षरकर्ता नहीं होने से विशेष रूप a. ऐसे क्षेत्रों में उन्नत नाभिकीय ऊर्जा अनुसंधान और
से अंतर्राष्ट्रीय संबंधों से संबंधित, भौगोलिक कारकों से विकास, जिन पर दोनों पक्षों के बीच सहमति हो;
प्रभावित राजनीति महत्त्व रुक कर ठप्प हो गया। 2003- b. परस्पर हित और क्षमता के परमाणु सुरक्षा मामले , जैसा
04 में जारी ‘‘सामरिक महत्त्व वाले कूटनीतिक 12 देशों कि अनुच्छेद 3 में निर्धारित किया गया है;
की साझेदारी में अगले चरण“ के माध्यम से “भारत और
c. यात्राओ ं, बैठकों, संगोष्ठियों और सहयोगी अनुसंधान के
अमेरिका के बीच बढ़ते रणनीतिक अभिसरण’’ द्वारा
लिए वैज्ञानिकों के आदान-प्रदान की सुविधा;
वैश्विक परमाणु अप्रसार निर्माण में भारतीय समावेश और
अखण्डता होने की मदद की गई थी। d. पक्षों या अधिकृत व्यक्तियों के बीच औद्योगिक या
वाणिज्यिक पैमाने पर प्रौद्योगिकी हस्तांतरण सहित
• यह दस्तावेज़ अप्रसार व्यवस्था में भारत के उत्थान को
परमाणु रिएक्टरों और संबद्ध परमाणु ईंधन चक्र के
प्रमुख भूमिका निभाने में सहायक सिद्ध होगा।
पहलु ओ ं को कवर करने वाली पूर्ण असैनिक परमाणु
• भारत की परमाणु विलगता, 2005 में अमेरिका के साथ सहयोग गतिविधियाँ;
एक असैन्य परमाणु समझौते पर हस्ताक्षर के साथ
e. भारत के रिएक्टरों के जीवनकाल में आपूर्ति में किसी भी
समाप्त हो गई।
व्यवधान से बचाव के लिए परमाणु ईंधन के रणनीतिक
भंडार का विकास;
123 समझौते (123 Agreement) f. जैविक अनुसंधान, चिकित्सा, कृषि और उद्योग, पर्यावरण
और जलवायु परिवर्तन सहित परमाणु विज्ञान में उन्नत
• असैन्य नाभिकीय ऊर्जा के मुद्दों पर भारत और अमेरिका
अनुसंधान और विकास;
के बीच घनिष्ठ सहयोग को सक्षम करने के लिए घरेलू
अमेरिकी कानूनी व्यवस्था में संशोधन करते हुए इस g. पार्टि यों के बीच आपूर्ति , चाहे वह पार्टि यों या तीसरे देशों के
समझौते ने भारत और अमेरिका के बीच अधिक गहन उपयोग के लिए या परमाणु सामग्री के लाभ के लिए हो;
जुड़ाव की सुविधा प्रदान की। h. अनुच्छेद 6 में दिए गए प्रावधान के अनुसार परमाणु
• 2005 के समझौते में अंतरराष्ट्रीय नाभिकीय ऊर्जा एजेंसी सामग्री के रूप या पदार्थ में परिवर्तन;

60
i. उपकरणों की पार्टि यों के बीच आपूर्ति , चाहे पार्टि यों या रिएक्टरों का निर्माण करने वाली कंपनियों की सुरक्षा
तीसरे देशों द्वारा उपयोग के लिए या उनके लाभ के लिए; के लिए एक बीमा पूल स्थापित करने पर भारत सहमत

j. बहुपक्षीय परियोजनाओ ं सहित नियंत्रित थर्मोन्यूक्लियर हुआ।

फ्यूजन; तथा • इस प्रकार, भारत किसी घटना के मामले में निवेशकों को

k. पारस्परिक हित के अन्य क्षेत्र जिसपर पक्षों द्वारा परस्पर भारी मुआवजे के भुगतान के खंड से राहत प्रदान करेगा।

सहमति हो सकती हैं। समझौते के कार्यान्वयन के संदर्भ • असैन्य नाभिकीय ऊर्जा सहयोग में द्विपक्षीय और
में, दोनों पक्षों ने सकारात्मक तरीके से संबंधों के माध्यम बहुपक्षीय संबंध स्थापित करने में नागरिक दायित्व
से आगे बढ़ने और विवाद की संभावनाओ ं को दूर करने का मुद्दा भारत और अन्य राज्यों के लिए समान रूप से
के साथ आगे बढ़ने का प्रयास किया। एक चुनौती है। भारत-अमेरिका बातचीत घरेलू और
अंतरराष्ट्रीय स्तर पर नवोन्मेषी समाधानों के साथ आने
की जरूरत का सुझाव देती है।
नागरिक दायित्व (Civil Liability)

• इस समझौते में नागरिक दायित्व का एक मुद्दा विवाद का


कारण भी बना है। भारत-फ्रांस (India-France)

• जैसा कि ऊपर उल्ले ख किया गया है और भोपाल गैस • 1950 में फ्रांसीसी नाभिकीय ऊर्जा आयोग (CEA)
त्रासदी के बाद चल रहे दायित्व के मुद्दे के मद्देनजर, ने असैन्य परमाणु नवाचार पर भारत को तकनीकी
भारतीय संसद ने 2010 में नागरिक दायित्व के लिए सहयोग की पेशकश की, जो 1951 में स्वीकृत हुई। दोनों
परमाणु क्षति अधिनियम अधिनियमित किया, जिसने देशों ने “बेरिलियम-संचालित रिएक्टरों के अनुसंधान
पीड़ितों के मुआवजे के लिए प्रक्रियातंत्र, आपूर्ति कर्ताओ ं और निर्माण के लिए“ एक द्विपक्षीय समझौते पर हस्ताक्षर
पर उपचार दायित्व पर प्रावधान (धारा 17बी) और अन्य किए।
कानूनों के तहत संभावित असीमित देयता 23 (धारा 46) • भारत के 1974 में शांतिपूर्ण परमाणु विस्फोट के बाद, फ्रांस
बनाया। इस घटना की सराहना करने वाला एकमात्र पश्चिमी देश
• कानून के इन पहलु ओ ं ने भारत-अमरीका परमाणु के रूप में उभरा, जो इसे परमाणु क्षेत्र में भारत की प्रगति
समझौते के संचालन में बाधा डाली है। के प्रतिबिं ब के रूप में इं गित करता है।

• इस गतिरोध ने नाभिकीय ऊर्जा को आगे बढ़ाने और • 1974 के शांतिपूर्ण परमाणु विस्फोट के विरोध में अमेरिका
अन्य आपूर्ति कर्ता देशों के साथ द्विपक्षीय संबंध स्थापित और कनाडा द्वारा अपने समझौतों को समाप्त करने के
करने की भारत की योजनाओ ं को प्रभावित किया है। बाद भारत के तारापुर परमाणु संयंत्र को ईंधन की निरंतर
अधिनियम की धारा 17बी ने एक उत्तरदायी ऑपरेटर को आपूर्ति में भारत के लिए फ्रांस का समर्थन पहले से ही
परमाणु दुर्घटना के मामले में आपूर्ति कर्ताओ ं से मुआवजे देखा गया था।
की वसूली करने की अनुमति दी, जिसमें संभावित • 1998 के परमाणु परीक्षणों के बाद फ्रांस ने भी भारत
अनिश्चित समय की देयता सुनिश्चित की गयी है। पर अमेरिकी प्रतिबंधों का खुलकर विरोध किया और
• भारत-अमेरिका 123 समझौते में यह एक प्राथमिक IAEA के साथ भारत की बातचीत के दौरान सकारात्मक
चुनौती बन गयी। यह परमाणु नागरिक दायित्व माहौल बनाने में मदद की।
सम्मेलनों में उपचार के अंतरराष्ट्रीय स्तर पर स्थापित • भारत और फ्रांस ने 30 सितंबर 2008 में एक समझौते
मानदंडों के खिलाफ गयी, जो संचालकों पर एकमात्र पर हस्ताक्षर किए। फ्रांस भारत-विशिष्ट NSG छूट के
जिम्मेदारी डालती है। बाद भारत के साथ असैन्य परमाणु समझौते पर हस्ताक्षर
करने वाला पहला देश बन गया।

• भारत और फ्रांस ने एक “इं डस्ट्रियल वे फारवर्ड“ नामक


मसला कैसे सुलझाया गया? (How the Issue was
Resolved?) समझौते पर भी हस्ताक्षर किए, जिसमें लागत, सुरक्षा
चिं ताओ ं और देनदारियों सहित कई मुद्दे शामिल हैं।
• भारत ने परमाणु क्षति के लिए पूरक मुआवजे पर समझौते के माध्यम से, इलै क्ट्रीसाइट डी फ्रांस (EDF)
सम्मेलन (CSC) नामक एक अंतरराष्ट्रीय संधि पर और न्यूक्लीयर पावर कॉरपोरेशन ऑफ इं डिया लिमिटे ड
हस्ताक्षर किए, जिससे अमेरिकी समकक्षों के बीच चिं ता (NPCIL) ने देनदारियों पर भारत के कानूनों को सुनिश्चित
बढ़ गई। अंतर्राष्ट्रीय संधि और भारत के घरेलू कानूनों के करने की प्रथा को अंतरराष्ट्रीय स्तर पर स्थापित (CAC)
अधिनियमन में एक विसंगति थी। हालांकि, तत्कालीन के दायरे में अधिनियमित किया है।
राष्ट्रपति बराक ओबामा की 2015 की भारत यात्रा के
दौरान दोनों पक्षों के बीच समझौता हो गया था। देश में

61
सहयोग में तेजी लाने और अगले 20 वर्षों में कम से कम
भारत-फ्रांस डील, 2008 (India-France Deal, 2008) 12 इकाइयों पर सहयोग करने का प्रयास करने के लिए
समझौते के अनुसार, फ्रांस 1,650 मेगावाट के छह यूरोपीय एक दूसरी साइट की पहचान करने पर सहमत हुआ।
दबाव रिएक्टर (EPR परमाणु रिएक्टर) का निर्माण करेगा • भारत और रूस दोनों तीसरे देशों के साथ नाभिकीय ऊर्जा
और 9,900 मेगावाट जैतपुर नाभिकीय ऊर्जा परियोजना के शांतिपूर्ण उपयोग पर सहयोग पर विचार करने पर भी
(JNPP) को तेजी से लागू करेगा। सहमत हुए।
उदाहरण के लिए, यह समझौता परमाणु क्षेत्र में उभरते कृषि • नाभिकीय ऊर्जा के शांतिपूर्ण उपयोग में सहयोग साझेदारी
विज्ञान, जीव विज्ञान और पृथ्वी विज्ञान जैसे क्षेत्रों पर केंद्रित का एक महत्वपूर्ण स्तंभ रहा है। रूस भारत का महत्वपूर्ण
था। भागीदार बना हुआ है और इसने 1974 और 1998 के
इस सौदे ने भारत के लिए फ्रांस और अमेरिका दोनों परमाणु परीक्षणों सहित कई महत्वपूर्ण मौकों पर भारत
की सहायता से ITER (इं टरनेशनल थर्मोन्यूक्लियर की मदद की है।
एक्सपेरिमेंटल रिएक्टर - एक अंतरराष्ट्रीय परमाणु
संलयन अनुसंधान और इं जीनियरिं ग 32 मेगाप्रोजेक्ट) का
हिस्सा बनने का अवसर भी पैदा किया। भारत-जापान (India-Japan)

• नाभिकीय ऊर्जा के शांतिपूर्ण उपयोग में सहयोग के लिए


भारत-जापान समझौते पर नवंबर 2016 में हस्ताक्षर किए
भारत-रूस (India-Russia) गए थे और जुलाई 2017 में इसे लागू किया गया था। इस
समझौते में सहयोग के कई पहलु ओ ं को शामिल किया
भारत और रूस के बीच परमाणु सहयोग 1960 के दशक का
गया था, जिसमें रिएक्टरों के डिजाइन और निर्माण में
है, जब भारत और सोवियत संघ ने कई समझौतों पर हस्ता-
सहायता के लिए सूचना और विशेषज्ञता का आदान-
क्षर किए थे। पहले 10 वर्षों में, इन समझौतों को विद्वानों के
प्रदान शामिल था।
अनुसंधानात्मक आदान-प्रदान को सुविधाजनक बनाने जैसे
वैज्ञानिक और तकनीकी सहयोग पर केंद्रित किया गया था।

भारत को लाभ (Benefits to India):


• परमाणु क्षेत्र में भारत के पोखरण-I के बाद भारत और
सोवियत संघ के बीच पहले वास्तविक द्विपक्षीय परमाणु • जापान असैन्य नाभिकीय ऊर्जा बाजार में एक प्रमुख
सहयोग समझौते पर हस्ताक्षर किए गए थे। खिलाड़ी है और एक नाभिकीय ऊर्जा सौदा वेस्टिं गहाउस
• सितंबर 1976 में हस्ताक्षरित एक द्विपक्षीय समझौते के इले क्ट्रिक कॉरपोरेशन जैसे अमेरिका स्थित परमाणु
माध्यम से राजस्थान नाभिकीय ऊर्जा स्टेशन (RAPS- संयंत्र निर्माताओ ं का पूरक होगा।
I&II) के लिए भारी जल की आपूर्ति करने के लिए • यह भारत में नाभिकीय ऊर्जा संयंत्रों की स्थापना की
सोवियत संघ सहमत हुआ था। संभावनाओ ं को आसान और बेहतर बनाने में मदद
• नवंबर 1988 में सोवियत संघ के साथ एक समझौते करेगा क्योंकि इन दोनों समूहों के पास जापानी निवेश है।
पर हस्ताक्षर किए गए थे ताकि दो “"दाबयुक्त व प्रत्येक • जापान का AP100 और EPR रिएक्टरों के रिएक्टर तत्वों
1000 मेगावाट के हल्के जल रिएक्टरों,“ से बने परमाणु और इसके सुरक्षा घटकों के साथ ही संरचनात्मक
बिजलीघर का निर्माण किया जा सके। प्रौद्योगिकी निर्माण पर "एक प्रकार से एकाधिकार" है।
• 1990 के दशक के दौरान, वैश्विक परमाणु संरचनात्मक • समझौते ने दोनों देशों के बीच ’विशेष और विशेषाधिकार
व्यवस्था के साथ भारत के जुड़ाव में सीमित प्रगति के प्राप्त रणनीतिक साझेदारी’ को बढ़ावा दिया है, जिसमें
समय, रूस भारत के लिए परमाणु ईंधन का एक प्रमुख पिछले एक दशक में भारी वृद्धि देखी गई है।
आपूर्ति कर्ता बना रहा।
• यह भारत के लिए एक सफलता थी क्योंकि यह जापान
• 5 दिसंबर 2008 को, दोनों देशों ने कुडनकुलम में चार के साथ असैन्य परमाणु सहयोग संधि में प्रवेश करने
अतिरिक्त इकाइयों के निर्माण और नई साइटों को वाला पहला गैर-NPT हस्ताक्षरकर्ता बन गया, इस प्रकार
विकसित करने के लिए एक समझौते पर हस्ताक्षर किए। परमाणु हथियार शक्ति के रूप में भारत की वास्तविक
• रूस ने ’नाभिकीय ऊर्जा भागीदारी के लिए वैश्विक केंद्र’ स्थिति को मान्यता दी गई।
स्थापित करने के भारत के निर्णय का स्वागत किया। यह
2010 में स्थापित किया गया था और दोनों देश भविष्य के
सहयोग पर चर्चा करने के लिए सहमत हुए। जापान को लाभ (Benefits to Japan):

• कुडनकुलम के साथ मिलकर, भारत रूस के साथ • 2011 फुकुशिमा परमाणु आपदा के बाद, परमाणु उद्योग

62

AKASH SINGH
को लगभग वैश्विक संकट का सामना करना पड़ा। सख्त • यह भारत-रूस सौदे के तहत पहली पहल है जहां दोनों
सुरक्षा नियमों ने नाभिकीय ऊर्जा संयंत्रों के निर्माण की देशों ने किसी तीसरे देश में नाभिकीय ऊर्जा परियोजना
लागत में वृद्धि की है, और कुछ देश नए परमाणु रिएक्टरों शुरू करने का फैसला किया है।
के बारे में अधिक सतर्क हो गए हैं।

• भारत के साथ समझौते से जापान असैन्य परमाणु उद्योग


को पुनर्जीवित करने पर असर पड़ने की संभावना है, जो नाभिकीय ऊर्जा से संबंधित मुद्दे (Issues Related
फुकुशिमा की घटना के झटके से अभी तक उबर नहीं to Nuclear Energy)
पाया है। हिताची, मित्सुबिशी और तोशिबा सभी नए संयंत्रों 1. योजना और संचालन के बीच लं बा समय अंतराल (Long
के निर्माण के बजाय मौजूदा संयंत्रों (सबसे निष्क्रिय) की Time Lag Between Planning and Operation)
मरम्मत और रखरखाव पर ध्यान केंद्रित कर रहे हैं।
• परमाणु रिएक्टर की योजना और संचालन के बीच के
समय के अंतराल में साइट की पहचान करने, साइट
परमिट प्राप्त करने, जमीन खरीदने या पट्टे पर देने,
सौदे में समस्याएं और समाधान (Issue in the Deal and
Resolution): निर्माण परमिट प्राप्त करने, निर्माण के लिए वित्तपोषण
और बीमा प्राप्त करने, ट्रांसमिशन स्थापित करने, बिजली
• यह एक विवादित पहलू ’निष्प्रभावी खंड’ है, जो परमाणु खरीद पर बातचीत करने, समझौता करने, परमिट प्राप्त
परीक्षणों का आयोजन करने वाले भारत में पक्षों के बीच करने, संयंत्र का निर्माण करने, इसे ट्रांसमिशन से जोड़ने
सहयोग को स्वचालित रूप से निलं बित कर देता है। और अंतिम संचालन लाइसेंस प्राप्त करने का समय
• “हालांकि, यह संधि के लिए एक अलग ज्ञापन संलग्न शामिल किया गया है।
करके हल किया गया था, जो निर्दिष्ट करता है कि जापान • एक नाभिकीय ऊर्जा संयंत्र के निर्माण में योजना चरण से
भारत के साथ सहयोग को निलं बित कर सकता है यदि ले कर संचालन तक लगभग 14 वर्ष लगते हैं।
भारत परमाणु आपूर्ति कर्ता समूह (NSG) के लिए अपनी
• दूसरी ओर, पवन और सौर उर्जा के लिए यह
'कोई परीक्षण नहीं' संकल्प का उल्लं घन करता है।“
अवसंरचनात्मक समय, योजना चरण से संचालन तक
इस प्रकार, यह समझौता दोनों पक्षों के बीच रणनीतिक औसतन केवल 2 से 5 वर्ष ही हैं।
सहयोग की गति को तेज करते हुए भारत और जापान के ऊर्जा • रूफटॉप सोलर पीवी परियोजनाएं केवल 6 महीने की
सुरक्षा लक्ष्यों को और बढ़ावा देगा क्योंकि वे एक सुरक्षित समयावधि तक सीमित की गयी हैं।
हिन्द-प्रशांत की स्थिति को सुनिश्चित करने में बड़ी भूमिका
निभाते हैं।
2. लागत (Cost)

• 2018 में ले जार्ड पर आधारित एक नए परमाणु संयंत्र के


भारत-रूस-बांग्लादेश (एक त्रिपक्षीय समझौता)
लिए ऊर्जा की स्तरीय लागत (LCOE) 151 डॉलर (112 से
(India-Russia-Bangladesh (A Tripartite
189)/मेगावाट घंटा थी। यह तटवर्ती हवा (ऑनशोर विं ड)
Agreement))
के लिए 43 डॉलर (29 से 56)/ मेगावाट घंटा और उसी
• बांग्लादेश में रूपपुर नाभिकीय ऊर्जा संयंत्र पर एक साथ स्रोत से उपयोगिता-पैमाने पर सोलर पीवी के लिए 41
काम करने के लिए रूस और बांग्लादेश के साथ एक डॉलर (36 से 46)/ मेगावाट घंटा के साथ तुलना करता
त्रिपक्षीय समझौते पर हस्ताक्षर करना भारत के लिए है।
सबसे महत्वपूर्ण था।

• इस ऐतिहासिक समझौते पर मार्च 2018 में हस्ताक्षर किए


3. उच्च परिचालन लागत (High Operating Costs)
गए और यह वैश्विक परमाणु समुदाय में एक जिम्मेदार
परमाणु शक्ति के रूप में भारत की स्वीकृति में बदलाव • बाजार की चुनौतीपूर्ण स्थितियों ने परमाणु उद्योग के लिए
का संकेत था। संघर्ष व प्रतिस्पर्धा के स्तर में वृद्धि कर दी है। रखरखाव,
• NPCIL क्षमता निर्माण, स्थापना और कार्य में सहायता स्टाफिंग स्तर, ऑपरेटर प्रशिक्षण और संयंत्र निरीक्षण पर
करने और रूस को सहयोग प्रदान करने के लिए भारतीय सख्त नियम उद्योग के लिए एक वित्तीय बोझ बन गए
पक्ष की ओर से कमांडिंग अथॉरिटी है, जो उपकरण के हैं।
डिजाइन, निर्माण और आपूर्ति और सुविधा के निर्माण में
अग्रणी होगा।

63
4. हथियार प्रसार जोखिम (Weapons Proliferation कचरा जमा करता है, रेडियोधर्मी रिसाव का खतरा उतना
Risk) ही अधिक होता है, जो जल की आपूर्ति , फसलों, जानवरों
और मनुष्यों को नुकसान पहुंचा सकता है।
• जलवायु परिवर्तन पर अंतरसरकारी पैनल (IPCC)
“रूबस्ट एवीडेन्स एं ड हाई एग्रीमेंट“ के साथ इस तथ्य को
पहचानता है कि परमाणु हथियार प्रसार चिं ता एक बाधा
परमाणु अप्रसार (Nuclear Non-proliferation)
और संभावित जोखिम है।
• परमाणु हथियारों के अप्रसार पर संधि (NPT) वैश्विक
अप्रसार व्यवस्था की आधारशिला है। इस व्यवस्था
5. मेल्टडाउन रिस्क (Meltdown Risk) में अंतर्राष्ट्रीय नाभिकीय ऊर्जा एजेंसी (IAEA) सुरक्षा
उपाय प्रणाली, द्विपक्षीय और बहुपक्षीय परमाणु सहयोग
• नाभिकीय ऊर्जा संयंत्र के लिए एक परमाणु दुर्घटना
समझौतों का एक नेटवर्क , बहुपक्षीय निर्यात नियंत्रण की
या पूर्णगलन (मेल्टडाउन) सबसे खराब स्थिति होती
प्रणाली और 1887 का प्रस्ताव सहित संयुक्त राष्ट्र सुरक्षा
है, जिससे पर्यावरण में व्यापक रूप से घातक विकिरण
परिषद के प्रस्तावों की एक श्रृंखला शामिल है।
प्रसारित होता है जो सैकड़ों मील दूर तक फैल सकता है
और वन्यजीवों के साथ-साथ मनुष्यों को भी प्रभावित
कर सकता है। विकिरण को देखा, सूंघा या चखा नहीं जा
सकता है और इसके संपर्क में आने से गंभीर दुष्प्रभाव हो NPT की उत्पत्ति (Origins of the NPT):
सकते हैं।
• अमेरिकी राष्ट्रपति ड्वाइट डी आइजनहावर के परमाणु
• मेल्टडाउन या तो विनाशकारी रहा है (जैसा कि 1986 शांति पहल के लिए, अंतरराष्ट्रीय नाभिकीय ऊर्जा एजेंसी
में चेरनोबिल, रूस में अनुभव किया गया था; 2011 (IAEA) का निर्माण, IAEA सुरक्षा उपायों का विकास, और
में फुकुशिमा, जापान में तीन रिएक्टर) या इसके नाभिकीय ऊर्जा के शांतिपूर्ण उपयोग का विस्तार।
हानिकारक (थ्री-माइल आइलैं ड, सेंट-लॉरेंट फ्रांस 1980
• 1965 में, जिनेवा निरस्त्रीकरण सम्मेलन ने परमाणु
में) प्रभाव अनुभव किए गए हैं।
अप्रसार संधि के मसौदे पर विचार करना शुरू किया।
सम्मेलन ने 1968 में अपनी वार्ता पूरी की, और 1 जुलाई,
1968 को हस्ताक्षर करने के लिए(NPT) को खोला गया।
6. माइनिं ग लं ग कैंसर का जोखिम (Mining Lung Cancer
Risk) • NPT 5 मार्च, 1970 को लागू हुआ।

• यूरनि
े यम का खनन करने से बड़ी संख्या में खनिकों में
फेफड़ों का कैंसर हो जाता है क्योंकि यूरनि
े यम खानों में
प्राकृतिक रेडॉन गैस होती है, जिनमें से कुछ क्षय उत्पाद तीन स्तंभ (The Three Pillars)
कैंसरकारी होते हैं। NPT के प्रावधान तीन मुख्य स्तंभों पर आधारित हैं:

• अप्रसार,
7. कार्बन-समतुल्य उत्सर्जन और वायु प्रदूषण (Carbon- • नाभिकीय ऊर्जा का शांतिपूर्ण उपयोग, और
Equivalent Emissions and Air Pollution)
• निरस्त्रीकरण।
• नाभिकीय ऊर्जा संयंत्र में शून्य- या शून्य के निकट
उत्सर्जन जैसी कोई संकल्पना नहीं होती है। संयंत्र के
लिए आवश्यक यूरनि
े यम के निरंतर खनन और शोधन
अप्रसार (Non-Proliferation):
के कारण मौजूदा संयंत्रों से भी उत्सर्जन होता रहता।
• NPT के अनुच्छेद 1 के तहत परमाणु हथियार वाले राज्य
किसी भी प्राप्तकर्ता को परमाणु हथियार या अन्य परमाणु
8. अपशिष्ट जोखिम (Waste Risk) विस्फोटक उपकरणों को हस्तांतरित नहीं करने या किसी
भी तरह से परमाणु हथियार के निर्माण या अधिग्रहण में
• अंत में, परमाणु संयंत्रों से खपत की हुई ईंधन छड़ें
किसी गैर-परमाणु हथियार राज्य को सहायता, प्रोत्साहित
रेडियोधर्मी अपशिष्ट होती हैं। अधिकांश ईंधन छड़ें
या प्रेरित न करने की प्रतिज्ञा करते हैं।
रिएक्टर के समान स्थान पर संग्रहित की जाती हैं। इसने
कई देशों में सैकड़ों रेडियोधर्मी अपशिष्ट स्थलों को जन्म • NPT एक परमाणु हथियार राज्य को एक ऐसे राज्य के
दिया है जिनकी देखभाल की जानी चाहिए और उन्हें वित्त रूप में परिभाषित करता है जिसने 1 जनवरी 1967 से पहले
पोषित किया जाना चाहिए। यह जितना अधिक परमाणु परमाणु हथियार या किसी अन्य परमाणु उपकरण का

64
निर्माण और विस्फोट किया था। ये राज्य चीन, फ्रांस, रूस, का सबसे व्यापक रूप से पालन किया जाने वाला अप्रसार
यूनाइटे ड किंगडम और अमेरिका हैं। अन्य सभी राज्य या हथियार नियंत्रण समझौता बनाता है।
संधि के तहत गैर-परमाणु हथियार वाले राज्य हैं।
• केवल चार राज्यों भारत, इज़राइल, दक्षिण सूडान
• NPT के अनुच्छेद 2 के तहत, गैर-परमाणु हथियार वाले पाकिस्तान ने संधि पर हस्ताक्षर/अनुमोदन/पुष्टि नहीं
राज्य परमाणु हथियारों या अन्य परमाणु विस्फोटक की है। केवल एक राज्य उत्तर कोरिया ने NPT से हटने या
उपकरणों पर नियंत्रण प्राप्त या प्रयोग नहीं करने और अपनी वापसी की घोषणा की है। भारत हमेशा NPT को
ऐसे उपकरणों के निर्माण में सहायता या सहयोग प्राप्त न भेदभावपूर्ण मानता था और उसने इस पर हस्ताक्षर करने
करने की प्रतिज्ञा करते हैं। से इनकार कर दिया था।

• संधि के अनुच्छेद 3 के तहत, गैर-परमाणु हथियार वाले


राज्य IAEA सुरक्षा उपायों को स्वीकार करने की प्रतिज्ञा
सफलताएं (Successes)
करते हैं ताकि यह सत्यापित किया जा सके कि उनकी
परमाणु गतिविधियां केवल शांतिपूर्ण उद्देश्यों को पूरा • NPT द्वारा प्रदान किए गए व्यापक लाभ ने अंतर्राष्ट्रीय
करती हैं। शांति और सुरक्षा में वृद्धि की है।

• NPT अंतरराष्ट्रीय परमाणु अप्रसार व्यवस्था की


आधारशिला है जिसमें प्रसार को रोकने के लिए कानूनी
शांतिपूर्ण उपयोग (Peaceful Uses):
प्रतिबंध, सुरक्षा उपाय, निर्यात नियंत्रण, अंतर्राष्ट्रीय
सहयोग और अन्य तंत्र शामिल हैं।
• NPT का अनुच्छेद 4 शांतिपूर्ण उद्देश्यों के लिए नाभिकीय
ऊर्जा विकसित करने और अंतर्राष्ट्रीय सहयोग से लाभ • नाभिकीय ऊर्जा के शांतिपूर्ण अनुप्रयोगों की संख्या और
उठाने और अपने अप्रसार दायित्वों के अनुरूप सभी पक्षों महत्व में वृद्धि हुई है, क्योंकि शांतिपूर्ण परमाणु कार्यक्रमों
के अधिकार को स्वीकार करता है। अनुच्छेद IV भी ऐसे वाले राज्यों की संख्या और नाभिकीय ऊर्जा के शांतिपूर्ण
सहयोग को प्रोत्साहित करता है। लाभों को अधिक से अधिक लोगों तक पहुंचाने के उद्देश्य
से अंतर्राष्ट्रीय सहयोग के स्तर में वृद्धि हुई है।

निरस्त्रीकरण (Disarmament):
एनपीटी में आईएईए की भूमिका (Role of IAEA In
• NPT के अनुच्छेद 5 के तहत, सभी पक्ष परमाणु हथियारों NPT):
की दौड़, परमाणु निरस्त्रीकरण, और सामान्य और पूर्ण
निरस्त्रीकरण से संबंधित प्रभावी उपायों पर सद्भावनापूर्ण • संयुक्त राष्ट्र प्रणाली के भीतर एक स्वायत्त अंतरराष्ट्रीय
बातचीत करने का वचन देते हैं। संगठन के रूप में 1957 में स्थापित, IAEA इसके शांतिपूर्ण
उपयोग की पुष्टि करते हुए समाज में परमाणु प्रौद्योगिकी
• ये स्तंभ आपस में जुड़े हुए हैं और परस्पर प्रबल हैं। एक
के योगदान को अधिकतम करने के लिए कार्यक्रम
प्रभावी अप्रसार व्यवस्था जिसके सदस्य अपने दायित्वों
चलाता है।
का पालन करते हैं, निरस्त्रीकरण पर प्रगति के लिए एक
आवश्यक आधार प्रदान करता है और नाभिकीय ऊर्जा • अंतर्राष्ट्रीय नाभिकीय ऊर्जा एजेंसी (IAEA) NPT का एक
के शांतिपूर्ण उपयोग पर अधिक सहयोग संभव बनाता है। पक्ष नहीं है। हालांकि, इसे इसके तहत एक महत्वपूर्ण
सत्यापन भूमिका सौंपी जाती है। NPT के तहत, IAEA की
• शांतिपूर्ण परमाणु प्रौद्योगिकी का लाभ प्राप्त करने के
अंतरराष्ट्रीय सुरक्षा उपायों के निरीक्षणालय के रूप में
अधिकार के साथ अप्रसार की जिम्मेदारी आती है।
एक विशिष्ट कार्यभूमिका रहती है।
• निरस्त्रीकरण पर प्रगति अप्रसार व्यवस्था को मजबूत
• अंतर्राष्ट्रीय नाभिकीय ऊर्जा एजेंसी (IAEA) दुनिया भर में
करने और दायित्वों के अनुपालन को लागू करने के
शांति और समृद्धि के लिए नाभिकीय ऊर्जा के योगदान
प्रयासों को पुष्ट करती है, जिससे शांतिपूर्ण परमाणु
को बढ़ाने के लिए काम करती है, हालांकि यह सुनिश्चित
सहयोग की सुविधा मिलती है।
करती है कि इसकी परमाणु सामग्री को परमाणु हथियारों
या अन्य विस्फोटक परमाणु उपकरणों की ओर मोड़ा
नहीं गया है।
एनपीटी की सफलताएं और चुनौतियां (NPT Successes
• एक विकसित IAEA सुरक्षा उपाय प्रणाली पर निर्भरता
and Challenges):
संधि के अनुच्छेद 3 में उस तंत्र के रूप में परिलक्षित होती है
• 1970 में NPT पालन करने वाले 43 दल थे जिनकी संख्या जिसके द्वारा गैर-परमाणु-हथियार राज्य अपने शांतिपूर्ण
अब बढ़कर लगभग 190 तक हो गयी है, जो इसे इतिहास उपयोग उपक्रमों के पालन को सत्यापित करते हैं।

65
AKASH SINGH
• अनुच्छेद 3 गैर-परमाणु-हथियार वाले राज्यों को परमाणु अप्रसार दायित्वों के साथ गैर-अनुपालन (Noncompliance
निर्यात के लिए IAEA सुरक्षा उपायों को लागू करने की With Non-Proliferation Obligations)
आवश्यकता के द्वारा निर्यात नियंत्रण के लिए सुरक्षा
उपायों को जोड़ता है। • कुछ NPT गैर परमाणु हथियार राज्यों द्वारा अप्रसार
दायित्वों के साथ गैर-अनुपालन जारी है जो संधि के लिए
• ज़ांगर समिति और परमाणु आपूर्ति कर्ता समूह दो निकाय
प्रमुख चुनौतियों में से एक है।
हैं जो नाभिकीय ऊर्जा के शांतिपूर्ण उपयोग पर सहयोग
को बाधित किए बिना परमाणु और परमाणु संबंधी निर्यात • NPT पक्षकारों का भारी बहुमत अपने अप्रसार दायित्वों
को शांतिपूर्ण प्रयोजन से हथियारों के उद्देश्यों में बदलने से का पालन करता है, ले किन निरंतर अनुपालन चुनौतियां
रोकने के लिए निर्यात नियंत्रण विकसित करने के लिए अंतर्राष्ट्रीय समुदाय को सतर्क रहने की आवश्यकता को
प्रतिबद्ध हैं। स्पष्ट करती हैं।

• गैर-अनुपालन वाले देशों को अनुपालन में वापस लाने


के अंतर्राष्ट्रीय प्रयासों को आगे बढ़ाने के लिए अप्रसार
आईएईए (IAEA) व्यवस्था मौजूद है।

IAEA अपने 164 सदस्य राज्यों के साथ काम करता है। • भारत की एक जिम्मेदार परमाणु देश के रूप में मान्यता
देने में असमर्थता (Inability to Recognize India as
IAEA का मुख्यालय वियना में वियना अंतर्राष्ट्रीय केंद्र,
a Responsible Nuclear State): संधि और मान्यता
ऑस्ट्रिया में है।
प्राप्त परमाणु देशों ने भारत को एक जिम्मेदार देश के रूप
परिचालन संपर्क और क्षेत्रीय कार्यालय जिनेवा, स्वि- में मान्यता देने की इच्छा नहीं दिखाई है, जिसने इसे समग्र
ट्जरलैं ड; न्यूयॉर्क , यूएसए; टोरंटो, कनाडा; और टोक्यो, रूप से अपनाने में बाधा उत्पन्न की है।
जापान में स्थित हैं।
• जिम्मेदारी और संसाधनों के बीच बेमेल (Mismatch
IAEA वियना और सीबर्सडॉर्फ , ऑस्ट्रिया और मोनाको Between Responsibility and Resources): IAEA
में वैज्ञानिक प्रयोगशालाएं चलाता है। सुरक्षा उपाय प्रणाली अपने कार्यभार और अपने संसाधनों
यह अनुसंधान केंद्रों का भी समर्थन करता है, जैसे ट्रा- के बीच बढ़ते असंतुलन की चुनौती का भी सामना कर
इस्टे, इटली में एक रही है। जैसे-जैसे एजेंसी की जिम्मेदारियां बढ़ी हैं, उसके
संसाधनों में आनुपातिक रूप से वृद्धि नहीं हुई है।
आईएईए की भूमिका (Role of IAEA)
• अघोषित परमाणु सामग्री और गतिविधियों का पता
IAEA अपने सदस्य देशों की सहायता करता है और वै-
लगाना (Detecting Undeclared Nuclear
ज्ञानिक और तकनीकी जानकारी के आदान-प्रदान
Materials and Activities): IAEA के पास एक देश में
को बढ़ावा देता है।
सभी परमाणु सामग्री (यानी देश की शुद्धता और पूर्णता
IAEA के कार्य अंतरराष्ट्रीय परमाणु सुरक्षा और सुरक्षा की घोषणा) के शांतिपूर्ण उपयोग को सत्यापित करने
व्यवस्था बनाने और इसे मजबूत करने के सहयोगी के लिए एक व्यापक सुरक्षा समझौते के तहत अधिकार
प्रयास हेतु और राज्यों की उनके अप्रसार (NPT के मौजूद है, इस तरह के समझौते के तहत IAEA के लिए
तहत उपक्रम) की पूर्ति पुष्टि के लिए एक फ्रेमवर्क कि उपलब्ध उपकरण सीमित हैं। दरअसल, IAEA के लिए
स्थापना भी शामिल है सबसे बड़ी चुनौतियों में से एक अघोषित परमाणु सामग्री
और गतिविधियों का पता लगाना है।
IAEA सचिवालय में 100 से अधिक देशों से लगभग
2500 बहु-अनुशासनात्मक पेशेवरों और सहयोगी • द नॉन स्टे ट एक्टर थ्रेट (The Non-State Actor
कर्मि यों की एक टीम है Threat):

» संयुक्त राज्य अमेरिका में 11 सितंबर, 2009 के हमलों


चुनौतियाँ (Challenges) सहित बड़े पैमाने पर नागरिक हताहतों के कारण होने
वाले आतंकवादी हमलों के 21वीं सदी के अनुभव का
• NPT और बृहत्तर अप्रसार व्यवस्था स्थिर नहीं रहती मतलब है कि गैर-राज्य अभिकर्ताओ ं को परमाणु
है। पिछले चालीस वर्षों में व्यवस्था ने कई प्रकार की हथियारों या परमाणु या रेडियोलॉजिकल सामग्री, या
चुनौतियों का सामना किया है और खुद को अनुकूलित परमाणु सुविधा पर हमला करना।
किया है।
» ए.क्यू. खान नेटवर्क का खुलासा और इसकी अवैध
गतिविधियों की सीमा ने आगे प्रसार के लिए गैर-राज्य
अभिकर्ताओ ं की क्षमता को स्पष्ट कर दिया।

66
सुरक्षा उपायों के अंतर्गत कौनसी सामग्री है? (What भारत का परमाणु सिद्धांत
Material is Subject to Safeguards?)
(India’s Nuclear
रक्षोपाय प्रणाली का उद्देश्य है: उस परमाणु सामग्री के
दिक्परिवर्तन का पता लगाना है जिसका संभावित रूप Doctrine):
से परमाणु हथियार उद्देश्यों के लिए दुरुपयोग किया जा
सकता है। ऐसी सामग्री में संवृद्धित यूरनि
े यम, प्लूटोनि-
परमाणु सिद्धांत क्या है? (What is a Nuclear
Doctrine?)
यम और यूरनि
े यम-233 शामिल हैं।

सुरक्षा उपायों के अधीन अन्य प्रकार की परमाणु • एक परमाणु सिद्धांत यह बताता है कि एक परमाणु
सामग्री में प्राकृतिक यूरनि
े यम और अवक्षयित यूरनि
े - हथियार वाला देश शांति और युद्ध के दौरान अपने
यम शामिल हैं, अवक्षयित यूरनि
े यम सामग्री का इस्ते- परमाणु हथियारों का इस्तेमाल कैसे करेगा।
माल आमतौर पर, उदाहरण के लिए, अस्पताल, उद्योग • परमाणु सिद्धांत विरोधियों के खिलाफ प्रतिरोध स्थापित
और कृषि में विकिरण स्रोतों के लिए परिरक्षण के रूप करने में मदद करता है।
में किया जाता है।

रेडियोधर्मी स्रोत जो परमाणु सामग्री युक्त नहीं हैं सुरक्षा


उपायों के अधीन नहीं हैं और रक्षोपाय समझौते के तहत ऐतिहासिक पृष्ठभूमि (Historical Background)
इनकी IAEA को सूचना देने की आवश्यकता नहीं है।
• नेहरू परमाणु हथियारों के खिलाफ थे इसलिए उन्होंने
व्यापक परमाणु निरस्त्रीकरण के लिए महाशक्तियों से
गुहार लगाई। हालाँकि, परमाणु शस्त्रागार लगातार ही
एनपीटी से संबंधित कुछ शब्द (Some Terms Related to बढ़ता रहा।
NPT)
• 1998 के परमाणु परीक्षणों के बाद, भारत ने एक विस्तृत
अतिरिक्त प्रोटोकॉल (एपी) (Additional Protocols- APs): परमाणु नीति अपनाई।

• ये वैश्विक परमाणु अप्रसार उद्देश्यों में योगदान के रूप • परमाणु सिद्धांत को अपनाने के बाद से, भारत ने लगातार
में प्रभावशीलता को मजबूत करने और सुरक्षा उपायों कहा है कि अगर निरोध विफल हो जाता है, उसके परमाणु
प्रणाली की दक्षता में सुधार करने के लिए डिज़ाइन किए हथियार चौंका देने वाले और दंडात्मक जवाबी कार्रवाई
गए प्रोटोकॉल हैं। पर आधारित थे।

• वे IAEA को घोषित परमाणु सामग्री के गैर-विचलन


और अघोषित परमाणु सामग्री और गतिविधियों की
भारत का परमाणु सिद्धांत (India’s Nuclear
अनुपस्थिति को आश्वस्त करने के लिए बेहतर उपकरणों
Doctrine)
से लै स करते हैं।
1. भारत के परमाणु सिद्धांत का मूल सिद्धांत “नो फर्स्ट यूज“
है। नीति के मुताबिक, सिर्फ भारतीय क्षेत्र या भारतीय
व्यापक सुरक्षा समझौते (Comprehensive safeguards सेनाओ ं पर कहीं भी परमाणु हमले के खिलाफ जवाबी
agreemants-CSAs): कार्रवाई में परमाणु हथियारों का इस्तेमाल किया जाएगा।
अगर कोई देश परमाणु मिसाइल से भारत पर हमला
• सभी शांतिपूर्ण परमाणु गतिविधियों में सभी परमाणु करता है तो उसकी जवाबी कार्रवाई काफी बड़े पैमाने
सामग्री पर सुरक्षा उपायों को लागू करने के लिए देशों पर और भयानक होगी ताकि दुश्मन को अस्वीकार्य
और IAEA के बीच समझौता होता है। नुकसान का अनुभव होगा और वह आसानी से उबर नहीं
• एक रिएक्टर, महत्वपूर्ण सुविधा, रूपांतरण संयंत्र, ईंधन पाएगा।
निर्माण संयंत्र, पुनर्संसाधन संयंत्र, आइसोटोप पृथक्करण 2. भारत को एक विश्वसनीय न्यूनतम प्रतिरोध बनाने और
संयंत्र या अलग भंडारण स्थापना; या कोई भी स्थान बनाए रखने की आवश्यकता है। इसमें शामिल है;
जहां एक प्रभावी किलोग्राम से अधिक मात्रा में परमाणु
• दुश्मन को अस्वीकार्य क्षति पहुंचाने के लिए पर्याप्त और
सामग्री का परंपरागत रूप से उपयोग किया जाता है।
जीवित रहने योग्य परमाणु बल और ताकतें ।

• परमाणु बलों को हमेशा प्रचालनात्मक एवं सक्रिय रूप से


तैयार रहना चाहिए।

• प्रभावी खुफिया और पूर्व चेतावनी क्षमताएं होनी चाहिए।

67
• शत्रु के प्रति प्रतिरोधक क्षमता की प्रतिक्रिया होनी चाहिए। एनएफयू के लाभ (Advantages of NFU)

3. गैर-परमाणु राज्यों के खिलाफ परमाणु हथियारों का


• नो फर्स्ट यूज (NFU) नीति सामरिक हथियारों और एक
इस्तेमाल नहीं किया जाएगा।
जटिल कमांड और नियंत्रण प्रणाली के बिना प्रतिबंधित
4. अगर भारत या भारतीय सुरक्षा बलों के खिलाफ कोई परमाणु हथियार कार्यक्रम की सुविधा प्रदान करती है।
रासायनिक या जैविक हमला होता है, तो भारत अपनी
• सिद्धांत अनावश्यक अराजकता की संभावना को भी
प्रतिक्रिया में परमाणु हमले का विकल्प खुला रखेगा।
कम करता है क्योंकि परमाणु उपयोग को बढ़ाने की
5. भारत परमाणु मुक्त विश्व बनाने की वैश्विक पहल का जिम्मेदारी विरोधी पर होती है।
समर्थन करना जारी रखेगा और भेदभाव मुक्त परमाणु
• सिद्धांत हेयर-ट्रिगर अलर्ट पर हथियारों की तैनाती से
निरस्त्रीकरण के विचार को आगे बढ़ाएगा।
बचने और हथियारों की दौड़ को नियंत्रण में रखकर
6. परमाणु और मिसाइल से संबंधित सामग्री और परमाणु उपयोग की संभावना को कम करता है
प्रौद्योगिकियों के निर्यात पर सख्त नियंत्रण की निरंतरता,
• सिद्धांत का कड़ाई से पालन करने से परमाणु आपूर्ति कर्ता
विखंडनीय सामग्री कटऑफ संधि वार्ता में भागीदारी,
समूह (NSG) और संयुक्त राष्ट्र सुरक्षा परिषद (UNSC) में
और परमाणु परीक्षणों पर रोक का निरंतर पालन किया
सदस्यता हासिल करने के भारत के प्रयासों को मजबूती
जाऐगा।
मिल सकती है।

परमाणु सिद्धांत के तहत निर्णय निर्माण


एनएफयू के खिलाफ तर्क (Arguments Against NFU)
(Decision-Making Under Nuclear
Doctrine) • परमाणु हथियारों को अक्सर पारंपरिक हीनता के लिए
• दुश्मन के खिलाफ परमाणु कार्रवाई करने का अधिकार एक मारक के रूप में देखा जाता है क्योंकि निम्न पक्ष
जनता के चुने हुए प्रतिनिधियों यानी देश का राजनीतिक परमाणु प्रतिक्रिया की धमकी देकर पारंपरिक हमले को
नेतृत्व द्वारा ही लिया जाएगा। हालांकि इसके लिए रोकने की कोशिश करेगा।
न्यूक्लियर कमांड अथॉरिटी का सहयोग जरूरी होगा। • परमाणु हथियारों के नो-फर्स्ट-यूज (NFU) के विचार को
• दूसरे शब्दों में; नौकरशाही दुश्मन पर परमाणु हमले का कुछ परमाणु हथियार देशों द्वारा खारिज कर दिया गया है
फैसला करने के लिए अधिकृत नहीं है। और केवल घोषणात्मक स्तर पर ही स्वीकार किया गया
है, यदि अन्य सभी द्वारा नहीं।
• न्यूक्लियर कमांड अथॉरिटी (NCA) में एक कार्यकारी
परिषद और एक राजनीतिक परिषद शामिल है। • भारत में NFU नीति पर सवाल उठाया गया है क्योंकि
राजनीतिक परिषद का अध्यक्ष प्रधान मंत्री होता है। यह यह पाकिस्तान को भारत के विकल्पों को सैन्य रूप से
एकमात्र निकाय है जो परमाणु हथियारों के उपयोग को प्रतिबंधित करते हुए पहल करने की अनुमति देता है और
अधिकृत कर सकता है। भारत को एक नुकसानदेह स्थिति में डालता है।

• कार्यकारी परिषद का नेतृत्व राष्ट्रीय सुरक्षा सलाहकार • पाकिस्तान की निम्न परमाणु सीमा और भारत में उप-
(NSA) करता है। कार्यकारी परिषद परमाणु कमान परंपरागत संघर्ष को बढ़ावा देने के लिए अपने परमाणु
प्राधिकरण (NSA) के निर्णय के लिए इनपुट प्रदान छत्र का उपयोग करने की उसकी नीति भारत की ’पहले
करती है और राजनीतिक परिषद द्वारा दिए गए निर्देशों उपयोग न करने’ की नीति पर बहस का प्रमुख कारण है।
को निष्पादित करती है।

• यह सच है कि दुश्मन पर परमाणु हमले का फैसला


भारत के लिए एनएफयू को छोड़ने के निहितार्थ
करने का अंतिम अधिकार केवल प्रधान मंत्री के पास है। (Implications of Abandoning NFU for India)
ले किन सीक्रेट कोड होने के बावजूद प्रधानमंत्री अकेले
फैसला नहीं ले सकते। • NFU नीति को वापस ले ने और घोषणा करने से एक
जिम्मेदार परमाणु शक्ति के रूप में भारत की स्थिति
• इस प्रकार, ऊपर वर्णि त बिं दु स्पष्ट करते हैं कि भारत का
प्रभावित हो सकती है।
परमाणु सिद्धांत किसी देश को धमकाने या आक्रमण
करने के लिए नहीं बल्कि देश की संप्रभुता और सीमाओ ं • ऐसा कदम परमाणु निरस्त्रीकरण के सार्वभौमिक लक्ष्य
की रक्षा करने के लिए है। के प्रति भारत की प्रतिबद्धता को समाप्त कर देगा और
उपमहाद्वीप में क्षेत्रीय संतुलन को बिगाड़ देगा।

• इसके अलावा, सिद्धांत को संशोधित करके चीन की


विस्तारवादी नीतियों को रोका नहीं जा सकता है,

68

AKASH SINGH
सिद्धांत को छोड़ने का निर्णय चीन को उकसावे का एक सहमत उद्देश्यों को आगे बढ़ाने के लिए क्या आवश्यक है।
जानबूझकर संकेत भेज सकता है।
• NWS ने एक विखंडनीय सामग्री उत्पादन कट-ऑफ
• परमाणु पूर्वधारणा एक महंगी नीति है क्योंकि इसके लिए संधि (FMCT) पर बातचीत करने और हथियारों के उद्देश्यों
हथियारों और वितरण प्रणालियों और खुफिया, निगरानी के लिए विखंडनीय सामग्री के उत्पादन पर स्वैच्छिक,
और टोही (ISR) बुनियादी ढांचे में बड़े पैमाने पर निवेश की एकतरफा रोक का पालन करने के लिए प्रतिबद्ध किया
आवश्यकता होती है। है।

• भारत को परमाणु हथियारों की एक बड़ी सूची की • एनडब्ल्यूएस प्रतिबद्धताओ ं और दायित्वों के तहत


आवश्यकता होगी, विशेष रूप से विरोधियों की परमाणु निर्धारित तीन आवश्यक समझौते हैंः
क्षमताओ ं को समाप्त करने के लिए कई हथियारों से
युक्त अपनी परमाणु संपत्ति को लक्षित वृद्धि करने की » परमाणु अप्रसार संधि (NPT) स्वयं,
आवश्यकता होगी। » परमाणु अप्रसार और निरस्त्रीकरण के सिद्धांतों और
उद्देश्यों पर 1995 का NPT समीक्षा सम्मेलन समझौता,
और
आगे का रास्ता (Way Forward)
» 2001 NPT समीक्षा सम्मेलन समझौता “NPT के अनुच्छेद
• चूंकि सुरक्षा एक गतिशील अवधारणा है और सभी 5 को लागू करने के लिए व्यवस्थित और प्रगतिशील
सिद्धांतों को समय-समय पर समीक्षा की आवश्यकता प्रयासों के लिए व्यावहारिक कदम।“
होती है।
• भारत की स्वयं की महत्वाकांछा NWS में सम्मिलित होने
• यदि भारतीय नीति निर्माताओ ं को राष्ट्र के परमाणु
और मान्यता प्राप्त करने की है। इस तरह की औपचारिक
सिद्धांत की समीक्षा करने की आवश्यकता महसूस होती
मान्यता न तो मिलनी चाहिए और न ही आने वाली है,
है, तो उन्हें इसमें शामिल लागतों से अवगत होना चाहिए।
ले किन निश्चित रूप से वैश्विक परिदृश्य में भारत के साथ
• साथ ही, भारत को सीमा पर अपने बुनियादी ढांचे का ऐसा व्यवहार करने की दिशा में एक आंदोलन सा चलता
निर्माण करके और निगरानी और चेतावनी क्षमताओ ं में दिख रहा है जैसे कि वह वस्तुतः एक NWS (De Facto
सुधार करके धीरे-धीरे ’सक्रिय निरोध’ की अपनी मुद्रा Nuclear Weapon State (DNWS) हो।
को ’अवरोधक प्रतिरोध’ में संशोधित करना चाहिए।

भारत की निरस्त्रीकरण प्रतिबद्धताएं (India’s


Disarmament Commitments)
निरस्त्रीकरण (Disarmament)
1. निरस्त्रीकरण (Disarmament)
• निरस्त्रीकरण का मतलब हथियारों को कम करने,
सीमित करने या समाप्त करने का कार्य होता है।
• परमाणु हथियार राष्ट्रों (NWS) की तरह, भारत ने NPT पर
निरस्त्रीकरण का अर्थ अक्सर सामूहिक विनाश (WMD),
कोई हस्ताक्षर नहीं किये हैं और इस प्रकार निरस्त्रीकरण
जैसे परमाणु हथियारों के हथियारों के कुल उं मूलन से
के लिए की गयी संधि के प्रति उसका कोई भी दायित्व
लिया जाता है।
नहीं है। हालांकि, निरस्त्रीकरण के लिए इसकी घोषित
• संयुक्त राष्ट्र महासभा इसे पूर्ण निरस्त्रीकरण या सभी प्रतिबद्धता परमाणु हथियारों के किसी भी दीर्घकालिक
WMD के उन्मूलन के रूप में परिभाषित करती है, साथ प्रतिधारण के खिलाफ वैश्विक मानदंड की मान्यता को
ही पक्षों की सुरक्षा को कम न करते हुए सशस्त्र बलों दर्शाती है।
और पारंपरिक आयुधों की संतुलित कमी के सिद्धांत पर
आधारित है, जो सभी राज्यों की सुरक्षा की आवश्यकता
को ध्यान में रखते हुए है।
2. सीटीबीटी (CTBT)

• परमाणु परीक्षण पर भारत की एकतरफा रोक को


परमाणु हथियार संपन्न राष्ट्रों के निरस्त्रीकरण द्विपक्षीय राजनीतिक प्रतिबद्धता बना दिया गया था, जिसे
दायित्व (एनडब्ल्यूएस) (Disarmament भारत CTBT को अस्वीकार करता रहता है, ले किन इसका
Obligations Of Nuclear Weapon States- तात्पर्य “सकारात्मक माहौल“ में है, यह इस पर हस्ताक्षर
NWS): करेगा, CTBT के अनुसमर्थन का सुझाव एनएसजी में
समझौते के लिए समर्थन के लिए एक न्यूनतम शर्त
• NWS ने स्वयं परिभाषित किया है कि वैश्विक परमाणु
होनी चाहिए।
निरस्त्रीकरण और अप्रसार के अंतरराष्ट्रीय स्तर पर

69
3. एफएमसीटी (FMCT) 6. अधिशेष विखंडनीय सामग्री की सुरक्षा (Safeguarding
Surplus Fissile Materials)
• भारत FMCT के लिए बातचीत का समर्थन करता है
ले किन हथियारों के उद्देश्यों के लिए विखंडनीय सामग्री
• भारत यह नहीं मानता कि वह हथियारों के उद्देश्य से
पर रोक में शामिल होने से इनकार करता है।
किसी भी विखंडनीय सामग्री को अपनी सैन्य जरूरतों के
लिए अधिशेष घोषित करने में सक्षम है।

4. एनएसए (NSA's)
7. रिपोर्टिं ग (Reporting)
• भारत वर्तमान में एक योग्य, एकतरफा NSA घोषणा के
साथ खड़ा हुआ है, और कानूनी रूप से बाध्यकारी NSA का
• भारत अपने परमाणु शस्त्रागार से संबंधित मामलों में
समर्थन करता है। इसकी NSA घोषणा उचित एवं तार्कि क
पारदर्शि ता को ले कर अत्यधिक सतर्क है। नागरिक
है क्योंकि यह परमाणु खतरे / हमले के साथ रासायनिक
कार्यक्रमों में पारदर्शि ता अमेरिका-भारत समझौते
या जैविक खतरों / हमलों का जवाब देने का अधिकार
का एक घोषित उद्देश्य है, ले किन भारत सैन्य परमाणु
सुरक्षित रखती है, इस प्रकार इसकी प्रतिबद्धता NSW द्वारा
कार्यक्रमों में पारदर्शि ता को “सभी राज्यों“ के साथ
की गई 1995 और 2000 की प्रतिबद्धताओ ं से कम है।
पारदर्शि ता उपायों पर समझौते से जोड़ता है।

5. कम परिचालन स्थिति (Reduced Operational भारत के परमाणु रुख पर विदेश सचिव (Foreign
Status) Secretary on India’s Nuclear Stance):

• भारतीय परमाणु बलों को काफी हद तक तत्परता • भारत के विदेश सचिव हर्षवर्धन श्रृंगला ने दोहराया कि
की स्थिति में कमी दिखाई देती है, जिसका अर्थ है कि चरण-दर-चरण गैर-भेदभावपूर्ण प्रक्रिया में परमाणु
भारत वर्तमान में NWS प्रतिबद्धता के साथ आवश्यक हथियारों को समाप्त कर दिया जाना चाहिए।
अनुपालन में है।

परमाणु हथियारों के निषेध पर संधि (TPNW)

• परमाणु हथियार निषेध संधि (TPNW), या परमाणु हथियार प्रतिबंध संधि, परमाणु हथियारों के पूर्ण उन्मूलन के साथ
परमाणु हथियारों को व्यापक रूप से प्रतिबंधित करने वाला पहला कानूनी रूप से बाध्यकारी अंतरराष्ट्रीय समझौता है।

• परमाणु हथियार निषेध संधि (TPNW) में किसी भी परमाणु हथियार गतिविधियों में भाग ले ने पर प्रतिबंधों का एक
व्यापक सेट शामिल है। इनमें परमाणु हथियारों का विकास, परीक्षण, उत्पादन, अधिग्रहण, अधिकार, भंडार, उपयोग या
उपयोग करने की धमकी नहीं देने के उपक्रम शामिल हैं। संधि राष्ट्रीय क्षेत्र पर परमाणु हथियारों की तैनाती और निषिद्ध
गतिविधियों के संचालन में किसी भी राज्य को सहायता के प्रावधान पर भी रोक लगाती है।

• राज्य के पक्ष TPNW के तहत प्रतिबंधित किसी भी गतिविधि को रोकने और दबाने के लिए बाध्य होंगे जो व्यक्तियों द्वारा
या इसके अधिकार क्षेत्र या नियंत्रण के तहत क्षेत्र में किया जाता है।

• इस संधि में अधिकांश राज्यों की भागीदारी, रेडक्रॉस की अंतर्राष्ट्रीय समिति और गैर-सरकारी संगठनों के सैकड़ों प्रति-
निधियों को शामिल किया गया, जो मुख्य रूप से परमाणु हथियारों (ICAN) को समाप्त करने के अंतरराष्ट्रीय अभियान
द्वारा समन्वित हैं, उन्होंने परमाणु निरस्त्रीकरण वार्ताओ ं को आगे बढ़ाने के लिए तत्काल कार्रवाई की मांग के निर्माण
में महत्वपूर्ण भूमिका निभाई।

70
अध्याय - 7

अंतरिक्ष प्रोद्योगिकी
(SPACE TECHNOLOGY)

परिचय (Introduction) मिशन डिजाइन, सामग्री, हार्डवेयर निर्माण, ठोस प्रणोदन


प्रौद्योगिकी, नियंत्रण बिजली संयंत्र, एवियोनिक्स, वाहन
एकीकरण चेकआउट और प्रक्षेपण संचालन के लिए
• डॉ. विक्रम साराभाई भारतीय अंतरिक्ष कार्यक्रम
क्षमता का निर्माण किया गया था।
के संस्थापक थे, जिन्होंने भारत के लिए अंतरिक्ष
प्रौद्योगिकियों के लाभों को शीघ्रता से पहचाना। उन्होंने • उपग्रह की कक्षा के लिए उपयुक्त नियंत्रण और मार्गदर्शन
भारतीय अंतरिक्ष कार्यक्रम का नेतृत्व करने के लिए देश प्रणाली के साथ मल्टीस्टेज रॉकेट सिस्टम विकसित
के कोने-कोने से सक्षम और प्रतिभाशाली वैज्ञानिकों, करना हमारे अंतरिक्ष कार्यक्रम में एक प्रमुख मील का
मानवविज्ञानी, संचारकों और सामाजिक वैज्ञानिकों को पत्थर था।
बुलाया। • जटिल संवर्धि त उपग्रह प्रक्षेपण यान (ASLV) के विकास
• स्थापना के बाद से, भारतीय अंतरिक्ष कार्यक्रम को अच्छी ने स्ट्रै प-ऑन, बल्बस हीट शील्ड, क्लोज्ड लू प मार्गदर्शन
तरह से व्यवस्थित किया गया है और इसमें तीन अलग- और डिजिटल ऑटोपायलट जैसी नई तकनीकों का भी
अलग तत्व हैं जैसे कि, प्रदर्शन किया। इसने जटिल मिशनों के लिए प्रक्षेपण यान
डिजाइन की कई बारीकियों को सीखने का मार्ग प्रशस्त
• संचार और सुदर
ू संवेदन के लिए उपग्रह
किया, जिससे PSLV और GSLV जैसे परिचालन प्रक्षेपण
• अंतरिक्ष परिवहन प्रणाली वाहनों की प्राप्ति हुई।
• अंतरिक्ष अनुप्रयोग कार्यक्रम • 90 के दशक में परिचालन चरण के दौरान, दो व्यापक
• डॉ. साराभाई को विश्वास था और उन्होंने कल्पना की थी वर्गों के तहत प्रमुख अंतरिक्ष बुनियादी ढांचे का निर्माण
कि अंतरिक्ष में संसाधन मनुष्य और समाज की वास्तविक किया गया था:
समस्याओ ं का समाधान कर सकते हैं।
» एक बहुउद्देश्यीय भारतीय राष्ट्रीय उपग्रह प्रणाली (इनसैट)
• 1967 में, अहमदाबाद में स्थित पहला 'प्रायोगिक उपग्रह के माध्यम से संचार, प्रसारण और मौसम विज्ञान के लिए।
संचार पृथ्वी केंद्र (ESCES)' चालू किया गया था, जो एक
» दूसरा भारतीय सुदर
ू संवेदी उपग्रह (IRS) प्रणाली के लिए।
प्रशिक्षण केंद्र के रूप में भी विस्तृत हुआ था।

• सैटेलाइट इं स्ट्रक्शनल टे लीविज़न एक्सपेरिमेंट • ध्रुवीय उपग्रह प्रक्षेपण यान (PSLV) का विकास और
(SITE), को 1975-76 के दौरान 'दुनिया का सबसे बड़ा संचालन और भू-तुल्यकालिक उपग्रह प्रक्षेपण यान
समाजशास्त्रीय प्रयोग' माना गया। 1977-79 के दौरान (GSLV) का विकास महत्वपूर्ण उपलब्धियां थीं।
फ्रेंको-जर्मन सिम्फनी उपग्रह का उपयोग करते हुए
इसरो और डाक और टे लीग्राफ विभाग (P&T) की एक
संयुक्त परियोजना, सैटेलाइट दूरसंचार प्रयोग परियोजना
(STEP) द्वारा साइट का अनुसरण किया गया था। साइट अंतरिक्ष विभाग
की अगली कड़ी के रूप में कल्पना की गई, जो टे लीविजन
पर केंद्रित थी, STEP दूरसंचार प्रयोगों के लिए था।
(Department of Space)
• इस अवधि के दौरान, पहला भारतीय अंतरिक्ष यान • अंतरिक्ष विभाग (DOS) का प्राथमिक उद्देश्य राष्ट्र के
'आर्यभट्ट' विकसित किया गया था और इसे सोवियत सर्वांगीण विकास में सहायता के लिए अंतरिक्ष विज्ञान
प्रक्षेपक का उपयोग करके प्रक्षेपण किया गया था। और प्रौद्योगिकी के विकास और अनुप्रयोग को बढ़ावा
देना है। इसे प्राप्त करने के लिए डॉस ने निम्नलिखित
• एक अन्य प्रमुख मील का पत्थर प्रक्षेपण वाहन एसएलवी
कार्यक्रम विकसित किए हैं।
-3 का विकास था, जिसमें पृथ्वी की निचली कक्षा (LEO)
में 40 किलोग्राम भार प्रक्षेपित करने की क्षमता थी,
जिसकी पहली सफल उड़ान 1980 में हुई थी।

• SLV-3 कार्यक्रम के माध्यम से समग्र वाहन डिजाइन,

71
1961 - नाभिकीय ऊर्जा विभाग को अंतरिक्ष अनुसंधान कार्य
सौंपा गया

प्रक्षेपण यान INSAT


1962 - INCOSPAR- DAE के अंतर्गत, भारतीय राष्ट्रीय अंतरिक्ष
कार्यक्रम, कार्यक्रम, अनुसंधान समिति का गठन किया गया और तिरुवनंतपुरम के
समीप थुम्बा इक्वेटोरियल रॅाकेट लांचिं ग स्टे शन (TERLS) पर
स्वदेशी होगा दूरसंचार हेतु
कार्य भी आरंभ कर दिया गया

1969 - INCOSPAR को भारतीय अंतरिक्ष अनुसन्धान संगठन


(ISRO)
सुदर
ू संवेदन अंतरिक्ष विज्ञान
कार्यक्रम, और प्रोद्योगिकी
उपग्रह अनुप्रयोग में अनुसंधान 1972 - भारत सरकार ने अंतरिक्ष आयोग का गठन किया और
अंतरिक्ष विभाग (DOS) की स्थापना की और ISRO को DOS
हेतु और विकास के अंतर्गत लाया गया

चित्र 7. 2: भारतीय अंतरिक्ष कार्यक्रम का विकास

चित्र 7.1 : अंतरिक्ष विभाग के कार्यक्रम • अंतरिक्ष आयोग (The Space Commission): यह
नीतियों को तैयार करता है और देश के सामाजिक-
आर्थि क लाभ के लिए अंतरिक्ष विज्ञान और प्रौद्योगिकी के
विकास और अनुप्रयोग को बढ़ावा देने के लिए भारतीय
अंतरिक्ष कार्यक्रम के कार्यान्वयन की देखरेख करता है।

• DOS - इन कार्यक्रमों को मुख्य रूप से कार्यान्वित करता


है,

1. भारतीय अंतरिक्ष अनुसंधान संगठन (ISRO)

2. भौतिक अनुसंधान प्रयोगशाला (PRL)

3. राष्ट्रीय वायुमंडलीय अनुसंधान प्रयोगशाला (NARL)

4. उत्तर पूर्वी-अंतरिक्ष अनुप्रयोग केंद्र (NE-SAC)

5. सेमी-कंडक्टर प्रयोगशाला (SCL)

• सरकारी स्वामित्व वाली कंपनी के रूप में 1992 में


स्थापित एं ट्रिक्स कॉर्पोरेशन, अंतरिक्ष उत्पादों और सेवाओ ं
का विपणन करता है।

• अंतरिक्ष अवधारणाओ ं को समझने के लिए कुछ मुख्य


शब्द।

मुख्य शब्द (Keywords)

शब्दावली जानकारी

एक ग्रहपथ या कक्षा अंतरिक्ष में एक नियमित, दोहराव वाला पथ होता है जहां एक वस्तु दूसरे के चारों
कक्षा/ग्रहपथ (Orbit)
ओर घूमती है। सौर मंडल में ग्रह, धूमकेतु, क्षुद्रग्रह और अन्य वस्तुएं सूर्य की परिक्रमा करती हैं।

72
ग्रहपथ में किसी वस्तु को उपग्रह कहते हैं। एक उपग्रह प्राकृतिक हो सकता है, जैसे पृथ्वी या चंद्रमा।
उपग्रह (Satellite) कई ग्रहों में चंद्रमा होते हैं जो उनकी परिक्रमा करते हैं। एक उपग्रह भी मानव निर्मि त हो सकता है, जैसे
अंतर्राष्ट्रीय अंतरिक्ष केन्द्र।

प्रक्षेपास्र एक वाहन है जिसका उपयोग किसी वस्तु को जोर देने के लिए किया जाता है। इसे मिसाइल
प्रक्षेपास्र (Rocket) या प्रक्षेपण यान के रूप में इस्तेमाल किया जा सकता है। इसमें ईंधन और ऑक्सीकारक होते हैं, जिन्हें
एक साथ प्रणोदक और इं जन कहा जाता है।

कक्षीय विमान पृथ्वी उपग्रह की कक्षा वाली एक काल्पनिक, विशाल सपाट प्लेट। कक्षीय तल पृथ्वी के केंद्र से
(Orbital Plane) होकर गुजरता है।

अंडाकार (Ellipse) एक चपटा वृत्त या अंडाकार।

यात्री और वस्तुएँ अंतरिक्ष में तैरती हैं तो सूक्ष्म गुरुत्वाकर्षण का प्रभाव देखा जा सकता है। सूक्ष्म गुरु-
त्वाकर्षण को अन्य तरीकों से भी अनुभव किया जा सकता है। सूक्ष्म गुरुत्वाकर्षण में, अंतरिक्ष यात्री
सूक्ष्म गुरुत्वाकर्षण अपने अंतरिक्ष यान में - या बाहर, स्पेसवॉक पर तैर सकते हैं। भारी वस्तुएं आसानी से इधर-उधर हो
(Microgravity) जाती हैं। उदाहरण के लिए, अंतरिक्ष यात्री अपनी उं गलियों से सैकड़ों पाउं ड वजन के उपकरण ले जा
सकते हैं। सूक्ष्म गुरुत्वाकर्षण को कभी-कभी "शून्य गुरुत्वाकर्षण" कहा जाता है, ले किन यह भ्रामक
है।

प्रक्षेपण यान एक प्रक्षेपण यान पृथ्वी के गुरुत्वाकर्षण से बचने के लिए एक अंतरिक्ष यान द्वारा आवश्यक वेग
(Launch Vehicle) प्रदान करता है और इसे अपने पाठ्यक्रम पर सेट करता है।

वह वेग है जो किसी पिं ड के लिए बिना किसी और त्वरण के आकर्षण के गुरुत्वाकर्षण केंद्र से बचने
पलायन वेग
के लिए पर्याप्त है। पलायन वेग ऊँचाई के साथ घटता जाता है और समान ऊँचाई पर वृत्ताकार कक्षा
(Escape Velocity)
बनाए रखने के लिए आवश्यक वेग के 2 (या लगभग 1.414) के वर्गमूल के बराबर होता है।

अंतरिक्ष यान अंतरिक्ष यान, पृथ्वी के निचले वायुमंडल के ऊपर एक नियंत्रित उड़ान पैटर्न में चालक दल के साथ
(Spacecraft) या उसके बिना संचालित करने के लिए डिज़ाइन किया गया वाहन।

पृथ्वी के निकटतम दृष्टिकोण को पेरिगी (ग्रीक पेरी = निकट से) के रूप में जाना जाता है। कक्षा में
उपभू (Perigee)
इस बिं दु पर, वस्तु अपनी अधिकतम गति से चलती है (केप्लर का दूसरा नियम)।

अपभू (Apogee) एक अण्डाकार कक्षा पथ, वह बिं दु जो एक उपग्रह पृथ्वी से सबसे दूर है।

दूरदर्शक यत्रं यह दूर की वस्तुओ ं को स्पष्ट रूप से प्रकट करने के लिए डिज़ाइन किया गया एक ऑप्टिकल उपक-
(Telescope) रण है। उदा: हबल दूरदर्शक यंत्र - नासा का सबसे प्रसिद्ध और सबसे बड़ा खगोलीय उपग्रह है।

वैन एलन विकिरण बेल्ट पृथ्वी के चुंबकीय क्षेत्र में उच्च ऊंचाई पर फंसे अत्यधिक ऊर्जावान आवेशित
वैन एले न विकिरण
कणों के डोनट के आकार के क्षेत्र हैं। अमेरिकी भौतिक विज्ञानी जेम्स ए. वान एलन, जिन्होंने 1958 में
बेल्ट (Van Allen
यू.एस. एक्सप्लोरर उपग्रह द्वारा प्रेषित डेटा का उपयोग करके उन्हें खोजा था, क्षेत्रों को नामित किया
Radiation Belts)
गया था।

73
लै ग्रेंज पॉइं ट अंतरिक्ष में ऐसी स्थितियाँ हैं जहाँ वहाँ भेजी जाने वाली वस्तुएँ रुकी रहती हैं। लै ग्रेंज
बिं दुओ ं पर, दो बड़े द्रव्यमानों का गुरुत्वाकर्षण खिं चाव एक छोटी वस्तु के साथ चलने के लिए आव-
श्यक अभिकेन्द्रीय बल के बराबर होता है। अंतरिक्ष में इन बिं दुओ ं का उपयोग अंतरिक्ष यान द्वारा
स्थिति में बने रहने के लिए आवश्यक ईंधन की खपत को कम करने के लिए किया जा सकता है।
पाँच विशेष बिं दु हैं जहाँ एक छोटा द्रव्यमान दो बड़े द्रव्यमानों के साथ एक स्थिर प्रतिरूप में परिक्रमा
कर सकता है।

पांच लै ग्रेंज बिं दुओ ं में से तीन अस्थिर हैं और दो स्थिर हैं। अस्थिर लै ग्रेंज बिं दु - L1, L2 और L3 ले बल -
दो बड़े द्रव्यमानों को जोड़ने वाली रेखा के साथ स्थित हैं। स्थिर लै ग्रेंज बिं दु - L4 और L5 ले बल - दो
समबाहु त्रिभुजों के शीर्ष का निर्माण करते हैं इसके शीर्षों पर बड़े द्रव्यमान होते हैं। L4 पृथ्वी की कक्षा
का नेतृत्व करता है और L5 अनुसरण करता है।

लै ग्रेंज प्वाइं ट
(Lagrange
Points)

हालो ऑर्बि ट (Halo यह लै ग्रेंज पॉइं ट्स के चारों ओर एक ऑर्बिट है।


Orbit)

ग्रहपथ या कक्षा (Orbit) • हमारे सौर मंडल में, चंद्रमा पृथ्वी की परिक्रमा करता है,
और पृथ्वी सूर्य की परिक्रमा करती है, इसका मतलब यह
नहीं है कि बड़ी वस्तु पूरी तरह से स्थिर रहती है।
• परिभाषा - एक ग्रहपथ अंतरिक्ष में एक घुमावदार पथ
है जहां एक वस्तु (जैसे एक तारा, ग्रह, चंद्रमा, क्षुद्रग्रह या • गुरुत्वाकर्षण के कारण, चंद्रमा द्वारा पृथ्वी को अपने
अंतरिक्ष यान) गुरुत्वाकर्षण के कारण किसी अन्य वस्तु केंद्र से थोड़ा खींचा जाता है (इसीलिए हमारे महासागरों
के चारों ओर ले जाती है में ज्वार आते हैं) और हमारे सूर्य को पृथ्वी और अन्य ग्रहों
द्वारा इसके केंद्र से थोड़ा खींचा जाता है।
• गुरुत्वाकर्षण के कारण अंतरिक्ष में द्रव्यमान वाली
वस्तुएं आस-पास की वस्तुओ ं की ओर आकर्षि त होती हैं। • जब रॉकेट उपग्रहों को प्रक्षेपित करते हैं, तो उन्हें वांछित
यदि आकर्षण उन्हें पर्याप्त गति के साथ एक साथ लाता ग्रहपथ में स्थापित कर दिया जाता है। वहां, गुरुत्वाकर्षण
है, तो वे कभी-कभी एक-दूसरे की परिक्रमा करना शुरू उपग्रह को उसकी आवश्यक ग्रहपथ में रखता है
कर सकते हैं।• समान द्रव्यमान की वस्तुएँ एक दूसरे की • उसी तरह गुरुत्वाकर्षण चंद्रमा को पृथ्वी के चारों ओर
परिक्रमा करती हैं और न ही केंद्र में कोई वस्तु होती है, ग्रहपथ में रखता है।
जबकि छोटी वस्तुएँ बड़ी वस्तुओ ं के चारों ओर परिक्रमा
• अंतरिक्ष में कोई हवा नहीं है और इसलिए कोई वायु घर्षण
करती हैं।

74

@ITAKASHSINGH
नहीं है, इसलिए गुरुत्वाकर्षण उपग्रह को लगभग बिना GEO किसके लिए प्रयोग किया जाता है ? (What is GEO
किसी सहायता के पृथ्वी के चारों ओर परिक्रमा करने Used for?)
देता है।
• GEO का उपयोग उपग्रहों द्वारा पृथ्वी पर एक विशेष स्थान
से लगातार ऊपर रहने के लिए किया जाता है, जैसे कि
दूरसंचार उपग्रह।
कक्षा के प्रकार (Types Of Orbit)
• इस तरह, पृथ्वी पर एक एं टीना को हमेशा बिना हिले -डुले
• प्रक्षेपण पर, एक उपग्रह या अंतरिक्ष यान को अक्सर उपग्रह की ओर इं गित करते रहने के लिए तय किया जा
पृथ्वी के चारों ओर कई कक्षाओ ं में से एक में रखा जाता सकता है।
है - या इसे एक अंतरग्रहीय यात्रा पर भेजा जा सकता है। • इसका उपयोग मौसम निगरानी उपग्रहों द्वारा भी किया
इं टरप्ले नेटरी यात्रा का मतलब है, उपग्रह अब पृथ्वी की जा सकता है क्योंकि वे लगातार विशिष्ट क्षेत्रों का
परिक्रमा नहीं करता है, बल्कि सूर्य की परिक्रमा तब तक निरीक्षण कर सकते हैं यह देखने के लिए कि मौसम की
करता है जब तक कि वह अपने अंतिम गंतव्य, जैसे मंगल प्रवृत्ति कैसे उभरती है।
या बृहस्पति पर नहीं पहुंच जाता।

GEO उपग्रहों का कवरेज (Coverage of GEO Satellites)


भूस्थिर कक्षा (GEO) (Geostationary orbit (GEO))
• GEO में उपग्रह पृथ्वी की एक बड़ी रेंज को कवर करते हैं
• GEO में उपग्रह पृथ्वी के घूर्णन के बाद पश्चिम से पूर्व की ताकि कम से कम तीन समान दूरी वाले उपग्रह वैश्विक
ओर भूमध्य रेखा के ऊपर पृथ्वी का चक्कर लगाते हैं - कवरेज प्रदान कर सकें।
23 घंटे 56 मिनट और 4 सेकंड - पृथ्वी के समान गति से
• जब कोई उपग्रह पृथ्वी से दूर होता है, तो वह एक साथ बड़े
यात्रा करते हुए।
हिस्से को कवर कर सकता है।
• यह GEO में उपग्रहों को एक निश्चित स्थिति पर 'स्थिर'
• यदि आप इससे एक सेंटीमीटर अधिक दूर हैं, तो इसकी
प्रतीत होता है।
तुलना में यह एक मीटर दूर से मानचित्र देखने जैसा है।
• पृथ्वी के घूर्णन से पूरी तरह मेल खाने के लिए, 35,786 GEO से पूरी पृथ्वी को एक साथ देखने के लिए कम
किमी की ऊंचाई पर GEO उपग्रहों की गति लगभग 3 ऊंचाई वाले उपग्रहों की तुलना में बहुत कम उपग्रहों की
किमी प्रति सेकंड होनी चाहिए। यह कई उपग्रहों की तुलना आवश्यकता होती है।
में पृथ्वी की सतह से काफी दूर है।

निम्न पृथ्वी कक्षा (LEO) (Low Earth Orbit (LEO))

• यह एक कक्षा है जो पृथ्वी की सतह के अपेक्षाकृत करीब


है। यह आमतौर पर 1000 किमी से कम की ऊंचाई पर
होता है, ले किन पृथ्वी से 160 किमी जितना कम हो सकता
है - जो कि अन्य कक्षाओ ं की तुलना में कम है।

• GEO में उपग्रहों के विपरीत (क्योंकि यह हमेशा पृथ्वी


के भूमध्य रेखा के साथ परिक्रमा करता है) LEO उपग्रहों
को हमेशा उसी तरह पृथ्वी के चारों ओर एक विशेष पथ
का अनुसरण नहीं करना पड़ता है - उनके विमान को
झुकाया जा सकता है। LEO में उपग्रहों के लिए अधिक
उपलब्ध मार्ग हैं, जो एक कारण है कि LEO आमतौर पर
उपयोग की जाने वाली कक्षा है।

चित्र 7. 3: भूस्थिर कक्षा

75
मध्यम पृथ्वी की कक्षा (Medium Earth Orbit- MEO)
मध्यम पृथ्वी की कक्षा में LEO और GEO के बीच कहीं भी,
कक्षाओ ं की एक विस्तृत श्रृंखला शामिल है।

• यह LEO की तरह है क्योंकि इसे पृथ्वी के चारों ओर


विशिष्ट पथ ले ने की आवश्यकता नहीं है, और इसका
उपयोग विभिन्न प्रकार के उपग्रहों द्वारा कई अलग-
अलग अनुप्रयोगों के साथ किया जाता है।

MEO किसके लिए प्रयोग किया जाता है ? (What Is MEO


Used for?)

• यह आमतौर पर नेविगेशन उपग्रहों द्वारा उपयोग किया


चित्र 7. 4: निम्न पृथ्वी कक्षा (LEO) जाता है।

LEO किसके लिए प्रयोग किया जाता है? (What is LEO ध्रुवीय कक्षा और सूर्य-तुल्यकालिक कक्षा (Polar Orbit
Used for?) and Sun-Synchronous Orbit- SSO)

• पृथ्वी से LEO की निकटता इसे कई कारणों से उपयोगी • ध्रुवीय कक्षाओ ं में उपग्रह आमतौर पर पृथ्वी के ध्रुवों के
बनाती है। ऊपर से गुजरते हुए पश्चिम से पूर्व की बजाय उत्तर से
• यह आमतौर पर उपग्रह इमेजिं ग के लिए उपयोग दक्षिण की ओर जाते हैं।
किया जाता है, क्योंकि सतह के पास होने से यह उच्च • ध्रुवीय कक्षा में उपग्रह ठीक से उत्तरी और दक्षिणी ध्रुव से
रिज़ॉल्यूशन के चित्र ले ने की अनुमति देता है। नहीं गुजरते हैं; यहां तक कि 20 से 30 डिग्री के भीतर भी
• यह अंतरराष्ट्रीय अंतरिक्ष स्टेशन (ISS) के लिए उपयोग विचलन को अभी भी ध्रुवीय कक्षा के रूप में वर्गीकृत
की जाने वाली कक्षा भी है, क्योंकि अंतरिक्ष यात्रियों के किया जाता है।
लिए इससे कम दूरी पर यात्रा करना आसान होता है। • ध्रुवीय कक्षाएँ एक निम्न पृथ्वी कक्षा प्रकार हैं, क्योंकि वे
• LEO में उपग्रह लगभग 7.8 किमी प्रति सेकंड की गति से 200 से 1000 किमी के बीच कम ऊंचाई पर हैं।
यात्रा करते हैं; इस गति से, एक उपग्रह पृथ्वी का चक्कर
लगाने में लगभग 90 मिनट का समय ले ता है, जिसका
सूर्य-तुल्यकालिक कक्षा (Sun-Synchronous Orbit-
अर्थ है कि ISS दिन में लगभग 16 बार पृथ्वी का चक्कर
SSO)
लगाता है।

• हालांकि, व्यक्तिगत LEO उपग्रह दूरसंचार कार्यों के लिए • यह एक विशेष प्रकार की ध्रुवीय कक्षा है। SSO में उपग्रह,
कम उपयोगी होते हैं क्योंकि वे आकाश में इतनी तेजी से ध्रुवीय क्षेत्रों में यात्रा करते हुए, सूर्य के साथ समकालिक
चलते हैं और इसलिए जमीनी स्टेशनों से ट्रै क करने के होते हैं।
लिए बहुत प्रयास की आवश्यकता होती है। • इसका मतलब है कि वे हमेशा सूर्य के सापेक्ष एक ही
• इसके बजाय, LEO संचार उपग्रह अक्सर निरंतर कवरेज 'स्थिर' स्थिति में रहने के लिए समकालिक होते हैं।
देने के लिए कई उपग्रहों के एक बड़े संयोजन या नक्षत्र के • इसका मतलब है कि उपग्रह हमेशा एक ही स्थान पर एक
हिस्से के रूप में काम करते हैं। ही स्थानीय समय पर जाता है - उदाहरण के लिए, हर दिन
• कवरेज बढ़ाने के लिए, कभी-कभी एक जैसे या एक जैसे दोपहर में ठीक बेंगलु रु शहर से गुजरना।
कई उपग्रहों से युक्त नक्षत्रों को पृथ्वी के चारों ओर एक
'जाल' बनाने के लिए एक साथ प्रक्षेपित किया जाता है।
यह एक साथ काम करके पृथ्वी के बड़े क्षेत्रों को एक साथ
कवर करता है।

76
नहीं रखा जाता है।

• उपग्रहों को अक्सर एक स्थानान्तरण कक्षा में रखा जाता


है: एक कक्षा जहाँ, अंतर्निर्मि त मोटरों से अपेक्षाकृत कम
ऊर्जा का उपयोग करके, उपग्रह या अंतरिक्ष यान एक
कक्षा से दूसरी कक्षा में जा सकता है।

• यह एक उपग्रह तक पहुँचने की अनुमति देता है, उदाहरण


के लिए, GEO जैसी उच्च-ऊंचाई वाली कक्षा में बिना
प्रक्षेपण यान की आवश्यकता के सभी आवश्यक ऊंचाई
तक जाने के लिए, जिसके लिए अधिक प्रयास की
आवश्यकता होगी।

• GEO तक पहुंचना सबसे आम स्थानांतरण कक्षाओ ं में


से एक का एक उदाहरण है, जिसे भूस्थैतिक स्थानांतरण
चित्र 7. 5: सूर्य तुल्यकालिक कक्षा
कक्षा (GTO) कहा जाता है।

• उपग्रह हमेशा दिन के एक ही समय में पृथ्वी पर एक बिं दु


का निरीक्षण करेगा, यह कई अनुप्रयोगों को पूरा करता
है; उदाहरण के लिए, वैज्ञानिक उपग्रह छवियों का उपयोग
तुलना करने के लिए कर सकते हैं कि एक विशेष क्षेत्र
समय के साथ कैसे बदलता है।

• ऐसा इसलिए है, क्योंकि यदि आप कई दिनों, हफ्तों,


महीनों, या यहां तक कि वर्षों में किसी विशेष स्थान की
छवियों की एक श्रृंखला ले कर किसी क्षेत्र की निगरानी
करना चाहते हैं, तो कहीं मध्यरात्रि और फिर दोपहर में
तुलना करना फायदेमंद नहीं होगा – आपको प्रत्येक
चित्र को यथासंभव पिछली तस्वीर के समान ले ने की
आवश्यकता है। इसलिए, वैज्ञानिक इस तरह की छवि
श्रृंखला का उपयोग यह जांचने के लिए करते हैं कि
मौसम का रूप कैसे उभरता है, तूफानों की भविष्यवाणी चित्र 7. 6 भूस्थैतिक स्थानांतरण कक्षा
करने में मदद करता है; जंगल की आग या बाढ़ जैसी
आपात स्थितियों की निगरानी करना; या वनों की कटाई
या बढ़ते समुद्र के स्तर जैसी दीर्घकालिक समस्याओ ं पर भारत में लॉन्च वाहनों के प्रकार (Types of
डेटा जमा करने के लिए। Launch Vehicles in India)
• सूर्य-समकालिक कक्षा में एक उपग्रह आमतौर पर ६००
• अंतरिक्ष यान को अंतरिक्ष में ले जाने के लिए प्रक्षेपकों
से ८०० किमी के बीच की ऊंचाई पर होगा। 800 किमी
या प्रक्षेपण यान का उपयोग किया जाता है। भारत में दो
पर, यह लगभग 7.5 किमी प्रति सेकंड की गति से यात्रा
ऑपरेशनल प्रक्षेपक हैं: पोलर सैटेलाइट लॉन्च व्हीकल
करेगा।
(PSLV) और जियोसिं क्रोनस सैटेलाइट लॉन्च व्हीकल
(GSLV)।

स्थानांतरण कक्षाएँ और भूस्थैतिक स्थानांतरण कक्षा • स्वदेशी क्रायोजेनिक अपर स्टेज के साथ जीएसएलवी ने
(Transfer Orbits and Geostationary Transfer 2 टन वर्ग के संचार उपग्रहों को प्रक्षेपित करने में सक्षम
Orbit GTO) बनाया है। GSLV का अगला संस्करण जीएसएलवी
एमके III है, जिसमें स्वदेशी उच्च जोर पिछले दशक में भू-
• स्थानांतरण कक्षाएँ एक विशेष प्रकार की कक्षा होती हैं
समकालिक स्थानांतरण कक्षा की ओर उपग्रह भेजने में
जिनका उपयोग एक कक्षा से दूसरी कक्षा में जाने के लिए
सफल रहा है। उदाहरण - चंद्रयान -1
किया जाता है।

• जब उपग्रहों को पृथ्वी से प्रक्षेपित किया जाता है और


एरियन 5 जैसे प्रक्षेपण यान के साथ अंतरिक्ष में ले जाया
जाता है, तो उपग्रहों को हमेशा उनकी अंतिम कक्षा में सीधे

77
पोलर सैटेलाइट लॉन्च व्हीकल (पीएसएलवी) (Polar जियोसिं क्रोनस सैटेलाइट लॉन्च व्हीकल मार्क (जीएसएलवी)
Satellite Launch Vehicle- PSLV) (Geosynchronous Satellite Launch Vehicle-
GSLV)
• ध्रुवीय उपग्रह प्रक्षेपण यान (PSLV) भारत की तीसरी पीढ़ी
का प्रक्षेपण यान है। • जियोसिं क्रोनस सैटेलाइट लॉन्च व्हीकल मार्क (GSLV
MK) चौथी पीढ़ी का लॉन्च व्हीकल है। यह एक तीन चरण
• यह तरल चरणों से लै स होने वाला पहला भारतीय प्रक्षेपण
का वाहन है जिसमें चार तरल स्ट्रै प-ऑन हैं।
यान है।
• स्वदेश में विकसित क्रायोजेनिक अपर स्टेज (CUS), जो
• PSLV को 1990 के दशक में इसरो द्वारा उपग्रहों (ज्यादातर
उड़ान सिद्ध है, GSLV MK II का तीसरा चरण है।
रिमोट सेंसिंग उपग्रहों) को ध्रुवीय और निकट ध्रुवीय (जैसे
सूर्य-तुल्यकालिक कक्षा) निचली पृथ्वी की कक्षाओ ं में • जनवरी 2014 से, वाहन ने लगातार चार सफलताएँ
स्थापित करने के लिए विकसित किया गया था। हासिल की हैं।

• हालांकि, पिछले एक दशक में कई PSLV मिशन उपग्रहों • GTO को पेलोड: 2,500 किग्रा - GSLV का प्राथमिक भार
को भू-समकालिक स्थानांतरण कक्षा में भेजने में सफल संचार उपग्रहों का इन्सैट वर्ग है यह भूस्थिर कक्षाओ ं से
रहे हैं। उदाहरण - चंद्रयान -1 संचालित होता है और इसलिए GSLV द्वारा भूतुल्यकाली
स्थानांतरण कक्षाओ ं में रखा जाता है।
• 2008 और मार्स ऑर्बिटर मिशन या मंगलयान - 2014
को PSLV का उपयोग करके लॉन्च किया गया था। • LEO को पेलोड: 5,000 किग्रा - इसके अलावा,
जीएसएलवी की कम पृथ्वी की कक्षाओ ं में 5 टन तक
• PSLV ने विभिन्न उपग्रहों को पृथ्वी की निचली कक्षाओ ं,
रखने की क्षमता भारी उपग्रहों से कई छोटे उपग्रहों तक
विशेषकर आईआरएस (इं डियन रिमोट सेंसिंग) श्रृंखला
शिपमेंट के दायरे को विस्तृत करती है।
के उपग्रहों तक लगातार पहुंचाने के माध्यम से 'इसरो का
कार्यक्षेत्र' की उपाधि प्राप्त की।

• पीएसएलवी का उपयोग IRNSS उपग्रह तारामंडल जियोसिं क्रोनस सैटेलाइट लॉन्च व्हीकल मार्क III (GSLV III)
(जीईओ में 3 और GSO में 4) को कक्षा में स्थापित करने (Geosynchronous Satellite Launch Vehicle Mark
III- GSLV III)
के लिए किया गया था।

• PSLV अपने पेलोड के द्रव्यमान और लक्ष्य कक्षा के • GSLV-III को उपग्रहों को भूस्थिर कक्षा में प्रक्षेपित करने
आधार पर विभिन्न विन्यासों में उड़ सकता है। के लिए डिज़ाइन किया गया है और भारतीय मानव
अंतरिक्ष उड़ान कार्यक्रम के तहत चालक दल के मिशन
• ये विन्यास रॉकेट के पहले चरण से जुड़े ठोस रॉकेट बूस्टर
के लिए एक प्रक्षेपण यान के रूप में अभिप्रेत है।
की संख्या और प्रकार में भिन्नता रखते हैं, जबकि चार
मुख्य चरण सभी संरचनाओ ं में समान रहते हैं। • GSLV-III में GSLV की तुलना में अधिक पेलोड क्षमता है।

• GSLV-III पेलोड LEO की क्षमता: 8,000 किग्रा।

• GSLV-III पेलोड क्षमता GTO की क्षमता: 4000 किग्रा।


ठोस-ईंधन तरल-ईंधन • GSLV MK III, जीसैट श्रृंखला के 4 टन वर्ग के उपग्रहों को
चरण चरण जियोसिं क्रोनस ट्रांसफर कक्षाओ ं में स्थापित करने में
सक्षम होगा।

• GSLV MK III का निर्णायक क्रायोजेनिक चरण इसे भारी


PSLV का PSLV का पेलोड को 600 किमी की ऊंचाई की निचली पृथ्वी की
प्रथम चरण द्वितीय चरण कक्षाओ ं में रखने में सक्षम बनाता है।

PSLV का PSLV का
तृतीय चरण चतुर्थ चरण

78
चित्र 7.7 भारत में लॉन्च वाहनों के प्रकार

• Pslv और Gslv के बीच में अंतर (Difference between Pslv and Gslv)

जीएसएलवी एमके III


विशेषता पीएसएलवी (PSLV) जीएसएलवी (GSLV)
(GSLV MK III)

द्वारा विकसित इसरो इसरो इसरो

पहला परीक्षण 1993 2001 2014

पीढ़ी 3 4 4

संचार-उपग्रहों को अत्यधिक भारत का मानव-चालित


पृथ्वी अवलोकन या सुदर
ू अंतरिक्ष मिशन
मुख्य रूप से के लिए उपयोग अण्डाकार जियोसिं क्रोनस
संवेदी उपग्रहों को वितरित
किया जाता है ट्रांसफर ऑर्बिट (GTO) में अंतरिक्ष कार्यक्रमों आदि में
करने के लिए।
पहुंचाने के लिए। आत्मनिर्भरता।

2008 में चंद्रयान -1 और


2013 में मार्स ऑर्बिटर
स्पेसक्राफ्ट। सबसे हालिया
PSLV-C46 मिशन, 22 मई, GSLV-डी1/जीसैट-अप्रैल
2019 आदि है। अप्रैल 2007 18, 2001, जी-SLV-एफ11/
शुरू चंद्रयान-2 मिशन।
में PSLV-C8 द्वारा इतालवी जीसैट-7 मिशन के लिए - 19
खगोलीय उपग्रह लॉन्च के दिसंबर, 2018, जीएसएलवी।
साथ, ISRO ने वाणिज्यिक
लॉन्च के अंतर्राष्ट्रीय बाजार
में प्रवेश किया।

LEO- 1750 किग्रा GTO- LEO- 4000 किग्रा GTO- LEO-8000 किग्रा GTO-
भार क्षमता
1425 किग्रा 2000 किग्रा 4000 किग्रा

चरणों की संख्या 4 3 3

ठोस-तरल- क्रायोजेनिक सॉलिड- लिक्विड- क्रायोजे-


ईंधन संरचना ठोस-तरल-ठोस-तरल
(रूसी उधार लिया गया) निक (स्वदेशी इं जन- C20)

79
@ITAKASHSINGH

उपग्रहों के प्रकार और उनके अनुप्रयोग (Types of Satellites


and Their Applications)
उपग्रह के प्रकार

उपयोग के आधार पर भार के आधार पर

सुदर
ू संवेदी नौवहन प्रतिरक्षा मौसम निगरानी माइक्रो मेक्रो

संवाद प्रायोगिक

उपग्रहों के प्रकार (Types of Satellites) • इन्सैट 3डी और 3डीआर जैसे भूस्थिर उपग्रहों से सौर ऊर्जा
का आकलन, स्कैटरोमीटर डेटा से समुद्री पवन ऊर्जा और
सुदर
ू संवेदी उपग्रह (Remote Sensing Satellites)
अल्टीमीटर डेटा और संख्यात्मक मॉडल से महासागर
• रिमोट सेंसिंग किसी क्षेत्र की भौतिक विशेषताओ ं का पता तरंग ऊर्जा का आकलन किया जा रहा है।
लगाने और निगरानी करने की प्रक्रिया है, जिसमें दूरी पर
(आमतौर पर उपग्रह या विमान से) इसके परावर्ति त और
उत्सर्जि त विकिरण को मापते हैं। वन और पर्यावरण (Forest and Environment)
• अद्वितीय कैमरे दूर से संवेदनत छवियों को एकत्र करते
सुदर
ू संवेदी • ट्री कैनोपी घनत्व विशेषताओ ं के आधार पर रिमोट
हैं, जो शोधकर्ताओ ं को पृथ्वी के बारे में "समझने" में मदद
सेंसिंग डेटा पर विचार करते हुए वन आवरण का अनुमान
करते हैं।
लगाना।

• वन कार्बन बायोमास और जंगल की आग का अनुमान


सुदर
ू संवेदी उपग्रहों के अनुप्रयोग (Applications of लगाने के लिए।
Remote Sensing Satellites)
• प्रवाल भित्तियाँ (Coral Reefs): उपग्रह आधारित समुद्री
कृषि और मृदा (Agriculture and Soil) सतह के तापमान (एसएसटी) डेटा ने बड़े पैमाने पर
प्रवाल विरंजन (एमसीबी) घटनाओ ं की भविष्यवाणी
कृषि और मृदा का उपयोग फसल क्षेत्र के आकलन, फसल की
करने में अपनी क्षमता साबित कर दी है।
उपज और उत्पादन के आकलन, फसल की स्थिति, आवश्यक
मृदा की जानकारी प्राप्त करने, फसल प्रणाली के अध्ययन, • आर्द्रभूमि (Wetlands): रिमोट सेंसिंग डेटा विशाल क्षेत्रों
प्रयोगात्मक फसल बीमा आदि के लिए किया जाता है। की निगरानी और मानचित्रण के लिए प्राथमिक डेटा
स्रोत है, जिसमें आर्द्रभूमि की सीमा, वितरण, आर्द्रभूमि
• मृदा मानचित्रण के प्रकार जैसे मीठे जल, पीट दलदल और गैर-जंगली
• फसलों के सूखे आकलन के लिए पीटलैं ड आदि शामिल हैं।

• कीट/रोग का पता लगाने और घटना का पूर्वानुमान • हिम और ग्लेशियर (Snow and Glaciers): सिं धु, गंगा
और ब्रह्मपुत्र घाटियों को कवर करने वाले हिमालय के
• अंतर्देशीय मत्स्य विकास (उदा: हायसिस)
ग्लेशियरों का मानचित्रण।

• भूजल निगरानी।

अक्षय ऊर्जा (Renewable Energy)

• पृथ्वी अवलोकन डेटा की सहायता से पवन, सौर और खनिज अन्वेषण (Mineral Exploration)
तरंग ऊर्जा संसाधनों का आकलन किया जा सकता है।
• सामान्य रूप से भूवैज्ञानिक मानचित्रण में और हीरे और

80
लोहे के भंडार की खोज के लिए उपयोग किया जाता है। और खतरनाक घटना से पहले और बाद में उपयोग की
जाने वाली तैयारी रणनीति बनाने के लिए किया जा
• कार्टोसैट और रिसैट श्रृंखला के उपग्रहों का उपयोग
सकता है।
संभावित खनिज क्षेत्रों को मैप करने के लिए किया जाता
है।

उत्तर पूर्वी अंतरिक्ष अनुप्रयोग (North Eastern Space


Application)
शासन (Governance)
• उमियाम, शिलांग में स्थित पूर्वोत्तर अंतरिक्ष अनुप्रयोग
• वाटरशेड निगरानी केंद्र (NESC), पूर्वोत्तर क्षेत्र (NER) में अंतरिक्ष प्रौद्योगिकी
• जियो-मनरेगा-सैटेलाइट व्युत्पन्न स्थान-आधारित अनुप्रयोगों को पूरा करने के लिए अंतरिक्ष विभाग (DOS)
सेवाओ ं का उपयोग मोबाइल-आधारित जियो-टै गिंग के और उत्तर पूर्वी परिषद (NEC) की एक संयुक्त पहल है।
माध्यम से सालाना लगभग 2 मिलियन संपत्तियों की प्रमुख उद्देश्य हैं:
योजना और निगरानी के लिए किया जा रहा है। i. प्राकृतिक संसाधन प्रबंधन के लिए रिमोट सेंसिंग
• स्वच्छ गंगा परियोजना की निगरानी के लिए और जीआईएस

• विरासत स्थलों की सुरक्षा ii. उपग्रह संचार आधारित अनुप्रयोग सेवाएं

• आवास - प्रधानमंत्री आवास योजना (PMAY) के तहत iii. आपदा प्रबंधन सहायता
पांच अलग-अलग चरणों में लाभार्थी घरों की प्रगति iv. अकादमिक इं टरफेस के माध्यम से वायुमंडलीय
की निगरानी के लिए एक भुवन-एमओएचयूए वेब विज्ञान में अनुसंधान एवं विकास
जियोपोर्टल विकसित किया गया है। निर्माण गतिविधि
v. क्षेत्र में भू-स्थानिक प्रौद्योगिकी के लिए क्षमता
की निगरानी नॉट स्टार्टेड, फाउं डेशन, लिंटे ल, रूफ और
निर्माण
पूर्ण जैसे निर्माण के आधार पर की जाती है।

भारतीय सुदर
ू संवेदी कार्यक्रम (Indian Remote Sensing
तटीय अनुप्रयोग (Coastal Applications) Program)

• तटरेखा परिवर्तन की निगरानी करें, तलछट परिवहन को • 1979 और 1981 में भास्कर-1 और भास्कर-2 के सफल
ट्रै क करें, और तटीय विशेषताओ ं का नक्शा बनाएं । डेटा प्रदर्शन के बाद, भारत ने IRS कार्यक्रम विकसित करना
का उपयोग तटीय मानचित्रण और कटाव की रोकथाम शुरू किया।
के लिए किया जा सकता है। • 1988 में IRS-1A से शुरू होकर, इसरो ने कई ऑपरेशनल
रिमोट सेंसिंग उपग्रहों को लॉन्च किया है।

• भारत में सुदर


ू संवेदी उपग्रहों का सबसे बड़ा समूह प्रचालन
महासागरीय अनुप्रयोग (Ocean Applications):
में है। उदाहरण: सूर्य-समकालिक कक्षा- रिसोर्ससैट-1, 2,
2ए कार्टोसैट-1, 2, 2ए, 2बी, रिसैट-1 और 2, ओशनसैट-2,
• समुद्र के संचलन और वर्तमान प्रणालियों की निगरानी
मेघा-उष्णकटिबंधीय, सरल और स्कैटसैट-1। भूस्थिर
करें, समुद्र के तापमान और लहर की ऊंचाई को मापें,
कक्षा- इन्सैट-3डी, कल्पना और इन्सैट 3ए, इन्सैट
और समुद्री बर्फ को ट्रै क करें। महासागरों को बेहतर ढं ग
-3डीआर।
से समझने के लिए डेटा का उपयोग किया जा सकता है
और समुद्री संसाधनों का सर्वोत्तम प्रबंधन कैसे किया जा
सकता है और संभावित मछली पकड़ने के क्षेत्रों, अल्गल
ब्लू म जोन और कोरल रीफ संरक्षण की पहचान की जा संचार उपग्रह (Communication Satellites)
सकती है।
• कवरेज और आउटरीच के मामले में अद्वितीय क्षमताओ ं
का दोहन करने के लिए टे लीविजन, डीटीएच प्रसारण,
डीएसएनजी और वीसैट जैसे विविध अनुप्रयोगों के लिए
जोखिम मूल्यांकन (Hazard Assessment)
उपग्रह संचार उपयोग पूरे देश में व्यापक और सर्वव्यापी
• तूफान, भूकंप, कटाव और बाढ़ को ट्रै क करें। डेटा का हो गया है।
उपयोग प्राकृतिक आपदा के प्रभावों का आकलन करने

81
संचार उपग्रहों के अनुप्रयोग (Applications of उपग्रह आधारित नेविगेशन सेवाएं प्रदान करने के लिए
Communication Satellites) प्रतिबद्ध है।

• दूरसंचार (Telecommunications): इन्सैट उपग्रह • नागर विमानन आवश्यकताओ ं को पूरा करने के लिए,
पारंपरिक रूप से आवाज और डेटा संचार प्रदान करने के इसरो भारतीय विमानपत्तन प्राधिकरण (एएआई) के साथ
लिए दूरसंचार अनुप्रयोगों का समर्थन करते रहे हैं। संयुक्त रूप से जीपीएस एडेड GEO ऑगमेंटेड नेविगेशन
(जी ए जी ए एन) प्रणाली स्थापित करने के लिए काम
• टे लीमेडिसिन (Telemedicine)
कर रहा है।
• टे ली-एजुकेशन (Tele-Education): 'एडुसैट', भारत
• स्वदेशी प्रणाली के आधार पर स्थिति, नेविगेशन और
का पहला विषयगत उपग्रह, जो विशेष रूप से शैक्षिक
समय सेवाओ ं की उपयोगकर्ता आवश्यकताओ ं को
सेवाओ ं के लिए समर्पि त है, का व्यापक रूप से एकतरफा
पूरा करने के लिए, इसरो एक क्षेत्रीय उपग्रह नेविगेशन
टीवी प्रसारण, वीडियो कॉन्फ्रेंसिं ग, कंप्यूटर कॉन्फ्रेंसिं ग,
प्रणाली स्थापित कर रहा है जिसे भारतीय क्षेत्रीय
वेब-आधारित निर्देश जैसे इं टरैक्टिव शैक्षिक वितरण मोड
नेविगेशन सैटेलाइट सिस्टम (IRNSS) कहा जाता है।
की एक विस्तृत श्रृंखला को पूरा करने के लिए उपयोग
किया गया था। आदि।

• मोबाइल उपग्रह सेवाएं (Mobile Satellite Services) भारतीय क्षेत्रीय नेविगेशन सैटेलाइट सिस्टम (NAVIC
• रेडियो नेटवर्किंग (Radio Networking) (Indian Regional Navigation Satellite System
(IRNSS): NAVIC)
• ग्राम संसाधन केंद्र (Village Resource Centre):
ग्रामीण क्षेत्रों को सीधे अंतरिक्ष-आधारित सेवाएं प्रदान • यह महत्वपूर्ण राष्ट्रीय अनुप्रयोगों के लिए एक स्वतंत्र
करने के लिए। भारतीय उपग्रह आधारित स्थिति प्रणाली है।

• उपग्रह की सहायता से खोज और बचाव (Satellite • मुख्य उद्देश्य अच्छी सटीकता प्रदान करने के लिए भारत
Aided Search and Rescue): भारत एल ई ओ और उसके पड़ोस में विश्वसनीय स्थिति, नेविगेशन और
एसएआर (लो अर्थ ऑर्बिट सर्च एं ड रेस्क्यू) उपग्रह प्रणाली समय सेवाएं प्रदान करना है।
के माध्यम से संकट चेतावनी और स्थिति स्थान सेवा • IRNSS दो प्रकार की सेवाएं प्रदान करेगा
प्रदान करने के लिए अंतरराष्ट्रीय कोसपास-सारसैट
कार्यक्रम का सदस्य है। इनसैट -3 ए और इनसैट -3 डी » मानक स्थिति निर्धारण सेवा (SPS)
एक खोज और बचाव पेलोड से लै स हैं जो समुद्री, विमानन » प्रतिबंधित सेवा (R S)
और भूमि उपयोगकर्ताओ ं के संकट बीकन से उत्पन्न
होने वाले अलर्ट संकेतों को उठाता है और रिले करता है। • अब तक, इसरो ने IRNSS श्रृंखला में नौ उपग्रहों का

• आपदा प्रबंधन (Disaster Management): सार्क निर्माण किया है; जिनमें से आठ अभी कक्षा में हैं। इनमें

उपग्रह (जियोसिं क्रोनस सैटेलाइट)। से तीन उपग्रह भूस्थिर कक्षा (GEO) में हैं जबकि शेष भू-
समकालिक कक्षाओ ं (GSO) में भूमध्यरेखीय तल पर
• टे लीविजन (Television)
29° का झुकाव बनाए रखते हैं।
• उपग्रह नेविगेशन कार्यक्रम (Satellite Navigation
• IRNSS तारामंडल को "नाविक" (भारतीय नक्षत्र के साथ
Program): ईंधन की बचत, उड़ान सुरक्षा, वायु अंतरिक्ष
नेविगेशन) नाम दिया गया था।
क्षमता में वृद्धि, दक्षता, विश्वसनीयता में वृद्धि, ऑपरेटरों के
लिए कार्य भार में कमी, हवाई यातायात नियंत्रण के लिए
समुद्री क्षेत्र का कवरेज, उच्च स्थिति सटीकता, आदि। जीपीएस एडेड जियो ऑगमेंटेड नेविगेशन (GPS Aided
GEO Augmented Navigation - GAGAN):

• यह भारतीय विमानपत्तन प्राधिकरण (AAI) के साथ


नेविगेशन उपग्रह (Navigation Satellites)
संयुक्त रूप से कार्यान्वित एक उपग्रह आधारित वृद्धि
• सैटेलाइट नेविगेशन सेवा वाणिज्यिक और रणनीतिक प्रणाली (SBAS) है।
अनुप्रयोगों के साथ एक उभरती हुई उपग्रह आधारित • GAGAN का मुख्य उद्देश्य नागरिक उड्डयन अनुप्रयोगों के
प्रणाली है। लिए आवश्यक सटीकता और अखंडता के साथ उपग्रह
• भारत में, इसरो नागरिक उड्डयन आवश्यकताओ ं की आधारित नेविगेशन सेवाएं प्रदान करना और भारतीय
उभरती मांगों को पूरा करने और स्वतंत्र उपग्रह नेविगेशन हवाई क्षेत्र में बेहतर हवाई यातायात प्रबंधन प्रदान करना
प्रणाली के आधार पर स्थिति, नेविगेशन और समय की है।
उपयोगकर्ता आवश्यकताओ ं को पूरा करने के लिए • प्रणाली अन्य अंतरराष्ट्रीय एसबीएएस प्रणालियों के साथ

82
अंतःक्रियाशील होगी और क्षेत्रीय सीमाओ ं के पार निर्बाध • मार्स ऑर्बिटर मिशन को एक चुनौतीपूर्ण तकनीकी
नेविगेशन प्रदान करेगी। मिशन और एक विज्ञान मिशन कहा जा सकता है जो
महत्वपूर्ण मिशन संचालन और प्रणोदन, संचार और अन्य
• गगन सिग्नल-इन-स्पेस (SIS) जीसैट-8 और जीसैट-10
अंतरिक्ष यान बस प्रणालियों पर कठोर आवश्यकताओ ं
के माध्यम से उपलब्ध है।
पर विचार कर रहा है।

• मिशन का प्राथमिक ड्राइविं ग तकनीकी उद्देश्य एक


अंतरिक्ष विज्ञान और अन्वेषण उपग्रह (Space Science & अंतरिक्ष यान को डिजाइन और साकार करना है जिसमें
Exploration Satellites) अर्थ बाउं ड पैंतरेबाज़ी (EBM), मार्टि यन ट्रांसफर ट्रैजेक्टरी
(MTT), और मार्स ऑर्बिट इं सर्शन (MOI) चरणों और
• भारतीय अंतरिक्ष कार्यक्रम में खगोल विज्ञान, खगोल
संबंधित गहरे अंतरिक्ष मिशन की योजना बनाने की और
भौतिकी, ग्रह और पृथ्वी विज्ञान, वायुमंडलीय विज्ञान और
लगभग 400 मिलियन किमी की दूरी पर संचार प्रबंधन
सैद्धांतिक भौतिकी में अनुसंधान शामिल है।
क्षमता है।
• गुब्बारों, परिज्ञापी रॉकेटों, अंतरिक्ष प्लेटफार्मों और जमीन
• ऑटोनॉमस फॉल्ट डिटे क्शन और रिकवरी भी मिशन के
पर आधारित सुविधाएं इन अनुसंधान प्रयासों का समर्थन
लिए महत्वपूर्ण हो जाती है।
करती हैं।

• वायुमंडलीय प्रयोगों के लिए परिज्ञापी राकेटों की श्रंखला


उपलब्ध है। विशेष रूप से आकाशीय एक्स-रे और गामा- चंद्रयान-1 (Chandrayaan-1)
रे फटने को निर्देशित करने के लिए उपग्रहों पर कई
वैज्ञानिक उपकरण उड़ाए गए हैं। • चंद्रयान-1, चंद्रमा के लिए भारत का पहला मिशन,
2008 में सतीश धवन अंतरिक्ष केंद्र, श्रीहरिकोटा से
• एस्ट्रोसैट, मार्स ऑर्बिटर मिशन, चंद्रयान 1, चंद्रयान 2
सफलतापूर्वक लॉन्च किया गया था।
इसरो के वैज्ञानिक उपग्रह और अंतरिक्ष यान हैं।
• चंद्रमा की रासायनिक, खनिज, और फोटो-भूगर्भि क
मानचित्रण के लिए अंतरिक्ष यान चंद्रमा की सतह से 100
एस्ट्रोसैट (Astrosat) किमी की ऊंचाई पर चंद्रमा के चारों ओर परिक्रमा कर
रहा था।
• एस्ट्रोसैट एक्स-रे, ऑप्टिकल और यूवी स्पेक्ट्रल बैंड में
आकाशीय स्रोतों का एक साथ अध्ययन करने वाला
पहला समर्पि त भारतीय खगोल विज्ञान मिशन है। चंद्रयान-2 (Chandrayaan-2)
• पेलोड पराबैंगनी (निकट और दूर), सीमित ऑप्टिकल
• चंद्रयान-2 चंद्रमा पर पिछले चंद्रयान-1 मिशन का उन्नत
और एक्स-रे व्यवस्था के ऊर्जा बैंड को कवर करते हैं।
संस्करण है।
• एस्ट्रोसैट मिशन की अनूठी विशेषताओ ं में से एक यह है
• चंद्रयान-2 को जियोसिं क्रोनस सैटेलाइट लॉन्च व्हीकल
कि यह एक ही उपग्रह के साथ विभिन्न खगोलीय पिं डों के
मार्क -III (GSLV MK-III) ले जाया गया
एक साथ बहु-तरंग दैर्ध्य अवलोकन को सक्षम बनाता है।
• चंद्रयान-2 को दो मॉड्यूल प्रणाली के रूप में कॉन्फ़िगर
• सतीश धवन अंतरिक्ष केंद्र, श्रीहरिकोटा से पीएसएलवी-
किया गया है जिसमें एक ऑर्बिटर क्राफ्ट मॉड्यूल (OC)
सी30 द्वारा 28 सितंबर, 2015 को 1515 किलोग्राम के
और इसरो द्वारा विकसित रोवर को ले जाने वाला एक
उत्थापन द्रव्यमान के साथ एस्ट्रोसैट का प्रमोचन किया
लैं डर क्राफ्ट मॉड्यूल (LC) शामिल है।
गया।
• मिशन का उद्देश्य एक रोवर को उतारकर आकाशीय पिं ड
के अज्ञात दक्षिणी ध्रुव का पता लगाना है।
मार्स ऑर्बि टर मिशन (Mars Orbiter Mission) • चंद्रयान-2 का 27 किलोग्राम वजनी रोबोटिक वाहन
'प्रज्ञान' चंद्रमा पर उतरने के स्थान से 500 मीटर तक की
• मार्स ऑर्बिटर मिशन मंगल ग्रह के लिए इसरो का पहला
दूरी तय कर सकता है और अपने कामकाज के लिए सौर
इं टरप्ले नेटरी मिशन है, जिसमें एक ऑर्बिटर क्राफ्ट है
ऊर्जा का इस्तेमाल करता है।
जिसे मंगल की कक्षा में 372 किमी x 80,000 किमी
की अण्डाकार कक्षा में परिक्रमा करने के लिए डिज़ाइन • हालांकि विक्रम मून लैं डर, चंद्र सतह पर उतरने से कुछ
किया गया है। सेकंड पहले ही वैज्ञानिकों से संपर्क खो बैठा।

83
प्रोजेक्ट नेत्रा - नेटवर्क फॉर स्पेस ऑब्जेक्ट ट्रैकिंग एं ड
एनालिसिस (Project Netra - Network for Space
Object Tracking and Analysis)

• इसरो ने भारतीय उपग्रहों के लिए मलबे और अन्य


खतरों का पता लगाने के लिए अंतरिक्ष में एक प्रारंभिक
चेतावनी प्रणाली 'प्रोजेक्ट नेत्रा' शुरू की थी।

• नेत्रा के तहत, इसरो ने कई अवलोकन सुविधाएं स्थापित


करने की योजना बनाई है जैसे:

• जुड़े हुए रडार और टे लीस्कोप

• डाटा प्रोसेसिंग इकाइयां

• नियंत्रण केंद्र

लाभ (Advantages)

चित्र 7.8 चंद्रयान 2 का उद्देश्य • स्थापित होने पर, यह परियोजना भारत को अन्य अंतरिक्ष
शक्तियों की तरह अंतरिक्ष स्थितिजन्य जागरूकता
(SAS) में अपनी क्षमता प्रदान करेगी, जिसका उपयोग
भारतीय उपग्रहों के लिए मलबे से होने वाले खतरों की
रक्षा उपग्रह (Defense Satellites)
'भविष्यवाणी' करने के लिए किया जाता है।
• इसरो ने क्षमताओ ं के साथ उपग्रहों की एक श्रृंखला का
• इसरो का कहना है कि हमारा SSA पहले निम्न-पृथ्वी
प्रक्षेपण किया जो सशस्त्र बलों के लिए महत्वपूर्ण उपयोग
की कक्षाओ ं या LEO के लिए होगा जिनके पास रिमोट-
हो सकता है।
सेंसिंग अंतरिक्ष यान है।
• कार्टोसैट और रीसैट ने भारत को सीमाओ ं पर पाकिस्तान
• SSA का एक सैन्य भागफल भी है और यह अंतरिक्ष और
की गतिविधियों पर नजर रखने की क्षमता प्रदान की।
रक्षा के रूप में देश की समग्र सुरक्षा में एक नया घेरा
• जून 2018 में कार्टोसैट 2ई+ के साथ, भारत ने अंतरिक्ष में जोड़ता है।
अपनी रिमोट सेंसिंग क्षमताओ ं को काफी बढ़ाया है।
• वे दूसरों के साथ-साथ, 10 सेमी जितनी छोटी वस्तुओ ं को
• कार्टोसैट 2ई+ को कई लोगों ने भारत का "आई इन द 3,400 किमी की सीमा तक और लगभग 2,000 किमी
स्काई" नाम दिया था। की अंतरिक्ष कक्षा के बराबर स्पॉट, ट्रै क और कैटलॉग
कर सकते हैं।
• भारत ने 2012 में रीसैट-1 लॉन्च किया; सुदर
ू संवेदी उपग्रह
में रडार टोही इमेजिं ग क्षमता थी। • यह देश के लिए मिसाइल या अंतरिक्ष हमले के खिलाफ
एक अघोषित चेतावनी के रूप में भी काम करता है।
• इन उपग्रहों में निगरानी और मानचित्रण क्षमताएं हैं और
ये जमीन और समुद्री सीमाओ ं पर दुश्मनों पर नजर रख
सकते हैं।
भविष्य की कुछ इसरो परियोजनाओ ं की झलक
• विश्वसनीय संचार सेवाएं प्रदान करने के लिए समर्पि त
(Glimpse of Few Future ISRO Projects)
रक्षा संचार उपग्रहों को लॉन्च किया जाता है। उदाहरण:
जीसैट -7 - नेवी, जीसैट -7 ए वायु सेना और सेना। आदित्य - एल 1 सूर्य का अध्ययन करने वाला पहला भारतीय
मिशन (Aditya - L1 First Indian Mission to Study
• एमिसैट एक भारतीय टोही उपग्रह है जो अंतरिक्ष आधारित the Sun)
इले क्ट्रॉनिक इं टे लिजेंस या एलिंट प्रदान करने के लिए
• इसरो आदित्य एल1 मिशन के माध्यम से सूर्य का
है। अंतरिक्ष यान भारतीय सशस्त्र बलों की स्थितिजन्य
अध्ययन करने के लिए अपना पहला वैज्ञानिक प्रोजेक्ट
जागरूकता में सुधार करने में मदद करेगा क्योंकि यह
तैयार कर रहा है।
डीआरडीओ की परियोजना कौटिल्य के तहत विकसित
दुश्मन राडार की जानकारी और स्थान प्रदान करेगा। • प्रारंभ में, आदित्य -1 (जिसे बाद में आदित्य L1 में संशोधित
किया गया) मिशन की कल्पना 400 किग्रा वर्ग के
उपग्रह के रूप में की गई थी जिसमें एक पेलोड, विज़िबल
एमिशन लाइन कोरोनग्राफ (VELC) था। इसे 800 किमी

84
कम पृथ्वी की कक्षा में लॉन्च करने की योजना थी। मिशन में शामिल चुनौतियां (Challenges Involved in
the Mission)
• ले किन सूर्य-पृथ्वी प्रणाली के लग्रांगियन बिं दु 1 (L1) के
चारों ओर प्रभामंडल कक्षा में रखे गए उपग्रह को आदित्य
• विशाल दूरी (Huge Distance): पृथ्वी और सूर्य के बीच,
1 में प्रस्तावित LEO पर रखने की तुलना में सूर्य को बिना
जो 149 मिलियन किलोमीटर . है
किसी ग्रहण / ग्रहण के लगातार देखने का महत्वपूर्ण
लाभ है। इसलिए, आदित्य -1 मिशन को अब "आदित्य- • अति-गर्म विकिरण और तापमान (Ultra-Hot
एल 1 मिशन" में संशोधित किया गया है और L1 के चारों Radiations and Temperatures): जैसे-जैसे उपग्रह
ओर एक प्रभामंडल कक्षा में डाला गया है, जो पृथ्वी से सूर्य की ओर बढ़ता है, तेजी से बढ़ता है, जिससे सूर्य के
1.5 मिलियन किमी दूर है। यह उपग्रह विज्ञान के बढ़े हुए निकट होना असंभव हो जाता है। हालाँकि, आदित्य एल 1
दायरे और उद्देश्यों के साथ अतिरिक्त छह नीतभार वहन बहुत दूर रहेगा, और बोर्ड पर लगे उपकरणों के लिए गर्मी
करता है। एक महत्वपूर्ण चिं ता का विषय होने की उम्मीद नहीं है।

गगनयान मिशन - भारत का पहला मानवयुक्त अंतरिक्ष


मिशन (GAGANYAAN Mission – India’s First
Manned Space Mission)

• गगनयान भारत का पहला मानव अंतरिक्ष मिशन होगा।


इसरो का लक्ष्य 2022 में भारत की स्वतंत्रता की 75वीं
वर्षगांठ से पहले मिशन को लॉन्च करना है।

• व्यय - गगनयान कार्यक्रम के लिए कुल निधि की


आवश्यकता लगभग रु.10,000 करोड़ है और इसमें
प्रौद्योगिकी विकास की लागत, उड़ान हार्डवेयर की
प्राप्ति और आवश्यक बुनियादी ढांचे के तत्व शामिल हैं।
गगनयान कार्यक्रम के हिस्से के रूप में दो बिना चालक
वाली उड़ानें और एक व्यक्ति की उड़ान शुरू की जाएगी।

प्रक्षेपण यान - जीएसएलवी एमके III (Launch Vehicle –


GSLV MK III)
चित्र 7.9: आदित्य - L1 मिशन
उद्दे श्य (Objectives)

प्रक्षेपण यान - आदित्य एल 1 को PSLV XL का उपयोग करके • देश में विज्ञान और प्रौद्योगिकी के स्तर को बढ़ाने के लिए
लॉन्च किया जाएगा। • राष्ट्रीय परियोजना में कई संस्थानों, शिक्षाविदों और
उद्योग को शामिल करना

उद्दे श्य (Objectives) • औद्योगिक विकास में सुधार के लिए

• प्रेरक युवा
• अध्ययन करने के लिए:
• सामाजिक लाभ के लिए प्रौद्योगिकी का विकास
» सूर्य का कोरोना (दृश्यमान और निकट अवरक्त किरणें) • अंतर्राष्ट्रीय सहयोग में सुधार
» सूर्य का प्रकाशमंडल (नम और कठोर एक्स-रे)

» सूर्य का वर्णमण्डल (UV)


महत्व (Significance)
» सौर उत्सर्जन, सौर हवाएं और भड़कना

» कोरोनल मास इजेक्शन (CME) • अनुसंधान और विकास को बढ़ावा देने के लिए - इस


कार्यक्रम से देश के भीतर विशिष्ट विज्ञान और प्रौद्योगिकी
» सूर्य की चौबीसों घंटे इमेजिं ग।
डोमेन में अनुसंधान और विकास को प्रोत्साहित करने
की उम्मीद है।

• चिकित्सा, कृषि, औद्योगिक सुरक्षा, प्रदूषण, अपशिष्ट

85
प्रबंधन, जल, खाद्य संसाधन प्रबंधन आदि में प्रौद्योगिकी गति को धक्का देगा। प्रक्षेपण चरण के दौरान कई चीजें
स्पिनऑफ की व्यापक संभावनाएं है। गलत हो सकती हैं, जिसमें रॉकेट का आग के गोले में

• भविष्य की प्रौद्योगिकियों के लिए प्रयोग और परीक्षण के फटना भी शामिल है। जीवन के नुकसान की संभावना

लिए अंतरिक्ष में मंच - मानव अंतरिक्ष उड़ान कार्यक्रम को कम से कम सुनिश्चित करने के लिए कई सुरक्षा

भविष्य की प्रौद्योगिकियों के लिए जांच और परीक्षण सुविधाओ ं को रॉकेट सिस्टम में बनाया जाना है। हालांकि,

करने के लिए एक अद्वितीय सूक्ष्म गुरुत्वाकर्षण मंच वास्तविक ऑपरेटिंग वातावरण में इन सभी प्रणालियों

प्रदान करेगा। का परीक्षण करना लगभग असंभव है और इस तरह के


मिशन को शुरू करते समय एक परिकलित जोखिम
• रोजगार (Employment): इस कार्यक्रम से रोजगार उठाना पड़ता है।
सृजन, मानव संसाधन विकास और बढ़ी हुई औद्योगिक
क्षमताओ ं के मामले में देश के भीतर आर्थि क गतिविधियों • विषम वातावरण (Hostile Environment): की

को प्रोत्साहन मिलने की उम्मीद है। इसरो प्रमुख के परिस्थितियाँ अत्यंत विषम हैं। कोई माहौल नहीं है। प्रेशर

अनुसार, गगनयान मिशन 15,000 नए रोजगार के न होने पर इं सान का खून खौलने लगता है। 'गगनयान'

अवसर पैदा करेगा, उनमें से 13,000 निजी उद्योग में और को एक छोटी मात्रा के अंदर पृथ्वी जैसा वातावरण बनाना

अंतरिक्ष संगठन को 900 के अतिरिक्त कार्यबल की है और पूरे मिशन में ऑक्सीजन की पर्याप्त आपूर्ति ,

आवश्यकता होगी। कार्बन-डाइऑक्साइड को हटाना और आरामदायक


तापमान और आर्द्रता के स्तर को बनाए रखना है।
• प्रतिष्ठा (Reputation): मानव अंतरिक्ष उड़ान क्षमता
भारत को दीर्घकालिक राष्ट्रीय लाभों के साथ भविष्य में
वैश्विक अंतरिक्ष अन्वेषण पहल में एक सहयोगी भागीदार
के रूप में भाग ले ने में सक्षम बनाएगी। अंतर्राष्ट्रीय अंतरिक्ष स्टे शन -
• प्रेरणा (Inspiration): यह कई युवा छात्रों को राष्ट्रीय
आईएसएस (International
विकास के लिए विज्ञान और प्रौद्योगिकी के क्षेत्र में
करियर बनाने के लिए प्रेरित करेगा। Space Station - ISS)
• अंतर्राष्ट्रीय अंतरिक्ष स्टेशन (ISS) एक बहु-राष्ट्र निर्माण
शामिल चुनौतियां (Challenges Involved) परियोजना है। यह मानव द्वारा अंतरिक्ष में डाली गई अब
तक की सबसे बड़ी एकल संरचना है।
• गुरुत्वाकर्षण क्षेत्र (Gravitational Field): एक
• यह एक घर के रूप में कार्य करता है जहां अंतरिक्ष यात्री
गुरुत्वाकर्षण क्षेत्र से दूसरे में संक्रमण करना मुश्किल है।
और अंतरिक्ष यात्री रहते हैं। यह एक रहने योग्य मानव
यह हाथ-आंख और सिर-आंख के समन्वय को प्रभावित
निर्मि त उपग्रह है जो पृथ्वी के चारों ओर परिक्रमा करता
करता है। नासा ने सीखा है कि गुरुत्वाकर्षण के बिना
है।
मानव शरीर पर काम करने से हड्डियाँ खनिज खो देती
हैं। अंतरिक्ष मिशन से लौटने के बाद भी, चालक दल को • यह एक अनूठी विज्ञान प्रयोगशाला भी है।
ऑस्टियोपोरोसिस से संबंधित फ्रैक्चर का अधिक खतरा
• अंतरिक्ष स्टेशन के निर्माण और उपयोग के लिए कई देशों
हो सकता है। यदि यूनिट व्यायाम नहीं करती है और ठीक
ने मिलकर काम किया।
से नहीं खाती है, तो वे मांसपेशियों की ताकत खो देंगे। टीम
को दृष्टि संबंधी समस्याएं भी हो सकती हैं। • नंगी आंखों से, आईएसएस को समय के विभिन्न बिं दुओ ं
पर पृथ्वी से देखा जा सकता है।
• एकांत (isolation): अलगाव के कारण, चालक दल को
अवसाद, थकान, नींद विकार और मानसिक विकारों का • अंतर्राष्ट्रीय अंतरिक्ष स्टेशन पृथ्वी की सतह से लगभग
सामना करना पड़ सकता है। 400 किमी (250 मील) से 20 किमी (260 मील) ऊपर
LEO में है। यह हर 90 मिनट में पृथ्वी की परिक्रमा करता
• विकिरण (Radiation): अंतरिक्ष स्टेशनों में, अंतरिक्ष
है। नासा अंतरिक्ष में रहने और काम करने के बारे में
यात्री पृथ्वी पर लोगों की तुलना में दस गुना अधिक
अधिक जानने के लिए अंतरिक्ष स्टेशन का उपयोग
विकिरण प्राप्त करते हैं। रेडिएशन के संपर्क में आने से
कर रहा है। आईएसएस कार्यक्रम भाग ले ने वाली पांच
कैंसर का खतरा बढ़ सकता है। यह केंद्रीय तंत्रिका तंत्र
अंतरिक्ष एजेंसियों के बीच एक संयुक्त परियोजना है
को नुकसान पहुंचा सकता है। विकिरण भी मतली, उल्टी,
एनोरेक्सिया और थकान का कारण बन सकता है। • नासा अंतरिक्ष में रहने और काम करने के बारे में अधिक
जानने के लिए अंतरिक्ष स्टेशन का उपयोग कर रहा है।
• रॉकेट (Rocket): रॉकेट में यात्रा करना एक विस्फोट
करने वाले बम पर बैठने जैसा है जो 30 मिनट से भी कम • आईएसएस कार्यक्रम भाग ले ने वाली पांच अंतरिक्ष
समय में 0 किमी प्रति घंटे से 29,000 किमी प्रति घंटे की एजेंसियों के बीच एक संयुक्त परियोजना है।

86

@ITAKASHSINGH
» नासा - यूएसए वोयेजर 2 (Voyager 2):
» रोस्कोस्मोस - रूस
• वोयेजर 1, और वोयेजर 2 हमारे सौर मंडल के किनारे को
» जाक्सा - जापान खोजने और उसका अध्ययन करने के लिए एनएए जांच

» ईएसए - यूरोप हैं।

» सीएसए - कनाडा • वोयेजर 2 इं टरस्टेलर स्पेस में प्रवेश करने वाला दूसरा
अंतरिक्ष यान है, जिसका लक्ष्य चार विशाल ग्रहों बृहस्पति,
• अंतरसरकारी संधियाँ और समझौते अंतरिक्ष स्टेशन के शनि, यूरन
े स और नेपच्यून का अध्ययन करना है।
स्वामित्व और उपयोग को स्थापित करते हैं। • नवंबर 2018 में यह 'इं टरस्टेलर स्पेस' में प्रवेश करने के
• स्टेशन को दो खंडों में बांटा गया है। रूसी कक्षीय खंड बाद अपने जुड़वां वोयेजर 1 में शामिल हो गया है। वोयेजर
(आरओएस) और संयुक्त राज्य कक्षीय खंड (यूएसओएस)। 1 एकमात्र मानव निर्मि त वस्तु है जो तारों के बीच अंतरिक्ष
में प्रवेश करती है।

इसरो ने मानव रहित गगनयान अंतरिक्ष यान में ह्यूमनॉइड वोयेजर 2 की उपलब्धियां (Achievements of Voyager
'व्योममित्र' भेजने के लिए इसरो ने अपनी पहली महिला 2):
अंतरिक्ष यात्री का अनावरण किया, जिसे व्योमित्र कहा
• इसने बृहस्पति पर 14वें चंद्रमा की खोज की है।
जाता है, जो मानव अंतरिक्ष मिशन, गगनयान की पहली
परीक्षण उड़ान में अंतरिक्ष की सवारी करेगी। • यूरन
े स और नेपच्यून को पार करने वाली पहली मानव
निर्मि त वस्तु।
व्योमित्र एक "अर्ध-ह्यूमनॉइड" है (एक ह्यूमनॉइड एक इं सान
की उपस्थिति वाला रोबोट है। इसरो के व्योममित्र (व्योम = • यूरन
े स में १० नए चंद्रमाओ ं और २ नए वलय की खोज की।
अंतरिक्ष, मित्र = मित्र) को अर्ध-ह्यूमनॉइड भी कहा जाता है • नेपच्यून में 5 चंद्रमा, चार वलय और एक "ग्रेट डार्क स्पॉट"
क्योंकि उसके पास केवल एक सिर, दो हाथ और एक होगा। की खोज की।
धड़, और निचले अंग नहीं होंगे।)

इनसाइट मिशन (Insight Mission):

• इनसाइट का अर्थ है 'भूकंपीय जांच, भूगणित और ऊष्मा


परिवहन का उपयोग कर आंतरिक अन्वेषण' नासा के
डिस्कवरी कार्यक्रम का हिस्सा है जिसका उद्देश्य मंगल
का अध्ययन करना है।

• यह मंगल के "आंतरिक अंतरिक्ष" का गहराई से अध्ययन


करने वाला पहला बाहरी अंतरिक्ष रोबोट एक्सप्लोरर है:
इसकी परत, मेंटल और कोर।

• यह विज्ञान के सबसे मौलिक प्रश्नों में से एक का उत्तर


देना चाहता है: स्थलीय ग्रहों का निर्माण कैसे हुआ?
वह पैनल संचालन स्विच कर सकती है, पर्यावरण नियं- • हाल ही में इसने मंगल ग्रह पर भूकंप और चुंबकीय स्पंदों
त्रण और जीवन समर्थन प्रणाली (ECLSS) कार्य कर सकती का पता लगाया है।
है, अंतरिक्ष यात्रियों के साथ बातचीत कर सकती है, उन्हें
• मंगल के पास टे क्टोनिक प्लेट नहीं हैं, ले किन इसमें
पहचान सकती है और उनके प्रश्नों को हल कर सकती है।
ज्वालामुखीय रूप से सक्रिय क्षेत्र हैं, जिनमें से एक सेर्बरस
यदि केबिन के भीतर का वातावरण बदलता है तो ह्यूमनॉइड फॉसे है, जो गड़गड़ाहट का कारण बन सकता है।
चेतावनी का पता लगा सकता है और चेतावनी दे सकता है।
• इनसाइट चौबीसों घंटे मौसम की जानकारी प्रदान कर
वह वास्तविक अंतरिक्ष यात्रियों के उड़ान भरने से पहले रहा है। प्रत्येक सोल (एक मंगल ग्रह का दिन) के प्रत्येक
अंतरिक्ष के लिए आवश्यक मानवीय कार्यों का अनुकरण सेकंड के दौरान डेटा और इसे दैनिक आधार पर पृथ्वी पर
करेगी। भेजता है।

87
अंतर्दृष्टि के घटक (Components of the Insight): और परिवहन करते हैं।

• भूकंपमापी- भूकंप का पता लगाने के लिए।

• सेंसर- हवा और हवा के दबाव को मापने के लिए। मलबा हटाना मिशन (Remove Debris
• मैग्नेटोमीटर और हीट फ्लो प्रोब- ग्रह का तापमान ले ने Mission):
के लिए।
• मलबा हटाना एक यूरोपीय संघ फ्रेमवर्क 7 अनुसंधान
परियोजना है जो एक कम लागत वाली कक्षा में प्रदर्शक
मिशन को विकसित करने और उड़ान भरने के लिए
पार्क र सौर जांच (Parker Solar Probe): है जिसका उद्देश्य भविष्य के सक्रिय मलबे को हटाने
(एडीआर) मिशनों के लिए आवश्यक प्रौद्योगिकियों को
• पार्क र सोलर प्रोब मिशन सूर्य को समझने के लिए नासा
जोखिम से मुक्त करना और सत्यापित करना है।
का ऐतिहासिक मिशन है।
• परियोजना का उद्देश्य वैश्विक/यूरोपीय सक्रिय मलबा
• यह नासा के "लिविं ग विद अ स्टार" कार्यक्रम का हिस्सा
हटाने के रोडमैप में योगदान देना है।
है जो सूर्य-पृथ्वी प्रणाली के विभिन्न पहलु ओ ं की पड़ताल
करता है। • यूरोपीय अंतरिक्ष एजेंसी मलबे के क्षेत्र में सबसे सक्रिय
में से एक है निष्कासन। इसने क्लीनस्पेस रोडमैप की
• सौर गतिविधि पर नया डेटा प्रदान करके यह सूर्य की
एक श्रृंखला तैयार की है, जो अंतरिक्ष मलबे के शमन और
बदलती परिस्थितियों के बारे में समझने में मदद करेगा
अंतरिक्ष मलबे के उपचार के लिए प्रौद्योगिकियों पर ध्यान
जो सौर मंडल में फैल सकता है, पृथ्वी और अन्य दुनिया
केंद्रित करती है।
को प्रभावित कर सकता है।
• यह अंतरिक्ष कचरे के चुने हुए टु कड़े को एक बार में अपने
• पार्क र सोलर प्रोब के निम्नलिखित 3 विज्ञान उद्देश्य हैं:
रोबोटिक हथियारों का उपयोग करके पकड़ ले गा और
• ऊर्जा के प्रवाह का पता लगाएं जो सौर कोरोना और सौर नियंत्रित वंश में वापस पृथ्वी की ओर गिरेगा।
पवन को गर्म और तेज करता है।
• जापान ने पहले एक मालवाहक जहाज लॉन्च किया था
• सौर पवन के स्रोतों पर प्लाज्मा और चुंबकीय क्षेत्रों की जो पृथ्वी की कक्षा से कुछ मलबे को हटाने के लिए 700
संरचना और गतिशीलता का निर्धारण करें। मीटर लं बे तार का उपयोग करेगा।
• उन तंत्रों का अन्वेषण करें जो ऊर्जावान कणों को गति

88
अध्याय - 8

रक्षा
(DEFENCE)

• रक्षा देश की सेनाओ ं का संगठन और सरकार तथा उसके एकीकृत निर्देशित मिसाइल विकास कार्यक्रम
हितों की सुरक्षा के लिए शस्‍त्रों का उपयोग है। रक्षा के (आईजीएमडीपी) (Integrated Guided Missile
माध्‍यम से संप्रभुता, स्‍वतंत्रता, एकता और क्षेत्रीय अखंडता Development Program, IGMDP)
को सुनिश्चित करने के लिए राज्‍य के संस्‍थानों और राज्‍यों
• मिसाईल प्रौद्योगिकी में आत्‍म–निर्भरता हासिल करने के
के संसाधनों की सक्रिय भागीदारी द्वारा राष्‍ट्रीय हितों की
लिए भारत ने आईजीएमडीपी आरंभ की है। सुरक्षा बलों
सुरक्षा की जाती है।
द्वारा विभिन्‍न मिसाइलों की आवश्‍यकताओ ं पर विचार
करते हुए आईजीएमडीपी ने पांच मिसाइल प्रणालियों के
विकास की आवश्‍यकता की पहचान की। यह रणनीतिक,
भारत में रक्षा (Defence in India) स्‍वदेशी मिसाईल प्रणालियों को आकार देने के लिए देश
• भारतीय अर्थव्‍यवस्‍था में रक्षा व्‍यय वाले महत्‍वपूर्ण क्षेत्रों के वैज्ञानिक समुदाय, शैक्षणिक संस्‍थानों, अनुसंधान और
में से एक है। भौगोलिक और स्थलाकृतिक विविधता, विकाश प्रयोगशालाओ ं, उद्योगों और तीन रक्षा सेवाओ ं को
विशेषकर 15,000 किमी. लं बी सीमा जिसे भारत सात साथ ले कर आया। और भारत को मिसाइल प्रौद्योगिकी
पड़ोसी देशों के साथ साझा करता है भारतीय रक्षा बलों के में आत्‍म-निर्भर बनाने का लक्ष्‍य हासिल करने के बाद
लिए अद्वितीय चुनौतियां पेश करता है। 8 जनवरी 2008 को डीआरडीओ ने आईजीएमडीपी के
सफलतापूर्वक पूरा होने की औपचारिक घोषणा की।
• भारत सरकार भारत की और उसके साथ प्रत्‍येक भाग
की सुरक्षा सुनिश्चित करने के लिए जिम्‍मेदार है। सशस्‍त्र
सेनाओ ं की सर्वोच्‍च कमान राष्‍ट्रपति के पास है। राष्‍ट्रीय
आईजीएमडीपी के भाग के रूप में निम्‍नलिखित
सुरक्षा की जिम्‍मेदारी कैबिनेट (मंत्रिमंडल) के पास है।
मिसाईलों का विकास किया गया (Following
इसे रक्षा मंत्रालय के माध्‍यम से किया जाता है।
Missiles were Developed as Part of
IGMDP)
पृथ्‍वी I (Prithvi I)
भारत का सुरक्षा वातावरण (India's Security
Environment) 1. यह आईजीएमडीपी के अंतर्गत भारत की पहली स्‍वदेश में
विकसित बैलिस्टिक मिसाइल है।
• भारत का सुरक्षा वातावरण क्षेत्रीय और वैश्विक आदेशों
और चुनौतियों की जटिल अन्‍योन्‍यक्रिया द्वारा परिभाषित 2. कम दूरी की सतह से सतह पर मार करने वाली मिसाइल।
है। देश का आकार और रणनीतिक अवस्थिति हमें सुरक्षा 3. फरवरी 1988 में लांच की गई पृथ्‍वी I, एक एकल-चरण,
गतिशीलता के केन्‍द्र में रखते हैं जो कि एक ओर क्षेत्रीय तरल-ईंधन वाली मिसाइल है।
और वैश्विक कनेक्टिविटी के सकारात्‍मक बलों द्वारा
4. इसकी रेंज 150 किमी है और माऊंटिंग क्षमता 1000
साथ-साथ प्रभावित होते हैं और दूसरी ओर निकटतम और
कि.ग्रा. है।
विस्‍तारित पड़ोस के भागों में अप्रत्‍याशितता, अस्थिरता,
और चंचलता से उठने वाले परिणामों द्वारा प्रभावित होते 5. इसे 1994 में भारतीय सेना में शामिल किया गया।
हैं। 6. यह फील्ड इं टरचेंजेबल वॉरहेड्स के साथ उच्च घातक
• विभिन्‍न भू-राजनीतिक और अंतर्निहित सुरक्षा चुनौतियों प्रभाव और उच्च स्तरीय क्षमता प्रदर्शि त करता है।
पर विचार करते हुए, भारत समय-समय पर अपनी रक्षा
का संवर्धन करता रहा है, इसलिए भारत अपने महत्‍वपूर्ण
राष्‍ट्रीय हितों और मूल्‍यों आदि की प्रभा‍वशाली तरीके से अग्नि I (Agni I)
सुरक्षा कर सकता है। इसे प्राप्त करने के लिए भारत ने
• एक नाभिकीय – सक्षम बैलिस्टिक मिसाइल।
इस अध्‍याय में उल्लिखित अनुसार अनेक रक्षा उत्‍पादों
और प्रद्योगिकियों का अधिग्रहण किया है। • यह रक्षा अनुसंधान तथा विकास संगठन द्वारा अग्नि
श्रृंखला की पांच मिसाईलों में से पहली है।

• इसकी रेंज 700 कि.मी. है।

89
• मध्‍यवर्ती रेंज की सतह से सतह पर मार करने वाली करने वाली मिसाइल है।
बैलिस्टिक मिसाइल।
2. यह सभी उपलब्‍ध वायुयान जैमरों के विरूद्ध इले क्‍ट्रॉनिक
• बाद में अपने रणनीतिक महत्‍व के कारण अग्नि मिसाइल उपायों से सुसज्जित है।
को IGMDP से अलग कर दिया गया।
3. इसकी रेंज 9 कि.मी. है।

4. इसका उपयोग कम ऊंचाई से होने वाले हमलों के विरूद्ध


आकाश (Akash) पोत से एक समुद्री स्किमर रोधी के रूप में किया जाता है।

1. यह एक मध्‍यम दूरी की सतह से हवा में मार करने वाली


मिसाईल है। नाग (Nag)
2. इं टरसेप्ट (अन्तररोध) रेंज – 30 कि.मी.
1. नाग तीसरी पीढ़ी की एक हिट-टू -किल एं टी-टैं क
3. इसमें बहु-लक्ष्‍य नियोजन क्षमता होती है और भारतीय मिसाइल है। इसका पहला परीक्षण 1990 में किया गया।
सेना तथा भारतीय वायु सेना के पास प्रचालनात्‍मक सेवा
2. नाग मिसाइलों को DRDO ने एकीकृत गाइडेड मिसाइल
में है।
विकास कार्यक्रम (Integrated Guided Missile
Development Program-IGMDP) के तहत विकसित
किया है।
त्रिशूल (Trishul)

1. त्रिशूल, कम दूरी की निचले स्‍तर की सतह से हवा में मार

भारत द्वारा स्‍वदेश में विकसित विभिन्‍न मिसाइलों का विवरण (Various Missiles - Indigenously Developed by
India in Detail.)

स्‍वदेश में विकसित

नाभिकीय
प्राक्षेपिक /
मिसाइल प्रकार रूप से रेंज अन्‍य विशेषताएं
क्रू ज आदि
सक्षम?

अग्नि मिसाइल श्रृंखला

उन्‍नत उच्‍च परिशुद्धता नौचालन प्रणाली से


सुसज्जित दो-चरणीय मिसाइल एक नवीन
मध्‍यवर्ती 2000-
अग्नि II सतह से सतह प्राक्षेपिक हां आधुनिक कमान तथा नियंत्रण प्रणाली द्वारा
2500 कि.मी.
निर्देशित की गई और एक ठोस रॉकेट प्रणोदन
प्रणाली द्वारा प्रणोदित थी।

पड़ोसी देशों के बहुत भीतर तक लक्ष्‍यों को नष्‍ट


मध्‍यवर्ती –
करने में सक्षम। इसकी मारक क्षमता में वृद्धि
अग्नि II सतह से सतह प्राक्षेपिक हां 3500-5000
करके इसे जून 2011 में सशस्‍त्र बलों में शामिल
किमी.
किया गया।

1,000 कि.ग्रा पेलोड ले जाने के लिए दो-चरणीय


मध्‍यवर्ती
प्रणोदन प्रणाली लगी है। स्‍वदेश में विकसित
अग्नि IV सतह से सतह प्राक्षेपिक हां 3500-5000
रिं ग ले ज़र गायरो और कम्‍पोजिट रॉकेट मोटर
किमी.
से सुसज्जित।

उच्‍च सड़क चालकता, तीव्र प्रतिक्रिया क्षमता


लं बी रेंज 5000
अग्नि V सतह से सतह प्राक्षेपिक हां युक्‍त भारत की पहली अंतर्महाद्वीपीय प्राक्षेपिक
+ किमी.
मिसाइल।

पृथ्‍वी मिसाइल श्रृंखला

90
यह एकल-चरणीय तरल ईंधन युक्‍त मिसाईल
है जो कि 500 कि.ग्रा. वारहेड ले जाने में सक्षम है।

मिसाइल की उच्‍च परिशुद्धता है, यह किसी भी


लक्ष्य पर प्रभावशाली ढ़ंग से प्रहार कर सकती
पृथ्‍वी II सतह से सतह प्राक्षेपिक हां 350 किमी. तक है, जो इसे युद्ध क्षेत्र का एक शस्‍त्र बनाता है।
रॉकेट में बैलेस्टिक मिसाइल को धोखा देने की
विशेषता है।

2002 में, इसका मिसाइल प्रबंधन, भारतीय वायु


सेना से भारतीय सेना में स्‍थानांतरित हो गया।

पृथ्‍वी III एक नौसेना रूपांतर मिसाइल है


जिसकी रेंज 350 किमी. है।

पृथ्‍वी III का प्रथम परीक्षण आईएनएस सुभद्रा,


सुकन्‍या श्रेणी निगरानी पोत से 2000 में
पृथ्‍वी III सतह से सतह प्राक्षेपिक हां 350 किमी. तक
आयोजित किया गया जो कि आंशिक रूप से
सफल था।

2004 में पृथ्‍वी III मिसाइल का पूर्ण प्रचालना-


त्‍मक परीक्षण पूरा किया गया।

यह समुद्र से संचालित की जा सकने वाली द्रव


प्रणोदित मिसाइल है इस पर पृथ्‍वी II प्राक्षेपित
मिसाइल के कम-रेंज के रूपांतर के रूप में
कम रेंज 350 विचार किया गया।
धनुष सतह से सतह प्राक्षेपिक हां
कि.मी.
इसका मार्च 2011 में एक नौसेना युद्धपोत से
सफलतापूर्वक टे स्‍ट फायर किया गया और के-
15 सागरिका की विरासत को आगे बढ़ाया।

अन्‍य मिसाईलें

शौर्य, जल क्षेत्र में लांच की गई भारत की के-


15 मिसाइल का भू-रूपांतर है और इसे नौसेना
की नाभिकीय शक्ति सम्‍पन्‍न पन्‍डुब्‍बी अरिहंत
में फिट किया गया है। नौसेना और सेना दोनों
हाईपरसो- ही रॉकेट का उपयोग कर सकते हैं क्‍योंकि यह
शौर्य सतह-से-सतह हां 700 किमी.
निक विभिन्‍न भूमिकाएं निभा सकता है। मिसाइल
की नाभिकीय क्षमता भारत की द्वितीय-मारक
क्षमता में वृद्धि करती है और रूस की महत्‍वपूर्ण
सहायता से निर्मि त के-15 प्राक्षेपिक मिसाइल
पर निर्भरता को कम करती है।

यह 1,000 कि.मी. तक पहुंचने के लिए एक भू-


भाग अनुसरणीय नौचालन प्रणाली का उपयोग
करती है।

सब निर्भय बहुल प्‍लेटफार्म, भूमि, समुद्र और हवा से


लं बी दूरी 1000
निर्भय सोनिक हां लांच किए जाने में सक्षम है।
कि.मी. तक
क्रूज
निर्भय के सफल लांच के साथ भारत, राष्‍ट्रों के
विशिष्‍ट वर्ग में शामिल हो गया। जिसके पास
अबसब-सोनिक क्रूज मिसाइल बनाने की
क्षमता मौजूद है।

91
उच्‍च परिचालनीयता, गतिवर्द्धन और परिशुद्ध-
ता।

कम दूरी 150 प्रारंभिक रूप से सेना के लिए एक युद्ध क्षेत्र


प्रहार सतह से सतह प्राक्षेपिक नहीं सहायता प्रणाली के तौर पर मिसाइल को
कि.मी.
रोड़-मोबाइल लांचरों से दागा जा सकता है और
यह अपने हल्‍के वजन के कारण युद्ध स्थिति में
अत्‍यधिक गतिशील है।

अस्‍त्र दृश्‍यता से परे की रेंज (बीवीआर) वाली


हवा से हवा में मार करने वाली (एएएम)
मिसाइल है जिसका मई 2011 में सफल परी-
क्षण किया गया। बीवीआर क्षमता के साथ अस्‍त्र
मिसाइल अत्यधिक शक्ति वर्द्धन का मार्ग प्र-
शस्त करेगी और नौसेना तथा वायु सेना की
मारक क्षमता में अत्‍यधिक वृद्धि करेगी।
सुपरसो- कम दूरी 80
अस्‍त्र हवा-से-हवा नहीं इसमें इले क्‍ट्रॉनिक प्रत्युत्तर क्षमताएं हैं।
निक कि.मी.
आकार और वजन के संदर्भ में अस्‍त्र डीआरडीओ
द्वारा विकसित सबसे छोटी मिसाइल है।

इसमें 80 कि.मी. की रेंज के भीतर हेड-ऑन-


मोड के साथ सुपरसोनिक गति से शत्रु के
वायुयान के अवरोधन और उसे नष्ट करने की
क्षमता है।

यह पनडुब्‍बी से लांच की जा सकने वाली अपनी


प्राक्षेपिक मिसाईल (SLBM) क्षमता के साथ-
साथ भारत की नाभिकीय रक्षा का महत्वपूर्ण
सागरिका तीसरा चरण बनाती है।
प्राक्षेपिक हां 750 किमी.
के -15
इसे भारत की नाभिकीय शस्‍त्र समर्थ अरिहंत
श्रेणी की पनडुब्‍बी के साथ एकीकृत किया गया
था।

इसका विकास भारत और रूस के बीच एक


संयुक्‍त उद्यम के तौर पर किया गया था और प्र-
चालन में यह विश्‍व की तीव्रतम पोत रोधी क्रूज
मिसाईल है।
सुपरसो- 450 कि.मी –
ब्रह्मोस सतह-से-सतह हां एक ‘फायर एं ड फॉर्गेट’ शस्‍त्र, यानि एक बार
निक क्रूज 600 कि.मी.
लक्ष्‍य निर्धारित करने के बाद नियंत्रण केन्‍द्र से
अधिक निर्देशन की कोई आवश्‍यकता नहीं।

ब्रह्मोस विश्‍व में अपनी श्रेणी की तीव्रतम क्रूज


मिसाईल है।

स्‍कॉर्पीन श्रेणी प्रहार पनडुब्बियां (Scorpene Class हस्‍तांतरण के लिए भारतीय नौसेना की परियोजना-75
Attack Submarines): के अंतर्गत फ्रांस के सहयोगी मैसर्स नैवल ग्रुप और
मज़गांव डॉक लिमिटे ड के बीच 2005 में एक करार पर
• परियोजना 75 आई (P75आई) भारतीय नौसेना की
हस्‍ताक्षर किए गए।
स्‍कॉर्पीन पनडुब्‍बी विकास परियोजना है। छह स्‍कॉर्पीन
श्रेणी की पनडुब्बियों के लिए प्रौद्योगिकी के निर्माण और • P75आई स्‍कॉर्पीन श्रेणी की डीज़ल-विद्युत/वायु –स्‍वतंत्र

92

@ITAKASHSINGH
प्रणोदन पनडुब्बियां स्‍कॉर्पीन श्रेणी की पनडुब्बियों पर • एएच-64 अपाचे अमरीकी सेना द्वारा उपयोग किया जाता
आधारित हैं। है और यह विश्‍व का सर्वाधिक उन्‍नत बहु-भूमिका वाला
लड़ाकू हेलिकॉप्‍टर है।
• इन स्‍कॉर्पीन्‍स में दबाव के प्रति क्षमता और जल के भीतर
छिपे रहने की योग्यता होती है। इसमें 25 से 31 के बीच • यह एक उन्‍नत बहु-मिशन हेलिकॉप्‍टर है जिसमें
कर्मीदल सदस्‍यों के साथ साथ 14 लड़ाकू गोताखोर भी नवीनतम प्रौद्योगिकियां लगी हैं, जो इसकी विश्‍व का
आ सकते हैं। सर्वश्रेष्‍ठ हमलावर हेलिकॉप्‍टर होने की क्षमता को बनाए
रखती हैं और यह ‘फ्लाइं ग टैं क’ के नाम से जाना जाता है।
• भारतीय नौसेना इन पनडुब्बियों का उपयोग आसूचना,
यह एक हर मौसम में काम करने वाला हेलिकॉप्‍टर है।
एकत्रिकरण, क्षेत्र की निगरानी, पनडुब्‍बी रोधी युद्ध
स्थिति, सतह रोधी युद्ध स्थिति और सुरग
ं बिछाने के • यह हेलिकॉप्‍टर विभिन्‍न प्रकार के शस्‍त्र की डिलीवरी
प्रचालनों जैसे मिशनों के लिए करना चाहती है। कर सकता है जिसमें एक हेलिकॉप्‍टर में 8 मिसाइलें ले
जाने की क्षमता है।
• आईएनएस कलवारी, आईएनएस खंडेरी, आईएनएस
करंज, वेला और वजीर पहले ही लांच किए जा चुके हैं। • इसमें बेहतर प्रतिबल और लिफ्ट, संयुक्‍त डिजिटल
प्रचालनीयता, उन्‍नत उत्‍तरजीविता और संज्ञानात्‍मक
निर्णय सहायक है।
आईएनएस नीलगिरी (INS Nilgiri):
• इसमें नेटवर्क केन्द्रित हवाई युद्धस्थिति में हेलिकॉप्‍टर को
• आईएनएस नीलगिरी परियोजना 17A के अंतर्गत बहुविज्ञता प्रदान करने के लिए आधुनिक इले क्‍ट्रॉनिक
विकसित सात छिपे हुए जलपोतों में पहला है जिसका युद्धस्थिति क्षमताएं हैं।
लक्ष्‍य उत्‍तरजीविता, समुद्र सुरक्षा, छिपाव और पोत • ये हेलिकॉप्‍टर अन्‍य समान मशीनों की तुलना में अधिक
परिचालनीयता में सुधार के लिए उन्‍नत विशेषताओ ं और नीचे और तेज उड़ने में सक्षम हैं जिससे जमीन पर चक्‍कर
स्‍वदेशी शस्‍त्रों और सेन्‍सरों से युक्‍त मौजूदा शिवालिक लगाते समय रेडार इन्‍हें नहीं पकड़ पाती।
श्रेणी के जलपोतों का डिजाइन और अद्यतन करना है।

• यह भारत में प्रथम प्रमुख युद्धपोत है जिसका निर्माण एक


एकीकृत निर्माण पद्धति से किया गया है जिसमें छोटे रूस्‍तम 2 ड्रोन (Rustom 2 Drone):
मॉड्यूल का निर्माण करना और उन्‍हें एक साथ असेम्‍बल
• यह डीआरडीओ द्वारा विकसित मध्यम–तुंगता,
करना शमिल है।
दीर्घकालिक स्थिरता वाला मानवरहित वायु वाहन है।

• रूस्‍तम 2 यूएवी की रूस्‍तम पंक्ति का एक भाग है जिसमें


एलसीए तेजस (LCA Tejas): रूस्‍तम-I, रूस्‍तम-H और रूस्‍तम – C शामिल है। इसे 24
घंटे कार्य करने की क्षमता के साथ सशस्‍त्र बलों के लिए
• हल्‍का लड़ाकू वायुयान (एलसीए) तेजस एक एकल निगरानी करने के लिए विकसित किया गया है।
इं जन, हल्‍के वजन, अत्‍यधिक दक्ष, बहु-भूमिका वाला
• डीआरडीओ द्वारा रूस्‍तम-2 की विशेषताओ ं को पहले से
लड़ाकू यान है। डेल्‍टा विं ग युक्‍त वायुयान को ‘हवाई
सेना में सेवारत इजराइली हेरोन के बराबर लाने पर कार्य
समाघात’ और ‘आक्रामक हवाई सहायता’ के लिए
करने की रिपोर्ट है।
डिजाइन किया गया है जिसमें उसकी गौण भूमिकाएं ‘टोह’
और ‘पोत-रोधी’ है।

• वायुयान का डिजाइन और विकास कार्यक्रम, हिन्‍दुस्‍तान सुखोई एस यू -30 मेक I (Sukhoi Su-30MKI):
एयरोनॉटिक्‍स लिमिटे ड, जो कि इसका प्रमुख औद्योगिक
संविदाकार है के साथ मिलकर भारतीय रक्षा विभाग की • यह ट्विन जेट बहु भूमिका वाला वायु श्रेष्‍ठता वाला फाइटर
वैमानिकीय विकास एजेन्‍सी (एडीए) के नेतृत्‍व में किया जेट है। यह एक बहुभूमिका वाला समाघात लड़ाकू
गया है। वायुयान है जिसका भारतीय वायु सेना (IAF) के लिए
सुखोई डिजाइन ब्‍यूरो और हिन्‍दुस्‍तान एयरोनॉटिक्‍स
• भारत के हल्के लड़ाकू वायुयान, तेजस के नौसेना रूपांतर
लिमिटे ड (HAL) द्वारा संयुक्‍त विकास किया गया है।
ने अपनी पहली नियंत्रित लैं डिं ग वायुयान कैरियर आई
एन एस विक्रमादित्य पर की। • वायुयान में टिटे नियम और उच्‍च सघनता वाले
एल्‍यूमीनियम मिश्रधातु से बना एयरोडायनेमिक
एयरफ्रेम होता है।
अपाचे हेलिकॉप्‍टर (एएच-64ई) (Apache Helicopters • वायुयान में एक फ्लाई-बाय-वायर (FBW) उड़ान नियंत्रण
(AH-64E)): प्रणाली एकीकृत होती है। इसके पिछले कॉकपिट में एक
विशाल मोनोक्रोमैटिक डिस्‍प्‍ले स्‍क्रीन लगी होती है जो

93
हवा से भूमि का मिसाइल निर्देशन प्रदान करता है। • उन्‍नत वायु रक्षा (एएडी) मिसाइल प्रणाली – अपनी
ओर आ रही मिसाईलों को 15-25 कि.मी. की तुंगता पर
• इसमें हवा-से सतह की मिसाइल और ले ज़र निर्देशित
अवरूद्ध करने के लिए।
युद्ध-सामग्री को निर्देशित करने के लिए एक पैसिव
इले क्‍ट्रॉनिकली स्‍कैन्‍ड ऐरे रेडार, ले ज़र-ऑप्टिकल
लोकेटर प्रणाली और लिस्निं ग टार्गेट डेजिग्‍नेशन पॉड भी
पृथ्‍वी वायु रक्षा (पीएडी) प्रणाली (Prithvi Air Defense
लगा होता है।
(PAD) System)
• इन जेट में ब्रह्मोस हवा से सतह पर मार करने वाली
• पीएडी पृथ्‍वी मिसाईल पर आधारित दो-चरणीय मिसाइल
मिसाइलों को ले जाने के लिए आशोधन किया गया
है। इसे प्रद्युम्‍न के नाम से भी जाना जाता है। प्रथम चरण
है जिससे इनमें लं बी दूरी तक सटीक प्रहार करने की
तरल ईंधन युक्‍त है, और दूसरा चरण ठोस ईंधन युक्‍त
क्षमता आ जाती है। 2.5 टन वजन वाली ब्रम्‍होस एसयू-30
है। रॉकेट पार्श्वि य गतिवर्द्धन के लिए परिचालनीय
मेक I लड़ाकू वायुयान पर तैनात किया जाने वाला सबसे
प्रतिबलकों से सुसज्जित है। यह मैक 5 की गति से
भारी शस्‍त्र है।
प्राक्षेपिक मिसाइलों (300 कि.मी. – 2,000 कि.मी. रेंज)
को तैनात कर सकती है।
भारत का बैलिस्टिक मिसाईल रक्षा कार्यक्रम (बीएमडी) • यह लक्ष्‍य अधिग्रहण और अग्नि नियंत्रण के लिए लं बी
(India’s Ballistic Missile Defence Programme, रेंज की ट्रैकिंग रेडार (एलआरटीआर) का उपयोग करती
BMD)
है। एलआरटीआर एक सक्रिय चरण ऐरे रेडार है जो कि
• अप्रत्याशित व दुर्भावनापूर्ण हमलों से देश की रक्षा करने के 600 कि.मी. की रेंज के भीतर 200 लक्ष्‍यों का पता लगाने
लिए एक बहु-परतीय बैलिस्टिक मिसाइल रक्षा प्रणाली में सक्षम है। प्रक्षेपपथ इष्‍टतमीकरण, ऊंची और निचली
का सृजन करने के लिए डीआरडीओ इसका विकास कर तुंगताओ ं दोनों पर अवरोधन को सक्षम बनाता है। पीएडी
रहा है। 1998 में पाकिस्‍तान के एकमात्र परमाणु परीक्षण मिसाइल का स्‍थान पृथ्‍वी रक्षा मिसाइल ने ले लिया है जो
और इस क्षेत्र में चीन के आगे बढ़ने के बाद 1999 में भारत कि 2000 किमी. रेंज तक आरबीएम तैनात कर सकती
का बीएमडी कार्यक्रम शुरू किया गया। है।

• बीएमडी बेजोड़ विशेषताओ ं वाली एक अत्‍याधुनिक


रक्षा प्रणाली है। बैलिस्टिक मिसाईल रक्षा कार्यक्रम का पृथ्‍वी रक्षा वाहन (पीडीवी) पृथ्‍वी के वातावरण से बाहर
उद्देश्‍य अपनी ओर आ रही शत्रु की प्राक्षेपिक और परमाणु लगभग 2000 कि.मी. की मारक रेंज के साथ एक आवक
मिसाईलों के विरूद्ध एक प्रभावी मिसाइल कवच प्रदान मिसाइल को नष्‍ट करने में सक्षम है। यह मौजूदा पृथ्‍वी वायु
करना है। बूस्‍ट प्‍वाइं ट (लांच), मध्‍य-मार्ग (आकाश रक्षा (पीएडी) का स्‍थान ले गी जिसकी अधिकतम इं टरसे-
में उड़ान के दौरान), या टर्मि नल चरण (वातावरणीय प्‍शन तुंगता 80 किलोमीटर है। इतनी उच्‍च तुंगता पर एक
अवरोहण के दौरान) पर एक शत्रु के रॉकेट का अवरोधन आवक मिसाइल को अवरूद्ध करने का लाभ यह है कि
किए जाने की आवश्‍यकता है। मलबा भूमि पर नहीं गिरेगा और कोई अतिरिक्‍त क्षति नहीं
होगी।

बीएमडी दो स्‍तरों पर कार्य करता है (The BMD Works on


Two Levels)
उन्नत वायु रक्षा (ए ए डी) (Advanced Air Defence,
AAD)
• अंतर्वातावरणीय स्‍तर: पृथ्‍वी के वातावरण के भीतर जो
अपनी ओर आने वाली मिसाइलों को अंतर्वातावरण में • यह डीआरडीओ द्वारा विकसित एक प्राक्षेपिक मिसाइल
नष्‍ट कर देता है। अवरोधक है जिसका उपयोग अपनी ओर आ रही
• अंत: वातावरणीय स्‍तर: पृथ्‍वी के वातावरण से परे फैला मिसाईलों को 30 कि.मी. की ऊंचाई पर अंतर्वातावरणीय
आकाश। यह अपनी ओर आ रही मिसाइलों को अंत: क्षेत्र में मोड़ने के लिए किया जाता है। ए ए डी में एक
वातावरण में नष्‍ट करने का कार्य करता है। एकल- चरण ठोस प्रणोदित निर्देशित मिसाइल होती है।
इसका निर्देशन पृथ्वी वायु रक्षा प्रणाली के समान है।
निर्देशन में जड़त्वीय नौचालन प्रणाली होती है और इसे
भारत में बीएमडी शस्‍त्रागार में निम्न शामिल है (India’s भू-आधारित रेडार प्रणाली से अर्ध-मार्ग अद्यतन प्राप्त
BMD Arsenal Consists of) होता है। टर्मि नल चरण में यह एक्टिव रेडार होमिं ग का
उपयोग करता है।
• पृथ्‍वी वायु रक्षा (पीएडी) मिसाइल प्रणाली - अपनी ओर
आ रही मिसाइलों को लगभग 80 कि.मी. की तुंगता पर • 2007 में, उन्नत वायु रक्षा ने अपनी ओर आ रही आशोधित
अवरूद्ध करने के लिए और पृथ्वी II प्राक्षेपिक मिसाइल को सफलतापूर्वक अवरूद्ध

94
किया। यह अवरोधन 15 कि.मी. की ऊंचाई पर किया गया भीतर छोड़ा जा सकता है।
जिसमें लक्ष्य नष्ट हो गया और कई टु कड़ों में टू ट गया। ए
ए डी इं टरसेप्टर मिसाइल के सफल लांच ने 150 कि.मी.
तक की रेंज वाली वर्धि त रेंज वाली सतह से हवा में मार बराक 8 (Barak 8)
करने वाली इं टरसेप्टर मिसाईलों के विकास के लिए
मार्ग खोल दिया है। अभी तक उन्नत वायु रक्षा इं टरसेप्टर • यह एक लं बी-रेंज वाली नौसेना की सतह से हवा में मार
मिसाइल के अनेक परीक्षण किए गए हैं। नवीनतम करने वाली मिसाइल (एलआरएसएएम) है जिसका
सफल परीक्षण 2018 में किया गया जिसमें 1500 कि.मी. भारत और इज़राईल द्वारा संयुक्‍त रूप से विकास किया
श्रेणी की प्राक्षेपिक मिसाइल का अनुरूपण करने वाले गया है।
बहु आवक लक्ष्यों में से एक को सफलतापूर्वक नष्ट • यह प्राक्षेपिक मिसाइलों और अन्‍य लक्ष्‍यों जैसे पोत-रोधी
किया गया। मिसाइलों, वायुयानों, हेलिकॉप्‍टरों, यूएवी, क्रूज मिसाइलों
आदि को भी नष्‍ट कर सकती है।

• हालांकि शक्तिशाली प्राक्षेपिक मिसाइलों को नष्‍ट करने


स्‍वॉर्डफिश रेडार (Swordfish Radar)
की इसकी क्षमता सीमित है।
• स्‍वॉ‍र्डफिश बीएमडी प्रणाली के लिए विकसित की गई एक
लं बी रेंज की ट्रैकिंग रेडार है। इसे इजराइल की ग्रीन पाईन
लं बी रेंज रेडार से लिया गया है। स्‍वॉर्डफिश 80 कि.मी. से परमाणु खनिज अन्‍वेषण और अनुसंधान निदेशालय
ऊपर की तुंगताओ ं पर हवाई लक्ष्‍यों को नष्‍ट करने के (Atomic Minerals Directorate for Exploration
and Research)
लिए अंत: वातावरणीय इं टरसेप्‍टर मिसाइल पीएडी को
निर्देशित करती है। यह रेडार 600 कि.मी. – 800 कि.मी. • परमाणु खनिज अन्‍वेषण और अनुसंधान निदेशालय का
की रेंज के भीतर छोटे लक्ष्‍यों की पहचान कर सकती है। प्रमुख कार्य देश के परमाणु ऊर्जा कार्यक्रम के सफल
कार्यान्‍वयन के लिए आवश्‍यक यूरनि
े यम संसाधनों की
पहचान और मूल्‍यांकन करना है।
भारत की प्राक्षेपिक मिसाईल रक्षा प्रणाली में अन्‍य • इसके कार्यान्‍वयन के लिए, नई दिल्‍ली, बेंगलु रू,
परिवर्धन (Other Additions in the Ballistic जमशेदपुर, शिल्‍लोंग, जयपुर, नागपुर, और हैदराबाद
Missile Defence System of India)
(मुख्‍यालय एवं दक्षिण मध्‍य क्षेत्र) में स्थित क्षेत्रीय अन्‍वेषण
एस-400 ट्रायम्‍फ मिसाइल रक्षा प्रणाली (S-400 Triumph एवं अनुसंधान केन्‍द्रों से देश भर में अनिवार्य कार्य जांच
Missile Defence System): की गई हैं।

• यह रूस के अलमाज़ सेन्‍ट्रल डिजाइन ब्‍यूरो द्वारा 1990 में


विकसित वायुयान रोधी शस्‍त्र प्रणाली है।
न्‍यूक्लियर पावर कार्पोरेशन ऑफ इं डिया
• यह शत्रु के अन्‍य लक्ष्‍यों जैसे फाइटर जेट, क्रूज मिसाइलों,
लिमिटे ड (एनपीसीआईएल) (Nuclear Power
यूएवी, आदि के साथ आवक प्राक्षेपिक मिसाईलों को नष्‍ट
Corporation of India Limited, NPCIL)
करने में सक्षम होगी।
• न्‍यूक्लियर पावर कार्पोरेशन ऑफ इं डिया लिमिटे ड
• यह श्रेष्‍ठ वायु रक्षा प्रणालियों में शामिल है।
(एनपीसीआईएल) परमाणु ऊर्जा विभाग (डीएई) भारत
• यह एक बहुप्रकार्यात्‍मक प्रणाली पर आधारित है। यह सरकार के अंतर्गत एक सार्वजनिक क्षेत्र का उपक्रम है।
विभिन्‍न गति पर मिसाईलें गिरा सकती है।
• इस कम्‍पनी को परमाणु ऊर्जा अधिनियम, 1962 के
• यह प्रणाली सुपरसोनिक और हाइपरसोनिक मिसाइलों तहत भारत सरकार की योजनाओ ं और कार्यक्रमों के
द्वारा वायुयान (पांचवी पीढ़ी तक), क्रूज और प्राक्षेपिक अनुपालन में विद्युत उत्‍पादन के लिए परमाणु शक्ति
मिसाइलों को लक्ष्‍य बना सकती है। संयंत्रों के परिचालन और परमाणु शक्ति परियोजनाओ ं
• यह आवक लक्ष्‍यों का 400 कि.मी. की रेंज तक और 40 के कार्यान्‍वयन के उद्देश्‍य से सितम्‍बर 1987 में कम्‍पनी
कि.मी. ऊंचाई तक पता लगा सकती है। अधिनियम, 1956 के अंतर्गत एक सार्वजनिक लिमिटे ड
कम्‍पनी के रूप में पंजीकृत किया गया।
• यह 400 कि.मी. की रेंज में एक समय में 36 लक्ष्‍यों को
निष्क्रिय कर सकती है। • एनपीसीआईएल की परमाणु ऊर्जा विभाग (डीएई) की
एक अन्‍य पीएसयू भाविनी में भी साम्‍या भागीदारी है जो
• इसकी रेडार संसूचन प्रणाली की रेंज 600 कि.मी. है और
कि देश में फास्‍ट ब्रीडर रिएक्‍टर प्रोग्राम को कार्यान्वित
100-300 लक्ष्‍यों को भेद सकती है।
करती है।
• इसमें मिसाइलों से लै स 12 लांचर है जिन्‍हें 5 मिनट के

95
• एनपीसीआईएल परमाणु शक्ति रिएक्‍टरों के डिजाइन, प्रत्‍येक कम्‍पनी और देश को एक करता है। परमाणु सुरक्षा
निर्माण, कमीशनिं ग, और परिचालन के लिए जिम्‍मेदार के सर्वोच्‍च संभव मानक प्राप्‍त करने के लिए इसका एक
है। प्रचालनीय व्‍यावसायिक परमाणु शक्ति संयंत्र है।

• संगठन प्रचालनात्‍मक सुरक्षा और विश्‍वसनीयता के


उच्‍चतम स्‍तरों को प्राप्‍त करने में अपने सदस्‍यों की विशुद्ध
अंतर्राष्‍ट्रीय परमाणु ऊर्जा एजेन्‍सी (The रूप से सहायता करने के लिए है।
International Atomic Energy Agency)
• इसे उच्‍चतम महत्‍व वाले प्रोग्रामों जैसे पीयर समीक्षाएं ,
• इसे संयुक्‍त राष्‍ट्र परिवार के भीतर 1957 में एक शांति और तकनीकी सहायता तक अभिगम और परिचालन
संगठन के लिए विश्‍व परमाणु के तौर पर स्‍थापित किया अनुभव का एक वैश्विक संग्रह की एक श्रृंखला के माध्‍यम
गया। से प्राप्‍त किया जाता है।

• संयुक्‍त राष्‍ट्र आम सभा और सुरक्षा परिषद दोनों को • लं दन में एक कार्यालय और मास्‍को, अटलांटा, टोक्‍यो,
रिपोर्ट करती है। तथा पेरिस में क्षेत्रीय केन्‍द्रों के साथ डब्‍ल्‍यूएएनओ
राजनीतिक बाधाओ ं और हितों से परे है।
• वियना, आस्ट्रिया में मुख्‍यालय।
• डब्‍ल्‍यूएएनओ एक अलाभ वाला संगठन है और परमाणु
• यह नाभिकीय क्षेत्र में वैज्ञानिक और तकनीकी सहयोग
उद्योग के लिए या उसकी ओर से वकालत नहीं करता।
के लिए विश्‍व केन्‍द्रीय अंतरसरकारी फोरम है।
• जबकि डब्‍ल्‍यूएएनओ अपने सदस्‍यों के साथ प्रत्यक्ष तौर
• यह नाभिकीय विज्ञान तथा प्रौद्योगिकी के सुरक्षित,
पर काम करता है, संगठन एक विनियामक निकाय
निश्चित और शांतिपूर्ण उपयोग के लिए कार्य करती है
नहीं है और कम्‍पनीयों को प्रारंभिक रिएक्‍टर डिजाइन
और अंतर्राष्‍ट्रीय शांति और सुरक्षा तथा संयुक्‍त राष्‍ट्र के
सिले क्‍शन के लिए कम्‍पनियों को सलाह नहीं देता।
धारणीय विकास लक्ष्‍यों में योगदान देती है।
• सुरक्षा का एकमात्र लक्ष्‍य होने के साथ, डब्‍ल्‍यूएएनओ
प्रभावी तरीके से संप्रेषण करने और खुले तौर पर सूचना
विश्‍व परमाणु प्रचालक संघ (डब्‍ल्‍यूएएनओ) साझा करने में परिचालकों की सहायता करता है।
(World Association of Nuclear Operators, • डब्‍ल्‍यूएएनओ की दूरदर्शि ता: डब्‍ल्‍यूएएनओ और उसके
WANO) सदस्‍य पूरे विश्‍वभर के नेता होंगे जो व्‍याव‍सायिक

• विश्‍व परमाणु प्रचालक संघ (डब्‍ल्‍यूएएनओ) विश्‍व में परमाणु शक्ति के लिए प्रचालनात्‍मक परमाणु सुरक्षा में
उत्‍कृ ष्‍टता की खोज में रहेंगे।

96
अध्याय - 9

कुछ उन्न
‍ त प्रौद्योगिकियां
(SOME ADVANCED TECHNOLOGIES)

कृत्रिम बुद्धिमता (Artificial • 1997 में डीप ब्‍लू , विश्‍व शतरंज चैम्पियन, गैरी कास्‍पारोव
को हराने वाला पहला कम्‍प्‍यूटर बन गया। 2008 में गूगल
Intelligence) ने वाक पहचान में (आज के आधुनिक वाक् अनुवादकों
में उपयोग की गई प्रौद्योगिकी) बड़ी सफलता हासिल की।
• सोचने और समझने की क्षमता बुद्धिमता है: जब मनुष्‍य 2004 में गूगल ने प्रथम स्‍वचालित‍ कारें बनाईं।
बुद्धिमता प्रदर्शि त करते हैं, इसमें अधिकतर कुछ बोध
और भावुकता शामिल होती है; हम निर्णय ले ने के लिए
भावनाओ ं को कारणों के साथ मिला देते हैं। कृत्रिम एआई के प्रकार (Types of AI)
बुद्धिमता मशीनों द्वारा प्रदर्शि त की गई बुद्धिमता है; यह
संज्ञानात्‍मक कार्यों की नकल करती है। हम मनुष्‍य के एआई को दो प्रकार में विभाजित किया गया है:
रूप में मानव मस्तिष्‍क के साथ जुड़े हैं। कृत्रिम बुद्धिमता
• सॉफ्टवेयर एआई: परोक्ष सहायक, सर्च इं जन, वाक् तथा
के प्रस्‍तावों ने इसे इस सिद्धांत पर विकसित किया है कि
चेहरा पहचान, आदि।
मानव बुद्धिमता की इतने आसान तरीके से व्‍याख्‍या की
जा सकती है कि एक मशीन इसकी नकल कर सकती है • सम्मिलित एआई: स्‍वचालित कारें, ड्रोन, रोबोट आदि।
और निष्‍पादित कर सकती है। कृत्रिम बुद्धिमता (एआई)
का संबंध उन मशीनों में मानव बुद्धिमता के अनुरूपण
से है जिन्‍हें मानव की तरह सोचने और उनके कार्यों की कृत्रिम बुद्धिमता का श्रेणीकरण (Categorization
नकल करने के लिए बनाया गया है। of Artificial Intelligence)

• कृत्रिम बुद्धिमता को दो भिन्‍न श्रेणियों में भी बांटा


जा सकता है: कमजोर और मजबूत। कमजोर कृत्रिम
एआई की उत्‍पत्ति (Evolution of AI) बुद्धिमता में एक कार्य करने के लिए डिजाइन की गई
• एलन तुरिं ग ने इतिहास बदल दिया जब उन्‍होंने एक सरल प्रणाली शामिल होती है। कमजोर एआई प्रणालियों में
प्रश्‍न पूछा, ‘’क्‍या मशीनें सोच सकती हैं।‘’ एलन तुरिं ग वीडियो गेम और वैयक्तिक सहायक जैसे कि अमेजन का
के परिकलन के सिद्धांत ने सुझाया कि शून्‍य और एक अले क्सा और ऐपल का सिरी शामिल हैं।
जैसे चिह्नों का स्‍थान बदलना किसी गणितीय घटाव • बहुआयामी कृत्रिम बुद्धिमता प्रणालियां वे प्रणालियां हैं जो
के किसी कल्‍पनीय कार्य का अनुरूपण कर सकता उन कार्यों को करती हैं जिन्‍हें मानव – जैसा समझा जाता
है जिसके फलस्‍वरूप ‘’चर्च-टू रिं ग अभिधारणा’’ बनी। है। ये अधिक जटिल और पेचीदा प्रणालियां प्रतीत होती
चर्च टू रिं ग की अभिधारणा इस बात पर बल देती है कि हैं। उनमें ऐसी परिस्थितियों को संभालने के लिए प्रोग्राम
डिजिटल कम्‍प्‍यूटर औपचारिक तर्क का प्रसंस्‍करण कर डाला गया है जिनकी उन्‍हें किसी व्‍यक्ति के हस्‍तक्षेप के
सकती है। परन्‍तु कृत्रिम बुद्धिमता शब्‍द 1956 में प्रकाश बिना समस्‍या सुलझाने के लिए आवश्‍यकता होगी। इस
में आया जब डार्टमाऊथ कॉले ज में एक कृत्रिम बुद्धिमता प्रकार की विधियां स्‍वचालित कारों या अस्‍पताल क
अनुसंधान कार्यशाला स्‍थापित की गई। शल्‍य-चिकित्‍सा कक्ष जैसे अनुप्रयोगों में पाई जा सकती
• हर्बर्ट सिमोन (CMU), जॉन मैकार्थी, मार्वि न मिं स्‍की, और हैं।
आर्थर सैमुअल कृत्रिम बुद्धिमता अनुसंधान के संथापक
बने। 1959 तक कम्‍प्‍यूटर बीजगणित में चैकर रणनीतियों
द्वारा वर्ड प्रोब्‍लम को सुलझाना सीख रहे थे और अंग्रेजी संकीर्ण एआई बनाम सामान्‍य एआई (Narrow AI Vs.
General AI):
बोल रहे थे। नई प्रौद्योगिकी जैसे कि वीएलएसआई CMOS
ट्रांजिस्‍टर प्रौद्योगिकी ने 1980 तक कम्‍प्‍यूटर विज्ञान की • संकीर्ण एआई एक ऐसे एआई को परिभाषित करती है जो
दुनिया को बदल दिया। एक सीमित कार्य या कार्यों की एक निर्धारित संख्‍या तक
• 1990 के दशक के अंत में और 21वीं शताब्‍दी की शुरूआत सीमित है। उदाहरण के लिए, IBM के डीप ब्‍लू की क्षमताएं ,
में एआई का उपयोग संभार-तंत्र डाटा माइनिं ग, चिकित्‍सा 1997 में शतरंज खेलने वाले कम्‍प्‍यूटर ने विश्‍व चैम्पियन
निदान और अन्‍य क्षेत्र में होना आरंभ हो गया। गैरी कास्‍पारोव को हरा दिया, केवल शतरंज खेलने तक
सीमित थीं। वह टिक-टै क-टो के एक गेम को जीतने में

97
सक्षम नहीं होता – या यह भी पता नहीं होता कि कैसे • उदारहण: बेंगलु रू स्थिति लिनकोड प्रयोगशाला स्‍मार्ट
खेलना है। फैक्ट्रियों के सृजन में सहायता कर रही है।

• सामान्‍य एआई एक ऐसे एआई को परिभाषित करती है


जिसका उपयोग पर्यावरण की व्‍यापक रेंज में कार्यों की
भोजन और कृषि (Food and Farming)
विशाल रेंज को पूरा करने के लिए किया जा सकता है।
इस प्रकार यह मानव बुद्धिमता के अत्‍यधिक निकट है। • एआई धारणीय कृषि व्‍यवसायों को अपनाने में सहायता
कर सकता है; यह उर्वरकों का उपयुक्‍त उपयोग सुझा
सकता है, रोबोट कीट हटाने में सहायता कर सकते हैं,
एआई की विभिन्‍न अनुप्रयोग (Various जिससे कीटनाशकों का कम उपयोग होगा।
Applications of AI)
• उदाहरण: भारतीय उद्योग फसल किसानों को वास्‍तविक
स्‍वास्‍थ्‍य (Health) समय डाटा, सटीक संसाधन अनुमान जैसे छिड़काव,
सिं चाई और अन्‍य बचाव उपाय प्रदान कर रहा है।
• स्‍वास्‍थ्‍य संबंधी एआई अनुप्रयोगों का प्रमुख उद्देश्‍य बचाव
या उपचार तकनीकों तथा मरीज के परिणाम के बीच
संबंधों का विश्‍ले षण करना है। एआई प्रोग्राम का निदान
ई-कॉमर्स प्रबंधन (E-commerce Management)
प्रक्रियाओ ं, उपचार प्रोटोकॉल विकास, औषधि विकास,
व्‍यक्तिगत औषधि तथा मरीज के अनुवीक्षण और देखभाल • उद्योग में इनवेंटरी नियोजन, संभार-तंत्र, उत्‍पाद
में उपयोग किया जाता है। इष्‍टतमीकरण आदि में उपयोग किया जा रहा है। इसका

• उदाहरण के लिए: पाथ एआई रोग विज्ञानियों को अधिक उपयोग लोगों को वैयक्तिक सिफारिशें प्रदान करने में

सटीक निदान करने में सहायता करने के लिए मशीन किया जा रहा है और उन्‍हें उनकी जरूरत के अनुसार

अध्‍ययन प्रौद्योगिकी का विकास कर रहा है। बोय हैल्‍थ उत्‍पाद सुझा रहा है।

एक एआई – आधारित लक्षण और उपचार चैकर है। यह • उदाहरण: भारतीय उद्योग ब्‍लू बर्च उन उत्‍पादों का निर्माण
बीमारी के निदान और इलाज के लिए अल्‍गोरिथम का करता है जो कि ग्राहकों द्वारा लौटा दिए जाते हैं और इन
उपयोग करता है। उत्‍पादों को पुन: बेचने पर ध्‍यान केन्द्रित करता है।

परिवहन (Transport) अनुवादक (Translators)

• एआई यातायात को अधिक कुशल बना सकता है, • भाषा अनुवाद सॉफ्टवेयर बहुत हद तक कृत्रिम
यातायात भीड़-भाड़ को कम कर सकता है, चालक का बुद्धिमता की क्षमताओ ं पर आ‍धारित होते हैं, स्‍वचालित
समय बचा सकता है, पार्किंग अधिक आसान बना सकता सबटाइटलिंग जैसे कार्य जिन्‍हें हम अक्‍सर यू ट्यूब के
है, तथा कार और राइड-शेयरिं ग को प्रोत्‍साहित कर सकता वीडियो में देखते हैं, वे भी एआई का उपयोग करते हैं।
है। चूंकि एआई सड़क यातायात को सुगम बनाए रखता है,
यह ईंधन खपत भी कम कर सकता है जो कि वाहनों के
चालू खड़े रहने से होता है और वायु गुणवत्‍ता तथा शहरी साइबर सुरक्षा (Cybersecurity)
नियोजन को सुधार सकता है।
• एआई साइबर हमलों की पहचान करने, उनका पता
लगाने और उनसे लड़ने में सहायता कर सकता है।
उदाहरण के लिए लॉगरिथम (अमरीकी सुरक्षा बुद्धिमता
निर्माण (Manufacturing)
कम्‍पनी) साइबर सुरक्षा का पता लगाने और उत्‍तर देने के
• एआई निर्माण, गुणवत्‍ता नियंत्रण कर सकता है, डिजाइन लिए कम्‍पनियों और संगठनों के लिए प्रारंभ से अंत तक
समय कम कर सकता है, सामग्री की बर्बादी कम कर हल प्रदान करती है।
सकता है, उत्‍पादन पुनरूपयोग में सुधार कर सकता
है, और भविष्‍यसूचक अनुरक्षण कर सकता है। एआई
अल्‍गोरिथम का उपयोग सप्‍लाई चेन के निर्माण का भारत और एआई (India and AI)
इष्‍टतमीकरण करने, कम्‍पनियों को बाजार बदलावों
का पूर्वानुमान लगाने में सहायता करने में भी किया जा • नीति आयोग ने एक त्रिआयामी दृष्टिकोण अपनाया
सकता है। रोबोट स्‍मार्ट फैक्ट्रियों में निर्माण करने, विक्रय है – अन्‍वेषणात्‍मक संकल्‍पना प्रमाण का जिम्‍मा ले ते
पथ के इष्‍टतमीकरण में या अनुरक्षण और ब्रेकडाऊन का हुए जिसका एआई विभिन्‍न क्षेत्रों में अनुमान लगाता है,
अनुमान लगाने में सहायता कर रहे हैं। भारत में एक गुंजायमान एआई पारिस्थितिकी के निर्माण

98

@ITAKASHSINGH
के लिए एक राष्‍ट्रीय रणनीति का निर्माण और विभिन्‍न ख. भारत में कृत्रिम बुद्धिमता के लिए केन्‍द्र – बिं दु क्षेत्र
विशेषज्ञों और हितधारकों के साथ सहयोग करना। नीति (Focus Area for Artificial Intelligence in India)
आयोग ने महत्‍वपूर्ण क्षेत्रों जैसे कि कृषि और स्‍वास्‍थ्‍य
स्‍वास्थ्‍य देखभाल (Healthcare)
में एआई परियोजनाओ ं के कार्यान्‍वयन के लिए अनेक
अग्रणी एआई प्रौद्योगिकी प्‍ले यर्स के साथ भगीदारी की है। • स्‍वास्‍थ्‍य देखभाल, भारत में सर्वाधिक क्रियाशील, फिर
भी चुनौ‍तीपूर्ण क्षेत्रों में से एक है, और इसके वर्तमान
100 बिलियन अमरीकी डॉलर से 16% के ऊपरगामी
रिपोर्ट में विशेष बल दिए गए बिं दु हैं: सीएजीआर से 2020 तक 280 बिलियन अमरीकी
डॉलर तक बढ़ने का अनुमान है। फिर भी इसे जनंसख्‍या
क. कृत्रिम बुद्धिमता और भारत (Artificial Intelligence
and India) के विशाल भाग के लिए गुणवत्‍ता, अभिगम्‍यता, और
वहनीयता की महत्‍वपूर्ण चुनौतियों का सामना करना
एआई रणनीति को एक ऐसे ढ़ांचे पर प्रारंभ करने की आव- पड़ता है:
श्‍यकता है जोकि भारत की बेजोड़ आवश्‍यकताओ ं और अभि-
क. योग्‍य स्‍वास्‍थ्‍य देखभाल व्‍यावसायियों और योग्‍य
लाषाओ ं के अनुकूल है; ऐसे ढ़ांचा निम्‍नलिखित तीन भिन्‍न,
डॉक्‍टरों, नर्सों, तकनीशियनों, और आधारभूत ढ़ांचे
फिर भी अंतर्संबद्ध घटकों का संचयन हो सकता है:
जैसी सेवाओ ं की कमी। देशभर में स्‍वास्‍थ्‍य देखभाल
तक अनियमित अभिगम्‍यता जिसमें शारीरिक
क. अवसर: भारत में कृत्रिम बुद्धिमता का उपयोग करने वाले
अभिगम्‍यता, बचाव और रोग निवारक स्‍वास्‍थ्‍य
आर्थि क प्रभाव के अनुमान के अनुसार भारत की वार्षि क
सेवाओ ं दोनों के लिए प्राथमिक बाधा बनी रहती है
वृद्धि दर, 2035 तक 1.3 प्रतिशत बढ़ जाएगी।
और ग्रामीण तथा शहरी भारत के बीच स्‍पष्‍ट अंतर
ख. अल्‍फोर ग्रेटर गुड: सामाजिक विकास और सम्मिलित बना रहता है।
वृद्धि: गुणवत्‍तापूर्ण स्‍वास्‍थ्‍य सुविधाओ ं तक वर्धित पहच
ुं
ख. वहनीयता अभी भी निजी व्‍यय के साथ एक समस्‍या
(स्थानीय पहुंच रूकावटों का विवरण शामिल), विशाल
है जो कि स्‍वास्‍थ्‍य देखभाल व्‍यय का 70% है जिसमें में
जनसंख्‍या भागों के लिए सम्मिलित वित्‍तीय विकास। इसे
62% खर्च की गई वास्‍तविक नकद राशि है, संभवत:
अब तक औपचारिक वित्‍तीय उत्‍पादों से बाहर रखा गया है।
विश्‍व में सर्वाधिक में से एक। ग्रामीण (~47%) और
किसानों को वास्‍तविक समय परामर्श प्रदान करना और शहरी भारत (~31%) दोनों में अस्‍पताल लागत के
उत्‍पादकता में वृद्धि करने की ओर अनदेखे कारकों का महत्‍वपूर्ण भाग को ऋण और सम्‍पत्तियों की बिक्री से
विवरण देने में सहायता, एक तेजी से शहरीकृत होती वित्‍तपोषण मिलता है।
आबादी की मांगों को पूरा करने के लिए स्‍मार्ट और
ग. मुख्‍य रूप से जागरूकता, सेवाओ ं तक पहुंच और
कुशल शहर और आधार भूत ढ़ांचे का निर्माण करना
व्‍यावहारिक कारकों की कमी के कारण अनिवार्य
इसके उदाहरण हैं। इसे गैर-वेतन-वृद्धि लाभों के माध्‍यम
स्‍वास्‍थ्‍य देखभाल तक प्रतिक्रियाशील दृष्टिकोण
से सर्वाधिक कुशलता से हल किया जा सकता है जैसा कि
का अर्थ है कि अधिकांश मरीज किसी अस्‍पताल में
एआई जैसी प्रौद्योगिकी प्रदान कर सकती है।
/ चिकित्‍सक के पास केवल तब जाते हैं जब कोई
ग. विश्‍व के 40% हिस्से के लिए एआई गैराज: बीमारी बहुत बढ़ जाती है यानि अपने उन्‍नत चरण
दुनिया के 40% के लिए एआई गैरज
े : सीधे शब्दों में कहें, पर पहुंच जाती है।
तो भारत के सन्दर्भ में इसे हल करने का अर्थ दुनिया के • प्रौद्योगिकी में वर्धि त प्रगतियां और नवप्रवर्तकों द्वारा रूचि
40% या उससे अधिक के लिए समाधान प्रस्तुत करना और गतिविधि भारत की जनसंख्‍या के एक विशाल वर्ग
है।। क्षयरोग के जल्‍दी निदान के लिए एक उन्‍नत एआई को उपयुक्‍त स्‍वास्‍थ्‍य देखभाल प्रदान करने में लं बे समय
आधारित हल (विश्‍वभर में मौतों के लिए शीर्ष – 10 से चली आ रही अपनी चुनौतियों को सुलझाने में भारत
कारणों में से एक), उदाहरण के लिए, एक बार भारत में के अवसर प्रदान करते हैं। रोबोटिक्‍स तथा चिकित्‍सा
विकसित और परिष्‍कृ त होने के बाद आसानी से दक्षिण सामग्रियों का इं टरनेट (आईओएमटी) के साथ मिलकर
पूर्वी एशिया या अफ्रीका के देशों में भेजा जा सकता है। एआई स्‍वास्‍थ्‍य देखभाल के लिए संभवत: नया तंत्रिका-
स्‍वास्‍थ्‍य सेवा से परे अन्‍य क्षेत्रों जिनमें कृषि, शिक्षा और तंत्र हो सकता है, जो स्‍वास्‍थ्‍य देखभाल समस्‍याओ ं पर
चालकता शामिल हैं में प्रौद्योगिकियां विश्‍व को बदलने ध्‍यान देने के लिए समाधान प्रस्‍तुत करता है और उद्देश्‍यों
के लिए तैयार हैं। विकसशील देशों में उपर्युक्‍त क्षेत्रों के को पूरा करने में सरकार की सहायता करता है।
बारे में मुद्दों की समानता बहुल बाजारों के लिए अपनाए
गए एआई समाधानों के विकास की आदर्श उपयोग
स्थिति प्रदान करती है।
कृषि (Agriculture)

• कृषि और संबद्ध क्षेत्र अभी भी भारत के कार्यबल का

99
49%, देश के सकल घरेलू उत्‍पाद (जीडीपी) 13 का 16% कारण आईसीटी आधारभूत ढ़ांचे के शिक्षक प्रशिक्षण
होता है और लगभग 1.3 बिलियन लोगों को खाद्य सुरक्षा प्रावधान में कमी है। जबकि सर्वेक्षण किए गए 83%
सुनिश्चित करता है। भारत सरकार ने हाल ही में एक शिक्षक कम्‍प्‍यूटर का उपयोग करते हैं, उपयोग मुख्‍य रूप
राष्‍ट्रीय एजेन्‍डा के तौर पर किसानों की आय को दोगुना से श्रृव्‍य / दृश्‍य डिस्‍प्‍ले , या विद्यार्थी अभ्‍यास तक सीमित है।
करने को प्राथमिकता दी है; उत्‍पादकता संवर्धन के
• एआई अध्‍ययन के संपूरण द्वारा और हितधारकों और
अतिरिक्‍त कृषि और बाजार विकास में आपूर्ति श्रृंखला
प्रशा‍सनिक स्‍तरों पर निर्णय ले ने की सूचना देने और
संदर्भों पर यथेष्‍ठ ध्‍यान दिया है।
सहायता करने के लिए प्रणालियां स्‍थापित करके इस
• भारत कुल मिलाकर संसाधन प्रगाढ़ कृषि अभ्‍यासों पर क्षेत्र में बदलाव लाता है
अपनी शोषक निर्भरता को हटाने में समर्थ नहीं हो सका
• विश्‍व के अन्‍य भागों में अनेक एआई साधनों का
है। भूमि का अपकर्षण, मिट्टी की घटती उर्वरकता, अधिक
सफलतापूर्वक उपयोग किया जा रहा है और विशिष्‍ट
उत्‍पादन के लिए अजैवी उर्वरकों पर बढ़ती निर्भरता, तेजी
चुनौती को लक्ष्‍य बनाने के लिए भारतीय संदर्भ में
से गिरता जल स्‍तर और उभरते कीटनाशक भारत के
अपनाया जा सकता है।
असंधारणीय कृषि अभ्‍यासों की अनेक अभिव्‍यक्तियों में
से कुछ हैं।

• ए आई का उपयोग ऊपर उल्लिखित समस्याओ ं को स्‍मार्ट चालकता और परिवहनीयता (Smart Mobility and
दूर कर सकता है और संसाधनों के कुशल उपयोग में Transportation)
सहायता कर सकता है।
• चालकता और परिवहनीयता अन्‍य क्षेत्रों के साथ अपने
संपर्कों और घरेलू तथा अंतर्राष्‍ट्रीय व्‍यापार में महत्‍व के
शिक्षा (Education) कारण आधुनिक अर्थव्‍यवस्‍था का आधार स्‍तंभ बनते
हैं। आज का समाज लोगों और वस्‍तुओ ं की कुशल और
• उभरते देशों में, विशेषकर, शिक्षा और जनसंख्‍या की सुरक्षित परिवहनीयता सक्षम बनाने के लिए विभिन्‍न
साक्षरता के स्‍तर, एक उन्‍नत अ‍र्थव्‍यवस्‍था के विकास प्रकार की उच्‍च डिग्री की चालकता की मांग करता है।
और उसकी तरफ सम्‍पूर्ण परिवर्तन में एक अनिवार्य
क. स्‍वायत्‍त परिचालन: परिचालन की स्‍वायत्‍त
भूमिका निभाते हैं। अनुमान दर्शाते हैं कि वर्तमान में देश
प्रौद्योगिकी में उस तरीके से बदलने की क्षमता है
की आधी से ज्‍यादा जनसंख्‍या 25 वर्ष की आयु से नीचे है।
जिससे हम आज वस्‍तुओ ं को इधर-उधर ले जाते हैं।
• भारत में विद्यालयी शिक्षा में केन्‍द्रीय और राज्‍य स्‍तरों एआई प्‍लाटू निं ग के माध्‍यम से सुरक्षा में वृद्धि करने
दोनों पर प्रयासों के साथ हाल के दशकों में महत्‍वपूर्ण और क्षमता में सुधार करने में सहायता कर सकता है।
प्रगति हुई है और नामांकन में महत्‍वपूर्ण लाभ प्राप्‍त किए
ख. आधुनिक परिवहन प्रणालियां: एक आधुनिक
गए हैं - ताजा आकड़ों के अनुसार प्रारंभिक स्‍तर पर
यातायात प्रबंधन प्रणाली जिसमें सेन्‍सर, सीसीटीवी
सकल नामांकन अनुपात (जीईआर) 97% है और गौण
कैमरे, ऑटोमैटिक नम्‍बर प्‍लेट संसूचन कैमरे
स्‍तर पर 80% है। हालांकि, निम्‍न अवधारण दरें और खराब
सिग्‍नल युक्‍त पैदल पारपथ और स्‍टॉप लाईन
अध्‍ययन परिणाम नामांकन में लाभ के प्रभाव को बिगाड़
उल्‍लं घन संसूचन प्रणालियां शामिल हैं के उपयोग
देते हैं।
द्वारा और एआई के उपयोग द्वारा यातायात आवागमन
• निम्‍न अवधारण दरें: बच्‍चों का नामांकन बहुत कम जैसे कि ले न मॉनीटरिं ग, निकास तक पहुंच, टॉल
उपयोगी है, यदि बच्‍चों को विद्यालयी प्रणाली में अवधारित की लागत, सार्वजनिक यातायात वाहनों को रास्‍ता
नहीं किया जाता। प्रारंभिक स्‍तर पर 70.7% की अवधारण देना, स्‍मार्ट टिकेटिंग के माध्‍यम से यातायात कानूनों
दर दर्शाती है कि एक-तिहाई नामांकित बच्‍चे कक्षा 8 पूरी को लागू करना आदि बनाए जा सकते हैं। एआई के
करने से पहले स्‍कू ल छोड़ देते हैं। साथ संभावित दुर्घटनाओ ं को रोकने के लिए पहले

• खराब अध्‍ययन परिणाम: स्‍कू लों में बच्‍चों के कम से ही उपयुक्‍त उपाय किए जा सकते हैं।

अध्‍ययन स्‍तरों के बारे में चिं ता बढ़ रही है और एक नया ग. यात्रा मार्ग / आवागमन ईष्‍टतमीकरण: नेटवर्क
राष्‍ट्रीय उपलब्धि सर्वेक्षण (एनएएस) दिखाता है कि कक्षा स्‍तर पर ट्रैफिक डाटा तक अधिगम के लिए, एआई
5 के 60% से अधिक विद्यार्थि यों ने सभी विषयों में कुल कुल यात्रा समय का ईष्‍टतमीकरण करके जिसमें
मिलाकर 50% से कम अंक प्राप्‍त किए; और जांच किए अधिगम समय, प्रतीक्षा समय और यात्रा समय
गए अधिकांश 31 राज्‍यों / संघ शासित प्रदेशों का प्रदर्शन शामिल है, सार्वजनिक यातायात यात्राओ ं के लिए
महत्‍वपूर्ण रूप से कम हो गया। स्‍मार्ट अनुमान लगा सकता है।

• एक ताजा सर्वेक्षण में पाया गया कि स्‍कू लों में प्रौद्योगिकी घ. रेलवे के लिए एआई: आधिकारिक आकड़ों के
को अपनाने का स्‍तर पिछड़ रहा है और इसका मुख्‍य अनुसार, 2012-2017 के बीच 500 से अधिक रेल

100
दुर्घटनाएं हुईं, 53% पटरी से उतरने के कारण हुईं। क. परिचालन योग्‍य बनाने वाली डाटा पारिस्थितिकियों
ट्रे न चालक वास्‍तविक समय परिचालन डाटा के की कमी
माध्‍यम से स्थितिजन्‍य जानकारी हासिल कर सकते
ख. एआई अनुसंधान की कम तीव्रता
हैं और तीन आयामों में उनका विश्‍ले षण कर सकते
हैं: स्‍थानिक, अस्‍थायी और नोडल। i. मूलभूत प्रौद्योगिकियों में महत्‍वपूर्ण अनुसंधान

हाल ही में, रेल मंत्रालय, भारत सरकार ने सिग्‍नलों ii. महत्‍वपूर्ण अनुसंधान को बाजार अनुप्रयोग में
की मॉनीटरिं ग के लिए गैर-अंतर्वेधी सेंसरों, परिवर्ति त करना
ट्रै क सर्किटों, एक्‍सल काऊंटरों और सबसिस्‍टम घ. एआई विशेषज्ञता, मानव संसाधन और कौशल
इं टरलॉकिंग, वोल्‍टेज तथा करंट ले वल, रिले , टाइमर अवसरों की अपर्याप्‍त उपलब्‍धता।
युक्‍त पावर सप्‍लाई प्रणालियों के उपयोग द्वारा दूरस्‍थ
च. व्‍यवसाय प्रक्रियाओ ं में एआई को अपनाने के लिए
परिस्थिति का जिम्‍मा ले ने के लिए एआई का उपयोग
उच्‍च संसाधन लागत और कम जागरूकता
करने का निर्णय लिया है।
छ. अस्‍पष्‍ट निजता, सुरक्षा और नैतिक विनियमन
ङ. समुदाय आधारित पार्किंग: पार्किंग की उपलब्‍धता
भारतीय शहरों के लिए एक महत्‍वपूर्ण मुद्दा है। एआई ज. एआई के अनुसंधान और अपनत्‍व को उत्‍प्रेरित करने
वाहन डाऊनटाइम को न्‍यूनतम करके ड्राइविं ग के लिए अनाकर्षक बौद्धिक सम्‍पदा सामाजिक
टाइम को अधिकतम करके संभवत: पार्किंग के व्‍यवस्‍था
इष्‍टतमीकरण में सहायता कर सकता है। इले क्ट्रिक • जबकि किसी भी साधन से सर्वांगीण इन चुनौतियों
वाहनों के विकास के साथ कॉम्‍प्‍ले क्‍स व्‍हीकल ग्रिड की यदि संगत हितधारकों द्वारा आयोजित सम्मिलित
इं टरेक्‍शन (वीजीआई) और चार्जिं ग इष्‍टतमीकरण में प्रयासों के माध्‍यम से तेजी से जानकारी दी जाती है,
मध्‍यस्‍थता के लिए एआई की आवश्‍यकता होगी। जिसमें सरकार एक अग्रणी भूमिका निभाती है, के
परिणामस्‍वरूप मूलभूत निर्माण ब्‍लॉक एआई में नेतृत्‍व
की ओर भारत की प्रगति का केन्‍द्र बनते हैं।
भारत में एआई को अपनाने में महत्‍वपूर्ण चुनौतियां (Critical
Challenges to Adoption of AI in India)
अनुशंसा (Recommendations):
• ध्‍यान देने वाले क्षेत्रों का प्रारंभिक विश्‍ले षण – स्‍वास्‍थ्‍य
देखभाल, कृषि, शिक्षा, स्‍मार्ट शहर और आधारभूत ढ़ांचा अनुसंधान (Research)
और स्‍मार्ट चालकता और परिवहन, भारतीय अर्थव्‍यवस्‍था
• भारत ने 2016 में अत्‍यधिक 2.6 मिलियन एसटीईएम
के क्षेत्रों और स्थिति को परिवर्ति त करने में एआई साधनों
स्‍नातक तैयार किए, जो कि चीन के बाद दूसरा स्‍थान है
और प्रौद्योगिकीयों की क्षमता पर प्रकाश डालते हैं।
और अमरीका के स्‍नातकों से 4 गुना है, और इस प्रकार
• स्‍वास्‍थ्‍य देखभाल क्षेत्र को एक उदाहरण के रूप में ले कर उभरती प्रौद्योगिकियों में नवोन्‍मेष लाने के लिए वांछित
विशाल स्‍तर पर अधिग्रहण को सक्षम बनाने में कम टै लेंट पूल उत्‍पन्‍न किया। हालांकि निराशाजनक बात
से कम निम्‍नलिखित कारकों की जानकारी देने की यह है कि उस टै लेंट पूल का अत्‍यधिक बहुमत नियमित
आवश्‍यकता होगी: आईटी विकास पर केन्द्रित है और अनुसंधान तथा
क. विभिन्‍न हितधारकों के बीच सहयोगात्‍मक प्रयास नवोन्‍मेष पर अधिक नहीं।
की अनुपस्थिति: जबकि भारत ने इले क्‍ट्रॉनिक • अंतर्मंत्रालयी राष्‍ट्रीय अंतर्वि षयी साइबर भौतिकी प्रणाली
स्‍वास्‍थ्‍य रिकॉर्ड (इएचआर) नीति अपना ली है, मिशन (आईएम-आईसीपीएस) ने प्रौद्योगिकी जीवन चक्र
विभिन्‍न अस्‍पताल श्रृंखला के बीच डाटा साझा करना के सभी पहलु ओ ं पर केन्द्रित अनुसंधान को बढ़ावा देने
अभी भी कार्य प्रगति पर है बना हुआ है, चूंकि विभिन्‍न के लिए निम्‍नलिखित चार-स्‍तरीय ढ़ांचे का प्रस्‍ताव दिया
अस्‍पताल श्रृंखलाओ ं ने ‘डिजिटाइजिं ग रिकॉर्ड' की है: अनुसंधान, प्रौद्योगिकी परिनियोजन, अनुवाद और
विभिन्‍न व्‍याख्‍याओ ं को अपनाया है। प्रबंधन:
ख. संगत डाटा अनुपलब्‍ध है और स्पष्ट खुले क्लिनिकल क. आईसीओएन (अंतर्राष्‍ट्रीय नवज्ञान केन्‍द्र): प्रारंभिक
डाटा सेट का अभाव है; और अनुसंधान के माध्‍यम से नए ज्ञान के सृजन पर
ग. निजता और डाटा की सुरक्षा की चिं ताएं । ध्‍यान केन्द्रित करना।

• इसी प्रकार प्रत्‍येक क्षेत्र की अपनी स्‍वयं की समस्‍याएं हैं। ख. सीआरओएसएस (उप-प्रणाली अनुसंधान केन्‍द्र):
हालांकि, ध्‍याने देने वाले क्षेत्रों के विश्‍ले षण पर चुनौतियां आईसीओएन स्‍तर और किसी अन्‍य स्रोत पर
निम्‍नलिखित के सामान्‍य विषयों के आसपास केन्द्रित है: विकसित कोर प्रौद्योगिकियों के विकास और
एकीकरण पर ध्‍यान और

101
ग. सीएएसटीएलई (उन्‍नत अध्‍ययन, स्‍थानांतरीय पहलों के साथ एकीकरण, नए प्‍लेटफार्मों का निर्माण जो
अनुसंधान तथा नेतृत्‍व केन्‍द्र): अनुप्रयोग आधा‍रित उन्‍नत अध्‍ययन को संभव बना सकती है, और एआई के
अनुसंधान के विकास और नियोजन पर ध्‍यान और प्रसार के माध्‍यम से विशाल स्‍तर पर रोजगार सृजन को

घ. सीईटीआईटी (उत्‍कृ ष्‍ट प्रोद्योगिकी, नवोन्‍मेष और स्‍वीकृति देने वाली नॉवेल विधियों की आवश्‍यकता होगी।

स्‍थानांतरण केन्‍द्र): विकसित प्रौद्योगिकियों के क. नौकरियों के सृजन को प्रोत्‍साहन देना जो नए


व्‍यावसायीकरण पर ध्‍यान सेवा उद्योग में शामिल हो सकता है: सेवा उद्योग

• अनुसंधान को बढ़ावा देने के लिए एक निर्बाध, लक्षित में स्‍थानांतरणों की चुनौतियों से निपटने के लिए

और जवाबदेह ढ़ांचा सुनिश्चित करने के लिए एक नौकरियों के सृजन की पहचान करना और बढ़ावा

अधिक सरलीकृत और दक्ष दृष्टिकोण की आवश्‍यकता देना अनिवार्य है जो कि भविष्‍य में परंपरागत आईटी

है। अत: एआई में कोर और अनुप्रयुक्‍त अनुसंधान दोनों – बीपीएम क्षेत्र की नौकरियों का स्‍थान ले सकता है।

को प्रोत्‍साहित करने के लिए निम्‍नलिखित दो-स्‍तरीय ख. अनौपचारिक प्रशिक्षण संस्‍थानों की पहचान और


एकीकृत दृष्टिकोण का प्रस्‍ताव है। मानकीकरण: उच्‍चतर शिक्षा संस्‍थानों के माध्‍यम से

क. सीओआरई (कृत्रिम बुद्धिमता उत्‍कृ ष्‍ट अनुसंधान मान्‍यता प्राप्‍त प्रमाणपत्र पाठ्यक्रमों का कार्यान्‍वयन

केन्‍द्र): सीओआरई, एआई के कोर अनुसंधान पर पुन: कौशल पर खर्च किए गए पहचाने गए संसाधनों

ध्‍यान केन्द्रित करेंगे और आईएम-आईसीपीएस ढ़ांचे के लिए एक महत्‍वपूर्ण बढ़ावा हो सकता है और ज्ञान

के आईसीओएन और सीआरओएसएस, दोनों की की डिलीवरी में इन संस्‍थानों को मानकों पर रोक

जिम्‍मेदारियों के निष्‍पादन को आच्‍छादित करेंगे। इस कर रखना।

प्रकार, सीओआरई मौलिक अनुसंधान के माध्‍यम ग. अध्‍ययन के लिए ओपन प्‍लेटफार्मों का सृजन:
से नए ज्ञान का सृजन करने में विशेषज्ञता हासिल नैसकॉम की भावी कौशल प्‍लेटफार्म जैसी पहलें
करेंगे और मूलभूत ज्ञान / प्रौद्योगिकियों का एक स्रोत नियोजित कार्यबल के कुछ महत्‍वपूर्ण भागों के
होंगे। यह भारत को अगली पीढ़ी की प्रौद्योगिकियों के अपेक्षित कौशल के विशाल स्‍तर पर प्रचार-प्रसार
लिए तैयार रखेगा। में महत्‍वपूर्ण भूमिका अदा करेंगी। ऑन-लाईन

ख. आईसीटी एआई (अंतर्राष्‍ट्रीय परिवर्तनीय कृत्रिम और स्‍व-अध्‍ययन प्‍लेटफार्म, जैसे कि कोरसेरा और

बुद्धिमता केन्‍द्र): आईसीटीएआई अनुप्रयोग ईडीएक्‍स विश्‍वभर में श्रेष्‍ठ विश्‍वविद्यालयों और

आधारित प्रौद्योगिकी विकास और नियोजन के संस्‍थानों तक पहुंचने में प्रशिक्षुओ ं की सहायता कर

लिए पारिस्थितिकी प्रदान करेंगे और आई एम – सकता है।

आईसीपीएस ढ़ांचे के अनुसार सीएएसटीएलई और घ. कर्मचारियों के पुन: कौशल के लिए वित्‍तीय


सीईटीआईटी की जिम्‍मेदारियों के निष्‍पादन को प्रोत्‍साहनों का सृजन करना: कर्मचारियों के पुन:
आच्‍छादित करने का काम करेगा। यह एक उद्योग कौशल या कर्मचारियों को पुन: कौशल पहलें
आधारित पहल होगी और पहचानी गई शीर्ष-स्‍तर प्राप्‍त करने की स्‍वीकृति देने की पहलों में निजी
की चुनौतियों या एआई आधारित समाधान वाली कम्‍पनियों की उच्‍च अवसर लागत होती है और
अंतर्मंत्रालयी परियोजनाओ ं का जिम्‍मा ले ने की उनकी स्‍वीकृति को प्रभावित कर सकती हैं। जिससे
आशा है। वे अपने कर्मचारियों को प्रक्रिया को एक स्‍तर तक
नियोजित करने देते हैं। इसलिए यह सुझाव दिया
जाता है कि सरकार और कम्‍पनियों के बीच के सह-
कौशल (Skilling) निधि वाले मॉडलों की खोज की जाए, विशेषकर
आईटी क्षेत्र में। निजी कम्‍पनियों के लिए वित्‍तीय
• नैसकॉम के अनुमान से 2022 तक एक चौंकानेवाला प्रोत्‍साहनों में पेरोल कर शामिल हो सकते हैं जो कि
46% भारतीय कार्यबल बिलकुल नई नौकरियों में प्रशिक्षण अवसरों की सहायता करने के प्रति समर्पि त
नियोजित होगा जो कि आज मौजूद नहीं हैं या वे नौकरियां हैं।
जिन्‍होंने मूलत: कौशल सेट बदल दिए हैं। कुछ अन्‍य स्रोत
• पिछले वर्ष कुंभ मेले के दौरान कृत्रिम बुद्धिमता विशेषकर
एआई की मांग का अनुमान लगाते हैं और 2021 तक
नोट करने योग्‍य थी, जहां प्रयागराज में सफल यातायात
भारत में मशीन द्वारा सीखने वाले विशेषज्ञों की संख्‍या
संचलन, सुरक्षा और भीड़ प्रबंधन के लिए स्‍मार्ट वीडियो
में 60% वृद्धि होने की आशा है। एक स्‍वतंत्र अध्‍ययन के
विश्‍ले षण का उपयोग किया गया था। उड़ीसा सरकार भी
अनुमान के अनुसार 2025 तक भारत को 2,00,000
अपराध डाटा के विश्‍ले षण और जांच में सहायता करने
डाटा विश्‍ले षक व्‍यावसायियों के मांग – आपूर्ति अंतर का
के लिए कृत्रिम बुद्धिमता, डाटा विश्‍ले षण, और मोबाईल
सामना करना पड़ेगा।
परिकलन का उपयोग करने की तैयारी कर रही है।
• वर्तमान कार्यबल के पुन: कौशल में संगत मौजूदा कौशल

102
कृत्रिम बुद्धिमता और निजता (Artificial Intelligence हालांकि भारत जैसे विशाल देश के लिए योजना, नीति
and Privacy): निर्माण, या देश में विभिन्‍न क्षेत्रों जैसे कृषि और शहरी
नियोजन के साथ डाटा विश्‍ले षण का सहायता के लिए
• एआई मानवता की प्रौद्योगिकिय उत्‍पत्ति में एक प्रमुख पूरी तरह से डाटा विश्‍ले षण का उपयोग नहीं किया गया
कारक होगा, जहां निर्णय ले ने के लिए मशीनों और है।
अलगोरिथम पर मानव निर्भरता पहले कभी इतनी गंभीर
• परियोजना अंतर्दृष्टि के एक भाग के रूप में डाटा माइनिं क
नहीं रही। अत: एआई को बढ़ावा देने में किसी रणनीति
तकनीकों का उपयोग, टै क्‍स चोरी की पहचान करना
दस्‍तावेज को एआई पारिस्थितिकी के संभावित कारकों
और शैल कम्‍पनियों पर शून्‍य –इन करने के लिए जिओ-
के बारे में आवश्‍यक रूप से सचेत रहने की आवश्‍यकता
टै गिंग डाटा के परिभ्रमण की संभावना पर भारत टै प्पिं ग
है जो कि नैतिक आचरण को दुर्बल बना सकता है, किसी
के उत्‍कृ ष्‍ट उदाहरण हैं।
की निजता का अतिक्रमण कर सकता है और सुरक्षा
प्रोटोकाल को दुर्बल बना सकता है। इन खतरों को • एक उदाहरण के तौर पर कृषि का उपयोग करके बिग
समाप्‍त करने के लिए कदमों को किसी भी ऐसी रणनीति डाटा फसल उगाने के पैटर्न के अध्‍ययन, उस श्रेष्‍ठतम
का एक अभिन्‍न भाग होने की आवश्‍यकता है। समय का विश्‍ले षण करने जो कि सर्वोच्‍च फसल उत्पादन
पैटर्न देगा, श्रेष्‍ठ रोपण समय के निर्धारण के लिए मौसम
और ऋतु पैटर्नके अध्‍ययन और अन्‍य में सहायता करेगा।
बिग डाटा (Big Data)

• बिगडाटा का एक समूह है जो कि मात्रा में बेहद विशाल है, वस्‍तुओ ं का इं टरनेट (Internet of Things):
और समय के साथ घातीय रूप से बढ़ रहा है। यह एक इतने
• वस्‍तुओ ं का इं टरनेट, या आईओटी का संबंध विश्‍वभर के
बड़े आकार और जटिलता वाला डाटा है कि परंपरागत
कई बिलियन भौतिक यंत्रों से है जो कि अब इं टरनेट से जुड़े
डाटा प्रबंधन साधनों में से कोई भी इसे सफलतापूर्वक
सभी डाटा एकत्र और साझा कर रहे हैं। इसलिए यदि हम
स्‍टोर या प्रोसेस नहीं कर सकता। बिग डाटा विशाल
भौतिक वस्‍तुओ ं को ले ते हैं और उन्‍हें सॉफ्टवेयरों, सेन्‍सरों,
आकार का डाटा होता है।
या किसी अन्‍य प्रौद्योगिकी की सहायता से जोड़ देते हैं, तब
• डाटा को कम अव्‍यक्‍तता के साथ प्राप्‍त करना, प्रबंधन हम उन्‍हें अपने बीच सम्‍प्रेषण करने में सक्षम बनाएं गे, हम
करना और संसाधित करना पुरानी शैली के परस्‍पर इसे ‘’इं टरनेट ऑफ थिं ग्‍स‘’ के तौर पर परिभाषित करेंगे।
अन्‍योन्‍यक्रिया वाले डाटाबेस की क्षमता से परे है। बिग
• कुछ वस्‍तुएं स्‍वयं भी अनेक छोटे आईओटी घटकों से
डाटा की एक या अधिक निम्‍नलिखित विशेषताएं होती
भरी जा सकती हैं जैसे कि डाटा एकत्र करने वाले हजारों
हैं: उच्‍च मात्रा, उच्‍च गति, उच्‍च विविधता। कृत्रिम बुद्धिमता
सेन्‍सरों से भरा जेट इं जन और यह सुनिश्चित करने के
(एआई) वस्‍तुओ ं का इं टरनेट (आईओटी) डाटा के नए रूपों
लिए कि यह कुशलतापूर्वक कार्य कर रहा है डाटा का
और स्रोतों के माध्‍यम से डाटा जटिलता को आगे बढ़ाती
सम्‍प्रेषण वापस करना।
हैं। उदाहरण के लिए, बिग डाटा सेन्‍सरों, यंत्रो, वीडियो/
आडियो, नेटवर्क , लॉग फाइलों, कार्य संपादन अनुप्रयोगों, • एक लाइट बल्‍ब जिसे एक स्‍मार्टफोन ऐप से स्विच ऑन
वेब और सोशल मीडिया से आता है – इसमें से अधिकांश किया जा सकता है, एक आईओटी यंत्र है।
वा‍स्‍तविक समय में और एक विशाल स्‍तर पर उत्‍पन्‍न
• आईओटी एक उभरते व्‍यवसाय और प्रौद्योगिकी में एक
किया गया है।
अग्रणी भूमिका निभाती है। जून 2020 में जिन्‍नोव द्वारा
• सैद्धांतिक अर्थ में बिग डाटा संरचनाबद्ध, उप संरचनाबद्ध जारी एक ताजा रिपोर्ट के अनुसार भारत में आईओटी
और गैर संरचनाबद्ध डाटा का एक युग्‍म है। संरचनाबद्ध निवेश 2019 में 5 बिलियन अमरीकी डॉलर के करीब
डाटा को एक स्थिर फॉर्मेट के रूप में स्‍टोर, अधिगम और था और 2021 में इसके 15 बिलियन अमरीकी डॉलर तक
संसाधित किया जा सकता है। अनभिज्ञ रूप या संरचना पहुंचने का अनुमान है।
वाले गैर-संरचनाबद्ध डाटा को गैर संरचनाबद्ध डाटा के
रूप में श्रेणीबद्ध किया जाता है। उप संरचना बद्ध डाटा
दोनों प्रकार का डाटा प्राप्‍त कर सकता है। 5G प्रौद्योगिकी का विकास (Evolution of 5G
Technology):

भारत में बिग डाटा (Big Data in India) 5G मोबाइल नेटवर्क की पांचवीं पीढ़ी है जिसे आधुनिक
समाजों की डाटा और कनेक्टिविटी में अत्‍यधिक विकास को
• बिग डाटा का उपयोग, देश में किसी भी तरह नया नहीं पूरा करने के लिए डिजाइन किया गया है। तीव्र संपर्क और
है जिसमें अब कुछ समय से भारत सरकार और निगमों बेहतर क्षमता डिलीवर करने के लिए 5G को प्राथमिकता दी
द्वारा डाटा माइनिं ग तकनीकों का उपयोग किया जाता है। गई है।

103

@ITAKASHSINGH
• 1G: यह वायरले स सेल्‍यूलर प्रौद्योगिकी की प्रथम पीढ़ी और ओएफडीएम (आर्थोगोनल फ्रीक्‍वेंसी डिवीजन
थी, वे एनालॉग प्रौद्योगिकी मानक थे जो कि 1980 में मल्टिप्‍लेक्सिंग)। सर्वाधिक महत्‍वपूर्ण 4G मानक
प्रारंभ किए गए थे और 2G प्रौद्योगिकी द्वारा प्रतिस्‍थापित WiMAX और LTE हैं।
किए जाने तक उपयोग की जाती रही। विश्‍व में प्रथम
• 4GLTE: जबकि 4GLTE 3G स्‍पीड में महत्‍वपूर्ण सुधार हैं,
व्‍यावसायिक 1G मोबाइल नेटवर्क 1979 में टोक्‍यों,
तकनीकी तौर पर यह 4G नहीं है, 4G मोबाइल नेटवर्क
जापान में निप्‍पोन टे लिफोन और टे लिग्राफी कम्‍पनी
कनेक्‍शनों के लिए पूर्वनिर्धारित मानक हैं। 4GLTE
द्वारा लांच किया गया था।
‘’फोर्थ जेनरेशन लॉंग-टर्म इवोल्‍यूशन’’ का संक्षिप्‍त रूप
• 2G: 1G प्रौद्योगिकी एनालॉग संचरण पर आधारित थी है। इसलिए वास्‍तव में यह दो शब्‍दों को जोड़कर बना है।
जबकि 2G प्रौद्योगिकी 2G क्षमताओ ं के बहुविधिकरण के पहला ‘’4G’’ मोबाईल प्रौद्योगिकी की चौथी पीढ़ी को
माध्‍यम से डिजिटल संचरण पर आधारित थी जिसमें बहु दर्शाता है, 3G के बाद अगला बड़ा विकास। और ‘’लॉंग-
उपयोक्‍ताओ ं को एकमात्र चैनल पर आने दिया गया। यह टर्म इवोल्‍यूशन’’ या ‘’एलटीई’’ (LTE) उद्योग जारगून है
एसएमएस और एमएमएस (मल्टिमीडिया मैसेज) को भी जिसका उपयोग विशेष प्रकार के 4G को परिभाषित
सक्षम बनाता है। करने के लिए किया गया है जो कि सबसे तेज मोबाईल
इं टरनेट अनुभव डिलीवर करता है।
• 3G: तीसरी पीढ़ी अपनी प्रमुख नेटवर्क संरचना के तौर
पर यूनिवर्सल मोबाईल टे लिकम्‍यूनिकेशन सिस्‍टम • 5G: 5G मोबाईल कम्‍यूनिकेशन नेटवर्क की पांचवीं
(यूएमटीएस) का उपयोग करती है। यह 14MBPS तक की पीढ़ी है, 5G की सर्वाधिक महत्‍वपूर्ण विशेषता इसकी कम
डाटा स्‍पीड प्राप्‍त करने के लिए सर्किट स्विचिं ग की बजाय ले टे न्‍सी है। ले टे न्‍सी वायरले स नेटवर्क पर एक दूसरे को
पैकेट स्‍विचिं ग का उपयोग करती है। उत्‍तर देने के लिए यंत्रों द्वारा लिया गया समय है, 3G का
प्रतिक्रिया समय 100 मिलीसेकंड है, 4G का प्रतिक्रिया
• 4G: 3G और 4G के बीच मुख्‍य अंतर डाटा रेट है, अनिवार्य
समय 30 मिलीसेकंड है, और 5G ले टे न्‍सी 1 मिलीसेकंड
प्रौद्योगिकियां जिनहोंने 4G को संभव बनाया है, वे हैं
जितनी कम होगी जो 5G को 60-120 गुना तेज बनाती है।
एमआईएमओ (मल्टिपल इनपुट मल्टिपल आऊटपुट)

तुलना 2G 3G 4G 5G

आरंभ होने का वर्ष 1993 2001 2009 2018

प्रौद्योगिकी जीएसएम डब्‍ल्‍यूसीडीएमए एलटीई, WiMAX MIMO, mm तरंगे

अभिगम प्रणाली TDMA,CDMA CDMA CDMA OFDM,BDMA

डाटा के लिए वॉयस


कुछ हस्‍तक्षेप को
स्विचिं ग प्रणाली और पैकेट स्विचिं ग के पैकेट स्विचिं ग पैकेट स्विचिं ग
छोड़कर पैकेट स्विचिं ग
लिए सर्किट स्विचिं ग

इं टरनेट सेवा नैरोबैंड ब्रॉडबैंड अल्‍ट्रा ब्रॉडबैंड वायरलै स वर्ल्ड वाइड वेब

बैंडविड्थ 25MHZ 25MHZ 100MHZ 30GHZ से 300GHZ तक

एसएमएस, एमए-
गति, उच्‍च गति, हैंड अति उच्‍च गति कम
लाभ मएस, इं टरनेट पहुंच इं टरनेशनल रोमिं ग
ऑफ्स ले टे न्‍सी
और सिम आरंभ हुई

उच्च गति अनु- उच्‍च गुणवत्‍ता वाली


वीडियो कॉफ्रेंसिं ग
वॉयस कॉल, शॉर्ट प्रयोग, मोबाइल वीडियो चलना, वाहनों का
अनुप्रयोग मोबाइल टीवी,
मैसेज टीवी, पहनने योग्‍य रिमोट से नियंत्रण, रोबोट
जीपीएस
तंत्र और चिकित्‍सा प्रक्रिया

104
भारत में 5G (5G in India)

• दूरसंचार विभाग (डीओटी) ने अगले 10 वर्षों के लिए रेडियो फ्रीक्‍वेंसी स्‍पेक्‍ट्रम की बिक्री और उपयोग पर संचार कम्‍पनि-
यों और उद्योग के अन्‍य विशेषज्ञों से इनपुट मांगे हैं जिसमें 5G बैंड शामिल हैं।

• 5G या पांचवी पीढ़ी लॉंग-टर्म इवोल्‍यूशन (एलटीई) मोबाईल ब्रॉडबैंड नेटवर्कों में नवीनतम अद्यतन है। 5G मुख्‍यत: 3 बैंड
नामत: निम्‍न, मध्‍यम और उच्‍च फ्रीक्‍वेंसी स्‍पेक्‍ट्रम में कार्य करता है।

• वैश्विक देशों के समतुल्‍य भारत ने 2018 में प्रद्योगिकी की बेहतर नेटवर्क गति और शक्ति का उपयोग करके शीघ्रतीशीघ्र
5G सेवाएं शुरू करने की योजना बनाई।

• सभी तीन निजी टे लिकॉम कम्‍पनियां रिलायंस जियो इं फोकॉम, भारती एयरटे ल और V! स्‍पेक्‍ट्रम आबंटन और 5G फ्री-
क्‍वेंसी बैंड की स्‍पष्‍ट रूपरेखा प्रस्‍तुत करने के लिए दूरसंचार विभाग (डीओटी) से अनुरोध कर रही हैं ताकि वे तदनुसार
अपनी सेवाओ ं को शुरू करने की योजना बना सकें। हालांकि एक बड़ी बाधा, तीन कम्‍पनियों में से कम से कम दो, नामत:
भारती एयरटे ल और वोडाफोन आ‍इडिया के पास नकद आहरण और पर्याप्‍त पूंजी की कमी है।

क्रिप्‍टोकरेंसी और ब्‍लॉकचेन प्रौद्योगिकी का उदय जिनका उपयोग वे एक सुरक्षित डिजिटल पहचान संदर्भ
(Evolution of Cryptocurrencies and उत्‍पन्‍न करने के लिए करते हैं।
Blockchain Technology)
• सुरक्षित पहचान ब्‍लॉकचेन प्रौद्योगिकी का सर्वाधिक
• ब्‍लॉकचेन प्रौद्योगिकी: ब्‍लॉकचेन ले नदेन का एक महत्‍पूर्ण पहलू होता है। क्रिप्‍टोकरेंसी की दुनिया में
डिजिटल खता बही है जिसकी अनुलिपि की जाती है और पहचान को ‘डिजिटल हस्‍ताक्षर’ कहा जाता है और इसका
जिसे कम्‍प्‍यूटर प्रणालियों के सम्‍पूर्ण नेटवर्क में वितरित उपयोग ले न देन को प्राधिकृत और नियंत्रित करने के
किया जाता है। यह एक प्रणा‍ली है जो इस तरह से सूचना लिए किया जाता है।
को रिकॉर्ड करती है जो प्रणाली को बदलने, चोरी करने • डिजिटल हस्‍ताक्षर को समकक्ष-के-समकक्ष नेटवर्क के
या धोखा देना कठिन या असंभव बनाता है। साथ मिला दिया जाता है; अनेक व्‍यक्ति जो प्राधिकारियों
• ब्‍लॉकचेन को समझने का सरल साम्‍यानुमान गूगल के तौर पर कार्य करते हैं अन्‍य मुद्दों के बीच ले न-देन पर
डॉक है। जब हम एक डॉक्‍यूमेंट बनाते हैं और इसे लोगों एक मतैक्‍य तक पहुंचने के लिए डिजिटल हस्‍ताक्षर का
के समूह के साथ साझा करते हैं। यह एक विकेन्द्रिकृत उपयोग करते हैं।
वितरण चेन बनाता है जो एक ही समय पर सभी की • जब वे एक सौदा प्राधिकृत करते हैं, इसे गणितिय
डॉक्‍यूमेंट तक पहुंच बनाता है, डॉक में किए गए सभी प्रमाणीकरण द्वारा प्रमाणित किया जाता है, जिसके
आशोधनों को वास्‍तविक – समय में रिकॉर्ड किया जाता परिणामस्‍वरूप दो नेटवर्क से जुड़ी पार्टि यों के बीच एक
है, तथा बदलावों को पूरी तरह पारदर्शी बनाया जाता है। सफल सुरक्षित ले न देन होता है। ब्‍लॉकचेन उपयोक्‍ता
पीयर-टू -पीयर नेटवर्क पर विभिन्‍न प्रकार के डिजिटल
ले न-देन करने के लिए क्रिप्‍टोग्राफिक कुंजियों को
ब्लॉकचेन कैसे कार्य करता है? (How does Blockchain नियोजित करते हैं।
Work?)
• एक ब्‍लॉकचने में प्रत्‍येक ब्‍लॉक में 4 मुख्‍य हैडर होते हैं।
ब्‍लॉकचेन तीन अग्रणी प्रौद्योगिकियों का एक युग्‍म है:
• पिछला हैश: हैश पता पिछले ब्‍लॉक का पता लगाता है।
1. क्रिप्‍टोग्राफिक कुंजियां। • ले न-देन विवरण: सभी ले न-देन के विवरण होने की
2. एक साझा खाता-बही युक्‍त समकक्ष-के-समकक्ष आवश्‍यकता है।
नेटवर्क । • नोन्‍स: ब्‍लॉक के हैश पते का अंतर बताने के लिए
3. नेटवर्क के कार्य-विवरणों और रिकॉर्डों को स्‍टोर करने क्रिप्‍टोग्राफी में दी गई एक यादृच्छिक संख्‍या।
के लिए परिकलन का एक साधन। • ब्‍लॉक का हैश पता: उपर्युक्‍त सभी (यानि, हैश से पहले ,
• क्रिप्‍टोग्राफिक कुंजियों में दो कुंजियां होती हैं। ले न-देन विवरण, और नोन्‍स) एक हैशिं ग अल्‍गोरिथम के
माध्‍यम से संचारित किए जाते हैं। यह एक आऊटपुट देता
• निजी कुंजी और सार्वजनिक कुंजी।
है जिसमें 256- बिट, 64 कैरेक्‍टर का लं बाई मान होता है
• ये कुंजियां दो पार्टि यों के बीच सफल ले न देन करने में जिसे बेजोड़ ‘हैश पता’ कहा जाता है।
सहायता करती हैं। प्रत्‍येक व्‍यक्ति के पास दो कुंजी होती हैं,

105
माइनिं ग (Mining) के लिए खतरनाक थे। एक विषय के रूप में रोबोटिक्‍स
विज्ञान, इं जीनियरिं ग और प्रौद्योगिकी का प्रतिच्‍छेद है जो
• ब्‍लॉकचेन प्रौद्योगिकी में वर्तमान डिजिटल / सार्वजनिक
मशीनों का उत्‍पादन करते हैं, जिन्‍हें रोबोट कहा जाता है।
खाता – बही में ले न-देन विवरण जोड़ने की प्रक्रिया
‘माइनिं ग’ कहलाती है। हालांकि यह शब्‍द बिटकॉइन से • प्रौद्योगिकी के साथ रोबोटिक्‍स के क्षेत्र का विस्‍तार हुआ
जुड़ा है, इसका उपयोग अन्‍य ब्‍लॉकचेन प्रौद्योगिकियों को है, आज हम रो‍बोटिक्‍स का एक विकसित और विस्‍तारित
भेजने के लिए भी किया जाता है। रूप देख रहे हैं।

• पहले रोबोट का उपयोग मुख्‍य रूप से कार फैक्ट्रियों जैसी


जगहों पर सरल बारंबारी के कार्य करने के लिए किया
क्रिप्‍टोकरेन्‍सी (Cryptocurrencies) जाता था। आज रोबोट का उपयोग पृथ्‍वी की विषमतम
परिस्थितियों की खोज करने में किया जाता है, वे कानून
• क्रिप्‍टोकरेन्‍सी धन का एक डिजिटल प्रारूप है जो एक लागू करने में सहायता कर रहे हैं और वे स्‍वास्‍थ्‍य देखभाल
नई मुद्रा प्रणाली पर चलती है, जो कि केन्‍द्रीकृत एजेन्‍सी के लगभग प्रत्‍येक क्षेत्र में सहायता कर रहे हैं।
द्वारा विनियमित या एक औपचारिक संस्‍थान द्वारा खोजी
नहीं जाती। यहां विभिन्‍न कार्यों वाली अनेक प्रकार
की क्रिप्‍टोकरेन्‍सी हैं। प्रत्‍येक डिजिटल करेन्‍सी एक रोबोटिक्‍स का अनुप्रयोग (Application of Robotics):
विकेन्‍द्रीकृत पीयर-टू -पीयर नेटवर्क द्वारा समर्थि त होती
है जिसे प्रत्‍येक कार्य के अनपेक्ष ब्‍लॉकचेन कहा जाता 1. स्‍वास्‍थ्‍य देखभाल: रोबोट ने पहले से ही विश्‍व भर में
है। ब्‍लॉकचेन प्रौद्योगिकी सुनिश्चित करती है कि सभी अस्‍पतालों में चिकित्‍सा देखभाल के विभिन्‍न क्षेत्रों में काम
क्रिप्‍टोकरेन्सियों से संपर्क रखा जाए, इस बात के अनपेक्ष करना शुरू कर दिया है, विशेषकर शारीरिक हलचलों को
की चाहे उन्‍हें एक डिजिटल वाले ट में रखा गया है। सही करने या सहायता करने के लिए सेन्‍सरी प्रोस्‍थेसिस
जैसी प्रक्रियाओ ं में शरीर के एक विशिष्‍ट भाग को दवाइयां
• बिटकॉइन बाजार में पहला था जिसने एक प्रणाली
डिलीवरी करने के लिए माइक्रो रोबोट और बहुत कुछ।
स्‍थापित की जिसमें लोग – प्रेषक और कॉईन के
प्राप्‍तकर्ता – को एक डिजिटल हस्‍ताक्षर बनाने के लिए उदाहरण के लिए, जयपुर के सवाई मान सिं ह अस्‍पताल
भुगतानों पर हस्‍ताक्षर करने चाहिए। प्रत्‍येक व्‍यक्ति एक ने कोरोना वायरस के मरीजों को भोजन और दवाई
सार्वजनिक और एक निजी कोडीकरण कुंजी होता डिलीवरी करने के लिए स्‍थानीय रूप से निर्मि त मानवीय
है। प्रत्‍येक ले न-देन का परिशुद्धता के लिए सत्‍यापन रोबोट का परीक्षण किया।
किया जाता है और प्रणाली अनाम और परदर्शी होती है। 2. कृषि: सम्‍पूर्ण लागत को कम करने और उत्‍पादकता
आधारभूत ढांचे के केन्‍द्र में खाता बही होता है। में वृद्धि करने के लिए कृषि उद्योग विभिन्‍न रोबोटिक
• एक क्रिप्‍टोकरेन्‍सी में एक खाता बही होता है जहां सभी प्रौद्योगिकी रूपों को अपनाने के लिए सक्रिय तौर पर कार्य
ले न-देन सार्वजनिक किए जाते हैं ताकि कुल दृश्‍यता कर रहा है। किसान पहले से ही ट्रे क्‍टरों और हारवेस्‍टरों का
प्रदान की जा सके। उपयोग कर रहे हैं जो कि जीपीएस द्वारा स्‍वनिर्देशित हैं।
हाल ही में स्‍वचालित प्रणालियों के प्रयोगात्‍मक उपयोग
• खाता बही एक डाटाबेस में प्रविष्टियों की सूची है जिसे
में वृद्धि हुई है जो कि विरलन, कटाई, छिड़काव और कीट
कोई भी विशेष शर्तों को पूरा किए बिना बदल नहीं
हटाने जैसे कार्य स्‍वत: करती हैं।
सकता। कोई भी खाता – बही या क्रिप्‍टोकरेन्‍सी ब्‍लॉक
चेन का स्‍वामी नहीं होता; बजाय इसके यह विकेन्‍द्रीकृत उदाहरण के लिए, एग्रीबोट एक कृषि बोट है जिसे बिट्स,
है जिसका अर्थ है कि यह बाहरी पार्टि यों के हस्‍तक्षेप के पिलानी, हैदराबाद के विद्यार्थि यों ने डिजाइन किया है,
बिना स्‍वचालित और स्‍वशासित है। जिसे कार्य की कार्यक्षमता, गति, उत्‍पादकता, उपयोग और
सटीकता बढ़ाने के लिए और किसानों का श्रम न्‍यूनतम
करने के लिए डिजाइन किया गया है। कृषि में शामिल
रोबोटिक्‍स (Robotics) इसका केन्‍द्रीय क्षेत्र कटाई, छिड़काव, बीज डालने और
कीट हटाने में शामिल है।
• रोबोटिक्‍स अनुसंधान और विकास का प्रारंभिक युग
3. स्‍थान:
प्रमुख रूप से एक औद्योगिक वातावरण के साथ 20वीं
शताब्‍दी का मध्‍य था जहां निरंतर संचलनों और भारी I. जैविक रूप से प्रेरित नवोन्‍मेष लोकोमोटिव संकल्‍पनाओ ं
मशीनों को उठाने ने मशीनों के उपयोग को मानव से पर आधारित गैर ढ़ांचागत असमान भूभाग के लिए रोबोट
अधिक आकर्षक बना दिया। प्रणाली का विकास

• रोबोट को मुख्‍य रूप से उन कार्यों के लिए नियोजित II. इन-साईट जांच और आधारभूत ढ़ांचे के अनुरक्षण के
किया गया था जो कि अत्‍यधिक गंदे, दूरस्‍थ और मनुष्‍य लिए उपयोगी बहुकार्य रोबोट टीम का विकास।

106
III. गृहीय खोज के लिए पुन:संरूपणीय प्रणाली। III. रोबोट उपकरण अत्‍यधिक लचीले होते हैं और जटिल
कार्य करने के लिए भी इन्‍हें अनुकूल बनाया जा सकता
IV. चित्र मूल्‍यांकन, वस्‍तु की पहचान और भू-भाग मॉडलिंग
है।
वैज्ञानिक अनुभवों के लिए एआई आधारित सहायता
6. शिक्षा: भारत में कम से कम 15 विश्‍वविद्यालय और
प्रणालियां: उदाहरण के लिए, ‘’व्‍याम मित्र’’ एक महिला
कॉले ज अब रोबोटिक्‍स पाठ्यक्रम करवा रहे हैं, जैसे कि
जैसा दिखने वाला अंतरिक्ष में घूमनेवाला हृयूमनॉइड
रोबोटिक्‍स में एम.टे क., ऑटोमेशन में एम.टै क.
रोबोट है जिसका विकास इसरो द्वारा एक कर्मीदल युक्‍त
कक्षीय अंतरिक्ष यान गगनयान पर काम करने के लिए
किया गया है।
रोबोटिक्‍स के लाभ और हानि (Advantage and
4. विद्युत चालकता: विद्युत चालित वाहनों के लिए अनुप्रयोग
Disadvantage of Robotics):
की जांच करने, बैट्री चार्जिं ग प्रौद्योगिकियां विकसित
करने और एक बुद्धिमान,पर्यावरण हितैषी और एकीकृ‍त • लागत प्रभाविता: रोबोट लागत प्रभावी होते हैं, वे विश्राम
शहरी चालकता उत्‍पन्‍न करने के लिए। नहीं करते, छुट्टियां, बीमारी की छुट्टी नहीं ले ते और उन्‍हें
उदाहरण के लिए: केयूकेए (जर्मनी की एक निर्माता लगातार काम पर लगाया जा सकता है, उन्‍हें एकमात्र
कंपनी) का इं जीनियरिं ग परियोजना प्रबंधन, प्रक्रिया चीज की आवश्‍यकता होती है, वह है नियमित रखरखाव।
ज्ञान, पावर टै रन
े सहित विद्युत चालित वाहनों के • उन्‍नत गुणवत्‍ता आश्‍वासन: यह बहुत स्‍वाभाविक है कि
स्‍वचालित उत्‍पाद के लिए कमीशनिं ग और सेवा में समय के साथ श्रमिक की एकाग्रता घट जाती है, एकाग्रता
अत्‍यधिक अनुभव और विशेषज्ञता हासिल है। भंग होती है, जिसके फलस्‍वरूप अधिकतर व्‍यवसाय के
5. निर्माण: रोबोटिक्‍स निर्माण के क्षेत्र में एक महत्‍वपूर्ण लिए महंगी गलतियां होती हैं।
भूमिका निभाता है। आज स्‍वचालित निर्माण समाधान • उन्नत उत्‍पादकता: रोबोटिक स्‍वचालन बारंबार के
किसी की कार्य का एक महत्‍वपूर्ण भाग है जहां अधिकतम कार्यों को संभालने में सहायता कर सकता है जो स्‍टाफ
कार्यक्षमता, सुरक्षा और बाजार में प्रतियोगितात्‍मक लाभ के सदस्‍यों को अपने कौशल का विस्‍तार करने के लिए
की जरूरत होती है। बारंबार के कार्यों का स्‍वचालन, पर्याप्‍त समय देता है और अनुसंधान के लिए नए रास्‍ते
गलतियां करने की सीमा को नगण्‍य दरों तक कम कर भी खोलता है।
देता है, और मानव श्रमिकों को अधिक कौशल विकास
• जोखिम भरे वातावरण में कार्य करें: रोबोट प्रभावी तरीके
और विविध कार्य रूपरेखा पर ध्‍यान केन्द्रित करने में
से जोखिम भरे वातावरण को संभाल सकते हैं विशेषकर
सक्षम बनाता है।
उत्‍पादन क्षेत्रों में जिनमें असामान्‍य उच्‍च या निम्‍न
I. यह कच्‍चे माल को संभालने से ले कर पूर्ण निर्मि त उत्‍पाद तापमानों की आवश्‍यकता होती है, कार्य की प्रकृति के
की पैकिंग तक कार्य क्षमता का सृजन करता है। कारण स्‍टाफ का अधिक टर्नओवर होता है। रोबोट सामग्री
II. इसे निरंतर उत्‍पादन के लिए 24x7 कार्य करने के लिए की बर्बादी को न्‍यूनतम कर सकते हैं और मानव द्वारा
प्रोग्राम किया जा सकता है। स्‍वयं को अनावश्‍यक खतरों में डालने की आवश्‍यकता
को खत्‍म करते हैं।

107 @ITAKASHSINGH
अध्याय - 10

बौद्धिक संपदा अधिकार


(INTELLECTUAL PROPERTY RIGHTS)

परिभाषा (Definition): सांस्कृतिक कल्याण के लिए एक उत्प्रेरक के रूप में


बौद्धिक संपदा की क्षमता के बारे में जानने में मदद कर
सकती है। बौद्धिक संपदा प्रणाली नवोन्मेषकों के हितों
• बौद्धिक संपदा (IP) से आशय मस्तिष्क की रचनाओ ं जैसे
और सार्वजनिक हित के बीच संतुलन बनाने में मदद
आविष्कार; साहित्यिक और कलात्मक कार्य; वाणिज्य में
करती है और एक ऐसा वातावरण प्रदान करती है जिसमें
प्रयुक्त प्रतीक, नाम और चित्र आदि है। बौद्धिक संपदा शब्द
रचनात्मकता और आविष्कार सभी के लाभ के लिए
विशिष्ट कानूनी अधिकारों को दर्शाता है, न कि बौद्धिक
विकसित हो सकें।
कार्य को। यह जानना आवश्यक है कि ये IP अधिकार
क्या हैं, इन्हें कैसे संरक्षित किया जा सकता है और समय
आने पर इनका लाभ कैसे प्राप्त किया जा सकता है।
IPR के प्रकार (Types of IPR)
• बौद्धिक संपदा को दो श्रेणियों में बांटा गया है:
• पेटेंट: एक पेटेंट एक आविष्कार को संरक्षण प्रदान करता
» औद्योगिक संपत्ति जिसमें आविष्कारों के लिए पेटेंट, है। यह धारक को एक निश्चित अवधि (आमतौर पर दाखिल
ट्रे डमार्क , औद्योगिक डिजाइन और भौगोलिक संकेत करने की तारीख से 20 साल) के लिए पेटेंट किए गए
शामिल हैं। आविष्कार को दूसरे के द्वारा बेचने, बनाने और उपयोग
» कॉपीराइट के अंतर्गत साहित्यिक कृतियाँ (जैसे उपन्यास, करने से रोकने का विशेष अधिकार देता है।
कविताएँ और नाटक), फ़िल्में, संगीत, कलात्मक कार्य • भारत में पेटेंट के संरक्षण और पंजीकरण से संबंधित
(जैसे, चित्र, पेंटिंग, तस्वीरें और मूर्ति यां) और वास्तुशिल्प कानून पेटेंट अधिनियम, 1970 और पेटेंट नियम 2003 हैं।
डिज़ाइन शामिल हैं।
• पेटेंट अधिनियम 1970 में 1999, 2002 और 2005 में तीन
(3) संशोधन हुए हैं।2005 के संशोधन से खाद्य, फार्मा और
रासायनिक आविष्कारों के लिए उत्पाद पेटेंट संरक्षण की
बौद्धिक संपदा का प्रोत्साहन शुरुआत हुई।

और संरक्षण क्यों? (Why • भारत में एक आविष्कार/उत्पाद को पेटेंट के लिए अर्हता


प्राप्त करने के लिए विभिन्न मानदंडों को पूरा करना
Promote and Protect the पड़ता है:

Intellectual Property?) » नया/ नवीनता: आविष्कार में कोई विशेषता होनी चाहिए
जो इसे पिछले आविष्कारों से अलग करती हो और जनता
इसके निम्नलिखित कारण हैं: के लिए अज्ञात हो।

• मानवता की प्रगति और भलाई प्रौद्योगिकी और संस्कृति » गैर-स्पष्टता: आविष्कार की नवीनता किसी ऐसे व्यक्ति

में नए कार्यों को करने और आविष्कार करने की उसकी के लिए स्पष्ट नहीं होनी चाहिए जिसके पास आविष्कार

क्षमता पर निर्भर करती है। के क्षेत्र में सामान्य कौशल है।

• नई रचनाओ ं का कानूनी संरक्षण आगे नवाचार के लिए » उपयोगिता: आविष्कार को उपयोगी होना चाहिए।

अतिरिक्त संसाधनों की उपलब्धता को प्रोत्साहित करता


• अन्य IP कानूनों की तरह पेटेंट संरक्षण क्षेत्रीय है। पेटेंट
है।
का पंजीकरण पूरे भारत में संरक्षण सुनिश्चित करता है।
• बौद्धिक संपदा का प्रचार और संरक्षण आर्थि क विकास यदि कोई किसी अन्य देश में अपने आविष्कार के संरक्षण
को बढ़ावा देता है, नए रोजगार और उद्योग उत्पन्न करता चाहता है तो उसे हर उस देश में आवेदन करना होगा जहां
है, और जीवन की गुणवत्ता और आनंद को बढ़ाता है। आवेदक अपने उत्पाद/आविष्कार के लिए पेटेंट संरक्षण
• एक कुशल और न्यायसंगत बौद्धिक संपदा प्रणाली चाहता है।
सभी देशों को आर्थि क विकास और सामाजिक और

108
पेटेंट के लिए अंतर्राष्ट्रीय संरक्षण (International सम्मान करने के लिए अपनाया गया था जो कन्वेंशन के
Protection for Patents) पक्षकार देशों के नागरिक हैं। विश्व बौद्धिक संपदा संगठन
(WIPO) कन्वेंशन का संचालन करता है।
• अंतरराष्ट्रीय पेटेंट संरक्षण पर विचार करते समय हुए
बाध्यकारी समझौते पेरिस कन्वेंशन फॉर प्रोटे क्टिं ग द
इं डस्ट्रियल प्रॉपर्टी और पेटेंट कोआपरेशन ट्रीटी (PCT) हैं।
ट्रे डमार्क (Trademarks):

• ट्रे डमार्क एक विशिष्ट संकेतक है जिसका उपयोग एक


1.पेरिस कन्वेंशन (The Paris Convention)
व्यवसाय के उत्पादों या सेवाओ ं को दूसरे से अलग
• औद्योगिक संपत्ति की सुरक्षा के लिए 1883 में स्थापित करने के लिए किया जाता है। ट्रे डमार्क ज्यादातर ब्रांडों से
पेरिस कन्वेंशन एक अंतरराष्ट्रीय बौद्धिक संपदा संधि है निकटता से जुड़े होते हैं।
जिसका पालन 100 से अधिक देशों द्वारा किया जाता है। • भारत में ट्रे डमार्क के संरक्षण और पंजीकरण से संबंधित
कानून ट्रे डमार्क अधिनियम, 1999 और ट्रे डमार्क नियम
2002 हैं। भारत में, ट्रे डमार्क पंजीकरण दस वर्षों के लिए
2. पेटेंट कोआपरेशन ट्रीटी (PCT) के माध्यम से अंतर्राष्ट्रीय वैध है। इसे समय-समय पर दस-दस वर्ष की अतिरिक्त
अनुप्रयोग (International Applications Through
अवधि के लिए नवीनीकृत किया जा सकता है।
the Patent Co-operation Treaty (PCT))

• पेटेंट कोआपरेशन ट्रीटी (PCT) सदस्य देशों में एक ही


आविष्कार पर पेटेंट के लिए आवेदन दाखिल करने की डिज़ाइन (Design):
सुविधा प्रदान करती है। पीसीटी प्रणाली अंतरराष्ट्रीय
पेटेंट प्रदान नहीं करती है, यह पेटेंट के लिए अंतरराष्ट्रीय • यह किसी वस्तु के बाहरी रूप या बनावट को संरक्षण
आवेदन दाखिल करने की एक प्रणाली है। प्रदान करता है और कार्यक्षमता या दिखाई न देने वाले
(आंतरिक) डिजाइन तत्वों का संरक्षण नहीं करता।

• डिजाइन के भारतीय कानून डिजाइन अधिनियम, 2000


कॉपीराइट (Copyright): और डिजाइन नियम, 2001 में निहित है। औद्योगिक
डिजाइन पंजीकरण मालिक को किसी भी उत्पाद को
• कॉपीराइट साहित्यिक या कलात्मक कार्य की अभिव्यक्ति बेचने, आयात करने और डिजाइन को लागू करने का
को संरक्षा प्रदान करता है। संरक्षण स्वतः उत्पन्न होता है विशेष अधिकार प्रदान करता है। भारत ने 'फर्स्ट टू फाइल'
जिससे धारक को पुनरूत्पादन या एडाप्शन को नियंत्रित प्रणाली को अपनाया है, जिसका अर्थ है कि अधिकार
करने का विशेष अधिकार प्राप्त होता है। धारक को जल्दी से जल्दी आवेदन करना चाहिए, जिससे
• भारतीय कानून किसी भी रचना के लिए ले खक के उस डिजाइन पर किसी और के दावे की संभावना को
जीवन काल और उसके बाद साठ (60) वर्षों के लिए खारिज किया जा सके।
कॉपीराइट सुरक्षा प्रदान करता है। कॉपीराइट अधिनियम, • कुछ कलात्मक कृतियाँ हैं जिन्हें डिजाइन के रूप में
1957 और कॉपीराइट नियम, 1958 भारत में कॉपीराइट पंजीकृत करना संभव नहीं है। उनमें पेंटिंग, मूर्ति कला,
की सुरक्षा प्रदान करते हैं। ड्राइं ग (आरेख, मानचित्र, चार्ट या योजना सहित), एक
• जैसे ही कोई रचना की जाती है, कानून के अंतर्गत आने उत्कीर्णन या तस्वीर (चाहे ऐसे किसी भी काम में
वाले किसी भी मूल रचना के ले खक को कॉपीराइट कलात्मक गुणवत्ता हो या न हो), वास्तुकला का काम,
अपने आप ‍मिल जाता है। पंजीकरण अनिवार्य नहीं है, और कलात्मक शिल्प कौशल का कोई अन्य काम
ले किन विवाद की स्थिति में स्वामित्व की सुरक्षा प्रदान शामिल है।
करता है। कॉपीराइट पंजीकरण एक कॉपीराइट धारक
जो उल्लं घन के खिलाफ सिविल या आपराधिक कार्रवाई
करना चाहता है, के लिए बहुत महत्वपूर्ण ‍हो जाता है। डाटाबेस (Database):
• भारत सरकार ने अंतर्राष्ट्रीय कॉपीराइट आदेश, 1999
• डाटाबेस अधिकार डेटाबेस के महत्वपूर्ण भागों की नकल
के तहत भारत में कॉपीराइट संरक्षण को पहले भारत के
करने से रोकता है। संरक्षा सूचना की अभिव्यक्ति के रूप
बाहर प्रकाशित की गई रचनाओ ं तक बढ़ा दिया है।
में नहीं बल्कि स्वयं सूचना की है, ले किन कई अन्य
• बर्न कन्वेंशन के तहत अंतर्राष्ट्रीय संरक्षण: साहित्यिक पहलु ओ ं में डाटाबेस अधिकार कॉपीराइट के समान है।
और कलात्मक कार्यों के संरक्षण के लिए बर्न कन्वेंशन
को पहली बार 1886 में उन सभी ले खकों के अधिकारों का

109
ट्रे ड सीक्रे ट (Trade Secrets): IPR के विकास में यूएनओ की भूमिका (Role of
UNO in Development of IPR):
• ट्रे ड सीक्रेट किसी व्यवसाय द्वारा प्रतिस्पर्धि यों पर लाभ
प्राप्त करने के लिए उपयोग ‍किया जाने वाला एक सूत्र, • यूएनओ विश्व बौद्धिक संपदा संगठन (WIPO) की मदद से
अभ्यास, प्रक्रिया, डिजाइन या जानकारी का संकलन है। आईपीआर के विकास और संरक्षण में महत्वपूर्ण भूमिका
ट्रे ड सीक्रेट की परिभाषा के अनुसार ये सामान्यत: दुनिया निभाता है, जो यूएनओ के तहत काम करने वाले 16
के सामने प्रकट नहीं किए जाते हैं। संगठनों में सबसे प्रभावशाली और महत्वपूर्ण संगठनों में
से एक है। WIPO का उद्देश्य दुनिया भर में आईपीआर को
बढ़ावा और संरक्षण देना और उपयोगिता मॉडल, चिह्न या
भौगोलिक संकेतक (जीआई) (Geographical औद्योगिक डिजाइन सुनिश्चित करना है।
Indication (GI))

• यह ऐसे "संकेतक" को संदर्भि त करता है जो किसी सदस्य सामान्य नियम (Common Rules):
के राज्यक्षेत्र, या अंचल या किसी स्थानीय इलाके में पैदा
होने वाले सामान की पहचान करता है, जहां सामान • पेरिस कन्वेंशन कुछ मानक नियम निर्धारित करता
की गुणवत्ता, प्रतिष्ठा या अन्य विशेषता अनिवार्य रूप से है जिनका पालन सभी अंनुबंधित राष्ट्रों को करना
इसके भौगोलिक मूल के कारण होती है। चाहिए।इनमें सबसे महत्वपूर्ण हैं:

• भारत में, भौगोलिक संकेतक व्यवस्था को माल • पेटेंट: एक ही आविष्कार के लिए विभिन्न अनुबंधित
का भौगोलिक संकेतक (पंजीकरण और संरक्षण) देशों में दिए गए पेटेंट एक-दूसरे से स्वतंत्र होते हैं: एक
अधिनियम, 1999 और माल का भौगोलिक संकेतक अनुबंधित देश में दिया गया अन्य अनुबंधित देशों को
(विनियमन और संरक्षण) नियम, 2002 द्वारा विनियमित पेटेंट प्रदान करने के लिए बाध्य नहीं करता है।
किया जाता है। भौगोलिक संकेतक को पंजीकृत कराने
की सिफारिश की जाती है क्योंकि पंजीकरण प्रमाण पत्र
किसी भी विवाद में अदालत में प्रथम दृष्टया साक्ष्य के रूप निशान (Marks):
में कार्य करता है। वैधता/स्वामित्व साबित करने के लिए
किसी अतिरिक्त सबूत की आवश्यकता नहीं है। • पेरिस कन्वेंशन घरेलू कानून द्वारा प्रत्येक अनुबंधित देश
में निर्धारित निशानों के दाखिल और पंजीकरण की शर्तों
• भारत में, एक भौगोलिक संकेतक को शुरू में दस वर्षों
को विनियमित नहीं करता है।
के लिए पंजीकृत किया जा सकता है, और इसे समय-
समय पर आगे 10 वर्षों की अवधि के लिए नवीनीकृत
किया जा सकता है। भौगोलिक संकेतक के अंतरराष्ट्रीय
औद्योगिक डिजाइन (Industrial Designs):
पंजीकरण से संबंधित दो समझौते लिस्बन एग्रीमेंट फॉर
द प्रोटे क्शन ऑफ एपीले शन ऑफ ऑरिजिन, 1958 और
• प्रत्येक अनुबंधित देश में औद्योगिक डिजाइनों को संरक्षित
पेरिस कन्वेंशन ऑन इं डस्ट्रियल प्रापर्टी, 1883 हैं।
किया जाना चाहिए, और संरक्षण को जमीनी तथ्यों पर
खारिज नहीं किया जा सकता है जिसमें डिजाइन शामिल
हैं जो उस देश में निर्मि त नहीं हैं।

IPR पर अंतर्राष्ट्रीय संधियाँ


(International Treaties on व्यापार के नाम (Trade Names):

IPR): • नाम दर्ज करने या पंजीकृत करने के दायित्व के बगैर


ही प्रत्येक अनुबंधित देश में व्यापारिक नामों को संरक्षण
• बौद्धिक संपदा के राष्ट्रीय और अंतर्राष्ट्रीय दोनों आयाम
प्रदान किया जाना चाहिए।
हैं। उदाहरण के लिए, पेटेंट राष्ट्रीय कानूनों और संबंधित
देश के नियमों द्वारा शासित होते हैं। इसके विपरीत, पेटेंट
पर अंतर्राष्ट्रीय कन्वेंशन न्यूनतम अधिकारों की रक्षा
स्रोत के संकेत (Indications of Source):
करते हैं और अनुबंधित राष्ट्रों द्वारा अधिकारों के प्रवर्तन
के लिए कुछ उपाय प्रदान करते हैं।
• प्रत्येक अनुबंधित देश को माल के स्रोत या उनके निर्माता,
या व्यापारी की पहचान के झूठे उपदर्शन के प्रत्यक्ष या
अप्रत्यक्ष उपयोग के खिलाफ उपाय करना चाहिए।

110

बर्न कन्वेंशन (साहित्यिक और
अनुचित प्रतिस्पर्धा: प्रत्येक अनुबंधित देश को अनुचित
प्रतिस्पर्धा के खिलाफ पर्याप्त सुरक्षा प्रदान करनी चाहिए।

• पेरिस कन्वेंशन ऑन प्रोटे क्शन ऑफ इं डस्ट्रियल प्रापर्टी: कलात्मक कार्यों का संरक्षण)


WIPO पेरिस कन्वेंशन का संचालन करता है।
(Berne Convention
• यह बौद्धिक संपदा कानूनों में कुछ अंतरराष्ट्रीय सद्भाव
लाने के लिए अस्तित्व में आया और 20 मार्च, 1883 को (Protection of Literary
पेरिस में अपनाया गया और 7 जुलाई, 1884 को लागू
किया गया।
and Artistic Works))
• यह बौद्धिक संपदा जैसे पेटेंट, उपयोगिता मॉडल, • भारत ने 1 अप्रैल, 1928 को बर्न कन्वेंशन पर हस्ताक्षरकर्ता
औद्योगिक डिजाइन, ट्रे डमार्क , सेवा चिह्न, व्यापार नाम, किए।
सूचना के स्रोत या अपील के संकेत की संरक्षा के लिए
• अंतरराष्ट्रीय स्तर पर कॉपीराइट संरक्षण के लिए पहला
बुनियादी दिशानिर्देश प्रदान करता है और उत्पीड़न और
कदम उन्नीसवीं सदी के मध्य में द्विपक्षीय संधियों के
अनुचित प्रतिस्पर्धा के राष्ट्रीय व्यवहार के लिए कुछ
आधार पर उठाया गया।
प्रावधान भी उपलब्ध कराता है।
• अधिकारों को पारस्परिक मान्यता प्रदान करने वाली
• भारत में यह संधि 7 दिसंबर,1998 को अस्तित्व में आई।
ऐसी कई संधियाँ हुई, ले किन वे न तो पर्याप्त रूप से
• कन्वेंशन के भेदभाव-रोधी सिद्धांत में, एक सदस्य देश व्यापक थीं और न ही समान।
को अन्य सदस्य देशों के नागरिकों को अपने नागरिकों
• (कन्वेंशन का अनुच्छेद 2(1)) कहता है कि जहां तक
के समान संरक्षण और लाभ प्रदान करने का अधिकार है।
काम का सवाल है, संरक्षा में "साहित्यिक, वैज्ञानिक
और कलात्मक क्षेत्र में किया गया हर काम, चाहे उसकी
अभिव्यक्ति का तरीका या रूप कुछ भी हो" शामिल होना
राष्ट्रीय व्यवहार (National Treatment): चाहिए।

• औद्योगिक संपत्ति के संरक्षण के संबंध में राष्ट्रीय व्यवहार • व्यक्तिगत अनुमत शर्तों, सीमाओ ं या अपवादों के अधीन,
को परिभाषित किया जा सकता है क्योंकि पेरिस निम्नलिखित अधिकार हैं। इन्हें संरक्षा प्रदान करने के
कन्वेंशन के प्रत्येक सदस्य देश को अन्य सदस्य देशों के अनन्य अधिकारों के रूप में मान्यता दी जानी चाहिए:
नागरिकों को उनके आविष्कार के लिए अपने नागरिकों
» अनुवाद करने का अधिकार,
के समान संरक्षण प्रदान करना चाहिए।
» कार्य के एडाप्टेशन और अरेन्जमेंट करने का अधिकार,
• राष्ट्रीय व्यवहार का सिद्धांत गारंटी देता है कि विदेशियों
की रक्षा की जाएगी और उनके साथ किसी भी आधार पर » सार्वजनिक नाटकीय, नाटकीय-संगीत और संगीत
भेदभाव नहीं किया जाएगा। कार्यों में प्रदर्शन करने का अधिकार,

» सार्वजनिक रूप से साहित्यिक कृतियों को सुनाने का


अधिकार,
प्राथमिकता का एक ढांचा (A Framework of Priority):
» जनता को ऐसे कार्यों के प्रदर्शन के बारे में बताने का
अधिकार,
• पेरिस सम्मेलन का एक अन्य मूलभूत सिद्धांत
'प्राथमिकता का ढांचा' है। पेरिस कन्वेंशन के तहत, एक » प्रसारण का अधिकार,
ही समय में एक आविष्कार को विभिन्न देशों में संरक्षित
» किसी भी तरीके या रूप में पुनरुत्पादन करने का
किया जा सकता है। इसके द्वारा विदेशी आवेदक राष्ट्रीय
अधिकार,
पेटेंट सिस्टम तक पहुंच सकते हैं।
» दृश्य-श्रव्य कार्य के आधार के रूप में इसका उपयोग
करने का अधिकार, और

» सार्वजनिक दृश्य-श्रव्य कार्य को पुन: प्रस्तुत करने,


वितरित करने, सार्वजनिक रूप से प्रदर्शन करने या
संचार करने का अधिकार।

111

@ITAKASHSINGH
यूनिवर्सल कॉपीराइट कन्वेंशन को एकीकृत करना।

» औद्योगिक संपत्ति के लिए एक अंतरराष्ट्रीय पंजीकरण


(The Universal Copyright सेवा प्रदान करना।

Convention, UCC) » बौद्धिक संपदा के बारे में सूचनाओ ं का आदान-प्रदान


करना।
• यूनिवर्सल कॉपीराइट कन्वेंशन (UCC), पहली बार 1952 » विकासशील देशों और अन्य को कानूनी और तकनीकी
में जिनेवा में बर्न कन्वेंशन के विकल्प के रूप में बनाया सहायता प्रदान करना।
गया था।
» लोगों के बीच बौद्धिक संपदा से संबंधित विवादों के
• कुछ देश बर्न कन्वेंशन में विशिष्ट ले खों के पक्ष में नहीं थे निपटारे के दौरान सहायता करना।
और बर्न कन्वेंशन की शर्तों पर हस्ताक्षर करने के लिए
सहमत नहीं थे। • 14 जुलाई 1967 को स्टॉकहोम में आयोजित कन्वेंशन
• बर्न कन्वेंशन का पालन करने से पहले अमेरिका को और कन्वेंशन के अनुच्छेद 2(viii) के अनुसार बौद्धिक
अपने कानूनों में कई बदलाव करने पड़े। संपदा अधिकारों में निम्नलिखित अधिकार शामिल हैं:

» साहित्यिक, कलात्मक और वैज्ञानिक कार्य;

» प्रदर्शन करने वाले कलाकारों, फोनोग्राम और प्रसारणों


विश्व बौद्धिक संपदा संगठन का प्रदर्शन;

(World Intellectual » मानव व्यवहार के सभी क्षेत्रों में आविष्कार;

Property Organization) » " वैज्ञानिक खोज;

» औद्योगिक डिजाइन;
• विश्व बौद्धिक संपदा संगठन (WIPO) एक अंतरराष्ट्रीय
» ट्रे डमार्क ;
संगठन है जो बौद्धिक संपदा अधिकारों के रचनाकारों
और मालिकों को विश्वव्यापी सुरक्षा प्रदान करता है। » सेवा चिह्न;

• इसे 14 जुलाई 1967 को स्टॉकहोम में अपनाया गया था » व्यावसायिक नाम और पदनाम;
और 26 अप्रैल 1970 को लागू किया गया था। वर्तमान » अनुचित प्रतिस्पर्धा से सुरक्षा;
में 184 देश WIPO के सदस्य हैं। भारत विश्व बौद्धिक
» औद्योगिक वैज्ञानिक, साहित्यिक या कलात्मक क्षेत्रों में
संपदा संगठन के सदस्य देशों में से एक है। किसी देश को
बौद्धिक गतिविधि से उत्पन्न अन्य सभी अधिकार; आदि।
जिनेवा में विश्व बौद्धिक संपदा संगठन के महानिदेशक
के पास अनुसमर्थन या परिग्रहण का एक साधन जमा
करना होगा।

• WIPO दो मुख्य उद्देश्यों के साथ स्थापित किया गया: WIPO के तहत अंतर्राष्ट्रीय
• दुनिया भर में बौद्धिक संपदा के संरक्षण को बढ़ावा देने संधियां (International
के लिए और; WIPO प्रशासन के अधीन संधियों द्वारा
स्थापित बौद्धिक संपदा संघों के बीच प्रशासनिक सहयोग Treaties Under WIPO):
बनाए रखना।
• विश्व बौद्धिक संपदा संगठन द्वारा ‍की गई अंतर्राष्ट्रीय
• 1974 में, WIPO संयुक्त राष्ट्र की एक विशेष एजेंसी बन संधियां
गई और 1996 में, WIPO ने विश्व व्यापार संगठन के साथ
• WIPO द्वारा 24 अंतर्राष्ट्रीय संधियां की गई हैं। कुछ का
एक सहयोग समझौता करके वैश्वीकृत व्यापार में अपनी
उल्ले ख नीचे किया गया है:
भूमिका का विस्तार किया। WIPO 25 संधियों (उनमें
से तीन अन्य अंतरराष्ट्रीय संगठनों के साथ संयुक्त रूप
» बर्न सम्मेलन
से) का संचालन करता है और अपने सदस्य राज्यों और
सचिवालय के माध्यम से कार्यों का एक समृद्ध और » लिस्बन समझौता
विविध कार्यक्रम संचालित करता है, » मैड्रिड समझौता
• इसकी अपेक्षाएं : » पेरिस सम्मेलन

» पेरिस कानून सम्मेलन


» बौद्धिक संपदा से संबंधित राष्ट्रीय कानूनों और प्रक्रियाओ ं

112
»
व्यापार संबंधी पहलू (TRIPS)
फोनोग्राम सम्मेलन

» रोम सम्मेलन

» वियना समझौता
(Trade-related Aspects
» वाशिं गटन संधि of Intellectual Property
Rights)
WIPO के अंग (Organs of • जिस समझौते ने विश्व व्यापार संगठन ("डब्ल्यूटीओ
समझौता") की स्थापना की, उसे 15 अप्रैल 1994 को
WIPO): माराकेच को लागू किया गया था।

• TRIPS समझौते (जो सभी डब्ल्यूटीओ सदस्यों के लिए


विश्व बौद्धिक संपदा संगठन के तीन मुख्य अंग हैं:
बाध्यकारी है) सहित डब्ल्यूटीओ समझौता, 1 जनवरी,
1995 को लागू किया गया। डब्ल्यूटीओ के सदस्य राज्यों
• आम सभा
को TRIPS समझौते को लागू करने के लिए बाध्य होने
• कान्‍फ्रें स से पहले डब्ल्यूटीओ की स्थापना के समझौते को लागू
• समन्वय समिति करने के बाद एक निश्चित समय दिया गया था।

• बौद्धिक संपदा अधिकारों के व्यापार से संबंधित पहलू


(TRIPS) बौद्धिक संपदा अधिकारों पर सबसे वैश्विक और
आम सभा (The General Assembly) आवश्यक अंतरराष्ट्रीय समझौता है।

• विश्व बौद्धिक संपदा संगठन की आम सभा में स्टेट पार्टी टू • विश्व व्यापार संगठन के सदस्य देश अपने आप से
कन्वेंशन शामिल है। प्रत्येक सदस्य देश की सरकार का समझौते से बंधे हैं।
प्रतिनिधित्व आम सभा में एक प्रतिनिधि द्वारा किया जाता • समझौते में अधिकांश बौद्धिक संपदा रूप जैसे पेटेंट,
है। सभा विश्व बौद्धिक संपदा संगठन के महानिदेशक की कॉपीराइट, ट्रे डमार्क , व्यापार रहस्य, भौगोलिक संकेत,
नियुक्ति करती है। औद्योगिक डिजाइन, और नई पौधों की किस्मों पर
बहिष्करण अधिकार शामिल हैं।

• TRIPS ने मुख्य रूप से सभी प्रकार की बौद्धिक संपदा के


कान्‍फ्रेंस (The Conference) संरक्षण और प्रवर्तन के लिए वैश्विक न्यूनतम मानक पेश
• कान्‍फ्रें स बौद्धिक संपदा में सामान्य हित के मामले पर किया, ले किन यह पेटेंट के लिए वैश्विक न्यूनतम मानक
चर्चा करती है और संघ की क्षमता और स्वायत्तता के लिए निर्दिष्ट करने में विफल रहा।
ऐसे मामलों से संबंधित सिफारिशों को अपना सकती है। • इसका मुख्य उद्देश्य बौद्धिक संपदा अधिकारों के पर्याप्त
संरक्षण को बढ़ावा देना और बौद्धिक संपदा अधिकारों
को लागू करने के उपायों और प्रक्रियाओ ं को सुनिश्चित
समन्वय समिति (The Coordination करना है कि वे स्वयं वैध व्यापार में बाधा न बनें।
Committee)

• समन्वय समिति संघ के अंगों, महासभा और कान्‍फ्रें स


TRIPS द्वारा कवर किए गए मुद्दे (Issues Covered
और महानिदेशक को सभी प्रशासनिक, वित्तीय और
by the TRIPS)
सामान्य हित के अन्य मामलों पर सलाह देती है।

• समन्वय समिति महासभा की मसौदा कार्यसूची तैयार • व्यापार प्रणाली और अन्य अंतरराष्ट्रीय बौद्धिक संपदा
करती है। समझौतों के बुनियादी सिद्धांतों को कैसे लागू किया
जाना चाहिए?

• बौद्धिक संपदा अधिकारों को पर्याप्त सुरक्षा कैसे दी जाए?

बौद्धिक संपदा अधिकारों के • देशों को उनके क्षेत्रों में पर्याप्त रूप से अधिकार कैसे लागू
करना चाहिए?

• सदस्यों के बीच बौद्धिक संपदा विवादों का निपटारा कैसे


करें?

113
• नई प्रणाली शुरू करने की अवधि के दौरान विशेष अवधि 2016 तक बढ़ा दी गई है।
संक्रमणकालीन व्यवस्था करना।

निष्कर्ष (Conclusion)

TRIPS की विशेषताएं • व्यावसायिक महत्व के लिए आवश्यक है कि बौद्धिक

(Features of The TRIPS) संपदा को पर्याप्त रूप से संरक्षित किया जाए। इस प्रकार
परिणामी बौद्धिक संपदा की सुरक्षा यह सुनिश्चित करने
के लिए महत्वपूर्ण हो जाती है कि निवेश को लाभप्रद
• ट्रिप्स समझौते की तीन मुख्य विशेषताएं हैं जो इस प्रकार
रूप से वसूल किया जाए। रचनाकार/आविष्कारक को
हैं:
दिया गया अधिकार उनके लिए ऐसे विचार उत्पन्न करने
» मानक के लिए एक प्रोत्साहन है जो समग्र रूप से समाज को
लाभान्वित करेगा।
» प्रवर्तन
• ये समझौते IPR के प्रवर्तन के लिए एक न्यूनतम मानक
» विवाद निपटान
प्रदान करते हैं यह अधिकार धारकों को सिविल कोर्ट या
प्रशासनिक कार्यवाही के माध्यम से अपने वैध हितों की
• मानक: बौद्धिक संपदा के व्यापार संबंधी पहलू का मुख्य
रक्षा करने की अनुमति देता है।
तत्व विषय वस्तु की रक्षा करना है।

• प्रवर्तन: दूसरा तत्व घरेलू प्रक्रिया और संबंधित उपायों के


बारे में है।

• विवाद निपटान: TRIPS दायित्वों से संबंधित विश्व व्यापार IPR की रक्षा और बढ़ावा देने के
संगठन के सदस्यों के बीच समझौता विवाद विश्व व्यापार लिए भारत के प्रयास (India’s
संगठन विवाद निपटान प्रक्रिया के अधीन है।
Efforts To Protect and
Promote IPRs)
TRIPS समझौते के अंतर्गत
• किसी भी ज्ञान अर्थव्यवस्था की प्रगति और विकास में
दायित्व (Obligations Under रचनात्मकता और नवाचार एक प्रमुख तत्व रहे हैं। भारत

TRIPS Agreement) में रचनात्मक और नवीन ऊर्जाओ ं की प्रचुरता है। भारत


में TRIPS के अनुरूप, मजबूत, न्यायसंगत और गतिशील
IPR व्यवस्था है।
• TRIPS समझौता बौद्धिक संपदा के विभिन्न महत्वपूर्ण
व्यवसाय-संबंधी पहलु ओ ं की रूपरेखा तैयार करता है।

• मानदंड किसी भी प्रकार की बौद्धिक संपदा पर


राष्ट्रीय IPR नीति (National IPR Policy)
एकाधिकार देने साथ ही अवधि सीमा, प्रवर्तन प्रावधान
और आईपी विवाद निपटान के तरीके के लिए न्यूनतम • राष्ट्रीय IPR नीति का औचित्य एक विपणन योग्य वित्तीय
मानक भी हैं। परिसंपत्ति और आर्थि क उपकरण के रूप में IPR के महत्व

• जब 1 जनवरी 1995 को TRIPS समझौता लागू किया गया, के बारे में जागरूकता पैदा करना है।

तो सभी विकसित देशों को इसके प्रावधानों को लागू


करने के लिए समझौते पर हस्ताक्षर करने के बाद बारह
लक्ष्य वक्तव्य (Vision Statement):
महीने का समय दिया गया था। विकासशील देशों और
संक्रमण अर्थव्यवस्थाओ ं (कुछ शर्तों के तहत) को 2000 • भारत का विकास करना जहां सभी के लाभ के लिए
तक पांच साल दिए गए थे। बौद्धिक संपदा द्वारा रचनात्मकता और नवाचार को प्रेरित
• 2006 तक, सबसे कम विकसित देशों (LDCs) को किया जाता है;
अनुपालन करने के लिए 11 साल का समय दिया गया • भारत का विकास करना जहां बौद्धिक संपदा विज्ञान और
था। कुछ देशों ने संकेत दिया है कि यह दीर्घकालिक रूप प्रौद्योगिकी, कला और संस्कृति, पारंपरिक ज्ञान और जैव
से हासिल किया जाना चाहिए। विविधता संसाधनों में उन्नति को बढ़ावा देती है;
• इन LDCs में फार्मास्युटिकल पेटेंट के लिए अनुपालन • भारत को विकसित करना जहां ज्ञान विकास का

114
प्राथमिक चालक है, और स्वामित्व वाले ज्ञान को साझा Administration and Management):
ज्ञान में बदल दिया जाता है।
• सेवा उन्मुख IPR प्रशासन का आधुनिकीकरण और
सुदृढ़ीकरण करना।
मिशन वक्तव्य (Mission Statement):
उद्दे श्य 5- आईपीआर का व्यावसायीकरण (Objective 5
भारत में एक गतिशील, जीवंत और संतुलित बौद्धिक संपदा Commercialization of IPR):
अधिकार प्रणाली को प्रोत्साहित करने के लिए:
• आईपी के मालिकों के लिए व्यावसायीकरण और
• रचनात्मकता और नवाचार को बढ़ावा देना और इस तरह आर्थि क पुरस्कार के माध्यम से IPR को कीमती बनाया
उद्यमिता, सामाजिक आर्थि क और सांस्कृतिक विकास जाए। उद्यमिता को प्रोत्साहित किया जाना चाहिए ताकि
को बढ़ावा देना, और IPR का वित्तीय मूल्य मिल सके।
• महत्वपूर्ण सामाजिक, आर्थि क और तकनीकी महत्व के
उद्दे श्य 6- प्रवर्तन और अधिनिर्णय (Objective 6
अन्य क्षेत्रों में स्वास्थ्य देखभाल, खाद्य सुरक्षा और पर्यावरण
Enforcement and Adjudication)
संरक्षण तक पहुंच बढ़ाने पर ध्यान केंद्रित करना।
• IPR उल्लं घनों का मुकाबला करने के लिए प्रवर्तन और
न्यायिक तंत्र को मजबूत करने के लिए, आम जनता
उद्दे श्य (Objectives) के बीच IPR के लिए सम्मान पैदा करने और आईपी के
• नीति द्वारा चिन्हित नोडल मंत्रालय/विभाग द्वारा उठाए आविष्कारकों और रचनाकारों को उनके अधिकारों के
जाने वाले कदमों के साथ सात उद्देश्यों को विस्तृत रूप संरक्षण और प्रवर्तन के उपायों के बारे में जागरूक करने
से ‍उल्लिखित किया गया है। उद्देश्यों का संक्षेप में उल्ले ख की आवश्यकता है।
नीचे किया गया है।
उद्दे श्य 7- मानव पूंजी विकास (Objective 7 Human
उद्दे श्य 1- IPR जागरूकता (Objective 1 IPR Awareness): Capital Development)

• आउटरीच और प्रचार - समाज के सभी वर्गों के बीच IPR • आर्थि क विकास के लिए IPR की पूरी क्षमता का उपयोग
के आर्थि क, सामाजिक और सांस्कृतिक लाभों के बारे में करने के लिए IPR में शिक्षण, प्रशिक्षण, अनुसंधान और
जन जागरूकता पैदा करना। कौशल निर्माण के लिए मानव संसाधन, संस्थानों और
क्षमताओ ं को मजबूत और विस्तारित करना, IPR पेशेवरों
उद्दे श्य 2- IPR का सृजन (Objective 2 Generation of एवं नीति और कानून जैसे क्षेत्रों के विशेषज्ञों के एक बढ़ते
IPRs): पूल को विकसित करना आवश्यक है।

• जबकि उद्योग और आंतरिक व्यापार संवर्धन विभाग


• IPR की पीढ़ी को प्रोत्साहित करने के लिए भारत में
(DIPP) भारत में IPR के कार्यान्वयन, मार्गदर्शन और
अनुसंधान एवं विकास संस्थानों, उद्यमों, विश्वविद्यालयों
भविष्य के विकास के समन्वय, और निगरानी के लिए
और तकनीकी संस्थानों में फैले वैज्ञानिक और तकनीकी
नोडल बिं दु होगा, रूपरेखा को अपने निर्धारित कार्य क्षेत्र में
प्रतिभा का एक बड़ा प्रतिभा पूल है। प्रचुर मात्रा में ज्ञान
लागू करने की जिम्मेदारी संबंधित मंत्रालयों / विभागों के
संसाधनों का दोहन करने और आईपी परिसंपत्तियों के
पास रहेगी। राज्य सरकारों सहित सार्वजनिक और निजी
निर्माण को प्रोत्साहित करने की आवश्यकता है।
क्षेत्र के संस्थान और अन्य हितधारक भी कार्यान्वयन
उद्दे श्य 3- कानूनी और विधायी ढांचा (Objective 3 Legal प्रक्रिया में शामिल होंगे।
and Legislative Framework):

• मजबूत और प्रभावी IPR कानून बनाना, जो अधिक पेटेंट सुविधा के लिए योजनाएं , नीतियां और कार्यक्रम
महत्वपूर्ण जनहित के साथ अधिकार मालिकों के हितों (Schemes, Policies and Program for Patent
को संतुलित करता है। Facilitation)

उद्दे श्य 4 - प्रशासन और प्रबंधन (Objective 4

115
(पीएफसी) की स्थापना की है, ताकि सरकारी संगठन,
डार्क मैटर (DARK MATTER) केंद्र और राज्य विश्वविद्यालय से आविष्कारकों को IPR की
1903 में फ़्रिट्ज़ ज़्विकी ने देखा कि कई आकाशगंगाएँ सै- रक्षा करने में सहायता प्रदान की जा सके।
द्धांतिक गणनाओ ं की तुलना में तेज़ी से आगे बढ़ रही थीं। • TIFAC विज्ञान और प्रौद्योगिकी विभाग के तहत एक
इसका अर्थ यह हुआ कि उन आकाशगंगाओ ं के केंद्र की स्वायत्त संगठन है, जो भारत में उभरते तकनीकी क्षेत्रों
ओर कुछ रहस्यमयी गुरुत्वाकर्षण खिं चाव था। इस तरह के में अत्याधुनिक प्रौद्योगिकी तक पहुँचने और प्रौद्योगिकी
खिं चाव को दूर करने के लिए आवश्यक मैटर की मात्रा प्रे- विकसित करने के लिए एक प्रौद्योगिकी दृष्टि तैयार
क्षित मैटर से कहीं अधिक है। यह अतिरिक्त मैटर जो अदृश्य करने के लिए अनिवार्य है।
और अनिर्धारित है, को डार्क मैटर कहा गया है।

धीरे-धीरे कई खगोलविदों ने डार्क मैटर पर शोध करना शुरू


कर दिया। यह तब था जब एं ड्रोमेडा आकाशगंगा को अपेक्षा 3. बौद्धिक संपदा सुविधा केंद्र (एनआरडीसी) (Intellectual
से अधिक तेजी से आगे बढ़ते हुए देखा गया था और डार्क Property Facilitation Centre, NRDC)
मैटर खगोलीय अनुसंधान का केंद्र बन गया। • एनआरडीसी में बौद्धिक संपदा सुविधा केंद्र (आईपीएफसी)
इसे अभी तक प्रत्यक्ष रूप से नहीं देखा गया है। यह पदार्थ के एनआरडीसी और एमएसएमई की एक संयुक्त पहल है
साथ परस्पर क्रिया नहीं करता है और प्रकाश और विद्युत चु- जिसका उद्देश्य भारत में उद्यमियों और एमएसएमई के
म्बकीय विकिरण के अन्य रूपों के लिए पूरी तरह से अदृश्य बीच जागरूकता पैदा करना है।
है जिससे इसका पता लगाना असंभव हो जाता है।

गुरुत्वाकर्षण लें सिं ग के रूप में जानी जाने वाली इस घटना


4. जैव प्रौद्योगिकी पेटेंट सुविधा प्रकोष्ठ (जैव प्रौद्योगिकी
में दूर की आकाशगंगाओ ं से प्रकाश विकृत और डार्क मैटर
विभाग) (Biotechnology Patent Facilitation Cell,
के अदृश्य बादलों द्वारा विकृत और आवर्धि त हो जाता है। Department of Biotechnology)
स्रोत: नासा
• जैव प्रौद्योगिकी विभाग ने वैज्ञानिकों और शोधकर्ताओ ं
के बीच IPR के बारे में जागरूकता पैदा करने के लिए
जागरूकता सह सुविधा तंत्र प्रदान करने के लिए जैव
केंद्र सरकार की योजनाएं और कार्यक्रम (Central प्रौद्योगिकी पेटेंट सुविधा सेल (बीपीएफसी) की स्थापना
Government Schemes and Programmes) की।

1. स्टार्ट-अप के बौद्धिक संपदा संरक्षण की सुविधा के


लिए योजना (Scheme for Facilitating Start-Ups
Intellectual Property Protection) 5. बौद्धिक संपदा सुविधा केंद्र (रक्षा उत्पादन विभाग)
(Intellectual Property Facilitation Centre,
• केंद्र सरकार स्टार्ट-अप्स के बौद्धिक संपदा अधिकारों की Department of Defence Production)
रक्षा और बढ़ावा देने और इस तरह उनमें नवाचार और
• रक्षा उत्पादन विभाग (डीडीपी) की स्थापना रक्षा के लिए
रचनात्मकता को प्रोत्साहित करने की दृष्टि से स्टार्ट-
आवश्यक हथियारों, प्रणालियों, प्लेटफार्मों और उपकरणों
अप्स बौद्धिक संपदा संरक्षण (SIPP) की सुविधा के लिए
के उत्पादन के लिए एक व्यापक उत्पादन बुनियादी ढांचे
एक योजना ले कर आई। SIPP की परिकल्पना नवोन्मेषी
को विकसित करने के लिए की गई थी।
और इच्छुक स्टार्ट-अप के पेटेंट, ट्रे डमार्क और डिजाइन के
संरक्षण को सुविधाजनक बनाने के लिए की गई है। • आईपीएफसी बनाने के उद्देश्य:

• योजना के तहत स्टार्ट-अप को सुविधा के लिए पंजीकृत » आईपीआर के बारे में सामान्य सलाह,
सूत्रधार को शुल्क का भुगतान नहीं करना पड़ता है, केंद्र
» आईपी सुरक्षा, आईपी जागरूकता और प्रशिक्षण, परामर्श
सरकार इसकी प्रतिपूर्ति करेगी।
और सलाहकार सेवाओ ं के रूप में सेवाएं प्रदान करना,

» पेटेंट खोजों, पेटेंट प्रारूपण, पेटेंट अभियोजन, आविष्कारों


2. पेटेंट सुविधा कार्यक्रम (Patent Facilitation के व्यावसायीकरण में सुविधा आदि के लिए सेवाएं प्रदान
Program, TIFAC) करना,

• विज्ञान और प्रौद्योगिकी विभाग के पेटेंट सुविधा कार्यक्रम » पेशेवर आईपी अटॉर्नी/आईपी सलाहकारों के एक पैनल
(पीएफपी) ने देश में IPR पर जागरूकता और प्रशिक्षण की पहचान करना और पेशेवर सेवाओ ं के लिए उनसे
कार्यक्रम बनाने के लिए TIFAC में पेटेंट सुविधा सेल परामर्श करने के लिए शुल्क संरचना तय करना।

116
राज्य सरकार की योजनाएं (State Government हिग्स बोसोन (Higgs Boson):
Schemes)
यह 2012 में जिनेवा, स्विट्जरलैं ड के पास CERN में बड़े
आंध्र प्रदेश (Andhra Pradesh) हैड्रॉन कोलाइडर (LHC) में ATLAS और CMS प्रयोगों द्वारा
खोजा गया एक उप-परमाणु कण है।
प्रौद्योगिकी और नवाचार सहायता केंद्र (Technology &
Innovation Support Centre, TISC) हिग्स बोसोन कण भौतिकी के मानक मॉडल में एक
प्राथमिक कण है, जो हिग्स क्षेत्र के क्वांटम उत्तेजना द्वारा
निर्मि त है, जो कण भौतिकी सिद्धांत के क्षेत्रों में से एक है।
गुजरात (Gujrat)
इसका नाम भौतिक विज्ञानी पीटर हिग्स के नाम पर रखा
बौद्धिक संपदा सुविधा केंद्र (Intellectual Property गया है, जिन्होंने 1964 में पांच अन्य वैज्ञानिकों के साथ हिग्स
Facilitation Centre) तंत्र को यह समझाने की कोशिश की थी कि कुछ कणों में
द्रव्यमान क्यों होता है। इस तंत्र के लिए आवश्यक है कि
एक स्पिन रहित कण जिसे बोसॉन के रूप में जाना जाता
हरियाणा (Haryana) है, हिग्स तंत्र सिद्धांत द्वारा वर्णि त गुणों के साथ मौजूद होना
चाहिए। इस कण को हिग्स बोसोन कहा गया।
हरियाणा उद्यमी और स्टार्टअप नीति, 2017 (Haryana
गुण (Properties):
Entrepreneur and Startup Policy, 2017)
मानक मॉडल में, हिग्स कण शून्य स्पिन के साथ एक
विशाल अदिश बोसॉन है।

इसका कोई विद्युत आवेश नहीं है, इसका कोई रंग आवेश
नहीं है।

यह भी बहुत अस्थिर है, अन्य कणों में लगभग तुरत


ं क्षय हो
रहा है।

117

@ITAKASHSINGH
विगत वर्षों के प्रश्न
(Previous Years
Questions)

118
मुख्य परीक्षा (Mains) • प्रो. सत्येंद्र नाथ बोस द्वारा कृत रचना ‘बोस आइं स्टीन
स्टैटिसटिक्स’ की चर्चा कीजिए और बताइए कि भौतिक
विज्ञान के क्षेत्र में इसने क्रांतिकारी परिवर्तन कैसे
• नैनो टे कनोलोजी से आप क्या समझते हैं और यह
किया? (150 शब्द, 10 अंक) (2018)
स्वास्थ्य क्षेत्र में किस प्रकार सहायक बनी हुई है? (150
शब्द, 10 अंक) (2020)

• हमारे देश में जैवप्रौद्योगिकी के क्षेत्र में इतनी सक्रियता


• विज्ञान हमारे जीवन से कितनी गहराई तक जुड़ा हुआ क्यों है? इस गतिविधि से बायोफार्मा के क्षेत्रों में लाभ
है? कृषि में विज्ञान आधारित तकनीक द्वारा उत्पन्न कैसे हुआ है? (250 शब्द, 15 अंक) (2018)
महत्वपूर्ण परिवर्तन क्या हैं? (150 शब्द, 10 अंक) (2020)

• क्या गतिशील ऊर्जा की आवश्यकताओ ं के साथ-साथ


• कोविड-19 वैश्विक महामारी विश्वभर में अभूतपूर्व भारत को अपनी नाभिकीय ऊर्जा योजना का विस्तार जारी
तबाही मचा चुकी है। तथापि, इस संकट पर विजय रखना चाहिए? नाभिकीय ऊर्जा से संबंधित तथ्यों और
पाने के लिए प्रद्यौगिकीय प्रगति का लाभ स्वेच्छा से आशंकाओ ं पर चर्चा कीजिए। (250 शब्द, 15 अंक)(2018)
लिया जा रहा है। इस महामारी के प्रबंधन के सहायतार्थ
प्रद्यौगिकी की खोज कैसे की गई, उसका एक
विवरण दीजिये। (250 शब्द, 15 अंक) (2020) • स्टेम कोशिका उपचार भारत में मेडिकल शर्तों की
व्यापक किस्मों के साथ-साथ ले किमिया, थैलेसीमिया,
क्षतिग्रस्त कॉर्नि या व जलन के विभिन्न घावों का इलाज
• पारंपरिक ऊर्जा उत्पादन की तुलना में सूर्य प्रकाश से कर लोकप्रियता प्राप्त कर रहा है। संक्षेप में वर्णन कीजिए
उत्पन्न होने वाली विद्युत ऊर्जा के लाभों का वर्णन कीजिए। कि स्टेम कोशिका उपचार क्या है और अन्य उपचारों से
इस उद्देश्य के लिए हमारी सरकार द्वारा किए गए प्रयास अलग इसके क्या लाभ हैं? (150 शब्द, 10 अंक) (2017)
कौन-से हैं? (250 शब्द, 15 अंक) (2020)

• भारत ने अंतरिक्ष के विशेष अज्ञात कार्यों सहित


• अपना अंतरिक्ष केंद्र करने को ले कर भारत की क्या चंद्रयान और मंगलयान कक्षित्र मिशन में उल्ले खनीय
योजना है और यह हमारे अंतरिक्ष कार्यक्रम के लिए किस कामयाबियाँ पाई हैं ले किन मानव युक्त अंतरिक्ष मिशनों
प्रकार लाभप्रद होगी? (150 शब्द, 10 अंक) (2019) के जोखिम में नहीं पड़ा। तकनीक और न्याय शास्त्र के
संबंध में विस्तारपूर्वक वर्णन कीजिए। (150 शब्द, 10
अंक) (2017)

• जैव प्रौद्योगिकी किसानों के जीवन-स्तर को बेहतर


बनाने में किस प्रकार सहायक हो सकती है? (250 शब्द,
15 अंक) (2019) • एले लोपैथी क्या है? संचित कृषि की प्रमुख फसल
प्रणाली में इसके योगदान की चर्चा कीजिए। (200
शब्द, 12.5 अंक) (2016)

• भारत में नाभिकीय विज्ञान की वृद्धि एवं विकास और


तकनीक पर वर्णन कीजिए। भारत में एक फास्ट
ब्रीडर रिएक्टर प्रोग्राम के क्या लाभ हैं? (250 शब्द, 15 • अंतरिक्ष विज्ञान एवं प्रौद्योगिकी के क्षेत्र में भारत की
अंक) (2019) उपलब्धियों पर चर्चा कीजिए। किस प्रकार इस तकनीक
के उपयोग से भारत के सामाजिक-आर्थि क विकास में
सहायता हुई है? (200 शब्द, 12.5 अंक) (2016)

• भारत सरकार किस प्रकार औषधि के परंपरागत ज्ञान


को औषधि निर्माता कंपनियों के एकस्व अधिकार से बचा
कर रही है? (250 शब्द, 15 अंक) (2019) • नैनो टे क्नोलॉजी 21वीं सदी की प्रमुख तकनीकों में
से एक क्यों है? नैनो विज्ञान एवं प्रौद्योगिकी व देश की
विकास प्रक्रिया में इसके उपयोग के विषय-क्षेत्र में इस पर
भारत सरकार के मिशन की मुख्य विशेषताओ ं का वर्णन
कीजिए। (200 शब्द, 12.5 अंक) (2016)

119
• ‘डिजिटल इं डिया’ योजना कृषि भूमि उत्पादकता और आय • भारत के विश्वविद्यालयों में वैज्ञानिक अनुसंधान का पतन
उन्नति में किस प्रकार किसानों की सहायता कर सकती हो रहा है क्योंकि विज्ञान क्षेत्र में जीविका इतनी आकर्षक
है? इसके लिए सरकार ने क्या कदम उठाए हैं? (200 शब्द, नहीं है जितनी कि व्यापार संचालन, अभियांत्रिकी
12.5 अंक) (2015) या प्रशासन में है और विश्वविद्यालय उपभोक्ता प्रधान
बनते जा रहे हैं। आलोचनात्मक टिप्पणी कीजिए। (200
शब्द, 12.5 अंक) (2014)
• हाल ही में उपकरण लागत में हुए आकस्मिक पतन
और सौर ऊर्जा की लागत में कौन-से तत्त्व उत्तरदायी
हो सकते हैं? इस प्रवृति के तापीय विद्युत उत्पादकों • वैश्विकृत दुनिया में बौद्धिक संपदा महत्तव रखते हैं और
और संबंधित उद्योगों के लिए क्या निहितार्थ हैं? मुकदमेबाजी़ का एक स्त्रोत हैं। कोपीराइट, पेटेंट और
(200 शब्द, 12.5 अंक) (2015) व्यापारिक भेद के बीच व्यापक रूप से अंतर स्पष्ट कीजिए।
(200 शब्द, 12.5 अंक) (2014)

• जीपीएस युग ‘मानक स्थिति प्रणाली’ व ‘सुरक्षा स्थिति


प्रणाली’ से आप क्या समझते हैं? महज सात उपग्रहों को • 2005 में उन परिस्थितियों को सामने लाते हुए जिन्होंने
नियोजित करने वाले महत्त्वकांक्षी आइआरएनएसएस भारतीय पेटेंट कानून, 1970 की धारा 3 (घ) में संशोधन
(IRNSS) से भारत को मिलने वाले लाभों पर चर्चा कीजिए। के लिए मजबूर किया, चर्चा कीजिए कि ‘ग्लिवेक’ के लिए
(200 शब्द, 12.5 अंक) (2015) नोवार्टि स के पेटेंट आवेदन को खारिज करने में सर्वोच्च
न्यायालय द्वारा अपने फैसले में इसका उपयोग कैसे किया
गया है। निर्णय के पक्ष और विपक्ष पर संक्षेप में चर्चा कीचिए।
(200 शब्द, 10 अंक) (2013)
• निषेधात्मक श्रम के वे कौन-से क्षेत्र हैं जिनका प्रबंध
रोबोटों के द्वारा स्थायी रूप से किया जा सकता है? उन
उपक्रमों की चर्चा कीजिए जो प्रमुख अनुसंधान संस्थानों
को लाभप्रद व वास्तविकता की नई पद्धति में शोध करने • फिक्स्ड-डोज़ ड्रग कोम्बिनेशन (FDCs) से आपका क्या
के लिए प्रेरित कर सकते हैं। (200 शब्द, 12.5 अंक) (2015) अभिप्राय है? इसके गुण और दोष पर चर्चा कीजिए। (200
शब्द, 10 अंक) (2013)

• सर्वरों की क्लाउड होस्टिं ग सुविधा व सुरक्षा के पहलु ओ ं


की तुलना में सरकारी कामकाज के लिए संस्थानिक • डिजिटल सिग्नेचर क्या है? इसके प्रमाणीकरण का क्या
अर्थ है? डिजिटल हस्ताक्षर की विभिन्न अन्तर्निहित मुख्य
मशीन आधारित होस्टिं ग पर चर्चा कीजिए। (200 शब्द,
विशेषताएँ बताइए। (100 शब्द, 5 अंक) (2013)
12.5 अंक) (2015)

• 3डी प्रिंटिंग तकनीक कैसे काम करती है? इस तकनीक


• भारत की ट्रेडिशनल नॉले ज डिजिटल लाइब्रेरी (TKDL)
से होने वाले लाभ और हानियों की सूची बनाइए। (100
जिसमें 20 लाख से अधिक औषधीय सूत्रीकरणों की
शब्द, 5 अंक) (2013)
डाटाबेस स्वरूपित सूचना है जो ग़लत पेटेंट के विरुद्ध
देश की लड़ाई में एक शक्तिशाली हथियार साबित होती
है। ओपन सोर्स लाइलें लनिं ग के तहत इस ड़ेटाबास को
सार्वजनिक रूप से उपलब्ध कराने के पक्ष-विपक्षों पर • एफआरपी (FRP) सम्मिश्रित पदार्थ क्या होता है? इसका
चर्चा कीजिए। (200 शब्द, 12.5 अंक) (2015) निर्माण कैसे किया जाता है? विमानन और मोटर-वाहन
उद्योगों में इसके उपयोग पर चर्चा कीजिए। (100 शब्द, 5
अंक) (2013)

• क्या भारत में डॉक्टर के निर्देशों के बिना एं टीबायोटिक


दवाओ ं की उपलब्धता और इनका अधिक उपयोग औषध
प्रतिरोधक रोगों को उभारने में सहयोगी हो सकता है?
निरीक्षण और नियंत्रण करने की उपलब्ध युक्तियाँ क्या हैं?
विभिन्न सम्बन्धित मुद्दों पर आलोचनात्मक चर्चा कीजिए।
(200 शब्द, 12.5 अंक) (2014)

120
प्रारंभिक परीक्षा (Prelims) 4. निम्नलिखित कथनों पर विचार कीजिए: (2020)

1. उन कोशिकाओ ं में आनुवंशिक परिवर्तन किए जा


1. कार्बन नैनोट्यूब के संदर्भ में निम्नलिखित कथनों पर सकते हैं जो एक संभावित माता-पिता के अंडे या
विचार कीजिए: (2020) शुक्राणु पैदा करते हैं।

1. इनका उपयोग मानव शरीर में दवाओ ं और प्रतिजनों 2. प्रारंभिक भ्रूण अवस्था में किसी व्यक्ति के जीनोम को
के वाहक के रूप में किया जा सकता है। जन्म से पहले संपादित किया जा सकता है।

2. इन्हें मानव शरीर के एक घायल हिस्से के लिए 3. मानव प्रेरित प्लु रिपोटें ट स्टेम सेल को सूअर के भ्रूण
कृत्रिम रक्त केशिकाओ ं में बनाया जा सकता है। में इं जेक्ट किया जा सकता है।

3. इनका उपयोग जैव रासायनिक सेंसर में किया जा उपरोक्त में से कौन-सा/से कथन सही है/हैं?
सकता है। (a) केवल 1
4. कार्बन नैनोट्यूब जैव-निम्नीकरणीय होते हैं। (b) केवल 2 और 3
उपरोक्त में से कौन-से कथन सही हैं? (c) केवल 2
(a) केवल 2, 3 और 4 (d) 1, 2 और 3
(b) केवल 1, 3 और 4

(c) केवल 1 और 2 5. भारत में न्यूमोकोकल कॉन्जुगेट वैक्सीन के प्रयोग की


(d) 1, 2, 3 और 4 क्या महत्ता है? (2020)

1. ये वैक्सीन निमोनिया के साथ-साथ मस्तिष्क ज्वर


व रक्तविषंणता के प्रति उपयोगी होती है।
2. निम्नलिखित गतिविधियों पर विचार कीजिए: (2020)
2. उन एं टीबायोटिक दवाओ ं पर अधीनता को घटाया जा
1. फसल के खेत पर कीटनाशकों का छिड़काव
सकता है जो औषध प्रतिरोधक जीवाणुओ ं के विपरीत
2. सक्रिय ज्वालामुखियों के क्रेटरों का निरीक्षण प्रभावशाली नहीं होती।
3. डीएनए विश्ले षण के लिए स्पाउटिंग व्हेल से सांस के 3. इन टीकों का कोई दुष्प्रभाव नहीं है और न इनसे कोई
नमूने एकत्र करना एलर्जी संबंधी दुष्प्रभाव होता।
प्रौद्योगिकी के वर्तमान स्तर पर ड्रोन का उपयोग करके निम्न कूट का प्रयोग कर सही उत्तर का चयन कीजिए:
उपरोक्त में से कौन-सी गतिविधि सफलतापूर्वक की जा
(a) केवल 1
सकती है?
(b) केवल 2 और 3
(a) केवल 1 और 2
(c) केवल 2
(b) केवल 2 और 3
(d) 1, 2 और 3
(c) केवल 1 और 3

(d) 1, 2 और 3
6. भारत में "पब्लिक की इं फ्रास्ट्रक्चर" की परिभाषा जिस
संदर्भ में प्रयोग की जाती है: (2020)
3. प्रयोग एक समबाहु त्रिभुज के आकार में उड़ान भरने
(a) डिजिटल सुरक्षा इन्फ्रास्ट्रक्चर
वाले अंतरिक्ष यान की तिकड़ी को नियोजित करेगा
जिसकी भुजाएँ दस लाख किलोमीटर लम्बी हैं जिसमें (b) खाद्य सुरक्षा इन्फ्रास्ट्रक्चर
यान के बीच ले ज़र चमकते हैं। प्रश्न में दिया गया प्रयोग (c) स्वास्थय सेवा और शिक्षा इं फ्रास्ट्रक्चर
संबंधित हैः (2020)
(d) दूरसंचार एवं परिवहन इन्फ्रास्ट्रक्चर
(a) वोयेजर-2

(b) न्यू होरिजन्स


7. पादप और जन्तु कोशिकाओ ं के बीच साधारण अंतर के
(c) लिसा पाथफाइं डर विषय में निम्नलिखित में से कौन-से कथन सही हैं? (2020)
(d) विकसित लिसा 1. पादप कोशिकाओ ं में सेल्यूलोज कोशिका भित्ति
होती हैं जबकि जन्तु कोशिकाओ ं में नहीं होती।

121
2. पादप कोशिकाओ ं में प्लाज़्मा झिल्ली नहीं होती 2. VLC को दूरमार ऑप्टिकल ताररहित संचार के रूप
इसके विपरीत जन्तु कोशिकाओ ं में होती है। में जाना जाता है

3. परिपक्व पादप कोशिका में एक बड़ी रिक्तिका होती 3. ब्लूटू थ की तुलना में VLC बड़ी मात्रा में तेज़ी से डाटा
है जबकि जन्तु कोशिका में छोटी-छोटी अनेक ट्रांसफर कर सकता है
रिक्तिकाएँ होती हैं।
4. VLC में विद्युतचुम्बकीय उत्सर्जन नहीं होता
निम्न कूट का प्रयोग कर सही उत्तर का चयन कीजिए:
निम्न कूट का प्रयोग कर सही उत्तर का चयन कीजिएः
(a) केवल 1 और 2
(a) केवल 1, 2 और 3
(b) केवल 2 और 2
(b) केवल 1, 2 और 4
(c) केवल 1 और 3
(c) केवल 1, 3 और 4
(d) 1, 2 और 3
(d) केवल 2, 3 और 4

8. मानव प्रजनन तकनीकी में अभिनव प्रगति के संदर्भ


11. सौर जल पंपों के संदर्भ में निम्नलिखित कथनों पर
में ‘‘प्राक्केंद्रिक स्थानांतरण’’ का प्रयोग किस लिये
विचार कीजिए: (2020)
किया जाता है? (2020)
1. सौर ऊर्जा का उपयोग सतह पंप चलाने के लिए
(a) इन विट्रो अंड के निषेचन के लिये दाता शुक्राणु का
किया जा सकता है, सबमर्सि बल पंपों के लिए नहीं
उपयोग
2. सौर ऊर्जा का उपयोग अपकेंद्री पंपों को चलाने के
(b) शुक्राणु उत्पन्न करने वाली कोशिकाओ ं का
लिए किया जा सकता है, पिस्टन वाले पंपों के लिए
आनुवंशिक रूपांतरण
नहीं।
(c) स्टेम कोशिकाओ ं का कार्यात्मक भ्रूणों में विकास
उपरोक्त में से कौन-सा/से कथन सही है/हैं?
(d) संतान में सूत्रकणिका वाले रोगों का निरोध
(a) केवल 1

(b) केवल 2
9. विकास की वर्तमान स्थिति के साथ-साथ कृत्रिम बुद्धिमत्ता
(c) दोनों 1 और 2
निम्नलिखित में से प्रभावी रूप से क्या कर सकती
है? (2020) (d) न तो 1, न ही 2

i. औद्योगिक इकाइयों में बिजली की खपत में गिरावट

ii. सार्थक लघुकथाओ ं व गीतों का सृजन 12. निम्नलिखित में से किसके माप/अनुमान
के लिए उपग्रह प्रतिबिं ब/रिमोट सेंसिंग
iii. रोग निदान
डेटा का उपयोग किया जाता है? (2019)
iv. टे क्स्ट-टू -स्पीच संवाद
1. एक विशिष्ट स्थान की वनस्पति में क्लोरोफिल
v. विद्युत ऊर्जा का बेतार संचरण अवयव

निम्न कूट का प्रयोग कर सही उत्तर का चयन कीजिएः 2. एक विशिष्ट स्थान से धान के खेतों से ग्रीनहाउस
गैस उत्सर्जन
(a) केवल 1, 2, 3 और 5
3. एक विशिष्ट स्थान की भूमि की सतह का तापमान
(b) केवल 1, 3 और 4
निम्न कूट का प्रयोग कर सही उत्तर का चयन कीजिए:
(c) केवल 2, 4 और 5
(a) केवल 1
(d) 1, 2, 3, 4 और 5
(b) केवल 2 और 3

(c) केवल 3
10. दृश्य प्रकाश संचार (VLC) तकनीक
के संदर्भ में निम्नलिखित में से (d) 1, 2 और 3
कौन-से कथन सही हैं? (2020)

1. VLC 375 से 380 एनएम तरंग-धैर्य विद्युतचुम्बकीय


13. निम्नलिखित पर विचार कीजिए: (2019)
स्पेक्ट्रम का उपयोग करती है
1. कार्बन मोनोऑक्साइड

122

@ITAKASHSINGH
2. मीथेन को अवशोषित करते हैं।

3. ओज़ोन (b) बादल पृथ्वी की विकिरण को परावर्ति त कर देते हैं।

4. सल्फर डाइऑक्साइड (c) बादल वाली रातों में पृथ्वी की सतह का तापमान
कम हो जाता है।
फसल/बायोमास के अवशेषों के जलने से वायुमंडल में
उपरोक्त में से क्या छोड़ा जाता है? (d) बादल बहती हवा को जमीनी स्तर पर विक्षेपित करते
हैं।
(a) केवल 1 और 2

(b) केवल 2, 3 और 4
17. संचार प्रौद्योगिकियों के संदर्भ में LTE
(c) केवल 1 और 4
(लॉन्ग-टर्म इवोल्यूशन) और VoLTE
(d) 1, 2, 3 और 4 (वॉयस ओवर लॉन्ग-टर्म इवोल्यूशन)
के बीच क्या अंतर है/हैं? (2019)

14. भारत में कार्बोफ्यूरन, मिथाइल पैराथियान, फोरेट और 1. आमतौर पर एलटीई (LTE) को 3जी और वीओएलटीई
ट्रायज़ोफोस के उपयोग को शंका की दृष्टि से देखा जाता (VoLTE) को अडवांस्ड 3जी के रूप में विपणित किया
है। इन रसायनों का उपयोग इस प्रकार किया जाता जाता है।
है: ( 2 0 1 9 ) 2. एलटीई (LTE) डेटा-ओनलि तकनीक है और
(a) कृषि में कीटनाशक वीओएलटीई (VoLTE) वोइस-ओनलि तकनीक है।

(b) संसाधित खाद्य पदार्थों में संरक्षक निम्न कूट का प्रयोग कर सही उत्तर का चयन कीजिए:

(c) फल पकाने वाले एजेंट (a) केवल 1

(d) सौंदर्य प्रसाधनों में मॉइस्चराइजिं ग एजेंट (b) केवल 2

(c) 1 और 2 दोनों

15. उपयोग के प्रस्तावों के संदर्भ में हाइड्रोजन-सम्पन्न CNG (d) न तो 1, न ही 2


(H-CNG) का प्रयोग सार्वजनिक परिवहन में बसों के
ईंधन के रूप में किया जाता है, निम्नलिखित कथनों पर
18. डेनिसोवन' शब्द का उल्ले ख कभी-कभी मीडिया में
विचार कीजिए: (2019)
निम्नलिखित के संदर्भ में किया जाता है: (2019)
1. एच-सीएनजी के उपयोग का मुख्य लाभ कार्बन
(a) एक प्रकार के डायनासोर के जीवाश्म
मोनोऑक्साइड उत्सर्जन का उन्मूलन है।
(b) एक प्रारंभिक मानव प्रजाति
2. ईंधन के रूप में एच-सीएनजी कार्बन डाइऑक्साइड
और हाइड्रोकार्बन उत्सर्जन को कम करता है। (c) उत्तर-पूर्व भारत में पाई जाने वाली एक गुफा प्रणाली

3. आयतन के अनुसार 1/5 भाग तक हाइड्रोजन को (d) भारतीय उपमहाद्वीप के इतिहास में एक भूवैज्ञानिक
सीएनजी से मिश्रित करके बसों के ईंधन के रूप में काल
उपयोग किया जा सकता है।

4. एच-सीएनजी ईंधन को सीएनजी से कम खर्चीला


19. हाल ही में हुए विज्ञान के विकास के संदर्भ में निम्नलिखित
बनाता है।
में से कौन-सा कथन सही नहीं है? (2019)
उपरोक्त में से कौन-सा/से कथन सही है/हैं?
(a) विभिन्न प्रजातियों की कोशिकाओ ं से लिए गए
(a) केवल 1 डीएनए के खंडों को जोड़कर कार्यात्मक गुणसूत्र
बनाए जा सकते हैं।
(b) केवल 2 और 3
(b) प्रयोगशालाओ ं में कृत्रिम क्रियात्मक डीएनए के
(c) केवल 4
टु कड़े बनाए जा सकते हैं।
(d) 1, 2, 3 और 4
(c) किसी जंतु कोशिका से निकाले गए डीएनए के एक
टु कड़े को प्रयोगशाला में जीवित कोशिका के बाहर
16. बादल वाली रात में ओस की बूंदें क्यों नहीं बनती हैं? (2019) रिप्लिकेट करने के लिए बनाया जा सकता है।

(a) बादल पृथ्वी की सतह से निकलने वाली विकिरणों (d) प्लाज्मा और जंतुओ ं से निकाली गई कोशिकाओ ं को

123
प्रयोगशाला पेट्री डिश में कोशिका विभाजन के लिए निम्न कूट का प्रयोग कर सही उत्तर का चयन कीजिए:
बनाया जा सकता है।
(a) 1, 2 और 4
20. निम्नलिखित कथनों पर विचार कीजिए: (2019)
(b) 2 और 3
एक डिजिटल हस्ताक्षर है
(c) 1 और 3
1. एक इले क्ट्रॉनिक रिकॉर्ड जो इसे जारी करने वाले
(d) केवल 1 और 4
प्रमाणीकरण प्राधिकारी की पहचान करता है

2. इं टरनेट पर सूचना या सर्वर तक पहुँचने के लिए


किसी व्यक्ति की पहचान के प्रमाण के रूप में उपयोग 23. हाल ही में वैज्ञानिकों ने पृथ्वी से अरबों प्रकाश
किया जाता है वर्ष दूर विशाल ब्लै कहोल के विलय को देखा। इस
पर्यवेक्षण का क्या महत्व है? (2019)
3. एक इले क्ट्रॉनिक रिकॉर्ड इले क्ट्रॉनिक दस्तावेज़ पर
हस्ताक्षर करने की विधि और यह सुनिश्चित करना (a) ‘हिग्स बोसोन कणों’ की खोज हुई।
कि मूल सामग्री अपरिवर्ति त है (b) ‘गुरुत्वाकर्षण तरंगों’ की खोज हुई।
उपरोक्त में से कौन-सा/से कथन सही है/हैं? (c) ‘वर्महोल’ के माध्यम से अंतर-गैलेक्टिक अंतरिक्ष
(a) केवल 1 यात्रा की संभावना की पुष्टि की गई।

(b) केवल 2 और 3 (d) इसने वैज्ञानिकों को ‘विलक्षणता’ समझने में सक्षम


बनाया।
(c) केवल 3

(d) 1, 2 और 3
24. भारत में माइक्रोबियल रोगाणुओ ं में बहु-दवा प्रतिरोध
की घटना के निम्नलिखित में कौन कौन-से कारण
21. वियरेबल प्रौद्योगिकी के संदर्भ में निम्नलिखित में से हैं? (2019)
कौन-सा/से कार्य वियरेबल उपकरणों द्वारा पूरा/पूरे
1. कुछ लोगों की आनुवंशिक प्रवृत्ति
किया/किए जाता/जाते है/हैं? (2019)
2. रोगों का उपचार करने के लिए एं टीबायोटिक दवाओ ं
1. किसी व्यक्ति की लोकेशन का पता
की गलत खुराक ले ना
2. किसी व्यक्ति की नींद की निगरानी
3. पशुधन खेती में एं टीबायोटिक दवाओ ं का उपयोग
3. सुनने में असक्षम व्यक्ति की सहायता
4. कुछ लोगों में एकाधिक दीर्घकालिक रोग
निम्न कूट का प्रयोग कर सही उत्तर का चयन कीजिए:
निम्न कूट का प्रयोग कर सही उत्तर का चयन कीजिए:
(a) केवल 1
(a) 1 और 2
(b) केवल 2 और 3
(b) केवल 2 और 3
(c) केवल 3
(c) 1, 3 और 4
(d) 1, 2 और 3
(d) 2, 3 और 4

22. ‘आरएनए इं टरफेरेंस (RNAI)’ प्रौद्योगिकी ने पिछले कुछ


25. Cas9 प्रोटीन क्या है जिसका उल्ले ख अक्सर
वर्षों में लोकप्रियता हासिल की है। क्यों? (2019)
समाचारों में किया जाता है? (2019)
1. इसका उपयोग जीन साइलें सिं ग थेरप
े ी को विकसित
(a) लक्षित जीन संपादन में प्रयुक्त एक आण्विक कैंची
करने में किया जाता है।
(b) रोगियों में रोगजनकों का सटीक पता लगाने में
2. इसका उपयोग कैंसर के उपचार को विकसित करने
उपयोग किया जाने वाला एक बायोसेंसर
के लिए किया जा सकता है।
(c) एक जीन जो पौधों को कीट प्रतिरोधी बनाता है
3. इसका उपयोग हार्मोन रिप्ले समेंट थेरप
े ी को
विकसित करने के लिए किया जा सकता है। (d) आनुवंशिक रूप से संशोधित फसलों में संश्लेषित
एक जड़ी-बूटी पदार्थ
4. इसका उपयोग ऐसे फसल पौधों को विकसित करने
के लिए किया जा सकता है जो वायरल पैथोजेंस के
लिए प्रतिरोधी हैं। 26. निम्नलिखित में से कौन-सा कथन सही नहीं है? (2019)

124
(a) हेपेटाइटिस बी विषाणु एचआईवी की तरह ही फैलता (d) कोई नहीं
है।
29. पहचान मंच 'आधार' स्पष्ट रूप से "एप्लिकेशन
(b) हेपेटाइटिस बी के विपरीत हेपेटाइटिस सी का कोई प्रोग्रामिं ग इं टरफेस (APIs)" प्रदान करता है। इसका क्या
टीका नहीं है। तात्पर्य है? (2018)

(c) विश्व स्तर पर हेपेटाइटिस बी और सी विषाणु से 1. इसे किसी भी इले क्ट्रॉनिक डिवाइस में इं टीग्रेट किया
संक्रमित लोगों की संख्या एचआईवी से संक्रमित जा सकता है।
लोगों की तुलना में कई गुणा अधिक है।
2. आईरिस के उपयोग से ऑनलाइन प्रमाणीकरण
(d) हेपेटाइटिस बी और सी विषाणु से संक्रमित होने वाले संभव है।
कुछ लोगों में कई वर्षों तक लक्षण दिखाई नहीं देते।
उपरोक्त में से कौन-सा/से कथन सही है/हैं?

(a) केवल 1
27. भारत के उपग्रह प्रक्षेपण वाहनों के संदर्भ में
(b) केवल 2
निम्नलिखित कथनों पर विचार कीजिए: (2018)
(c) 1 और 2 दोनों
1. पीएसएलवी (PSLVs) पृथ्वी संसाधनों की निगरानी
रखने के लिए उपग्रहों को लॉन्च करते हैं जबकि (d) न तो 1, न ही 2
जीएसएलवी (GSLVs) मुख्य रूप से संचार उपग्रहों
को लॉन्च करने के लिए बनाए गए हैं।
30. निम्नलिखित में से किसमें “3D प्रिंटिंग” की एप्लिकेशंस
2. पीएसएलवी (PSLVs) द्वारा लॉन्च उपग्रह आकाश में हैं? (2018)
उसी स्थिति में स्थायी रूप से स्थिर प्रतीत होते हैं जैसा
1. कन्फेक्शनरी वस्तुओ ं की तैयारी
कि पृथ्वी पर किसी विशेष स्थान से देखा जाता है।
2. बायोनिक कानों का निर्माण
3. जीएसएलवी एमके III (GSLV Mk III) चार चरणों
वाला एक प्रक्षेपण यान है जिसमें पहले और तीसरे 3. मोटर वाहन उद्योग
चरण में ठोस रॉकेट मोटर्स का उपयोग किया जाता 4. पुनर्निर्माण सर्जरी
है; और दूसरे और चौथे चरण में तरल रॉकेट इं जनों
5. डाटा प्रोसेसिंग प्रौद्योगिकियाँ
का उपयोग किया जाता है।
निम्न कूट का प्रयोग कर सही उत्तर का चयन कीजिए:
उपरोक्त में से कौन-सा/से कथन सही है/हैं?
(a) केवल 1, 3 और 4
(a) केवल 1
(b) केवल 2, 3 और 5
(b) 2 और 3
(c) केवल 1 और 4
(c) 1 और 2
(d) 1, 2, 3, 4 और 5
(d) केवल 3

31. कभी-कभी समाचारों में प्रयुक्त की जाने वाली ‘वाना क्राई,


28. भारतीय क्षेत्रीय नेविगेशन सैटेलाइट सिस्टम (IRNSS) के
पेट्या और इटरनल ब्लू ’ अभिव्यक्तियाँ संबंधित हैं: (2018)
संदर्भ में निम्नलिखित कथनों पर विचार कीजिए: (2018)
(a) एक्सो-ग्रह से
1. IRNSS के भू-स्थिर में तीन उपग्रह और भू-
समकालिक कक्षाओ ं में चार उपग्रह हैं। (b) क्रिप्टो-मुद्रा से

2. IRNSS पूरे भारत को ढकता है और इसकी सीमाओ ं (c) साइबर हमले से


से लगभग 5500 वर्ग कि.मी. तक फैला है। (d) मिनी उपग्रह से
3. 2019 के मध्य भारत के पास पूर्ण वैश्विक कवरेज के
साथ अपनी उपग्रह नेविगेशन प्रणाली होगी।
32. निम्नलिखित में से किस क्षेत्र में जीपीएस
उपरोक्त में से कौन-सा/से कथन सही है/हैं? तकनीक का उपयोग किया जा सकता है? (2018)
(a) केवल 1 1. मोबाइल फोन संचालन
(b) केवल 1 और 2 2. बैंकिंग संचालन
(c) केवल 2 और 3 3. पावर ग्रिड नियंत्रण

125
निम्न कूट का प्रयोग कर सही उत्तर का चयन कीजिए: सरसों) के संदर्भ में निम्नलिखित कथनों पर विचार
कीजिए: (2018)
(a) केवल 1
1. जीएम सरसों में एक मृदा जीवाणु के जीन होते हैं जो
(b) केवल 2 और 3
पौधे को कीटों की विस्तृत विविधता के लिए कीट-
(c) केवल 1 और 3 प्रतिरोधी गुण देते हैं।
(d) 1, 2 और 3 2. जीएम सरसों में ऐसे जीन होते हैं जो पौधे को पार-
परागण और संकरण की अनुमति देते हैं।

33. सुबह जब आपके स्मार्टफोन का अलार्म बजता है, तो आप 3. जीएम सरसों को आईएआरआई और पंजाब कृषि
उठते हैं और अलार्म को टै प करके बंद कर देते हैं जिससे विश्वविद्यालय द्वारा संयुक्त रूप से विकसित किया
आपका गीजर स्वचालित हो जाता है। आपके बाथरूम में गया है।
स्मार्ट मिरर दिन के मौसम और आपके ओवरहेड टैं क में उपरोक्त में से कौन-सा/से कथन सही है/हैं?
जल के स्तर को दर्शाता है। जब आप नाश्ता बनाने के
(a) केवल 1 और 3
लिए अपने रेफ्रिजरेटर से कुछ सामान ले ते हैं तो यह
उसमें स्टॉक की कमी को पहचानता है और ताजा वस्तुओ ं (b) केवल 2
की पूर्ति के लिए एक आदेश देता है। जब आप अपने घर
(c) केवल 2 और 3
से बाहर निकलकर दरवाजा बंद करते हैं तो सभी लाइट,
पंखे, गीजर और ए.सी. मशीनें अपने आप बंद हो जाती (d) 1, 2 और 3
हैं। कार्यालय जाते हुए आपकी कार आगे यातायात की
भीड़ के बारे में आपको संकेत देती है और एक वैकल्पिक
36. निम्नलिखित पर विचार कीजिए: (1) पक्षी (2) उड़ती धूल
मार्ग सुझाती है और यदि आपको किसी मीटिंग के लिए
(3) वर्षा (४) बहती हवा।
देर हो जाती है तो यह तदनुसार आपके कार्यालय में एक
संदेश भेजती देती है। उभरती हुई संचार प्रौद्योगिकियों उपरोक्त में से कौन/क्या पादप रोग फैलाता है? (2018)
के संदर्भ में निम्नलिखित में से कौन-सा शब्द (a) केवल 1 और 3
“उपरोक्त परिदृश्य पर सबसे सटीक बैठता है?” (2018)
(b) केवल 3 और 4
(a) बॉर्डर गेटवे प्रोटोकॉल
(c) केवल 1, 2 और 4
(b) इं टरनेट ऑफ थिं ग्स
(d) 1, 2, 3 और 4
(c) इं टरनेट प्रोटोकॉल

(d) वर्चुअल प्राइवेट नेटवर्क


37. निम्नलिखित कथनों पर विचार कीजिए: (2018)

1. पृथ्वी का चुंबकीय क्षेत्र कुछ हर सौ हजार वर्षों में


34. मरुस्थलीय क्षेत्रों में जल हानि को रोकने उलट गया है।
के लिए निम्नलिखित में से कौन-
2. जब पृथ्वी का निर्माण 4 हजार मिलियन से अधिक
सा/से पत्ती परिवर्तन होता/होते है/हैं?
वर्ष पहले हुआ उस समय 54% ऑक्सीजन और कोई
(2018)
कार्बन डाइऑक्साइड नहीं थी।
1. कठोर और मोमी पत्ते
3. जब जीवित प्राणियों की उत्पत्ति हुई तो उन्होंने पृथ्वी
2. छोटे पत्ते के प्रारंभिक वातावरण को बदल दिया।
3. पत्तियों की जगह काँटे उपरोक्त में से कौन-सा/से कथन सही है/हैं?
निम्न कूट का प्रयोग कर सही उत्तर का चयन कीजिए: (a) केवल 1
(a) केवल 2 और 3 (b) केवल 2 और 3
(b) केवल 2 (c) केवल 1 और 3
(c) केवल 3 (d) 1, 2 और 3
(d) 1, 2 और 3

38. कभी-कभार समाचार बनाम प्रसंग/विषय में देखे जाने


35. भारत में विकसित आनुवंशिक संशोधित सरसों (जीएम वाली अभिव्यक्तियों के निम्नलिखित युग्मों पर विचार

126
कीजिए: (2018) 2. जीका विषाणु रोग का यौन संचरण संभव है।

1. बेले II प्रयोग: आर्टिफिशियल इं टे लिजेंस उपरोक्त में से कौन-सा/से कथन सही है/हैं?

2. ब्लॉकचैन तकनीक: डिजिटल/क्रिप्टोकरेंसी (a) केवल 1

3. सीआरआईएसपीआर (CRISPR) H Cas9: पार्टि कल (b) केवल 2


फिजिक्स
(c) 1 और 2 दोनों
उपरोक्त में से कौन-सा/से युग्म सही सुमेलित है/हैं?
(d) न तो 1, न ही 2
(a) केवल 1 और 3

(b) केवल 2
42. कई उपकरणों में डिजिटल डिस्प्ले बनाने के लिए ऑर्गेनिक
(c) केवल 2 और 3 लाइट एमिटिंग डायोड (OLEDs) का उपयोग किया जाता
है। लिक्विड क्रिस्टल डिस्प्ले पर OLED डिस्प्ले के क्या
(d) 1, 2 और 3
फायदे हैं? (2017)

1. OLED डिस्प्ले को लचीले प्लास्टिक सबस्ट्रे ट्स पर


39. निम्नलिखित परिघटनाओ ं पर विचार कीजिए: (2018) गढ़ा जा सकता है।
1. प्रकाश गुरुत्वाकर्षण से प्रभावित होता है। 2. OLEDs का उपयोग कर कपड़ों में एम्बेडेड रोल-अप
2. ब्रह्मांड का विस्तार लगातार हो रहा है। डिस्प्ले बनाई जा सकती हैं।

3. पदार्थ अपने आस-पास के अंतरिक्ष-समय को विकृत 3. OLEDs के उपयोग से पारदर्शी डिस्प्ले बनाना संभव
करता है। हैं।

उपरोक्त में से कौन-सी अल्बर्ट आइं स्टीन के सापेक्षता के निम्न कूट का प्रयोग कर सही उत्तर चयन कीजिए:
सामान्य सिद्धांत की भविष्यवाणी/भविष्यवाणियाँ है/हैं (a) केवल 1 और 3
जिसकी/जिनकी चर्चा अक्सर मीडिया में होती है?
(b) केवल 2
(a) केवल 1 और 2
(c) 1, 2 और 3
(b) केवल 3
(d) उपरोक्त में से कोई भी कथन सही नहीं है
(c) केवल 1 और 3

(d) 1, 2 और 3
43. भारत में निम्नलिखित में से किसके लिए साइबर सुरक्षा
घटनाओ ं पर रिपोर्ट करना कानूनी रूप से अनिवार्य
40. निम्नलिखित युग्मों पर विचार कीजिए: (2017) है? (2017)

आमतौर पर प्रयुक्त होने वाली सामग्री बनाम उनमें पाए 1. सेवा प्रदाता
जाने वाले अनिष्ट या विवादास्पद रसायन 2. डेटा सेंटर
1. लिपस्टिक: ले ड 3. कॉरपोरेट बॉडी
2. शीतल पेय: ब्रोमिनेटेड वनस्पति तेल निम्न कूट का प्रयोग कर सही उत्तर चयन कीजिए:
3. चीनी फास्ट फूड: मोनोसोडियम ग्लूटामेट (a) केवल 1
उपरोक्त में से कौन-सा/से युग्म सही सुमेलित है/हैं? (b) केवल 1 और 2
(a) केवल 1 (c) केवल 3
(b) केवल 2 और 3 (d) 1, 2 और 3
(c) केवल 1 और 3

(d) 1, 2 और 3 44. विकसित ले जर इं टरफेरोमीटर स्पेस एं टीना (eLISA)'


परियोजना का क्या उद्देश्य है? (2017)

41. निम्नलिखित कथनों पर विचार कीजिए: (2017) (a) न्यूट्रिनो का पता लगाना

1. उष्णकटिबंधीय क्षेत्रों में जीका विषाणु रोग डेंगू (b) गुरुत्वाकर्षण तरंगों का पता लगाना
फैलाने वाले मच्छर से ही फैलता है।

127
(c) मिसाइल रक्षा प्रणाली की परिणामिता का पता 49. भारत द्वारा शुरू की गई खगोलीय वेधशाला ‘एस्ट्रोसैट’ के
लगाना संदर्भ में निम्नलिखित में से कौन सा/से कथन सही है/
हैं? (2016)
(d) हमारी संचार प्रणालियों पर सौर फ्ले यर्स के प्रभाव
का अध्ययन करना 1. संयुक्त राज्य अमेरिका और रूस के अलावा भारत
एकमात्र ऐसा देश है जिसने अंतरिक्ष में एक समान
वेधशाला लॉन्च की है।
45. इवेंट होरिज़न, विलक्षणता, स्ट्रिंग सिद्धांत और मानक
2. एस्ट्रोसैट 2000 किलो का एक उपग्रह है जिसे पृथ्वी
मॉडल अभिव्यक्तियाँ कभी-कभी समाचारों में किस के
की सतह से 1650 कि.मी. ऊपर कक्षा में रखा गया है।
संदर्भ में देखी जाती हैं: (2017)
निम्न कूट का प्रयोग कर सही उत्तर का चयन कीजिए:
(a) ब्रह्मांड का पर्यवेक्षण और तालमेल
(a) केवल 1
(b) सौर और चंद्र ग्रहण का अध्ययन
(b) केवल 2
(c) उपग्रहों को पृथ्वी की कक्षा में स्थापित करना
(c) 1 और 2 दोनों
(d) पृथ्वी पर जीवित प्राणियों की उत्पत्ति और विकास
(d) न तो 1, न ही 2

46. कभी-कभी समाचारों में देखे जाने वाला 'प्रोजेक्ट लू न'


संबंधित है: (2016) 50. भारत ‘अंतर्राष्ट्रीय थर्मोन्यूक्लियर प्रायोगिक रिएक्टर’
का एक महत्वपूर्ण सदस्य है। यदि यह प्रयोग सफल
(a) अपशिष्ट प्रबंधन प्रौद्योगिकी
होता है, तो भारत के लिए तत्काल लाभ क्या है? (2016)
(b) वायरले स संचार प्रौद्योगिकी
(a) यह बिजली उत्पादन के लिए यूरनि
े यम के स्थान पर
(c) सौर ऊर्जा उत्पादन प्रौद्योगिकी थोरियम का उपयोग कर सकता है
(d) जल संरक्षण प्रौद्योगिकी (b) यह उपग्रह नेविगेशन में एक वैश्विक भूमिका प्राप्त
कर सकता है

47. हाल ही में चर्चि त 'Li-Fi' के संदर्भ में, निम्नलिखित में से कौन- (c) यह बिजली उत्पादन में अपने विखंडन रिएक्टरों की
सा/से कथन सही है/हैं? (2016) दक्षता में काफी सुधार कर सकता है

1. यह हाई-स्पीड डेटा ट्रांसमिशन के लिए प्रकाश को (d) यह बिजली उत्पादन में फ्यूजन रिएक्टरों का निर्माण
माध्यम के रूप में उपयोग करता है। कर सकता है

2. यह एक वायरले स तकनीक है और 'वाईफाई' से कई


गुना तेज है। 51. निम्नलिखित कथनों पर विचार कीजिए: (2016)
निम्न कूट का प्रयोग कर सही उत्तर का चयन कीजिए: इसरो द्वारा लॉन्च किए गए मंगलयान
(a) केवल 1 1. को मार्स ऑर्बिटर मिशन भी कहा जाता है।
(b) केवल 2 2. ने भारत को संयुक्त राज्य अमेरिका के बाद मंगल
(c) 1 और 2 दोनों ग्रह की कक्षा में अंतरिक्ष यान रखने वाला दूसरा देश
बना दिया।
(d) न तो 1, न ही 2
3. ने भारत को पहले ही प्रयास में अपना अंतरिक्ष यान
बनाकर मंगल की परिक्रमा करने वाला एकमात्र
48. हाल ही की चर्चा में रहा 'ग्रीस्ड लाइटनिं ग-10 (GL-10)' क्या सफल देश बना दिया।
है? (2016)
उपरोक्त में से कौन-सा/से कथन सही है/हैं?
(a) नासा द्वारा परीक्षित किया गया इले क्ट्रिक विमान
(a) केवल 1
(b) जापान द्वारा डिजाइन किया गया सौर ऊर्जा से चलने
(b) केवल 2 और 3
वाला दो सीटों वाला विमान
(c) केवल 1 और 3
(c) चीन द्वारा शुरू की गई अंतरिक्ष वेधशाला
(d) 1, 2 और 3
(d) इसरो द्वारा बनाया गया पुन: प्रयोज्य रॉकेट

128
52. जैव सूचना विज्ञान के विकास के संदर्भ में ‘ट्रांसस्क्रिप्टोम’ (a) केवल 1 और 2
अभिव्यक्ति जिसे अक्सर समाचारों में सुनते है, संबंधित
(b) केवल 3
है: (2016)
(c) केवल 1 और 3
(a) जीनोम संपादन में प्रयुक्त एं जाइमों की एक श्रृंखला
(d) 1, 2 और 3
(b) एक जीव द्वारा व्यक्त एमआरएनए अणुओ ं की पूरी
श्रृंखला

(c) जीन अभिव्यक्ति के रचनातंत्र का विवरण 56. स्वास्थ्य क्षेत्र में नैनो प्रौद्योगिकी के उपयोग के संदर्भ में,
निम्नलिखित में से कौन-सा/से कथन सही है/हैं? (2015)
(d) कोशिकाओ ं में होने वाले आनुवंशिक परिवर्तन का
एक तंत्र 1. नैनोटे क्नोलॉजी द्वारा लक्षित दवा वितरण संभव हो
गया है

2. नैनोटे क्नोलॉजी जीन थेरप


े ी में काफी हद तक
53. भारत सरकार द्वारा शुरू किया गया 'मिशन इं द्रधनुष' संबधं ित
योगदान कर सकती है
है: (2016)
निम्न कूट का प्रयोग कर सही उत्तर का चयन कीजिए:
(a) बच्चों और गर्भवती महिलाओ ं का प्रतिरक्षीकरण
(a) केवल 1
(b) देश भर में स्मार्ट शहरों का निर्माण
(b) केवल 2
(c) बाहरी अंतरिक्ष में पृथ्वी जैसे ग्रहों के लिए भारत की
अपनी खोज (c) 1 और 2 दोनों

(d) नई शिक्षा नीति (d) न तो 1, न ही 2

54. खाद्य सुरक्षा और मानक (पैकेजिं ग और ले बलिंग) 57. ईंधन के रूप में कोयले का उपयोग करने वाले बिजली
विनियम, 2011 के अनुसार भारत में पहले से पैक की गई संयंत्रों द्वारा उत्पादित 'फ्लाई ऐश' के संदर्भ में निम्नलिखित
वस्तुओ ं के संदर्भ में निर्माता को निम्नलिखित में से कौन- में से कौन सा/से कथन सही है/हैं? (2015)
सी जानकारी मुख्य ले बल पर रखनी अनिवार्य है? (2016) 1. भवन निर्माण के लिए ईंटों के उत्पादन में फ्लाई ऐश
1. एडिटिव्स सहित अवयवों की सूची का उपयोग किया जा सकता है

2. पोषण संबंधी जानकारी 2. फ्लाई ऐश का उपयोग कंक्रीट की कुछ पोर्टलैं ड


सीमेंट सामग्री के स्थान पर किया जा सकता है
3. किसी एलर्जी की प्रतिक्रिया की संभावना के बारे में
चिकित्सा व्यवसाय द्वारा की गई सिफारिशें, यदि हों 3. फ्लाई ऐश सिलिकॉन डाइऑक्साइड और कैल्शियम
तो। ऑक्साइड से बना है और इसमें कोई विषैले तत्व नहीं
हैं।
4. शाकाहारी/माँसाहारी
निम्न कूट का प्रयोग कर सही उत्तर का चयन कीजिए:
निम्न कूट का प्रयोग कर सही उत्तर का चयन कीजिएः
(a) 1 और 2
(a) 1, 2 और 3
(b) केवल 2
(b) 2, 3 और 4
(c) 1 और 3
(c) 1, 2 और 4
(d) केवल 3
(d) केवल 1 और 4

58. हाल ही में इबोला विषाणु के प्रकोप के लिए निम्नलिखित


55. निम्नलिखित में से कौन-सा/से कथन सही है/हैं? (2016)
में से अक्सर किसका उल्ले ख समाचारों में किया गया
विषाणु संक्रमित कर सकते हैं था? (2015)

1. बैक्टीरिया (a) सीरिया और जॉर्डन

2. कवक (b) गिनी, सिएरा लियोन और लाइबेरिया

3. पौधे (c) फिलीपींस और पापुआ न्यू गिनी

निम्न कूट का प्रयोग कर सही उत्तर का चयन कीजिए:

129
(d) जमैका, हैती और सूरीनाम (a) भारतीय रक्षा में शामिल स्वदेशी रूप से विकसित
एक रडार प्रणाली

(b) हिं द महासागर रिम के देशों को सेवाएं प्रदान करने


59. आधुनिक वैज्ञानिक अनुसंधान के संदर्भ में, हाल ही में चर्चि त
के लिए भारत का उपग्रह
दक्षिणी ध्रुव पर स्थित एक कण संसूचक 'आइसक्यूब' के
बारे में निम्नलिखित कथनों पर विचार कीजिए: (2015) (c) अंटार्कटिक क्षेत्र में भारत द्वारा स्थापित एक वैज्ञानिक
संस्थान
1. यह दुनिया का सबसे बड़ा न्यूट्रिनो डिटे क्टर है जो
एक घन किलोमीटर तक बर्फ समेटे हुए है। (d) भारत की अन्तर्जलीय प्रयोगशाला में वैज्ञानिक रूप
से आर्कटिक क्षेत्र का अध्ययन
2. यह डार्क मैटर खोजने के लिए एक शक्तिशाली
टे लीस्कोप है।

3. यह बर्फ में बहुत नीचे तक दबाया गया है। 63. H1N1 विषाणु का उल्ले ख कभी-कभी
समाचारों में निम्नलिखित में से किस
ऊपर दिए गए कथनों में से कौन-सा/से सही है/हैं?
रोग के संदर्भ में किया जाता है? (2015)
(a) केवल 1
(a) एड्स
(b) केवल 2 और 3
(b) बर्ड फ्लू
(c) केवल 1 और 3
(c) डेंगू
(d) केवल 1, 2 और 3
(d) स्वाइन फ्लू

60. 'नियर फील्ड कम्युनिकेशन (NFC) टे क्नोलॉजी' के संदर्भ


64. भारतीय रेलवे द्वारा उपयोग किए जाने वाले जैव-
में निम्नलिखित में से कौन-सा/से कथन सही है/हैं? (2015)
शौचालयों के संदर्भ में निम्नलिखित कथनों पर
1. यह एक संपर्क रहित संचार तकनीक है जो विद्युत विचार कीजिए: (2015)
चुम्बकीय रेडियो क्षेत्रों का उपयोग करती है
1. जैव-शौचालयों में मानव अपशिष्ट का अपघटन एक
2. एनएफसी को उन उपकरणों के उपयोग के लिए कवक इनोकुलम द्वारा किया जाता है।
बनाया गया है जिनके बीच एक मीटर की भी दूरी
2. इस अपघटन में अमोनिया और जल वाष्प ही एकमात्र
हो सकती है
अंतिम उत्पाद हैं जो वायुमंडल में छोड़े जाते हैं।
3. एनएफसी संवेदनशील जानकारी भेजते समय
उपरोक्त में से कौन-सा/से कथन सही है/हैं?
एन्क्रिप्शन का उपयोग कर सकता है।
(a) केवल 1
निम्न कूट का प्रयोग कर सही उत्तर का चयन कीजिए:
(b) केवल 2
(a) केवल 1 और 2
(c) 1 और 2 दोनों
(b) केवल 3
(d) न तो 1, न ही 2
(c) केवल 1 और 3

(d) 1, 2 और 3
65. ‘ईंधन सेल’ के संदर्भ में जिनसे बिजली उत्पन्न करने के लिए
हाइड्रोजन युक्त ईंधन और ऑक्सीजन का उपयोग किया
61. 'गोल्डीलॉक्स ज़ोन' अभिव्यक्ति अक्सर समाचारों में जाता है। निम्नलिखित कथनों पर विचार कीजिए: (2015)
निम्नलिखित के संदर्भ में देखी जाती है: (2015)
1. यदि शुद्ध हाइड्रोजन का उपयोग ईंधन के रूप में किया
(a) पृथ्वी सतह के ऊपर रहने योग्य क्षेत्र सीमा जाए तो ईंधन सेल गर्मी और जल को उप-उत्पादों के
(b) बाहरी अंतरिक्ष में पृथ्वी जैसे ग्रहों के अंदर के क्षेत्र रूप में उत्सर्जि त करेगा।

(c) बाहरी अंतरिक्ष में पृथ्वी जैसे ग्रहों की खोज 2. फ्यूल सेल का उपयोग इमारतों को बिजली देने के
लिए किया जा सकता है न कि लै पटॉप, कंप्यूटर जैसे
(d) कीमती धातुओ ं वाले उल्कापिं डों की खोज
छोटे उपकरणों के लिए।

3. ईंधन सेल प्रत्यावर्ती धारा (AC) के रूप में बिजली का


62. कभी-कभी समाचारों में देखा जाने वाली अभिव्यक्ति उत्पादन करते हैं।
'IndARC' किसका नाम है? (2015)

130
उपरोक्त में से कौन-सा/से कथन सही है/हैं? (c) 1, 2 और 3

(a) केवल 1 (d) कोई नहीं

(b) केवल 2 और 3

(c) केवल 1 और 3 69. फिंगरप्रिंट स्कैनिं ग के अलावा, किसी व्यक्ति की


बायोमेट्रिक पहचान में निम्नलिखित में से किसका उपयोग
(d) 1, 2 और 3
किया जा सकता है? (2014)

1. आईरिस स्कैनिं ग
66. निम्नलिखित में से कौन-सा/से रासायनिक परिवर्तन
2. रेटिनल स्कैनिं ग
का उदाहरण है/हैं? (2014)
3. वोइस रिकग्निशन
1. सोडियम क्लोराइड का क्रिस्टलीकरण
निम्न कूट का प्रयोग कर सही उत्तर का चयन कीजिए:
2. बर्फ का पिघलना
(a) केवल 1
3. दूध का खट्टा होना
(b) केवल 2 और 3
निम्न कूट का प्रयोग कर सही उत्तर का चयन कीजिए:
(c) केवल 1 और 3
(a) केवल 1 और 2
(d) 1, 2 और 3
(b) केवल 3

(c) 1, 2 और 3
70. निम्नलिखित में से कौन-सा/से युग्म सही सुमेलित है/
(d) कोई नहीं
हैं? (2014)

अंतरिक्ष यान उद्देश्य


67. निम्नलिखित रोगों पर विचार कीजिए: (2014)

1. डिप्थीरिया शुक्र की परिक्रमा करना और पृथ्वी पर


कैसिनी-ह्यूजेंस
डाटा भेजना
2. चिकनपोक्स

3. स्मॉलपाोक्स मैसेंजर मानचित्रण और बुध की जाँच करना


उपरोक्त में से कौन-से रोग भारत में समाप्त कर दी गई
है/हैं? वोयेजर 1 और 2 बाहरी सौरमंडल की खोज
(a) केवल 1 और 2
निम्न कूट का प्रयोग कर सही उत्तर का चयन कीजिए:
(b) केवल 3
(a) केवल 1
(c) 1, 2 और 3
(b) केवल 2 और 3
(d) कोई नहीं
(c) केवल 1 और 3

(d) 1, 2 और 3
68. निम्नलिखित युग्मों पर विचार कीजिए: (2014)

विटामिन की कमी से होने वाला रोग


71. टै टू गोदने के माध्यम से निम्नलिखित में से कौन-
1. विटामिन सी स्कर्वी सी बीमारी एक व्यक्ति से दूसरे व्यक्ति में फैल सकती
2. विटामिन डी रिकेट्स है? (2013)

3. विटामिन ई रतौंधी 1. चिकनगुनिया

उपरोक्त में से कौन-सा/से युग्म सही सुमेलित है/हैं? 2. हेपेटाइटिस बी

(a) केवल 1 और 2 3. एचआईवी-एड्स

(b) केवल 3 निम्न कूट का प्रयोग कर सही उत्तर का चयन कीजिए:

(a) केवल 1

(b) केवल 2 और 3

131
(c) केवल 1 और 3 75. अन्न और तिलहन के अनुचित संचालन और
भंडारण के परिणामस्वरूप एफ्लाटॉक्सिन के रूप
(d) 1, 2 और 3
में जाने जाने वाले विषाक्त पदार्थों का उत्पादन
होता है जो आम तौर पर सामान्य खाना पकाने
72. निम्नलिखित खनिजों पर विचार कीजिए: (2013) की प्रक्रिया से नष्ट नहीं होते हैं। एफ्लाटॉक्सिन
किसके द्वारा निर्मि त होते हैं: (2013)
1. कैल्शियम
(a) बैक्टीरिया
2. आयरन
(b) प्रोटोजोआ
3. सोडियम
(c) मोल्ड्स
उपरोक्त खनिजों में से कौन-सा/से खनिज मानव शरीर
द्वारा माँसपेशियों के संकुचन के लिए आवश्यक है/हैं? (d) विषाणु

(a) केवल 1

(b) केवल 2 और 3 76. कोल्ड चेंबर में संग्रहीत फल लं बे समय तक भंडारण में
सही रखे रहते हैं क्योंकि: (2013)
(c) केवल 1 और 3
(a) धूप के संपर्क में नहीं आते
(d) 1, 2 और 3
(b) कार्बन वातावरण में सांद्रता डाइऑक्साइड की वृद्धि
हो जाती है
73. निम्नलिखित में से कौन सा/से कथन सही है/हैं? (2013)
(c) श्वसन दर कम हो जाती है
1. विषाणु ऊर्जा उत्पादन के लिए आवश्यक एं जाइमों
(d) आर्द्रता में वृद्धि रहती है
को कम करते हैं।

2. किसी भी सिं थेटिक माध्यम में विषाणु को सुसंस्कृत


किया जा सकता है। 77. कई प्रतिरोपित पौध नहीं उगती क्योंकि: (2013)

3. विषाणु एक जीव से दूसरे जीव में केवल जैविक (a) नई मृदा में अनुकूल खनिज नहीं होते
वेक्टरों द्वारा ही संचरित होते हैं। (b) अधिकांश मूल रोम नई मृदा को बहुत मुश्किल से
निम्न कूट का प्रयोग कर सही उत्तर का चयन कीजिए: पकड़ते हैं

(a) केवल 1 (c) प्रत्यारोपण के दौरान अधिकांश मूल रोम खो जाते हैं

(b) केवल 2 और 3 (d) प्रत्यारोपण के दौरान पत्तियाँ टू ट जाती हैं

(c) केवल 1 और 3

(d) 1, 2 और 3 78. मरुस्थलीय क्षेत्रों में जल हानि को रोकने के लिए


निम्नलिखित में से कौन-सा/से पत्ती संशोधन होता/होते
है/हैं? (2013)
74. पुनः संयोजक डीएनए प्रौद्योगिकी (जेनेटिक इं जीनियरिं ग)
1. कठोर और मोमी पत्ते
जीन को स्थानांतरित करती है: (2013)
2. छोटे पत्ते या बिना पत्ते
1. पौधों की विभिन्न प्रजातियों में
3. पत्तियों के बजाय काँटे
2. जंतुओ ं से पौधों में
निम्न कूट का प्रयोग कर सही उत्तर का चयन कीजिए:
3. सूक्ष्मजीवों से उच्च जीवों में
(a) केवल 1 और 2
निम्न कूट का प्रयोग कर सही उत्तर का चयन कीजिए:
(b) केवल 2
(a) केवल 1
(c) केवल 1 और 3
(b) केवल 2 और 3
(d) 1, 2 और 3
(c) केवल 1 और 3

(d) 1, 2 और 3
79. साइकिल, कार आदि में बॉल बेयरिं ग का उपयोग किया
जाता है क्योंकि: (2013)

132
(a) पहिये और धुरी के बीच के संपर्क का वास्तविक क्षेत्र (c) दुर्बल नाभिकीय बल रेडियोएक्टिविटी बनाता है
बढ़ जाता है
(d) प्रबल नाभिकीय बल एक परमाणु के न्यूक्लियर के
(b) पहिये और धुरी के बीच के संपर्क का प्रभावी क्षेत्र बढ़ अंदर प्रोटॉन और न्यूट्रॉन को नियंत्रित करता है।
जाता है

(c) पहिये और धुरी के बीच के संपर्क का प्रभावी क्षेत्र कम


83. हाल ही में हिग्स बोसोन कण के अस्तित्व का पता लगाने
हो जाता है
के लगातार प्रयास चर्चा में हैं। इस कण की खोज का/के
(d) कोई भी कथन सही नहीं है क्या महत्व है/हैं? (2013)

1. इससे हम यह जानने में सक्षम होंगे कि प्राथमिक


कणों का द्रव्यमान क्यों होता है।
80. निम्नलिखित परिघटनाओ ं पर विचार कीजिए: (2013)
2. यह हमें निकट भविष्य में पदार्थों को उनके बीच
1. संध्या के समय सूर्य का आकार
के भौतिक स्थान को पार किए बिना एक स्थान से
2. उदय के समय सूर्य का रंग दूसरे स्थान पर स्थानांतरित करने की तकनीक को
3. उदय के समय चंद्रमा का दिखाई देना विकसित करने में सक्षम करेगा।

4. आकाश में तारों का टिमटिमाना 3. यह हमें परमाणु विखंडन के लिए बेहतर ईंधन बनाने
में सक्षम करेगा।
5. आकाश में ध्रुव तारे का दिखाई पड़ना
निम्न कूट का प्रयोग कर सही उत्तर का चयन कीजिए:
उपरोक्त में से ऑप्टिकल भ्रम हैं?
(a) केवल 1
(a) 1, 2 और 3
(b) केवल 2 और 3
(b) 3, 4 और 5
(c) केवल 1 और 3
(c) 1, 2 और 4
(d) 1, 2 और 3
(d) 2, 3 और 5

84. तेजी से बढ़ रही ऊर्जा की मांग को पूरा करने के लिए कुछ
81. बारिश की बूंदों पर सूरज की रोशनी पड़ने से इं द्रधनुष
का मानना है कि भारत को नाभिकीय ऊर्जा के ईंधन
बनता है। निम्नलिखित में से कौन-सी भौतिक घटनाएँ
के रूप में थोरियम पर शोध और विकास करना चाहिए।
इसके लिए उत्तरदायी हैं? (2013)
इस संदर्भ में थोरियम यूरनि
े यम के किन लाभों को
1. विक्षेपण रोकता है? (2013)
2. अपवर्तन 1. प्रकृति में यूरनि
े यम से अधिक प्रचुर मात्रा में थोरियम
3. आंतरिक परावर्तन पाया जाता है

निम्न कूट का प्रयोग कर सही उत्तर का चयन कीजिएः 2. खनन खनिज के प्रति द्रव्यमान इकाई के आधार पर
प्राकृतिक यूरनि
े यम की तुलना में थोरियम अधिक
(a) केवल 1 और 2
ऊर्जा उत्पन्न कर सकता है।
(b) केवल 2 और 3
3. यूरनि
े यम के मुकाबले थोरियम कम हानिकारक
(c) केवल 1 और 3 अपशिष्ट उत्पन्न करता है।
(d) 1, 2 और 3 उपरोक्त में से कौन-सा/से कथन सही है/हैं?

(a) केवल 1
82. सृष्टि के बलों को चार भागों में विभाजित किया जा (b) केवल 2 और 3
सकता है, जो हैं, गुरुत्वाकर्षण, विद्युत चुंबकत्व, दुर्बल
(c) केवल 1 और 3
नाभिकीय बल और प्रबल नाभिकीय बल। इनके संदर्भ में,
निम्नलिखित में से कौन-सा कथन सही नहीं है? (2013) (d) 1, 2 और 3

(a) चारों में गुरुत्वाकर्षण सबसे सबसे शक्तिशाली है

(b) विद्युत चुंबकत्व केवल विद्युत आवेश वाले कणों पर 85. ग्राफीन आजकल चर्चा में है। इसका क्या महत्व है?
कार्य करता है (2013)

133
1. यह एक द्वि-आयामी सामग्री है और इसमें विद्युत (d) 1, 2 और 3
सुचालकता है।

2. यह अब तक के परीक्षण किेए गए सबसे पतले और


88. हाल ही में, ‘दुर्ल भ पृथ्वी धातु’ नामक तत्वों के एक समूह
सबसे मजबूत पदार्थों में से एक है।
की कम आपूर्ति पर चिं ता व्यक्त की गई है। क्यों? (2013)
3. यह पूरी तरह से सिलिकॉन से बना है और इसमें उच्च
1. इन तत्त्वों के सबसे बड़े उत्पादक चीन ने इनके
ऑप्टिकल पारदर्शि ता है।
निर्यात पर कुछ प्रतिबंध लगा दिए हैं।
4. इसे टच स्क्रीन, एलसीडी और ऑर्गेनिक एलईडी
2. चीन, ऑस्ट्रेलिया, कनाडा और चिली के अलावा ये
के लिए आवश्यक 'कंडक्टिं ग इले क्ट्रोड' के रूप में
तत्त्व किसी भी देश में नहीं पाए जाते।
उपयोग किया जा सकता है।
3. विभिन्न प्रकार की इले क्ट्रॉनिक वस्तुओ ं के निर्माण
उपरोक्त में से कौन-से कथन सही हैं?
के लिए पृथ्वी की दुर्ल भ धातुएँ आवश्यक हैं और इन
(a) केवल 1 और 2 तत्त्वों की मांग बढ़ रही है।

(b) केवल 3 और 4 उपरोक्त में से कौन-सा/से कथन सही है/हैं?

(c) केवल 1, 2 और 4 (a) केवल 1

(d) 1, 2, 3 और 4 (b) केवल 2 और 3

(c) केवल 1 और 3

86. ले ड खाना या सांस द्वारा ले ना स्वास्थ्य के लिए घातक है। (d) 1, 2 और 3


पेट्रोल में ले ड मिलाने पर प्रतिबंध लगने के बाद भी ले ड
विषाक्तिकरण के क्या स्रोत हैं? (2013)
89. भारतीयों सहित ब्रुकहेवन नेशनल ले बोरेटरी में वैज्ञानिकों
1. स्मेलटिंग यूनिट
की एक टीम ने सबसे भारी एं टीमैटर (एं टी-हीलियम
2. पेन और पेंसिल न्यूक्लियस) का निर्माण किया है। विरोधी पदार्थ के निर्माण
का/के निहितार्थ क्या है/हैं? (2013)
3. पेंट
1. यह खनिज पूर्वेक्षण और तेल की खोज को आसान
4. बालों के तेल और सौंदर्य प्रसाधन
और सस्ता बनाएगा।
निम्न कूट का प्रयोग कर सही उत्तर का चयन कीजिए:
2. यह एं टी-मैटर से बने तारों और आकाशगंगाओ ं के
(a) केवल 1, 2 और 3 अस्तित्व की संभावना की खोज में मदद करेगा।
(b) केवल 1 और 3 3. यह ब्रह्मांड के उद्भव को समझने में सहायता करेगा।
(c) केवल 2 और 4 निम्न कूट का प्रयोग कर सही उत्तर का चयन कीजिए:
(d) 1, 2, 3 और 4 (a) केवल 1

(b) केवल 2 और 3
87. अक्सर चर्चा में 'स्टेम सेल' के संदर्भ में, निम्नलिखित में (c) केवल 3
से कौन-सा/से कथन सही है/हैं? (2013)
(d) 1, 2 और 3
1. स्टेम सेल केवल स्तनधारियों से प्राप्त किए जा
सकते हैं
90. अंतरिक्ष से कई सौ कि.मी./सेकंड की गति से यात्रा करने
2. स्टेम सेल का उपयोग नई दवाओ ं की जाँच के लिए
वाले विद्युत आवेशित कण यदि पृथ्वी की सतह पर पहुँच
किया जा सकता है।
जाते हैं तो वे जीवित प्राणियों को गंभीर रूप से क्षति पहुँचा
3. स्टेम सेल का उपयोग चिकित्सा उपचार के लिए सकते हैं। उन्हें पृथ्वी की सतह तक पहुँचने से क्या रोकता
किया जा सकता है। है? (2013)
निम्न कूट का प्रयोग कर सही उत्तर का चयन कीजिए: (a) पृथ्वी का चुंबकीय क्षेत्र उन्हें अपने ध्रुवों की ओर मोड़
(a) केवल 1 और 2 देता है

(b) केवल 2 और 3 (b) पृथ्वी के चारों ओर ओजोन परत उन्हें वापस बाहरी
अंतरिक्ष में परावर्ति त कर देती है
(c) केवल 3

134
(c) वायुमंडल की ऊपरी परतों की नमी उन्हें पृथ्वी की 4. इसे टच स्क्रीन, एलसीडी और ऑर्गेनिक एलईडी
सतह तक पहुँचने से रोकती है के लिए आवश्यक 'कंडक्टिं ग इले क्ट्रोड' के रूप में
उपयोग किया जा सकता है।
(d) ऊपर दिए गए कथन (a), (b) और (c) में से कोई भी
सही नहीं है उपरोक्त में से कौन-से कथन सही हैं?

91. निम्नलिखित कथनों पर विचार कीजिए: (a) केवल 1 और 2

यदि केशिकात्व की कोई घटना नहीं होती (2013) (b) केवल 3 और 4

1. मृदा के तेल का उपयोग करना मुश्किल होता (c) केवल 1, 2 और 4

2. शीतल पेय का सेवन करने के लिए कोई स्ट्रॉ का (d) 1, 2, 3 और 4


उपयोग नहीं कर पाता

3. ब्लाटिंग पेपर काम करना बंद कर देते


94. भारतीयों सहित ब्रुकहेवन नेशनल ले बोरेटरी में
4. जिन बड़े-बड़े पेड़ों को हम अपने चारों ओर देखते हैं, वे वैज्ञानिकों की एक टीम ने सबसे भारी एं टीमैटर (एं टी-
पृथ्वी पर नहीं उगते हीलियम न्यूक्लियस) का निर्माण किया है। विरोधी पदार्थ
के निर्माण का/के निहितार्थ क्या है/हैं? (2012)
उपरोक्त में से कौन-सा कथन सही है?
1. यह खनिज पूर्वेक्षण और तेल की खोज को आसान
(a) केवल 1, 2 और 3
और सस्ता बनाएगा।
(b) केवल 1, 3 और 4
2. यह एं टी-मैटर से बने तारों और आकाशगंगाओ ं के
(c) केवल 2 और 4 अस्तित्व की संभावना की खोज में मदद करेगा।
(d) 1, 2, 3 और 4 3. यह ब्रह्मांड के उद्भव को समझने में सहायता करेगा।

निम्न कूट का प्रयोग कर सही उत्तर का चयन कीजिए:


92. परोपजीवी के प्रतिरोध के अतिरिक्त और कौन-सी (a) केवल 1
संभावनाएँ हैं जिनके लिए आनुवंशिक रूप से व्यवस्थित
(b) केवल 2 और 3
पौधों का निर्माण किया गया है? (2012)
(c) केवल 3
1. उन्हें सूखे का सामना करने में सक्षम बनाने के लिए
(d) 1, 2 और 3
2. उपज के पोषक मूल्य में वृद्धि करने के लिए

3. उन्हें अंतरिक्ष यान और अंतरिक्ष स्टेशनों में प्रकाश


संश्लेषण और विकास करने में सक्षम बनाने के 95. वैज्ञानिकों द्वारा ब्रह्मांड के निरंतर विस्तार के लिए
लिए निम्नलिखित में से कौन-सा/से प्रमाण उद्धृत किया/
किये गया/गए है/हैं? (2012)
4. उनके शेल्फ जीवन को बढ़ाने के लिए।
1. अंतरिक्ष में माइक्रोवेव की खोज
निम्न कूट का प्रयोग कर सही उत्तर का चयन कीजिए:
2. अंतरिक्ष में रेडशिफ्ट घटना का अवलोकन
(a) केवल 1 और 2
3. अंतरिक्ष में क्षुद्रग्रहों की गति
(b) केवल 3 और 4
4. अंतरिक्ष में सुपरनोवा विस्फोटों की घटना
(c) केवल 1, 2 और 4
निम्न कूट का प्रयोग कर सही उत्तर का चयन कीजिए:
(d) 1, 2, 3 और 4
(a) 1 और 2

(b) केवल 2
93. ग्राफीन आजकल चर्चा में है। इसका क्या महत्व है? (2012)
(c) 1, 3 और 4
1. यह एक द्वि-आयामी सामग्री है और इसमें विद्युत
सुचालकता है। (d) उपरोक्त में से कोई भी प्रमाण के रूप में उद्धृत नहीं
किया जा सकता
2. यह अब तक के परीक्षण किेए गए सबसे पतले और
सबसे मजबूत पदार्थों में से एक है।

3. यह पूरी तरह से सिलिकॉन से बना है और इसमें उच्च 96. अंतरिक्ष से कई सौ कि.मी./सेकंड की गति से यात्रा करने
ऑप्टिकल पारदर्शि ता है। वाले विद्युत आवेशित कण यदि पृथ्वी की सतह पर पहुँच

135
जाते हैं तो वे जीवित प्राणियों को गंभीर रूप से क्षति (d) 1, 2 और 3
पहुँचा सकते हैं। उन्हें पृथ्वी की सतह तक पहुँचने से क्या
रोकता है? (2012)
99. ले ड खाना या सांस द्वारा ले ना स्वास्थ्य के लिए घातक
(a) पृथ्वी का चुंबकीय क्षेत्र उन्हें अपने ध्रुवों की ओर मोड़
है। पेट्रोल में ले ड मिलाने पर प्रतिबंध लगने के बाद भी
देता है
ले ड विषाक्तिकरण के क्या स्रोत हैं? (2012)
(b) पृथ्वी के चारों ओर ओजोन परत उन्हें वापस बाहरी
1. स्मेलटिंग यूनिट
अंतरिक्ष में परावर्ति त कर देती है
2. पेन और पेंसिल
(c) वायुमंडल की ऊपरी परतों की नमी उन्हें पृथ्वी की
सतह तक पहुँचने से रोकती है 3. पेंट

(d) ऊपर दिए गए कथन (a), (b) और (c) में 4. बालों के तेल और सौंदर्य प्रसाधन
से कोई भी सही नहीं है निम्न कूट का प्रयोग कर सही उत्तर का चयन कीजिए:

(a) केवल 1, 2 और 3
97. अक्सर चर्चा में 'स्टेम सेल' के संदर्भ में, निम्नलिखित में (b) केवल 1 और 3
से कौन-सा/से कथन सही है/हैं? (2012)
(c) केवल 2 और 4
1. स्टेम सेल केवल स्तनधारियों से प्राप्त किए जा
(d) 1, 2, 3 और 4
सकते हैं

2. स्टेम सेल का उपयोग नई दवाओ ं की जाँच के लिए


किया जा सकता है। 100. एक सीएफएल (CFL) और एलईडी (LED) लैं प में क्या
अंतर होता है? (2011)
3. स्टेम सेल का उपयोग चिकित्सा उपचार के लिए
किया जा सकता है। 1. प्रकाश उत्पन्न करने के लिए एक सीएफएल लै म्प
पारा वाष्प और फॉस्फोर का उपयोग करता है जबकि
निम्न कूट का प्रयोग कर सही उत्तर का चयन कीजिए:
एक एलईडी लैं प सेमीकंडक्टर पदार्थ का उपयोग
(a) केवल 1 और 2 करता है।
(b) केवल 2 और 3 2. एक सीएफएल का औसत जीवन काल एलईडी लैं प
(c) केवल 3 की तुलना में कहीं अधिक होता है।

(d) 1, 2 और 3 3. एलईडी लैं प की तुलना में सीएफएल कम ऊर्जा


सक्षम है।

उपरोक्त में से कौन-सा/से कथन सही है/हैं?


98. तेजी से बढ़ रही ऊर्जा की मांग को पूरा करने के लिए
कुछ का मानना है कि भारत को नाभिकीय ऊर्जा के (a) केवल 1
ईंधन के रूप में थोरियम पर शोध और विकास करना (b) केवल 2 और 3
चाहिए। इस संदर्भ में थोरियम यूरनि
े यम के किन लाभों
(c) केवल 1 और 3
को रोकता है? (2012)
(d) 1, 2 और 3
1. प्रकृति में यूरनि
े यम से अधिक प्रचुर मात्रा में थोरियम
पाया जाता है

2. खनन खनिज के प्रति द्रव्यमान इकाई के आधार पर 101. ब्लू -रे डिस्क (BD) के नाम से पहचाना जाने वाला एक
प्राकृतिक यूरनि
े यम की तुलना में थोरियम अधिक नया ऑप्टिकल डिस्क प्रारूप लोकप्रिय हो रहा है। यह
ऊर्जा उत्पन्न कर सकता है। पारंपरिक DVD से किस प्रकार भिन्न है? (2011)

3. यूरनि
े यम के मुकाबले थोरियम कम हानिकारक 1. डीवीडी स्टैंडर्ड डेफिनिशन वीडियो सपोर्ट करता है
अपशिष्ट उत्पन्न करता है। जबकि बीडी हाई डेफिनिशन वीडियो सपोर्ट करता है।

उपरोक्त में से कौन-सा/से कथन सही है/हैं? 2. डीवीडी की तुलना में, बीडी फोरमैट में भंडारण की
क्षमता कई गुना अधिक होती है।
(a) केवल 1
3. बीडी की मोटाई 2.4 मि.मी. होती है जबकि डीवीडी की
(b) केवल 2 और 3
मोटाई 1.2 मि.मी. होती है।
(c) केवल 1 और 3

136
उपरोक्त में से कौन-सा/से कथन सही है/हैं? उपयोगकर्ता एन्क्रिप्टेड जानकारी प्रसारित कर
सकते हैं
(a) केवल 1
(b) यह सार्वजनिक इं टरनेट पर एक कंप्यूटर नेटवर्क
(b) केवल 1 और 2
है जो संचरित सूचना की सुरक्षा को बनाए हुए
(c) केवल 2 और 3 उपयोगकर्ताओ ं को उनके संगठन के नेटवर्क तक
(d) 1, 2 और 3 पहुँच प्रदान कराता है

(c) यह एक कंप्यूटर नेटवर्क है जिसमें उपयोगकर्ता एक


सेवा प्रदाता के माध्यम से कंप्यूटिंग संसाधनों के
102. दूरसंचार रिले के लिए उपयोग किए जाने वाले उपग्रहों
एक साझा पूल तक पहुँच सकते हैं।
को भू-स्थिर कक्षा में रखा जाता है। एक उपग्रह को
एसी कक्षा में तब माना जाता है जब: (2011) (d) ऊपर दिए गए कथन (a), (b) और (c) में से कोई भी
वर्चुअल प्राइवेट नेटवर्क का सही विवरण नहीं है
1. कक्षा भू-समकालिक होती है।

2. कक्षा गोलाकार होती है।


105. बीटी-बैंगन के रूप में पहचाना जाने
3. कक्षा पृथ्वी के भूमध्य रेखा के तल में स्थित होती है।
वाला बैंगन का आनुवंशिक व्यवस्थित
4. कक्षा 22,236 किमी की ऊँचाई पर होती है। रूप बन गया है। इसका उद्देश्य है (2011)
नीचे दिए गए कूट का प्रयोग कर सही उत्तर का चयन (a) इसे कीट-प्रतिरोधी बनाना
कीजिए:
(b) इसके स्वाद और पोषक गुणों में सुधार करना
(a) केवल 1, 2 और 3
(c) इसे शुष्क-प्रतिरोधी बनाना
(b) केवल 1, 3 और 4
(d) इसकी शेल्फ-लाइफ को बढ़ाना
(c) केवल 2 और 4

(d) 1, 2, 3 और 4
106. एक दम्पत्ति ने एक लड़के को गोद लिया, कुछ साल बाद
एक उन्हें जुड़वाँ लड़के पैदा हुए। पति-पत्नी का ब्लड ग्रुप
103. ब्लूटू थ और वाई-फाई उपकरणों में क्या अंतर होता है? AB पॉजिटिव और O नेगेटिव है। तीनों बेटों का ब्लड ग्रुप
(2011) A पॉजिटिव, B पॉजिटिव और O पॉजिटिव है। दत्तक पुत्र
का रक्त समूह है (2011)
(a) ब्लूटू थ 2.4 गीगाहर्ट्ज़ रेडियो फ़्रीक्वेंसी बैंड का
उपयोग करता है जबकि-वाई-फाई 2.4 गीगाहर्ट्ज़ (a) O पॉजिटिव
और 5 गीगाहर्ट्ज़ फ़्रीक्वेंसी बैंड का उपयोग कर (b) A पॉजिटिव
सकता है।
(c) B पॉजिटिव
(b) ब्लूटू थ का उपयोग केवल वायरले स लोकल एरिया
(d) दिए गए डेटा के आधार पर निर्धारित नहीं किया जा
नेटवर्क (WLAN) के लिए किया जाता है जबकि
सकता
वाई-फाई का उपयोग केवल वायरले स वाइड एरिया
नेटवर्क (WWAN) के लिए किया जाता है।

(c) जब ब्लू -टू थ तकनीक के उपयोग से दो उपकरणों के 107. वर्तमान में, वैज्ञानिक एक गुणसूत्र में जीन के अनुक्रमों
बीच सूचना प्रसारित की जाती है तो दोनों उपकरणों की सापेक्ष स्थिति या डीएनए अनुक्रमों की व्यवस्था
का एक-दूसरे की दृष्टि रेखा में होना अनिवार्य होता निर्धारित कर सकते हैं। यह जानकारी हमें किस प्रकार
है ले किन जब वाई-फाई तकनीक का उपयोग किया लाभ पहुँचाती है? (2011)
जाता है तो उपकरणों का एक-दूसरे की दृष्टि रेखा में 1. पशुधन की वंशावली जानना संभव है।
होना आवश्यक नहीं होता।
2. मानव के सभी रोगों के कारणों को समझना संभव
(d) इस संदर्भ में उपरोक्त कथन (a) और (b) सही हैं है।

3. रोग प्रतिरोधी पशु नस्लों को विकसित करना संभव


104. “वर्चुअल प्राइवेट नेटवर्क ” क्या है? (2011) है।

(a) यह एक संगठन का एक निजी कंप्यूटर नेटवर्क है उपरोक्त कथनों में से कौन-सा/से कथन सही है/हैं?
जिससे इस संगठन के सर्वर के माध्यम से दूरस्थ

137
(a) केवल 1 और 2 (c) भारत की 3डी इमेजिं ग क्षमताओ ं के साथ इसरो का
एक जियोपोर्टल
(b) केवल 2
(d) भारत द्वारा विकसित एक अंतरिक्ष दूरबीन
(c) केवल 1 और 3

(d) 1, 2 और 3
112. हाल ही में, दृष्टि सुधार के लिए लै सिक (लै सर असिस्टेड
108. कड़ाके की ठं ड में झील की सतह जम जाती है ले किन
इन सीटू केराटोमाइल्यूसिस) प्रक्रिया को लोकप्रिय
उसके नीचे का जल फिर भी तरल अवस्था में ही होता है।
बनाया जा रहा है। इस संदर्भ में निम्नलिखित में से
क्या कारण है? (2011)
कौन-सा कथन सही नहीं है? (2010)
(a) बर्फ गर्मी की कुचालक होती है
(a) लै सिक प्रक्रिया का उपयोग आँख के अपवर्तन दोष
(b) जब झील की सतह हवा के समान तापमान पर होती को ठीक करने के लिए किया जाता है
है तो ऊष्मा नहीं घटती
(b) यह एक ऐसी प्रक्रिया है जो कॉर्नि या के आकार को
(c) जल का घनत्व अधिकतम 4 डिग्री सेल्सियस पर स्थायी रूप से बदल देती है
होता है
(c) यह चश्मे या कॉन्टैक्ट लें स पर किसी व्यक्ति की
(d) ऊपर दिए गए कथन (a), (b) और (c) में से कोई भी निर्भरता को कम करती है
सही नहीं है
(d) यह है एक प्रक्रिया जो किसी भी उम्र के व्यक्ति पर की
जा सकती है
109. परमाणु रिएक्टर में भारी जल का कार्य होता है: (2011)

(a) न्यूट्रॉनों की गति धीमा करना। 113. कैंसर ट्यूमर के इलाज के संदर्भ में एक साइबर
(b) न्यूट्रॉनों की गति बढ़ाना। चाकू नामक उपकरण चर्चा में रहा है। इस संदर्भ में,
निम्नलिखित में से कौन-सा कथन सही नहीं है? (2010)
(c) रिएक्टर को ठं डा करना।
(a) यह एक रोबोट छवि निर्देशित प्रणाली है
(d) नाभिकीय प्रतिक्रिया बंद करना।
(b) यह विकिरण का एक अत्यंत सटीक काम करता है

(c) इसमें उप-मिलीमीटर सटीकता प्राप्त करने की


110. निम्नलिखित कथनों पर विचार कीजिए: (2010)
क्षमता है
भारत द्वारा लॉन्च किया गया उपग्रह ओशनसैट-2
(d) यह शरीर में ट्यूमर के प्रसार को मैप कर सकता है
1. वातावरण में जल वाष्प तत्व का अनुमान लगाने में
सहायक है
114. Mon 863 मक्का की एक किस्म है। यह निम्नलिखित
2. मानसून की शुरुआत की भविष्यवाणी करता है
कारण से चर्चा में था: (2010)
3. समुद्र तटीय प्रदूषण की निगरानी करता है
(a) यह आनुवंशिक रूप से संशोधित बौना किस्म है जो
उपरोक्त में से कौन-सा/से कथन सही है/हैं? सूखे के लिए प्रतिरोधी है
(a) 1 और 2 (b) यह आनुवंशिक रूप से संशोधित किस्म है जो कीट
(b) केवल 2 प्रतिरोधी है

(c) 1 और 3 (c) यह आनुवंशिक रूप से संशोधित किस्म है जो आम


मक्का फसल की तुलना में दस गुना अधिक प्रोटीन
(d) 1, 2 और 3
देती है

(d) यह जैव-ईंधन उत्पादन के लिए विशेष रूप से उपयोग


111. अंतरिक्ष प्रौद्योगिकी के संदर्भ में हाल में चर्चा में रहा की जाने वाली आनुवंशिक रूप से संशोधित किस्म है
"भुवन" क्या है? (2010)

(a) भारत में दूरस्थ शिक्षा को बढ़ावा देने के लिए इसरो


115. आनुवंशिक हेर-फेर द्वारा प्राकृतिक आइसोले ट्स से
द्वारा लॉन्च किया गया एक छोटा उपग्रह
विकसित बैक्टीरिया का कौन-सा स्ट्रे न तेल रिसाव के
(b) चंद्रयान-2 के लिए अगले मून इमपैक्ट प्रोब का उपचार के लिए इस्तेमाल किया जा सकता है? (2010)
दिया गया नाम
(a) एग्रोबैक्टीरियम

138
(b) क्लोस्ट्रीडियम (b) केवल 2

(c) नाइट्रोसोमोनस (c) 1 और 2 दोनों

(d) स्यूडोमोनास (d) न तो 1, न ही 2

116. आनुवंशिक रूप से संशोधित "गोल्डन राइस" को मानव 119. चावल की व्यापक खेती के कारण, कुछ क्षेत्र ग्लोबल
पोषण संबंधी आवश्यकता को पूरा करने के लिए व्यवस्थित वार्मिं ग में योगदान दे रहे सकते हैं। यह किस/किन
किया गया है। निम्नलिखित में से कौन-सा कथन संभावित कारण/कारणों के कारण है? (2010)
गोल्डन राइस को सर्वोत्तम रूप से योग्य बनाता है? (2010)
1. चावल की खेती से जुड़ी अवायवीय अवस्था मीथेन के
(a) अन्य उच्च उपज देने वाली किस्मों की तुलना में प्रति उत्सर्जन का कारण बनती हैं
एकड़ तीन गुना अधिक अनाज उपज प्रदान करने के
2. जब नाइट्रोजन आधारित उर्वरकों का उपयोग
लिए अनाज को जीन के साथ सशक्त किया गया है
किया जाता है तो जुताई मृदा से नाइट्रस ऑक्साइड
(b) इसके अनाज में प्रो-विटामिन ए होता है जो मानव निकलती है।
शरीर के विटामिन ए में मिल जाता है
उपरोक्त में से कौन-सा/से कथन सही है/हैं?
(c) इसके संशोधित जीन सभी आवश्यक 9 अमीनो
(a) केवल 1
एसिड़ों के संश्लेषण का कारण बनते हैं
(b) केवल 2
(d) इसके संशोधित जीन विटामिन-डी के साथ इसके
अन्न के दृढ़ीकरण का कारण बनते हैं (c) 1 और 2 दोनों

(d) न तो 1 और न ही 2

117. नीले -हरे शैवाल की कुछ प्रजातियों की कौन-सी


विशेषताएँ उन्हें जैव-उर्वरक के रूप में बढ़ावा देने में मदद 120. चार ऊर्जा फसलों के नाम नीचे दिए गए हैं। (2010)
करती हैं? (2010)
इनमें से किसकी खेती इथेनॉल के लिए की जा सकती
(a) वे वायुमंडलीय मीथेन को अमोनिया में परिवर्ति त है?
करते हैं जिसे फसल के पौधे आसानी से अवशोषित
(a) जटरोफा
कर ले ते हैं
(b) मक्का
(b) वे फसल के पौधों को एं जाइम उत्पन्न करने के
लिए प्रेरित करते हैं जो वायुमंडलीय नाइट्रोजन को (c) पोंगामिया
नाइट्रे ट्स में बदलने में मदद करते हैं (d) सूरजमुखी
(c) उनके पास वायुमंडलीय नाइट्रोजन को ऐसी अवस्था
में बदलने की युक्ति होती है जिसे फसली पौधे
121. निम्नलिखित में से कौन वायरले स प्रौद्योगिकी के
आसानी से अवशोषित कर सकते हैं
जीएसएम परिवार से संबंधित नहीं है/हैं? (2010)
(d) वे फसली पौधों की जड़ों को बड़ी मात्रा में मृदा नाइट्रेट
(a) ईडीजीई
अवशोषित करने के लिए प्रेरित करते हैं
(b) एलटीई

(c) डीएसएल
118. जटरोफा करकस के अलावा भारत में बायो-डीजल के
उत्पादन के लिए पोंगामिया पिन्नाटा को भी एक अच्छा (d) ईडीजीई और एलटीई दोनों
विकल्प क्यों माना जाता है? (2010)

1. भारत के अधिकांश शुष्क क्षेत्रों में पोंगामिया पिनाटा 122. निम्नलिखित पर विचार कीजिए: (2010)
स्वाभाविक रूप से बढ़ता है।
1. ब्लूटू थ डिवाइस
2. पोंगामिया पिनाटा के बीज लिपिड सामग्री से भरपूर
2. कॉर्डले स फोन
होते हैं जिनमें लगभग आधा ओलिक एसिड होता है।
3. माइक्रोवेव ओवन
उपरोक्त में से कौन-सा/से कथन सही है/हैं?
4. वाई-फाई डिवाइस
(a) केवल 1
उपरोक्त में से क्या 2.4 और 2.5 गीगाहर्ट्ज रेंज के रेडियो

139
फ्रीक्वेंसी बैंड के बीच काम कर सकता है? (a) आलू से

(a) 1 और 2 (b) चावल से

(b) 3 और 4 (c) गन्ने से

(c) 1, 2 और 4 (d) गेहूँ से

(d) 1, 2, 3 और 4 128. निम्नलिखित में से किसका उपयोग विस्फोटक के रूप


में किया जाता है? (2009)

(a) फास्फोरस ट्राइक्लोराइड


123. हाइड्रोजन ईंधन सेल वाहन “इग्ज़होस्ट” के रूप
में निम्नलिखित में से उत्पादन करते हैं (2010) (b) मर्क्यूरिक ऑक्साइड

(a) NH3 (c) ग्रेफाइट

(b) CH4 (d) नाइट्रोग्लिसरीन

(c) H2O

(d) H2O2 129. निम्नलिखित में से किस एक में उच्चतम ऊर्जा है? (2009)

(a) नीली रोशनी

124. निम्नलिखित कथनों पर विचार कीजिए: (2009) (b) हरी रोशनी

1. भारत में थोरियम का कोई भंडार नहीं है (c) लाल रोशनी

2. केरल की मोनाजाइट रेत में यूरनि


े यम होता है (d) पीली रोशनी

उपरोक्त में से कौन-सा/से कथन सही है/हैं?

(a) केवल 1 130. एक शुष्क सेल (बैटरी) में निम्नलिखित में से किनका
उपयोग इले क्ट्रोलाइट्स के रूप में किया जाता है? (2009)
(b) केवल 2
(a) अमोनियम क्लोराइड और जिं क क्लोराइड
(c) 1 और 2 दोनों
(b) सोडियम क्लोराइड और कैल्शियम क्लोराइड
(d) न तो 1, न ही 2
(c) मैग्नीशियम क्लोराइड और जिं क क्लोराइड

(d) अमोनियम क्लोराइड और कैल्शियम क्लोराइड


125. WIMAX निम्नलिखित में से किससे संबंधित है? (2009)

(a) जैव प्रौद्योगिकी


131. निम्नलिखित में से किस वैज्ञानिक ने यह सिद्ध किया
(b) अंतरिक्ष प्रौद्योगिकी
कि सूर्य के द्रव्यमान के 1.44 गुणा से कम द्रव्यमान
(c) मिसाइल प्रौद्योगिकी वाले तारे जब खत्म होने हैं तो सूर्य का द्रव्यमान सफेद
(d) संचार प्रौद्योगिकी बौने तारों के रूप में समाप्त हो जाता है? (2009)

(a) एडविन हबल

126. वर्ष 2008 में, निम्नलिखित में से किसने एक जटिल (b) एस चंद्रशेखर
वैज्ञानिक प्रयोग किया जिसमें उप-परमाणु कणों को (c) स्टीफन हॉकिंग
प्रकाश की गति के लगभग त्वरित किया गया था? (2009)
(d) स्टीवन वेनबर्ग
(a) यूरोपीय अंतरिक्ष एजेंसी

(b) यूरोपीय संगठन परमाणु अनुसंधान


132. निम्नलिखित कथनों पर विचार कीजिए: (2009)
(c) अंतर्राष्ट्रीय नाभिकीय ऊर्जा एजेंसी
1. मीठे संतरे के पौधे को ग्राफ्टिं ग तकनीक द्वारा उगाया
(d) राष्ट्रीय वैमानिकी और अंतरिक्ष प्रशासन जाता है।

2. चमेली के पौधे को ले यरिं ग तकनीक द्वारा उगाया


127. ऊर्जा के वैकल्पिक स्रोतों के संदर्भ में, इथेनोल एक जीवक्षम जाता है।
जैव-ईंधन के रूप में प्राप्त किया जा सकता है (2009) उपरोक्त में से कौन-सा/से कथन सही है/हैं?

140
(a) केवल 1 साथ देखा जाता है तो यह उसकी विजुअल ब्राइटनेस
निर्धारित करता है।
(b) केवल 2
2. बुध का एल्बीडो पृथ्वी के एल्बीडो से काफी बड़ा है।
(c) 1 और 2 दोनों
उपरोक्त में से कौन-सा/से कथन सही है/हैं?
(d) न तो 1, न ही 2
(a) केवल 1
133. आनुवंशिक विकारों के संदर्भ में, निम्नलिखित पर
विचार कीजिए: (2009) (b) केवल 2

एक महिला वर्णान्धता से पीड़ित है जबकि उसका पति (c) 1 और 2 दोनों


इससे पीड़ित नहीं है। उनका एक पुत्र और एक पुत्री है। इस
(d) न तो 1, न ही 2
संदर्भ में, निम्नलिखित में से कौन-सा कथन संभवतः
अधिक सटीक है?

(a) दोनों बच्चे वर्णान्धता से पीड़ित हैं। 138. प्रोबायोटिक भोजन के बारे में निम्नलिखित कथनों पर
विचार कीजिए: (2008)
(b) पुत्री वर्णान्धता से पीड़ित है जबकि पुत्र इससे पीड़ित
नहीं है। 1. प्रोबायोटिक भोजन में जीवित बैक्टीरिया होते हैं जो
मनुष्यों के लिए गुणकारी माने जाते हैं।
(c) दोनों बच्चे वर्णान्धता से पीड़ित नहीं हैं।
2. प्रोबायोटिक भोजन जठरांत्र वनस्पति को बनाए
(d) पुत्र वर्णान्धता से पीड़ित है जबकि पुत्री इससे पीड़ित
रखने में मदद करता है।
नहीं है।
उपरोक्त में से कौन-सा/से कथन सही है/हैं?

(a) केवल 1
134. पांडा उसी परिवार से संबंधित है जिससे: (2009)
(b) केवल 2
(a) भालू
(c) 1 और 2 दोनों
(b) बिल्ली
(d) न तो 1, न ही 2
(c) कुत्ता

(d) खरगोश
139. सामान्य व्यक्ति के रक्त का pH स्तर कितना होता है?
(2008)
135. निम्नलिखित में से किस ग्रह में प्राकृतिक उपग्रहों या
(a) 4.5-4.5
चंद्रमाओ ं की संख्या सबसे अधिक है? (2009)
(b) 6.45-6.55
(a) बृहस्पति
(c) 7.35-7.45
(b) मंगल
(d) 8.25-8.35
(c) शनि

(d) शुक्र
140. निम्नलिखित में से कौन वानर नहीं है? (2008)

(a) गिब्बन
136. नाइट विज़न उपकरण में निम्नलिखित में से किस प्रकार
की तरंगों का उपयोग किया जाता है ? (2009) (b) गोरिल्ला

(a) रेडियो तरंगें (c) लं गूर

(b) माइक्रोवेव (d) ऑरंगुटान

(c) अवरक्त तरंगें

(d) उपरोक्त में से कोई नहीं 141. निम्नलिखित में से कौन-सा धातु युग्म क्रमशः सबसे हल्की
धातु और सबसे भारी धातु बनाता है? (2008)

(a) लिथियम और पारा


137. निम्नलिखित कथनों पर विचार कीजिए: (2008)
(b) लिथियम और ऑस्मियम
1. किसी वस्तु का अलबीडो जब परावर्ति त रोशनी के
(c) एल्यूमिनियम और ऑस्मिम

141
(d) एल्यूमिनियम और पारा (d) एक्साइमर ले जर

142. निम्नलिखित में से किस गैस युग्म का मिश्रण खदानों 147. जेट इं जन और रॉकेट के संबंध में निम्नलिखित कथनों
में अधिकांश विस्फोट होने का कारण होता है? (2008) पर विचार कीजिए: (2008)

(a) हाइड्रोजन और ऑक्सीजन 1. एक जेट इं जन अपनी ऑक्सीजन आपूर्ति के लिए


आसपास की हवा का उपयोग करता है इसलिए यह
(b) ऑक्सीजन और एसिटिलीन
अंतरिक्ष में गति के लिए अनुपयुक्त है
(c) मीथेन और वायु
2. एक रॉकेट गैस और ईंधन के रूप में ऑक्सीजन की
(d) कार्बन डाइऑक्साइड और मीथेन आपूर्ति स्वयं करता है

उपरोक्त में से कौन-सा/से कथन सही है/हैं?


143. निम्नलिखित में से किसे स्ट्रेंजर गैस भी कहा जाता है? (a) केवल 1
(2008)
(b) केवल 2
(a) आर्गन
(c) 1 और 2 दोनों
(b) नियॉन
(d) न तो 1, न ही 2
(c) क्सीनन

(d) नाइट्रस ऑक्साइड


148. हाल ही में चर्चा में रहे अमेरिकी अंतरिक्ष एजेंसी के थेमिस
मिशन का क्या उद्देश्य है? (2008)
144. माणिक्य और नीलम के रासायनिक रूप को क्या कहा (a) मंगल ग्रह पर जीवन की संभावना का अध्ययन
जाता है? (2008) करना
(a) सिलिकॉन डाइऑक्साइड (b) शनि के उपग्रहों का अध्ययन करना
(b) एल्यूमिनियम ऑक्साइड (c) उच्च अक्षांश आकाश के कलरफुल डिस्प्ले का
(c) ले ड टे ट्रोक्साइड अध्ययन करना

(d) बोरॉन नाइट्राइड (d) तारकीय विस्फोटों का अध्ययन करने के लिए एक


अंतरिक्ष प्रयोगशाला का निर्माण करना

145. निम्नलिखित कथनों पर विचार कीजिए: (2008)


149. निम्नलिखित में से एक अंतरिक्ष यान कौन-सा है? (2008)
1. विशाल क्षेत्र में उपयोग किए जाने वाले डायटोनिक
स्केल नामक सप्तक में सात आवृत्तियाँ होती हैं। (a) एपोफिस

2. Note Sā की आवृत्ति 256 Hz होती है वहीं Nī की (b) कैसिनी


आवृत्ति 512b Hz होती है। (c) स्पिट्जर
उपरोक्त में से कौन-सा/से कथन सही है/हैं? (d) टे कस्टार
(a) केवल 1

(b) केवल 2 150. निम्नलिखित में से कौन-सा पौधा कीटभक्षी है? (2008)
(c) 1 और 2 दोनों (a) सलीबी पौधा
(d) न तो 1, न ही 2 (b) पिचर प्लांट

(c) नाइट क्वीन


146. ले जर प्रिंटर में निम्नलिखित में से किस प्रकार के ले जर (d) जंगल की लौ
का उपयोग किया जाता है? (2008)

(a) डाई ले जर
151. जीवों के निम्नलिखित बच्चों में से किसमें वह घटना पाई
(b) गैस ले जर जाती है जिसमें मैथुन करने के बाद मादा नर को मार
(c) सेमीकंडक्टर ले जर देती है? (2008)

142
(a) व्याध पतंग 156. निम्नलिखित में से कौन-सा होर्मोन मानव शरीर में रक्त
कैल्शियम और फॉस्फेट को नियंत्रित करता है? (2007)
(b) मधुमक्खी
(a) ग्लू कागन
(c) मकड़ी
(b) ग्रोथ हार्मोन
(d) पिट वाइपर
(c) पैराथायराइड
152. बिस्फेनॉल ए (BPA) क्या है? (2008)
(d) थायरोक्सिन
(a) कैंसर की जाँच करने के लिए एक चिकित्सा परीक्षण

(b) एथलीटों द्वारा अच्छा प्रदर्शन करने के लिए दवाओ ं


के प्रयोग की जाँच करने के लिए परीक्षण 157. अधिकांश कीट श्वसन क्रिया कैसे करते हैं? (2007)

(c) खाद्य-पैकेजिं ग सामग्री के विकास के लिए इस्तेमाल (a) त्वचा के माध्यम से


किया जाने वाला एक रसायन
(b) गलफड़ों के माध्यम से
(d) एक विशेष प्रकार का मिश्र धातु इस्पात
(c) फेफड़ों द्वारा

(d) श्वासनली प्रणाली द्वारा


153. मानव शरीर की आंतरिक संरचना के दृश्य के लिए
नियोजित कंप्यूटे ड टोमोग्राफी द्वारा निम्न में से कौन-
सा प्रकार उपयोग किया जाता है? (2007) 158. मनुष्य में, सामान्यतया, निम्नलिखित में से किस भाग में
शुक्राणु डिं ब को निषेचित करता है? (2007)
(a) एक्स-रे
(a) गर्भाशय
(b) ध्वनि तरंगें
(b) फैलोपियन
(c) चुंबकीय अनुनाद
(c) गर्भाशय का निचला भाग
(d) रेडियोआइसोटोप
(d) गर्भाशय का ऊपरी भाग

154. गैलीलियो परियोजना क्या है जो हाल ही में चर्चा में


रही है? (2007) 159. निम्नलिखित में से मानव मस्तिष्क का कौन-सा भाग
निगलने और उल्टी करने का विनियम केंद्र है? (2007)
(a) संयुक्त राज्य अमेरिका द्वारा बनाई गई मिसाइल
शील्ड का एक अंतर्देशीय कार्यक्रम (a) सेरिबैलम

(b) कनाडा की सहायता से भारत द्वारा बनाई गई एक (b) सेरब्र


े म
परियोजना (c) मेडुला ऑबोंगटा
(c) जापान द्वारा बनाई जा रही एक पर्यावरण संरक्षण (d) पोंस
परियोजना

(d) बहु-उपग्रह नेविगेशन यूरोपीय संघ द्वारा बनाई जा


160. निम्नलिखित में से किसका उत्पादन यकृत का
रही एक परियोजना
प्रकार्य है? (2007)

(a) लाइपेज
155. मानव शरीर में छोटी आँत के नीचे तीन संरचनात्मक
(b) यूरिया
भागों की घटती लं बाई के क्रम में निम्नलिखित में
से कौन-सा क्रम सही है? (2007) (c) बलगम

(a) जेजुनम - डुओडेनम - इलियम (d) हाइड्रोक्लोरिक एसिड

(b) इलियम - डुओडेनम - जेजुनम

(c) जेजुनम - इलियम - डुओडेनम 161. मानव तंत्र में निम्नलिखित में से कौन-सा पाचक
एं जाइम नहीं है? (2007)
(d) इलियम - जेजुनम - डुओडेनम
(a) ट्रिप्सिन

143
(b) गैस्ट्रिन क्षेत्र) के साथ मिलान कीजिए और नीचे दिए गए सूची
कूट का प्रयोग कर सही उत्तर का चयन कीजिए: (2006)
(c) इं डिगो और पीला
सूची-I
(d) पीला और बैंगनी
(a) C.N.R. राव

(b) जगदीश भगवती


162. निम्नलिखित में से कौन-से प्रकाश पौधों द्वारा दृढ़ता से
अवशोषित किया जाता है? (2007) (c) G.N. रामचंद्रन

(a) बैंगनी और नारंगी (d) अशोक

(b) नीला और लाल सूची-II

(c) इं डिगो और पीला 1. दूरसंचार

(d) पीला और बैंगनी 2. भौतिकी

3. अर्थशास्त्र

163. निम्नलिखित में से कौन-सा युग्म सही सुमेलित नहीं 4. ठोस अवस्था रसायन विज्ञान और पदार्थ विज्ञान
है? (2007)
कूट:
(a) कॉस्मिक बैकग्राउं ड एक्सप्लोर (COBE): सैटेलाइट
ABCD
प्रोग्राम
(a) 4 3 2 1
(b) फाल्कन: अंडर-सी केबल सिस्टम
(b) 2 1 4 3
(c) डिस्कवरी: स्पेस शटल
(c) 4 1 2 3
(d) अटलांटिस: स्पेस स्टेशन
(d) 2 3 4 1

164. आमाशय और छोटी आँत से यकृत में पोषक तत्वों से भरपूर


रक्त पहुंचाने वाली नलिका का क्या नाम है? (2007) 167. निम्नलिखित कथनों पर विचार कीजिए: (2005)

(a) बाईं यकृत धमनी 1. परासरण की प्रक्रिया के दौरान विलायक सांद्र


विलयन से तनु विलयन की ओर गमन करता है।
(b) यकृत शिरा
2. रिवर्स परासरण में तनु विलयन पर बाह्य दाब लगाया
(c) दाहिनी यकृत धमनी
जाता है।
(d) यकृत पोर्टल शिरा
कौन-सा/से कथन सही है/हैं?

(a) केवल 1
165. मानव शरीर के संदर्भ में निम्नलिखित कथनों पर
(b) केवल 2
विचार कीजिए: (2006)
(c) 1 और 2 दोनों
1. सामान्य पित्त नली अपने अवयवों को आमाशय तक
पहुँचाती है। (d) न तो 1, न ही 2

2. अग्नाशयी वाहिनी अपने अवयवों को ग्रहणी तक


पहुँचाती है। 168. निम्नलिखित कथनों पर विचार कीजिए: (2005)
उपरोक्त में से कौन-सा/से कथन सही है/हैं? 1. डेंगू एक प्रोटोजोआ रोग है जो मच्छरों से फैलता है।
(a) केवल 1 2. रेट्रो-ऑर्बिटल दर्द डेंगू का लक्षण नहीं है।
(b) केवल 2 3. त्वचा पर लाल चकत्ते और नाक व मसूड़ों से खून
(c) 1 और 2 दोनों आना डेंगू के कुछ लक्षण हैं।

(d) न तो 1, न ही 2 कौन-सा/से कथन सही है/हैं?

(a) 1 और 2

166. सूची-I (भारतीय वैज्ञानिक/विद्वान) का सूची-II (कार्य का (b) केवल 3

144
(c) केवल 2 (a) केवल 1

(d) 1 और 3 (b) 1 और 3

(c) 2 और 3

169. निम्नलिखित में से दुधारु पशुओ ं के कौन-से रोग (d) 1, 2 और 3


संक्रामक हैं? (2005)

1. मुँहपका-खुरपका रोग
Answer key
2. एं थ्रेक्स
1) d 2) d 3) d 4) d 5) b 6) a 7) c 8) d 9) d 10) c 21) d
3. ब्लै क क्वार्टर
22) a 23) b 24) d 25) a 26) b 27) a 28) a 29) c 30) d
4. चेचक 11) d 12) d 13) d 14) a 15) b 16) b 17) d 18) b 19) a 20) c
सही उत्तर का चयन कीजिए: 31) c 32) d 33) b 34) d 35) b 36) d 37) c 38) b 39) d
40) d 169 41) c 42) c 43) d 44) b 45) a 46) b 47) c 48)
(a) 1, 2 और 3
a 49) d 50) d 51) c 52) b 53) a 54) c 55) d 56) c 57)
(b) 2, 3 और 4 a 58) b 59) d 60) c 61) c 62) d 63) d 64) d 65) a 66)
(c) 1 और 4 b 67) b 68) a 69) d 70) b 71) b 72) d 73) a 74) d 75) c
76) c 77) c 78) d 79) d 80) c 81) d 82) a 83) a 84) d
(d) 1, 2, 3 और 4
85) b 86) c 87) b 88) c 89) b 90) a 91) b 92) c 93) c
94) b 95) b 96) a 97) b 98) d 99) b 100) c 101) b 102)
170. निम्नलिखित कथनों पर विचार कीजिए: (2005) a 103) c 104) b 105) a 106) a 107) d 108) c 109) a 110)
d 111) c 112) d 113) d 114) b 115) c 116) b 117) c 118) c 119)
1. विशेष रूप से बनाए गए जलयानों में भंडारण या
a 120) b 121) c 122) d 123) c 124) d 125) d 126) b 127) c
परिवहन की सुविधा के लिए अधिक ठं डे तापमान
128) d 129) a 130) a 131) b 132) c 133) d 134) a 135) a
और उच्च दाब के अंतर्गत तरलीकृत प्राकृतिक गैस
136) c 137) a 138) c 139) c 140) c 141) b 142) c 143) c
(LNG) को लिक्विफाई किया जाता है।
144) b 145) a 146) c 147) c 148) b 149) b 150) b 151) c
2. भारत में पहला एलएनजी टर्मि नल हसन में बनाया 152) c 153) a 154) d 155) d 156) c 157) d 158) b 159) c
गया था। 160) b 161) b 162) b 163) d 164) d 165) b 166) a 167) b
3. प्राकृतिक गैस तरल पदार्थों (NGL) को एलपीजी 168) b 169) c 170) b 170
(LPG) से अलग किया जाता है जिनमें ईथेन, प्रोपेन,
ब्यूटे न और प्राकृतिक गैसोलीन घुली होती है।

इनमें से कौन-सा/से कथन सही है/हैं?

contact me @ITAKASHSINGH

Akash Singh

145

You might also like